Está en la página 1de 257

My Collection

of
Maths Olympiad
Materials
- BENNY VARGHESE
Hi

Am BENNY VARGHESE PGT MATHEMATICS JNV PATHANAMTHITTA KERALA

My collection of Maths Olympiad Materials is given below :

The syllabus for Mathematical Olympiad (regional, national and international) is pre-degree college
mathematics. The areas covered are arithmetic of integers, geometry, quadratic equations and expressions,
trigonometry, co-ordinate geometry, system of linear equations, permutations and combination,
factorization of polynomial, inequalities, elementary combinatorics, probability theory and number theory,
finite series and complex numbers and elementary graph theory. The syllabus does not include calculus and
statistics. The major areas from which problems are given are algebra, combinatorics, geometry and
number theory . The syllabus is in a sense spread over Class XI to Class XII levels, but the problems under
each topic involve high level of difficulty and sophistication. The difficulty level increases from RMO to
INMO to IMO.

Major Change in Mathematical Olympiads


Programme 2017-2018
Queries regarding PRMO may be sent by email to prmo@hbcse.tifr.res.in. Queries will not
be replied to individually, but via the FAQ section on this website.

Important Information concerning the Mathematics Olympiad


Programme:
1. The National Board for Higher Mathematics (NBHM) which is in charge of the mathematical
olympiad activity, has entrusted its implementation to the Homi Bhabha Centre for Science
Education (HBCSE) to be carried out in coordination with the MO-Cell and the National
Coordinator appointed by NBHM, supervised by a Committee constituted by NBHM. All major
policy decisions with regard to the Mathematical Olympiads programme are made with the
concurrence of NBHM. The following MAJOR changes are announced this year.
2. A pre-Regional Mathematical Olympiad (PRMO) exam will be held on August 20, 2017 at
different centres all over India. In order to be eligible for the RMO it is mandatory that
students qualify through this centrally administered PRMO. No other independently
administered examinations will be recognized from this year onwards.
3. Eligibility: All Indian students who are born on or after August 1, 1998 and, in addition, are
in Class XI or below are eligible to appear for the PRMO 2017.
4. The PRMO will be a machine-correctable test of 30 questions. Each question has an answer
which is a number with one or two digits. Sample PRMO questions and Sample OMR sheet
showing marked answers can be downloaded from here.
5. The PRMO exam will be organized this year by IAPT (the Indian Association of Physics
Teachers), the same association that also organizes the National Standards Examination,
which is the first step for participation in the International Physics, Chemistry, Biology,
Astronomy and Junior Science olympiads. The website for PRMO is available
at http://www.iapt.org.in/
6. A link to a portal is available at the IAPT webpage where schools can register as centres;
this portal will be open from May 20 to June 20, 2017. Any school with at least 5 registrations
can register on the portal as a “registered centre”. However, the list of approved exam
centres will be issued by IAPT. Students who register through a given school which is a
registered centre may be re-assigned to another nearby exam centre.
7. All Kendriya Vidyalaya, Jawahar Navodaya Vidyalaya and Atomic Energy Central Schools
may register as registered centres regardless of the number of students.
8. The list of registered centres will be available on the IAPT website from June 20, 2017.
9. Students can approach one of the approved registered centres and through the centre
register for the PRMO exam between June 20 and July 25, 2017 for a fee of Rs. 200. Kendriya
Vidyalaya Sangathan has made its schools available as centres free of cost; for KV students
there is a provision to register with payment of a fee of Rs. 100.
10. A carbon copy of the answersheet (OMR) will be given to the student after the exam; students
can check their answers as against the answers which will be uploaded on the HBCSE
website.
11. The top 300 students from each region will be eligible to write the Regional Mathematics
Olympiad exam. For the KV, CBSE, JNV schools which have a countrywide coverage, the
number to be selected is upto 5 per cent of the number of registrations or 300.
12. Each regional co-ordinator (including the co-ordinators for the groups KV, CBSE and JNV)
will receive the list of students eligible to write RMO by September 15th. Each Regional Co-
ordinator shall conduct the RMO in her/his region on October 8th, 2017.
13. Based on the RMO exam, the final list of 30 top students plus the next 5 girl students will be
sent by each regional co-ordinator to HBCSE by November 30, 2017. This should be done
after completing re-evaluation (provision for which is mandatory).
14. HBCSE will announce the list of students eligible to appear for INMO 2018 from all regions
by December 07, 2017.
15. The INMO exam will be held on January 21, 2018 and will be conducted by each regional co-
ordinator and the papers sent to HBCSE.
16. The regional co-ordinators will be sent brochures/posters and will be requested to advertise
the information on their webpages or homepages. As there are major changes, students will
be unaware of these and the support of Regional co-ordinators in this regard is crucial.
17. This is a preliminary announcement and more details will be announced and communicated
to all regional co-ordinators in the next few days. Regional co-ordinators are requested to
circulate the announcement and the weblink widely.

The center registration and student enrollment for all National Standard Examinations will start from
August 15th on wards. This year poster and brochure will be available shortly. For details as of now please
refer to last years students brochure.

ANNOUNCEMENT FOR PRE RMO

IAPT in association with National Board for Higher Mathematics (NBHM) is happy to announce the
first ever Pre Regional Mathematics Olympiad PRMO as the first selection examination for the team to
represent India in International Mathematics Olympiad. For information

please visit http://olympiads.hbcse.tifr.res.in/?p=1447

Important dates to note

Registration of Centers 20th MAY -20th JUNE 2017

Registration of Candidates 20th JUNE- 25th JULY 2017

Down loading of Admit Cards by the Center in-charge 1 AUG -12th AUG 2017

Date of Examination 20th AUG 2017 between 10 AM and 12.30 PM


PLEASE NOTE THE REGISTRATION FOR NATIONAL STANDARD EXAMINATION WILL BE ANNOUNCED LATTER.

The Indian Association of Physics Teachers (IAPT) was established in the year 1984 by the great
visionary, (Late) Dr. D. P. Khandelwal, with active support from some Physics teachers, with the aim of
upgrading the quality of Physics teaching and Physics teachers at all levels. It has now grown into a major
organisation with about 6500 life members spread all over about 1500 organisations throughout the
country including about 100 members from abroad. The members include school, college, university
teachers, research workers, science administrators and science savvy enthusiasts. For its grass root
working, the country is divided into 20 regions, each with a regional council. The apex excecutive council,
co-ordinates and directs the effort at the national level.

All IAPT work is voluntary, No remuneration is paid to its members for any IAPT activity.

NEW ADDRESS OF THE EXAMINATION OFFICE

From 1st July 2016,the IAPT examinations will be at the following address

Prof. G. Venkatesh

IAPT Bangalore Centre 2

Indian Academy Degree

College Hennur Cross, Hennur

Main Road Bangalore-560 043

e- mail : iapt.nse@gmail.com

Helpline Number. 080 - 49087030


National Board for Higher Mathematics &
Homi Bhabha Centre for Science Education

Mathematical Olympiad 2017-18


Winning Olympiad Medals – a dream of young achievers

NOTE
IMO
In 2017-18 PRMO has been introduced as the first step of the 2018
Mathematical Olympiad Programme INMO
2018

RM
201O
Pre-Regional 
 7

PRM
Mathematical Olympiad (PRMO) 201 O
7

For information please visit: http://olympiads.hbcse.tifr.res.in


For registration please visit: http://www.iapt.org.in

Important dates for PRMO 2017


Registration of Centers : 20 May - 20 June 2017
Registration of Candidates : 20 June - 25 July 2017
Downloading of Admit Cards by the Center in-charge : 1 August - 12 August 2017
Date of Examination : 20 August 2017
Time of Examination : 10 AM - 12.30 PM

These are the only Olympiads that lead to participation in the International Mathematical
Olympiad (IMO). No other contests are recognised for this purpose.

Homi Bhabha Centre for Science Education


Tata Institute of Fundamental Research
V. N. Purav Marg, Mankhurd, Mumbai - 400 088
Tel: 022 - 2556 2132, 2556 7711, 2558 0036 Fax: 022 - 2558 5660, 2556 6803
Sample Questions for PRMO 2017
a b
1. Two positive integers a and b are such that a + b = + . What is the value of a2 + b2 ?
b a
[Ans: 02]
2. The equations x2 − 4x + k = 0 and x2 + kx − 4 = 0, where k is a real number, have exactly
one common root. What is the value of k? [Ans: 03]
3. Let P (x) be a non-zero polynomial with integer coefficients. If P (n) is divisible by n for each
positive integer n, what is the value of P (0)? [Ans: 00]
4. A natural number k is such that k 2 < 2014 < (k + 1)2 . What is the largest prime factor of
k? [Ans: 11]

5. How many two-digit positive integers N have the property that the sum of N and the number
obtained reversing the order of the digits of N is a perfect square? [Ans: 08]
6. What is the greatest possible perimeter of a right-angled triangle with integer side lengths if
one of the sides has length 12? [Ans: 84]

7. In rectangle ABCD, AB = 8 and BC = 20. LetP be a point on AD such that ∠BP C = 90◦ .
If r1 , r2 , r3 are the radii of the incircles of triangles AP B, BP C and CP D, what is the value
of r1 + r2 + r3 ? [Ans: 08]
8. Let n be the largest integer that is the product of exactly 3 distinct prime numbers, x, y and
10x + y, where x and y are digits. What is the sum of the digits of n? [Ans: 12]

9. A subset B of the set of first 100 positive integers has the property that no two elements of
B sum to 125. What is the maximum possible number of elements in B? [Ans: 62]
10. The circle ω touches the circle Ω internally at P . The centre O of Ω is outside ω. Let XY
be a diameter of Ω which is also tangent to ω. Assume P Y > P X. Let P Y intersect ω at
Z. If Y Z = 2P Z ,what is the magnitude of ∠P Y X in degrees? [Ans: 15]

1
Instruction: Invigilator to please fold at perforation and then tear after the test is over and handover the carbon copy to the candidate.

IAPT-PRMO OMR ANSWER SHEET


20 AUGUST 2017 ORIGINAL COPY
1. STUDENT’S NAME STUDENT NAME
2. ROLL NO. 3. DATE OF BIRTH INSTRUCTIONS FOR MARKING

A P K 0 1 2 3 4 4 4 0 8 1 1 2 0 0 4 1. MARKING SHOULD BE DARK AND COMPLETELY FILL


USING BLUE/BLACK BALL POINT PEN ONLY AS SHOWN
A A K 0 0 0 0 0 0 0 0 0 0 0 1998 BELOW.
B B M 1 1 1 1 1 1 1 1 1 1 1 1999 WRONG METHODS
C C 2 2 2 2 2 2 2 2 2 2 2000
D D 3 3 3 3 3 3 3 3 3 3 2001 0 1 2 3 4 5 6 7 8 9
E E 4 4 4 4 4 4 4 4 4 2002 0 1 2 3 4 5 6 7 8 9
F F 5 5 5 5 5 5 5 5 5 2003
G G 6 6 6 6 6 6 6 6 6 0 1 2 3 4 5 6 7 8 9
2004
H H 7 7 7 7 7 7 7 7 7
I I 8 8 8 8 8 8 8 8 8
CORRECT METHOD
J J 9 9 9 9 9 9 9 9 9
K K 0 1 2 3 4 5 6 7 8 9
L L
M M
4. STANDARD 2. MARK YOUR ANSWER ONLY IN THE APPROPRIATE SPACE.

PY
N N 3. ANY STRAY MARKING/TAMPERING WITH THE ANSWER
O O SHEET WOULD BE TREATED AS MALPRACTICE.
P P
IX
Q Q
R R
5. GENDER
VIII VIII
S S
T T IX IX MALE

SHEET NO.
U U
V V X X
W
X
Y
Z
W
X
Y
Z
Xi XI CO M

F
MALE

FEMALE

6. NAME OF THE INSTITUTE

KENDRIYA VIDYALAYA NO. 1


E
MARK YOUR ANSWERS
Q. NO ANSWERS Q. NO ANSWERS Q. NO ANSWERS
PL

0 1 2 3 4 5 6 7 8 9 0 1 2 3 4 5 6 7 8 9 0 1 2 3 4 5 6 7 8 9
1 0 1 2 3 4 5 6 7 8 9 11 0 1 2 3 4 5 6 7 8 9 21 0 1 2 3 4 5 6 7 8 9

0 1 2 3 4 5 6 7 8 9 0 1 2 3 4 5 6 7 8 9 0 1 2 3 4 5 6 7 8 9
2 0 1 2 3 4 5 6 7 8 9 12 0 1 2 3 4 5 6 7 8 9 22 0 1 2 3 4 5 6 7 8 9

0 1 2 3 4 5 6 7 8 9 0 1 2 3 4 5 6 7 8 9 0 1 2 3 4 5 6 7 8 9
3 0 1 2 3 4 5 6 7 8 9 13 0 1 2 3 4 5 6 7 8 9 23 0 1 2 3 4 5 6 7 8 9
M

0 1 2 3 4 5 6 7 8 9 0 1 2 3 4 5 6 7 8 9 0 1 2 3 4 5 6 7 8 9
4 0 1 2 3 4 5 6 7 8 9 14 0 1 2 3 4 5 6 7 8 9 24 0 1 2 3 4 5 6 7 8 9

0 1 2 3 4 5 6 7 8 9 0 1 2 3 4 5 6 7 8 9 0 1 2 3 4 5 6 7 8 9
5 15 25
SA

0 1 2 3 4 5 6 7 8 9 0 1 2 3 4 5 6 7 8 9 0 1 2 3 4 5 6 7 8 9

0 1 2 3 4 5 6 7 8 9 0 1 2 3 4 5 6 7 8 9 0 1 2 3 4 5 6 7 8 9
6 0 1 2 3 4 5 6 7 8 9 16 0 1 2 3 4 5 6 7 8 9 26 0 1 2 3 4 5 6 7 8 9

0 1 2 3 4 5 6 7 8 9 0 1 2 3 4 5 6 7 8 9 0 1 2 3 4 5 6 7 8 9
7 0 1 2 3 4 5 6 7 8 9
17 0 1 2 3 4 5 6 7 8 9
27 0 1 2 3 4 5 6 7 8 9

0 1 2 3 4 5 6 7 8 9 0 1 2 3 4 5 6 7 8 9 0 1 2 3 4 5 6 7 8 9
8 0 1 2 3 4 5 6 7 8 9
18 0 1 2 3 4 5 6 7 8 9
28 0 1 2 3 4 5 6 7 8 9

0 1 2 3 4 5 6 7 8 9 0 1 2 3 4 5 6 7 8 9 0 1 2 3 4 5 6 7 8 9
9 0 1 2 3 4 5 6 7 8 9
19 0 1 2 3 4 5 6 7 8 9
29 0 1 2 3 4 5 6 7 8 9

0 1 2 3 4 5 6 7 8 9 0 1 2 3 4 5 6 7 8 9 0 1 2 3 4 5 6 7 8 9
10 0 1 2 3 4 5 6 7 8 9
20 0 1 2 3 4 5 6 7 8 9
30 0 1 2 3 4 5 6 7 8 9

STUDENT’S INVIGILATOR’S
SIGNATURE SIGNATURE
NATIONAL BOARD FOR HIGHER MATHEMATICS
AND
HOMI BHABHA CENTRE FOR SCIENCE EDUCATION
TATA INSTITUTE OF FUNDAMENTAL RESEARCH

Pre-REGIONAL MATHEMATICAL OLYMPIAD, 2015


Mumbai Region

October 4, 2015

QUESTION PAPER SET: A

• There are 20 questions in this question paper. Each question carries 5 marks.
• Answer all questions.
• Time allotted: 2.5 hours.

QUESTIONS
3 1
1. A man walks a certain distance and rides back in 3 hours; he could ride both ways in 2
4 2
hours. How many hours would it take him to walk both ways? [5]
2. Positive integers a and b are such that a + b = a/b + b/a. What is the value of a2 + b2 ? [2]
3. The equations x2 − 4x + k = 0 and x2 + kx − 4 = 0, where k is a real number, have exactly
one common root. What is the value of k? [3]
4. Let P (x) be a non-zero polynomial with integer coefficients. If P (n) is divisible by n for each
positive integer n, what is the value of P (0)? [0]
5. How many line segments have both their endpoints located at the vertices of a given cube?
[28]
6. Let E(n) denote the sum of the even digits of n. For example, E(1243) = 2 + 4 = 6. What
is the value of E(1) + E(2) + E(3) + · · · + E(100)? [400]
7. How many two-digit positive integers N have the property that the sum of N and the number
obtained by reversing the order of the digits of N is a perfect square? [8]
8. The figure below shows a broken piece of a circular plate made of glass.

A B
C

C is the midpoint of AB, and D is the midpoint of arc AB. Given that AB = 24 cm and
CD = 6 cm, what is the radius of the plate in centimetres? (The figure is not drawn to
scale.) [15]

1
9. A 2 × 3 rectangle and a 3 × 4 rectangle are contained within a square without overlapping
at any interior point, and the sides of the square are parallel to the sides of the two given
rectangles. What is the smallest possible area of the square? [25]
10. What is the greatest possible perimeter of a right-angled triangle with integer side lengths if
one of the sides has length 12 ? [84]

11. In rectangle ABCD, AB = 8 and BC = 20. Let P be a point on AD such that 6 BP C = 90◦ .
If r1 , r2 , r3 are the radii of the incircles of triangles AP B, BP C and CP D, what is the value
of r1 + r2 + r3 ? [8]
12. Let a, b, and c be real numbers such that a − 7b + 8c = 4 and 8a + 4b − c = 7. What is the
value of a2 − b2 + c2 ? [1]

13. Let n be the largest integer that is the product of exactly 3 distinct prime numbers, x, y and
10x + y, where x and y are digits. What is the sum of the digits of n? [12]
14. At a party, each man danced with exactly four women and each woman danced with exactly
three men. Nine men attended the party. How many women attended the party? [12]

15. If 3x + 2y = 985 and 3x − 2y = 473, what is the value of xy? [48]


16. In acute-angled triangle ABC, let D be the foot of the altitude from A, and E be the midpoint
of BC. Let F be the midpoint of AC. Suppose 6 BAE = 40◦ . If 6 DAE = 6 DF E, what is
the magnitude of 6 ADF in degrees? [40]

17. A subset B of the set of first 100 positive integers has the property that no two elements of
B sum to 125. What is the maximum possible number of elements in B? [62]
18. Let a, b and c be such that a + b + c = 0 and

a2 b2 c2
P = + 2 + 2
2a2 + bc 2b + ca 2c + ab

is defined. What is the value of P ? [1]


19. The circle ω touches the circle Ω internally at P . The centre O of Ω is outside ω. Let XY
be a diameter of Ω which is also tangent to ω. Assume P Y > P X. Let P Y intersect ω at
Z. If Y Z = 2P Z, what is the magnitude of 6 P Y X in degrees? [15]
20. The digits of a positive integer n are four consecutive integers in decreasing order when read
from left to right. What is the sum of the possible remainders when n is divided by 37?
[217]

2
NATIONAL BOARD FOR HIGHER MATHEMATICS
AND
HOMI BHABHA CENTRE FOR SCIENCE EDUCATION
TATA INSTITUTE OF FUNDAMENTAL RESEARCH

Pre-REGIONAL MATHEMATICAL OLYMPIAD, 2015


Mumbai Region

October 4, 2015

QUESTION PAPER SET: B

• There are 20 questions in this question paper. Each question carries 5 marks.
• Answer all questions.
• Time allotted: 2.5 hours.

QUESTIONS
3 1
1. A man walks a certain distance and rides back in 3 hours; he could ride both ways in 2
4 2
hours. How many hours would it take him to walk both ways? [5]
2. The equations x2 − 4x + k = 0 and x2 + kx − 4 = 0, where k is a real number, have exactly
one common root. What is the value of k? [3]
3. Positive integers a and b are such that a + b = a/b + b/a. What is the value of a2 + b2 ? [2]
4. How many line segments have both their endpoints located at the vertices of a given cube?
[28]
5. Let P (x) be a non-zero polynomial with integer coefficients. If P (n) is divisible by n for each
positive integer n, what is the value of P (0)? [0]
6. How many two-digit positive integers N have the property that the sum of N and the number
obtained by reversing the order of the digits of N is a perfect square? [8]
7. Let E(n) denote the sum of the even digits of n. For example, E(1243) = 2 + 4 = 6. What
is the value of E(1) + E(2) + E(3) + · · · + E(100)? [400]
8. The figure below shows a broken piece of a circular plate made of glass.

A B
C

C is the midpoint of AB, and D is the midpoint of arc AB. Given that AB = 24 cm and
CD = 6 cm, what is the radius of the plate in centimetres? (The figure is not drawn to
scale.) [15]

1
9. What is the greatest possible perimeter of a right-angled triangle with integer side lengths if
one of the sides has length 12 ? [84]
10. A 2 × 3 rectangle and a 3 × 4 rectangle are contained within a square without overlapping
at any interior point, and the sides of the square are parallel to the sides of the two given
rectangles. What is the smallest possible area of the square? [25]

11. Let a, b, and c be real numbers such that a − 7b + 8c = 4 and 8a + 4b − c = 7. What is the
value of a2 − b2 + c2 ? [1]
12. In rectangle ABCD, AB = 8 and BC = 20. Let P be a point on AD such that 6 BP C = 90◦ .
If r1 , r2 , r3 are the radii of the incircles of triangles AP B, BP C and CP D, what is the value
of r1 + r2 + r3 ? [8]

13. At a party, each man danced with exactly four women and each woman danced with exactly
three men. Nine men attended the party. How many women attended the party? [12]
14. If 3x + 2y = 985 and 3x − 2y = 473, what is the value of xy? [48]
15. Let n be the largest integer that is the product of exactly 3 distinct prime numbers, x, y and
10x + y, where x and y are digits. What is the sum of the digits of n? [12]
16. In acute-angled triangle ABC, let D be the foot of the altitude from A, and E be the midpoint
of BC. Let F be the midpoint of AC. Suppose 6 BAE = 40◦ . If 6 DAE = 6 DF E, what is
the magnitude of 6 ADF in degrees? [40]

17. Let a, b and c be such that a + b + c = 0 and

a2 b2 c2
P = + 2 + 2
2a2 + bc 2b + ca 2c + ab

is defined. What is the value of P ? [1]


18. A subset B of the set of first 100 positive integers has the property that no two elements of
B sum to 125. What is the maximum possible number of elements in B? [62]
19. The digits of a positive integer n are four consecutive integers in decreasing order when read
from left to right. What is the sum of the possible remainders when n is divided by 37?
[217]
20. The circle ω touches the circle Ω internally at P . The centre O of Ω is outside ω. Let XY
be a diameter of Ω which is also tangent to ω. Assume P Y > P X. Let P Y intersect ω at
Z. If Y Z = 2P Z, what is the magnitude of 6 P Y X in degrees? [15]

2
Problems and Solutions: CRMO-2012, Paper 1
1. Let ABC be a triangle and D be a point on the segment BC such that DC = 2BD.
Let E be the mid-point of AC. Let AD and BE intersect in P . Determine the ratios
BP/P E and AP/P D.

Solution: Let F be the midpoint of DC,


so that D, F are points of trisection of BC.
Now in triangle CAD, F is the mid-point of
CD and E is that of CA. Hence CF/F D =
1 = CE/EA. Thus EF k AD. Hence we find
that EF k P D. Hence BP/P E = BD/DF .
But BD = DF . We obtain BP/P E = 1.

In triangle ACD, since EF k AD we get EF/AD = CF/CD = 1/2. Thus AD = 2EF .


But P D/EF = BD/BF = 1/2. Hence EF = 2P D. Therefore A
This gives
AP = AD − P D = 3P D.
We obtain AP/P D = 3.
(Coordinate geometry proof is also possible.)

2. Let a, b, c be positive integers such that a divides b3 , b divides c3 and c divides a3 .


Prove that abc divides (a + b + c)13 .
Solution: If a prime p divides a, then p | b3 and hence p | b. This implies that p | c3
and hence p | c. Thus every prime dividing a also divides b and c. By symmetry,
this is true for b and c as well. We conclude that a, b, c have the same set of prime
divisors.
Let px || a, py || b and pz || c. (Here we write px || a to mean px | a and px+1 6| a.) We may
assume min{x, y, z} = x. Now b | c3 implies that y ≤ 3z; c | a3 implies that z ≤ 3x. We
obtain
y ≤ 3z ≤ 9x.
Thus x + y + z ≤ x + 3x + 9x = 13x. Hence the maximum power of p that divides abc
is x + y + z ≤ 13x. Since x is the minimum among x, y, z, whence px divides each of
a, b, c. Hence px divides a + b + c. This implies that p13x divides (a + b + c)13 . Since
x + y + z ≤ 13x, it follows that px+y+z divides (a + b + c)13 . This is true of any prime p
dividing a, b, c. Hence abc divides (a + b + c)13 .

3. Let a and b be positive real numbers such that a + b = 1. Prove that

aa bb + ab ba ≤ 1.

Solution: Observe
1 = a + b = aa+b ba+b = aa bb + ba bb .
Hence
1 − aa bb − ab ba = aa bb + ba bb − aa bb − ab ba = (aa − ba )(ab − bb )
Now if a ≤ b, then aa ≤ ba and ab ≤ bb . If a ≥ b, then aa ≥ ba and ab ≥ bb . Hence the
product is nonnegative for all positive a and b. It follows that

aa bb + ab ba ≤ 1.
4. Let X = {1, 2, 3, . . . , 10}. Find the the number of pairs {A, B} such that A ⊆ X,
B ⊆ X, A 6= B and A ∩ B = {2, 3, 5, 7}.

Solution: Let A ∪ B = Y , B \ A = M , A \ B = N and X \ Y = L. Then X is the


disjoint union of M , N , L and A ∩ B. Now A ∩ B = {2, 3, 5, 7} is fixed. The remaining
six elements 1, 4, 6, 8, 9, 10 can be distributed in any of the remaining sets M , N , L.
This can be done in 36 ways. Of these if all the elements are in the set L, then
A = B = {2, 3, 5, 7} and which this case has to be deleted. Hence the total number
of pairs {A, B} such that A ⊆ X, B ⊆ X, A 6= B and A ∩ B = {2, 3, 5, 7} is 36 − 1.

5. Let ABC be a triangle. Let BE and CF be internal angle bisectors of ∠B and ∠C


respectively with E on AC and F on AB. Suppose X is a point on the segment CF
such that AX ⊥ CF ; and Y is a point on the segment BE such that AY ⊥ BE. Prove
that XY = (b + c − a)/2 where BC = a, CA = b and AB = c.

Solution: Produce AX and AY to meet


BC is X 0 and Y 0 respectively. Since BY
bisects ∠ABY 0 and BY ⊥ AY 0 it follows
that BA = BY 0 and AY = Y Y 0 . Similarly,
CA = CX 0 and AX = XX 0 . Thus X and
Y are mid-points of AX 0 and AY 0 respec-
tively. By mid-point theorem XY = X 0 Y 0 /2.
But

X 0 Y 0 = X 0 C + Y 0 B − BC = AC + AB − BC = b + c − a.
Hence XY = (b + c − a)/2.

6. Let a and b be real numbers such that a 6= 0. Prove that not all the roots of ax4 +
bx3 + x2 + x + 1 = 0 can be real.

Solution: Let α1 , α2 , α3 , α4 be the roots of ax4 + bx3 + x2 + x + 1 = 0. Observe none of


these is zero since their product is 1/a. Then the roots of x4 + x3 + x2 + bx + a = 0 are
1 1 1 1
β1 = , β2 = , β3 = , β4 = .
α1 α2 α3 α4
We have
4
X X
βj = −1, 1 ≤ j < k ≤ 4βj βk = 1.
j=1

Hence  2  
4
X 4
X X
βj2 =  βj  − 2  βj βk  = 1 − 2 = −1.
j=1 j=1 1≤j<k≤4

This shows that not all βj can be real. Hence not all αj ’s can be real.

———-00000———-
Problems and Solutions: CRMO-2012, Paper 2
1. Let ABCD be a unit square. Draw a quadrant of a circle with A as centre and B, D
as end points of the arc. Similarly, draw a quadrant of a circle with B as centre and
A, C as end points of the arc. Inscribe a circle Γ touching the arc AC internally, the
arc BD internally and also touching the side AB. Find the radius of the circle Γ.
Solution: Let O be the centre of Γ. By
symmetry O is on the perpendicular bi-
sector of AB. Draw OE ⊥ AB. Then
BE = AB/2 = 1/2. If r is the radius of Γ,
we see that OB = 1 − r, and OE = r. Using
Pythagoras’ theorem
2
(1 − r)2 = r2 + 21 .
Simplification gives r = 3/8.
2. Let a, b, c be positive integers such that a divides b4 , b divides c4 and c divides a4 .
Prove that abc divides (a + b + c)21 .
Solution: If a prime p divides a, then p | b4 and hence p | b. This implies that p | c4
and hence p | c. Thus every prime dividing a also divides b and c. By symmetry,
this is true for b and c as well. We conclude that a, b, c have the same set of prime
divisors.
Let px || a, py || b and pz || c. (Here we write px || a to mean px | a and px+1 6| a.) We may
assume min{x, y, z} = x. Now b | c4 implies that y ≤ 4z; c | a4 implies that z ≤ 4x. We
obtain
y ≤ 4z ≤ 16x.
Thus x + y + z ≤ x + 4x + 16x = 21x. Hence the maximum power of p that divides abc
is x + y + z ≤ 21x. Since x is the minimum among x, y, z, px divides a, b, c. Hence px
divides a + b + c. This implies that p21x divides (a + b + c)21 . Since x + y + z ≤ 21x,
it follows that px+y+z divides (a + b + c)21 . This is true of any prime p dividing a, b, c.
Hence abc divides (a + b + c)21 .

3. Let a and b be positive real numbers such that a + b = 1. Prove that

aa bb + ab ba ≤ 1.

Solution: Observe
1 = a + b = aa+b ba+b = aa bb + ba bb .
Hence
1 − aa bb − ab ba = aa bb + ba bb − aa bb − ab ba = (aa − ba )(ab − bb )
Now if a ≤ b, then aa ≤ ba and ab ≤ bb . If a ≥ b, then aa ≥ ba and ab ≥ bb . Hence the
product is nonnegative for all positive a and b. It follows that

aa bb + ab ba ≤ 1.

4. Let X = {1, 2, 3, . . . , 12}. Find the the number of pairs {A, B} such that A ⊆ X,
B ⊆ X, A 6= B and A ∩ B = {2, 3, 5, 7, 8}.

Solution: Let A ∪ B = Y , B \ A = M , A \ B = N and X \ Y = L. Then X is the disjoint


union of M , N , L and A ∩ B. Now A ∩ B = {2, 3, 5, 7, 8} is fixed. The remaining seven
elements 1, 4, 6, 9, 10, 11, 12 can be distributed in any of the remaining sets M , N , L.
This can be done in 37 ways. Of these if all the elements are in the set L, then
A = B = {2, 3, 5, 7, 8} and this case has to be omitted. Hence the total number of
pairs {A, B} such that A ⊆ X, B ⊆ X, A 6= B and A ∩ B = {2, 3, 5, 7, 8} is 37 − 1.

5. Let ABC be a triangle. Let D, E be a points on the segment BC such that BD =


DE = EC. Let F be the mid-point of AC. Let BF intersect AD in P and AE in Q
respectively. Determine BP/P Q.
Solution: Let D be the mid-point of BE.
Join AD and let it intersect BF in P . Ex-
tend CQ and EP to meet AB in S and T
respectively. Now

BS [BQC] [BQC]/[AQB]
= =
SA [AQC] [AQC]/[AQB]
CF/F A 1
= = = 2.
EC/BE 1/2
Similarly,
AQ [ABQ] [ACQ] [ABQ] + [ACQ] [ABQ] [ACQ] AF AS 1 3
= = = = + = + =1+ = .
QE [EBQ] [ECQ] [BCQ] [BCQ] [BCQ] FC SB 2 2
And
AT [AP E] [AP E] [AP B] DE AQ 3 3
= = · = · =1· = .
TB [BP E] [AP B] [BP E] DB QE 2 2
Finally,
BP [BP E] [BP A] [BP E] + [BP A] [BP E] [BP A] BT BD 2 5
= = = = + = + = +1 = .
PQ [QP E] [AP E] [AP E] [AP E] [AP E] T A DE 3 3

(Note: BS/SA, AT /T B can also be obtained using Ceva’s theorem. A solution can
also be obtained using coordinate geometry.)

6. Show that for all real numbers x, y, z such that x + y + z = 0 and xy + yz + zx = −3,
the expression x3 y + y 3 z + z 3 x is a constant.
Solution: Consider the equation whose roots are x, y, z:
(t − x)(t − y)(t − z) = 0.
This gives t3 − 3t − λ = 0, where λ = xyz. Since x, y, z are roots of this equation, we
have
x3 − 3x − λ = 0, y 3 − 3y − λ = 0, z 3 − 3z − λ = 0.
Multiplying the first by y, the second by z and the third by x, we obtain
x3 y − 3xy − λy = 0,
y 3 z − 3yz − λz = 0,
z 3 x − 3zx − λx = 0.
Adding we obtain
x3 y + y 3 z + z 3 x − 3(xy + yz + zx) − λ(x + y + z) = 0.
This simplifies to
x3 y + y 3 z + z 3 x = −9.
(Here one may also solve for y and z in terms of x and substitute these values in
x3 y + y 3 z + z 3 x to get −9.)

———-00000———-
Problems and Solutions: CRMO-2012, Paper 3
1. Let ABCD be a unit square. Draw a quadrant of a circle with A as centre and B, D
as end points of the arc. Similarly, draw a quadrant of a circle with B as centre
and A, C as end points of the arc. Inscribe a circle Γ touching the arcs AC and BD
both externally and also touching the side CD. Find the radius of the circle Γ.
Solution: Let O be the centre of Γ. By sym-
metry O is on the perpendicular bisector of
CD. Draw OL ⊥ CD and OK ⊥ BC. Then
OK = CL = CD/2 = 1/2. If r is the radius
of Γ, we see that BK = 1 − r, and OE = r.
Using Pythagoras’ theorem
2
(1 + r)2 = (1 − r)2 + 12 .
Simplification gives r = 1/16.
2. Let a, b, c be positive integers such that a divides b5 , b divides c5 and c divides a5 .
Prove that abc divides (a + b + c)31 .
Solution: If a prime p divides a, then p | b5 and hence p | b. This implies that p | c4
and hence p | c. Thus every prime dividing a also divides b and c. By symmetry,
this is true for b and c as well. We conclude that a, b, c have the same set of prime
divisors.
Let px || a, py || b and pz || c. (Here we write px || a to mean px | a and px+1 6| a.) We may
assume min{x, y, z} = x. Now b | c5 implies that y ≤ 5z; c | a5 implies that z ≤ 5x. We
obtain
y ≤ 5z ≤ 25x.
Thus x + y + z ≤ x + 5x + 25x = 31x. Hence the maximum power of p that divides abc
is x + y + z ≤ 31x. Since x is the minimum among x, y, z, px divides a, b, c. Hence px
divides a + b + c. This implies that p31x divides (a + b + c)21 . Since x + y + z ≤ 31x,
it follows that px+y+z divides (a + b + c)31 . This is true of any prime p dividing a, b, c.
Hence abc divides (a + b + c)31 .

3. Let a and b be positive real numbers such that a + b = 1. Prove that

aa bb + ab ba ≤ 1.

Solution: Observe
1 = a + b = aa+b ba+b = aa bb + ba bb .
Hence
1 − aa bb − ab ba = aa bb + ba bb − aa bb − ab ba = (aa − ba )(ab − bb )
Now if a ≤ b, then aa ≤ ba and ab ≤ bb . If a ≥ b, then aa ≥ ba and ab ≥ bb . Hence the
product is nonnegative for all poitive a and b. It follows that

aa bb + ab ba ≤ 1.

4. Let X = {1, 2, 3, . . . , 10}. Find the the number of pairs {A, B} such that A ⊆ X,
B ⊆ X, A 6= B and A ∩ B = {5, 7, 8}.

Solution: Let A ∪ B = Y , B \ A = M , A \ B = N and X \ Y = L. Then X is the


disjoint union of M , N , L and A ∩ B. Now A ∩ B = {5, 7, 8} is fixed. The remaining
seven elements 1, 2, 3, 4, 6, 9, 10 can be distributed in any of the remaining sets M ,
N , L. This can be done in 37 ways. Of these if all the elements are in the set L, then
A = B = {5, 7, 8} and this case has to be omitted. Hence the total number of pairs
{A, B} such that A ⊆ X, B ⊆ X, A 6= B and A ∩ B = {5, 7, 8} is 37 − 1.

5. Let ABC be a triangle. Let D, E be a points on the segment BC such that BD =


DE = EC. Let F be the mid-point of AC. Let BF intersect AD in P and AE in
Q respectively. Determine the ratio of the area of the triangle AP Q to that of the
quadrilateral P DEQ.
Solution: If we can find [AP Q]/[ADE],
then we can get the required ratio as

[AP Q] [AP Q]
=
[P DEQ] [ADE] − [AP Q]
1
=  .
[ADE]/[AP Q] − 1

Now draw P M ⊥ AE and DL ⊥ AE. Ob-


serve
[AP Q] = 12 AQ · P M , [ADE] = 12 AE · DL.

Further, since P M k DL, we also get P M/DL = AP/AD. Using these we obtain

[AP Q] AP AQ
= · .
[ADE] AD AE
We have
AQ [ABQ] [ACQ] [ABQ] + [ACQ] [ABQ] [ACQ] AF AS
= = = = + = + .
QE [EBQ] [ECQ] [BCQ] [BCQ] [BCQ] FC SB
However
BS [BQC] [BQC]/[AQB] CF/F A 1
= = = = = 2.
SA [AQC] [AQC]/[AQB] EC/BE 1/2
Besides AF/F C = 1. We obtain
AQ AF AS 1 3 AE 3 5 AQ 3
= + =1+ = , =1+ = , = .
QE FC SB 2 2 QE 2 2 AE 5
Since EF k AD (since DE/EC = AF/F C = 1), we get AD = 2EF . Since EF k P D, we
also have P D/EF = BD/DE = 1/2. Hence EF = 2P D. Thus AD = 4P D. This gives
and AP/P D = 3 and AP/AD = 3/4. Thus

[AP Q] AP AQ 3 3 9
= · = · = .
[ADE] AD AE 4 5 20
Finally,
[AP Q] 1 1 9
=  = = .
[P DEQ] [ADE]/[AP Q] − 1 (20/9) − 1 11
(Note: BS/SA can also be obtained using Ceva’s theorem. Coordinate geometry
solution can also be obtained.)

6. Find all positive integers n such that 32n + 3n2 + 7 is a perfect square.
Solution: If 32n + 3n2 + 7 = b2 for some natural number b, then b2 > 32n so that
b > 3n . This implies that b ≥ 3n + 1. Thus
32n + 3n2 + 7 = b2 ≥ (3n + 1)2 = 32n + 2 · 3n + 1.
This shows that 2 · 3n≤ 3n2 + 6. If n ≥ 3, this cannot hold. One can prove this eithe
by induction or by direct argument:
If n ≥ 3, then
 
2 · 3n = 2(1 + 2)n = 2 1 + 2n + n(n − 1)/2) · 22 + · · · > 2 + 4n + 4n2 − 4n
= 3n2 + (n2 + 2) ≥ 3n2 + 11 > 3n2 + 6.

Hence n = 1 or 2.
If n = 1, then 32n + 3n2 + 7 = 19 and this is not a perfect square. If n = 2, we obtain
32n + 3n2 + 7 = 81 + 12 + 7 = 100 = 102 . Hence n = 2 is the only solution.

———-00000———-
Problems and Solutions: CRMO-2012, Paper 4
1. Let ABCD be a unit square. Draw a quadrant of a circle with A as centre and B, D
as end points of the arc. Similarly, draw a quadrant of a circle with B as centre and
A, C as end points of the arc. Inscribe a circle Γ touching the arc AC externally, the
arc BD internally and also touching the side AD. Find the radius of the circle Γ.
Solution: Let O be the centre of Γ and r
its radius. Draw OP ⊥ AD and OQ ⊥ AB.
Then OP = r, OQ2 = OA2 − r2 = (1 − r)2 −
r2 = 1 − 2r. We also have OB = 1 + r and
BQ = 1 − r. Using Pythagoras’ theorem we
get
(1 + r)2 = (1 − r)2 + 1 − 2r.
Simplification gives r = 1/6.
2. Let a, b, c be positive integers such that a divides b2 , b divides c2 and c divides a2 .
Prove that abc divides (a + b + c)7 .
Solution: If a prime p divides a, then p | b2 and hence p | b. This implies that p | c2
and hence p | c. Thus every prime dividing a also divides b and c. By symmetry,
this is true for b and c as well. We conclude that a, b, c have the same set of prime
divisors.
Let px || a, py || b and pz || c. (Here we write px || a to mean px | a and px+1 6| a.) We may
assume min{x, y, z} = x. Now b | c2 implies that y ≤ 2z; c | a2 implies that z ≤ 2x. We
obtain
y ≤ 2z ≤ 4x.
Thus x + y + z ≤ x + 2x + 4x = 7x. Hence the maximum power of p that divides abc
is x + y + z ≤ 7x. Since x is the minimum among x, y, z, px divides a, b, c. Hence px
divides a + b + c. This implies that p7x divides (a + b + c)7 . Since x + y + z ≤ 7x, it
follows that px+y+z divides (a + b + c)7 . This is true of any prime p dividing a, b, c.
Hence abc divides (a + b + c)7 .

3. Let a and b be positive real numbers such that a + b = 1. Prove that

aa bb + ab ba ≤ 1.

Solution: Observe
1 = a + b = aa+b ba+b = aa bb + ba bb .
Hence
1 − aa bb − ab ba = aa bb + ba bb − aa bb − ab ba = (aa − ba )(ab − bb )
Now if a ≤ b, then aa ≤ ba and ab ≤ bb . If a ≥ b, then aa ≥ ba and ab ≥ bb . Hence the
product is nonnegative for all positive a and b. It follows that

aa bb + ab ba ≤ 1.

4. Let X = {1, 2, 3, . . . , 11}. Find the the number of pairs {A, B} such that A ⊆ X,
B ⊆ X, A 6= B and A ∩ B = {4, 5, 7, 8, 9, 10}.

Solution: Let A ∪ B = Y , B \ A = M , A \ B = N and X \ Y = L. Then X is the


disjoint union of M , N , L and A ∩ B. Now A ∩ B = {4, 5, 7, 8, 9, 10} is fixed. The
remaining 5 elements 1, 2, 3, 6, 11 can be distributed in any of the remaining sets M ,
N , L. This can be done in 35 ways. Of these if all the elements are in the set L, then
A = B = {4, 5, 7, 8, 9, 10} and this case has to be omitted. Hence the total number of
pairs {A, B} such that A ⊆ X, B ⊆ X, A 6= B and A ∩ B = {4, 5, 7, 8, 9, 10} is 35 − 1.

5. Let ABC be a triangle. Let E be a point on the segment BC such that BE = 2EC.
Let F be the mid-point of AC. Let BF intersect AE in Q. Determine BQ/QF .
Solution: Let CQ and ET meet AB in S
and T respectively. We have

[SBC] BS [SBQ]
= = .
[ASC] SA [ASQ]

Using componendo by dividendo, we ob-


tain
BS [SBC] − [SBQ] [BQC]
= = .
SA [ASC] − [ASQ] [AQC]
Similarly, We can prove

BE [BQA] CF [CQB]
= , = .
EC [CQA] FA [AQB]

But BD = DE = EC implies that BE/EC = 2; CF = F A gives CF/F A = 1. Thus

BS [BQC] [BQC]/[AQB] CF/F A 1


= = = = = 2.
SA [AQC] [AQC]/[AQB] EC/BE 1/2

Now
BQ [BQC] [BQA] [BQC] + [BQA] [BQC] + [BQA]
= = = = .
QF [F QC] [F QA] [F QC] + [F QA] [AQC]
This gives

BQ [BQC] + [BQA] [BQC] [BQA] BS BE


= = + = + = 2 + 2 = 4.
QF [AQC] [AQC] [AQC] SA EC

(Note: BS/SA can also be obtained using Ceva’s theorem. One can also obtain the
result by coordinate geometry.)

6. Solve the system of equations for positive real numbers:


1 x 1 y 1 z
= + 1, = + 1, = + 1.
xy z yz x zx y

Solution: The given system reduces to


z = x2 y + xyz, x = y z + xyz, y = z 2 x + xyz.
Hence
z − x2 y = x − y 2 z = y − z 2 x.
If x = y, then y 2 z = z 2 x and hence x2 z = z 2 x. This implies that z = x = y. Similarly,
x = z implies that x = z = y. Hence if any two of x, y, z are equal, then all are equal.
Suppose no two of x, y, z are equal. We may take x is the largest among x, y, z so
that x > y and x > z. Here we have two possibilities: y > z and z > y.
Suppose x > y > z. Now z − x2 y = x − y 2 z = y − z 2 x shows that
y 2 z > z 2 x > x2 y.
But y 2 z > z 2 x and z 2 x > x2 y give y 2 > zx and z 2 > xy. Hence
(y 2 )(z 2 ) > (zx)(xy).
This gives yz > x . Thus x < xyz = (xz)y < (y 2 )y = y 3 . This forces x < y contradict-
2 3

ing x > y.
Similarly, we arrive at a contradiction if x > z > y. The only possibility is x = y = z.

For x = y = z, we get only one equation x2 = 1/2. Since x > 0, x = 1/ 2 = y = z.

———-0000———-
Paper 1 Regional Mathematical Olympiad 2013 December 1, 2013

1. Let ABC be an acute-angled triangle. The circle Γ with BC as diameter intersects AB and
AC again at P and Q, respectively. Determine ∠BAC given that the orthocentre of triangle
AP Q lies on Γ.

2. Let f (x) = x3 + ax2 + bx + c and g(x) = x3 + bx2 + cx + a, where a, b, c are integers with
c 6= 0. Suppose that the following conditions hold:

(a) f (1) = 0;
(b) the roots of g(x) = 0 are the squares of the roots of f (x) = 0.

Find the value of a2013 + b2013 + c2013 .

3. Find all primes p and q such that p divides q 2 − 4 and q divides p2 − 1.

4. Find the number of 10-tuples (a1 , a2 , . . . , a10 ) of integers such that |a1 | ≤ 1 and

a21 + a22 + a23 + · · · + a210 − a1 a2 − a2 a3 − a3 a4 − · · · − a9 a10 − a10 a1 = 2 .

5. Let ABC be a triangle with ∠A = 90◦ and AB = AC. Let D and E be points on the segment
BC such that BD : DE : EC = 3 : 5 : 4. Prove that ∠DAE = 45◦ .

6. Suppose that m and n are integers such that both the quadratic equations x2 + mx − n = 0
and x2 − mx + n = 0 have integer roots. Prove that n is divisible by 6.

——— ? ———
Paper 1 Regional Mathematical Olympiad 2013 December 1, 2013

1. Let ABC be an acute angled triangle. The circle Γ with BC as diameter intersects AB and
AC again at P and Q, respectively. Determine ∠BAC given that the orthocenter of triangle
AP Q lies on Γ.

Solution. Let K denote the orthocenter of triangle AP Q. Since triangles ABC and AQP
are similar it follows that K lies in the interior of triangle AP Q.
Note that ∠KP A = ∠KQA = 90◦ −∠A. Since BP KQ is a cyclic quadrilateral it follows that
∠BQK = 180◦ − ∠BP K = 90◦ − ∠A, while on the other hand ∠BQK = ∠BQA − ∠KQA =
∠A since BQ is perpendicular to AC. This shows that 90◦ − ∠A = ∠A, so ∠A = 45◦ .

2. Let f (x) = x3 + ax2 + bx + c and g(x) = x3 + bx2 + cx + a, where a, b, c are integers with
c 6= 0. Suppose that the following conditions hold:

(a) f (1) = 0;
(b) the roots of g(x) are squares of the roots of f (x).

Find the value of a2013 + b2013 + c2013 .

Solution. Note that g(1) = f (1) = 0, so 1 is a root of both f (x) and g(x). Let p and q be the
other two roots of f (x), so p2 and q 2 are the other two roots of g(x). We then get pq = −c and
p2 q 2 = −a, so a = −c2 . Also, (−a)2 = (p + q + 1)2 = p2 + q 2 + 1 + 2(pq + p + q) = −b + 2b = b.
Therefore b = c4 . Since f (1) = 0 we therefore get 1 + c − c2 + c4 = 0. Factorising, we
get (c + 1)(c3 − c2 + 1) = 0. Note that c3 − c2 + 1 = 0 has no integer root and hence
c = −1, b = 1, a = −1. Therefore a2013 + b2013 + c2013 = −1.

3. Find all primes p and q such that p divides q 2 − 4 and q divides p2 − 1.

Solution. Suppose that p ≤ q. Since q divides (p − 1)(p + 1) and q > p − 1 it follows that q
divides p + 1 and hence q = p + 1. Therefore p = 2 and q = 3.
On the other hand, if p > q then p divides (q − 2)(q + 2) implies that p divides q + 2 or
q − 2 = 0. This gives either p = q + 2 or q = 2. In the former case it follows that that q
divides (q +2)2 −1, so q divides 3. This gives the solutions p > 2, q = 2 and (p, q) = (5, 3).

4. Find the number of 10-tuples (a1 , a2 , . . . , a10 ) of integers such that |a1 | ≤ 1 and

a21 + a22 + a23 + · · · + a210 − a1 a2 − a2 a3 − a3 a4 − · · · − a9 a10 − a10 a1 = 2 .

Solution. Let a11 = a1 . Multiplying the given equation by 2 we get

(a1 − a2 )2 + (a2 − a3 )2 + · · · (a10 − a1 )2 = 4 .

Note that if ai − ai+1 =P ±2 for some i = 1, . . . , 10, then aj − aj+1 = 0 for all j 6= i which
contradicts the equality 10 i=1 (ai − ai+1 ) = 0. Therefore ai − ai+1 = 1 for exactly two values
of i in {1, 2, . . . , 10}, ai − ai+1 = −1 for two other values of i and ai − ai+1 = 0 for all other
values of i. There are 10 8
2 × 2 = 45 × 28 possible ways of choosing these values. Note
that a1 = −1, 0 or 1, so in total there are 3 × 45 × 28 possible integer solutions to the given
equation.

1
Paper 1 Regional Mathematical Olympiad 2013 December 1, 2013

5. Let ABC be a triangle with ∠A = 90◦ and AB = AC. Let D and E be points on the segment
BC such that BD : DE : EC = 3 : 5 : 4. Prove that ∠DAE = 45◦ .

Solution. Rotating the configuraiton about A by 90◦ , the point B goes to the point C. Let
P denote the image of the point D under this rotation. Then CP = BD and ∠ACP =
∠ABC = 45◦ , so ECP is a right-angled triangle with CE : CP = 4 : 3. Hence P E = ED.
It follows that ADEP is a kite with AP = AD and P E = ED. Therefore AE is the angular
bisector of ∠P AD. This implies that ∠DAE = ∠P AD/2 = 45◦ .

6. Suppose that m and n are integers such that both the quadratic equations x2 + mx − n = 0
and x2 − mx + n = 0 have integer roots. Prove that n is divisible by 6.

Solution. Let a be an integer. If a is not divisible by 3 then a2 ≡ 1 (mod 3), i.e., 3 divides
a2 − 1, and if a is odd then a2 ≡ 1 (mod 8), i.e., 8 divides a2 − 1.
Note that the discriminants of the two quadratic polynomials are both squares of integers.
Let a and b be integers such that m2 − 4n = a2 and m2 + 4n = b2 . Therefore 8n = b2 − a2
and 2m2 = a2 + b2 . If 3 divides m then 3 divides both a and b, so 3 divides n. On the other
hand if 3 does not divide m then 3 does not divide a or b. Therefore 3 divides b2 − a2 and
hence 3 divides n.
If m is odd, then so is a, and therefore 4n = m2 − a2 is divisible by 8, so n is even. On
the other hand, if m is even then both a and b are even. Further (m/2)2 − n = (a/2)2 and
(m/2)2 + n = (b/2)2 , so (b − a)/2 is even. In particular, n = (b2 − a2 )/4 is even.

——— ? ———

2
Paper 2 Regional Mathematical Olympiad 2013 December 1, 2013

1. Prove that there do not exist natural numbers x and y, with x > 1, such that

x7 − 1
= y5 + 1 .
x−1

2. In a triangle ABC, AD is the altitude from A, and H is the orthocentre. Let K be the centre
of the circle passing through D and tangent to BH at H. Prove that the line DK bisects AC.

3. Consider the expression


20132 + 20142 + 20152 + · · · + n2 .
Prove that there exists a natural number n > 2013 for which one can change a suitable
number of plus signs to minus signs in the above expression to make the resulting expression
equal 9999.

4. Let ABC be a triangle with ∠A = 90◦ and


√ AB = AC. Let D and E be points on the segment
BC such that BD : DE : EC = 1 : 2 : 3. Prove that ∠DAE = 45◦ .

5. Let n ≥ 3 be a natural number and let P be a polygon with n sides. Let a1 , a2 , . . . , an be the
lengths of the sides of P and let p be its perimeter. Prove that
a1 a2 an
+ + ··· + < 2.
p − a1 p − a2 p − an

6. For a natural number n, let T (n) denote the number of ways we can place n objects of weights
1, 2, . . . , n on a balance such that the sum of the weights in each pan is the same. Prove that
T (100) > T (99).

——— ?? ———
Paper 2 Regional Mathematical Olympiad 2013 December 1, 2013

1. Prove that there do not exist natural numbers x and y, with x > 1, such that

x7 − 1
= y5 + 1 .
x−1

Solution. Simple factorisation gives y 5 = x(x3 + 1)(x2 + x + 1). The three factors on the
right are mutually coprime and hence they all have to be fifth powers. In particular, x = r5
for some integer r. This implies x3 + 1 = r15 + 1, which is not a fifth power unless r = −1 or
r = 0. This implies there are no solutions to the given equation.

2. In a triangle ABC, AD is the altitude from A, and H is the orthocentre. Let K be the centre
of the circle passing through D and tangent to BH at H. Prove that the line DK bisects AC.

Solution. Note that ∠KHB = 90◦ . Therefore ∠KDA = ∠KHD = 90◦ − ∠BHD =
∠HBD = ∠HAC. On the other hand, if M is the midpoint of AC then it is the circumcenter
of triangle ADC and therefore ∠M DA = ∠M AD. This proves that D, K, M are collinear
and hence DK bisects AC.

3. Consider the expression


20132 + 20142 + 20152 + · · · + n2 .
Prove that there exists a natural number n > 2013 for which one can change a suitable
number of plus signs to minus signs in the above expression to make the resulting expression
equal 9999.

Solution. For any integer k we have

−k 2 + (k + 1)2 + (k + 2)2 − (k + 3)2 = −4 .

Note that 9999 − (20132 + 20142 + 20152 + 20162 + 20172 ) = −4m for some positive integer
m. Therefore, it follows that

9999 =(20132 + 20142 + 20152 + 20162 + 20172 )


m
X
−(4r + 2014)2 + (4r + 2015)2 + (4r + 2016)2 − (4r + 2017)2 .

+
r=1

4. Let ABC be a triangle with ∠A = 90◦ and


√ AB = AC. Let D and E be points on the segment
BC such that BD : DE : EC = 1 : 2 : 3. Prove that ∠DAE = 45◦ .

Solution. Rotating the configuraiton about A by 90◦ , the point B goes to the point C. Let
P denote the image of the point D under this rotation. Then CP √ = BD and ∠ACP =

∠ABC = 45 , so ECP is a right-angled triangle with CE : CP = 3 : 1. Hence P E = ED.
It follows that ADEP is a kite with AP = AD and P E = ED. Therefore AE is the angular
bisector of ∠P AD. This implies that ∠DAE = ∠P AD/2 = 45◦ .

1
Paper 2 Regional Mathematical Olympiad 2013 December 1, 2013

5. Let n ≥ 3 be a natural number and let P be a polygon with n sides. Let a1 , a2 , . . . , an be the
lengths of the sides of P and let p be its perimeter. Prove that
a1 a2 an
+ + ··· + < 2.
p − a1 p − a2 p − an

Solution. If r and s are positive real numbers such that r < s then r/s < (r + x)/(s + x)
for any positive real x. Note that, by triangle inequality, ai < p − ai , so
ai 2ai
< ,
p − ai p
for all i = 1, , 2 . . . , n. Summing this inequality over i we get the desired inequality.

6. For a natural number n, let T (n) denote the number of ways we can place n objects of weights
1, 2, . . . , n on a balance such that the sum of the weights in each pan is the same. Prove that
T (100) > T (99).

Solution. Let S(n) denote the collection of subsets A of X(n) = {1, 2, . . . , n} such that
the sum of the elements of A equals n(n + 1)/4. Then the given inequality is equivalent to
|S(100)| > |S(99)|. We shall give a map f : S(99) → S(100) which is one-to-one but not
onto. Note that this will prove the required inequality.
Suppose that A is an element of S(99). If 50 ∈ A then define f (A) = (A \ {50}) ∪ {100}.
Otherwise, define f (A) = A ∪ {50}. If A and B are elements of S(99) such that f (A) = f (B)
then either 50 belongs to both these sets or neither of these sets. In either of the cases we
have A = B. Therefore f is a one-to-one function.
Note that every element in the range of f contains exactly one of 50 and 100. Let Bi =
{i, 101 − i}. Then B1 ∪ B2 ∪ · · · B24 ∪ B50 is an element of S(100). Clearly, this is not in the
range of f . Thus f is not an onto function.

——— ?? ———

2
Paper 3 Regional Mathematical Olympiad 2013 December 1, 2013

1. Find the number of eight-digit numbers the sum of whose digits is 4.

2. Find all 4-tuples (a, b, c, d) of natural numbers with a ≤ b ≤ c and a! + b! + c! = 3d .

3. In an acute-angled triangle ABC with AB < AC, the circle Γ touches AB at B and passes
through C intersecting AC again at D. Prove that the orthocentre of triangle ABD lies on
Γ if and only if it lies on the perpendicular bisector of BC.

4. A polynomial is called a Fermat polynomial if it can be written as the sum of the squares
of two polynomials with integer coefficients. Suppose that f (x) is a Fermat polynomial such
that f (0) = 1000. Prove that f (x) + 2x is not a Fermat polynomial.

5. Let ABC be a triangle which it not right-angled. Define a sequence of triangles Ai Bi Ci ,


with i ≥ 0, as follows: A0 B0 C0 is the triangle ABC; and, for i ≥ 0, Ai+1 , Bi+1 , Ci+1 are the
reflections of the orthocentre of triangle Ai Bi Ci in the sides Bi Ci , Ci Ai , Ai Bi , respectively.
Assume that ∠Am = ∠An for some distinct natural numbers m, n. Prove that ∠A = 60◦ .

6. Let n ≥ 4 be a natural number. Let A1 A2 · · · An be a regular polygon and X = {1, 2, . . . , n}.


A subset {i1 , i2 , . . . , ik } of X, with k ≥ 3 and i1 < i2 < · · · < ik , is called a good subset
if the angles of the polygon Ai1 Ai2 · · · Aik , when arranged in the increasing order, are in an
arithmetic progression. If n is a prime, show that a proper good subset of X contains exactly
four elements.

——— ? ? ? ———
Paper 4 Regional Mathematical Olympiad 2013 December 1, 2013

1. Let Γ be a circle with centre O. Let Λ be another circle passing through O and intersecting
Γ at points A and B. A diameter CD of Γ intersects Λ at a point P different from O. Prove
that
∠AP C = ∠BP D .

2. Determine the smallest prime that does not divide any five-digit number whose digits are in
a strictly increasing order.

3. Given real numbers a, b, c, d, e > 1 prove that

a2 b2 c2 d2 e2
+ + + + ≥ 20 .
c−1 d−1 e−1 a−1 b−1

1 1
4. Let x be a non-zero real number such that x4 + 4 and x5 + 5 are both rational numbers.
x x
1
Prove that x + is a rational number.
x
5. In a triangle ABC, let H denote its orthocentre. Let P be the reflection of A with respect to
BC. The circumcircle of triangle ABP intersects the line BH again at Q, and the circumcircle
of triangle ACP intersects the line CH again at R. Prove that H is the incentre of triangle
P QR.

6. Suppose that the vertices of a regular polygon of 20 sides are coloured with three colours –
red, blue and green – such that there are exactly three red vertices. Prove that there are three
vertices A, B, C of the polygon having the same colour such that triangle ABC is isosceles.

——— ? ? ? ? ———
Paper 4 Regional Mathematical Olympiad 2013 December 1, 2013

1. Let Γ be a circle with centre O. Let Λ be another circle passing through O and intersecting
Γ at points A and B. A diameter CD of Γ intersects Λ at a point P different from O. Prove
that
∠AP C = ∠BP D .

Solution. Suppose that A0 is a point on Λ such that ∠A0 P C = ∠BP D. Then the segments
OA0 and OB subtends same angle in the respective minor arcs, so OA0 = OB. This shows
that A lies on Γ and hence A0 = A. This proves that ∠AP C = ∠BP D.

2. Determine the smallest prime that does not divide any five-digit number whose digits are in
a strictly increasing order.

Solution. Note that 12346 is even, 3 and 5 divide 12345, and 7 divides 12348. Consider a 5
digit number n = abcde with 0 < a < b < c < d < e < 10. Let S = (a + c + e) − (b + d). Then
S = a + (c − b) + (e − d) > a > 0 and S = e − (d − c) − (b − a) < e ≤ 10, so S is not divisible
by 11 and hence n is not divisible by 11. Thus 11 is the smallest prime that does not divide
any five-digit number whose digits are in a strictly increasing order.

3. Given real numbers a, b, c, d, e > 1 prove that

a2 b2 c2 d2 e2
+ + + + ≥ 20 .
c−1 d−1 e−1 a−1 b−1

a2
Solution. Note that (a − 2)2 ≥ 0 and hence a2 ≥ 4(a − 1). Since a > 1 we have ≥ 4.
a−1
By applying AM-GM inequality we get
s
a2 b2 c2 d2 e2 a2 b2 c2 d2 e2
+ + + + ≥55 ≥ 20 .
c−1 d−1 e−1 a−1 b−1 (a − 1)(b − 1)(c − 1)(d − 1)(e − 1)

1 1
4. Let x be a non-zero real number such that x4 + 4 and x5 + 5 are both rational numbers.
x x
1
Prove that x + is a rational number.
x

Solution. For a natural number k let Tk = xk + 1/xk . Note that T4 T2 = T2 + T6 and


T8 T2 = T10 + T6 . Therefore T2 (T8 − T4 + 1) = T10 . Since T2k = Tk2 + 2 it follows that T8 , T10
are rational numbers and hence T2 , T6 are also rational numbers. Since T5 T1 = T4 + T6 it
follows that T1 is a rational number.

5. In a triangle ABC, let H denote its orthocentre. Let P be the reflection of A with respect to
BC. The circumcircle of triangle ABP intersects the line BH again at Q, and the circumcircle
of triangle ACP intersects the line CH again at R. Prove that H is the incentre of triangle
P QR.

1
Paper 4 Regional Mathematical Olympiad 2013 December 1, 2013

Solution. Since RACP is a cyclic quadrilateral it follows that ∠RP A = ∠RCA = 90◦ − ∠A.
Similarly, from cyclic quadrilateral BAQP we get ∠QP A = 90◦ − ∠A. This shows that P H
is the angular bisector of ∠RP Q.
We next show that R, A, Q are collinear. For this, note that ∠BP C = ∠A. Since ∠BHC =
180◦ − ∠A it follows that BHCP is a cyclic quadrilateral. Therefore ∠RAP + ∠QAP =
∠RCP + ∠QBP = 180◦ . This proves that R, A, Q are collinear.
Now ∠QRC = ∠ARC = ∠AP C = ∠P AC = ∠P RC. This proves that RC is the angular
bisector of ∠P RQ and hence H is the incenter of triangle P QR.

6. Suppose that the vertices of a regular polygon of 20 sides are coloured with three colours –
red, blue and green – such that there are exactly three red vertices. Prove that there are three
vertices A, B, C of the polygon having the same colour such that triangle ABC is isosceles.

Solution. Since there are exactly three vertices, among the remaining 17 vertices there are
nine of them of the same colour, say blue. We can divide the vertices of the regular 20-gon
into four disjoint sets such that each set consists of vertices that form a regular pentagon.
Since there are nine blue points, at least one of these sets will have three blue points. Since
any three points on a pentagon form an isosceles triangle, the statement follows.

2
Regional Mathematical Olympiad-2014
Time: 3 hours December 07, 2014
Instructions:
• Calculators (in any form) and protractors are not allowed.
• Rulers and compasses are allowed.
• Answer all the questions.
• All questions carry equal marks. Maximum marks: 102.
• Answer to each question should start on a new page. Clearly
indicate the question number.

1. Let ABC be a triangle and let AD be the perpen-


dicular from A on to BC. Let K, L, M be points
on AD such that AK = KL = LM = M D. If the
sum of the areas of the shaded regions is equal to the
sum of the areas of the unshaded regions, prove that
BD = DC.
2. Let a1 , a2 , . . . , a2n be an arithmetic progression of positive real numbers with
common difference d. Let
(i) a21 + a23 + · · · + a22n−1 = x, (ii) a22 + a24 + · · · + a22n = y, and
(iii) an + an+1 = z.
Express d in terms of x, y, z, n.
3. Suppose for some positive integers r and s, the digits of 2r is obtained by
permuting the digits of 2s in decimal expansion. Prove that r = s.

4. Is it possible to write the numbers 17, 18, 19, . . . , 32


in a 4 × 4 grid of unit squares, with one number in
each square, such that the product of the numbers in
each 2 × 2 sub-grids AM RG, GRN D, M BHR and
RHCN is divisible by 16?
5. Let ABC be an acute-angled triangle and let H be its ortho-centre. For
any point P on the circum-circle of triangle ABC, let Q be the point of
intersection of the line BH with the line AP . Show that there is a unique
point X on the circum-circle of ABC such that for every point P 6= A, B,
the circum-circle of HQP pass through X. P2014
6. Let x1 , x2 , . . . , x2014 be positive real numbers such that j=1 xj = 1. De-
termine with proof the smallest constant K such that
2014
X x2j
K ≥ 1.
j=1
1 − xj

———-0———-
Solutions to RMO-2014 problems
1. Let ABC be a triangle and let AD be the
perpendicular from A on to BC. Let K, L, M
be points on AD such that AK = KL =
LM = M D. If the sum of the areas of
the shaded regions is equal to the sum of
the areas of the unshaded regions, prove that
BD = DC.
Solution: let BD = 4x, DC = 4y and AD = 4h. Then the sum of the areas of the
shaded regions is
1  h(6x + 10y)
h x + (y + 2y) + (2x + 3x) + (3y + 4y) = .
2 2
The sum of the areas of the unshaded regions is
1  h(10x + 6y)
h y + (x + 2x) + (2y + 3y) + (3x + 4x) = .
2 2
Therefore the given condition implies that

6x + 10y = 10x + 6y.

This gives x = y. Hence BD = DC.

2. Let a1 , a2 , . . . , a2n be an arithmetic progression of positive real numbers with com-


mon difference d. Let
(i) a21 + a23 + · · · + a22n−1 = x, (ii) a22 + a24 + · · · + a22n = y, and
(iii) an + an+1 = z.
Express d in terms of x, y, z, n.

Solution: Observe that

y − x = (a22 − a21 ) + (a24 − a23 ) + · · · + (a22n − a22n−1 ).

The general difference is


    
a22k − a22k−1 = a2k + a2k−1 d = 2a1 + (2k − 1) + (2k − 2) d d.

Therefore
 
y − x = 2na1 + (1 + 2 + 3 + · · · (2n − 1))d d = nd 2a1 + (2n − 1)d .

We also observe that

z = an + an+1 = 2a1 + (2n − 1)d.

It follows that y − x = ndz. Hence d = (y − x)/nz.


3. Suppose for some positive integers r and s, the digits of 2r is obtained by permuting
the digits of 2s in decimal expansion. Prove that r = s.
Solution: Suppose s ≤ r. If s < r then 2s < 2r . Since the number of digits in 2s
and 2r are the same, we have 2r < 10 × 2s < 2s+4 . Thus we have 2s < 2r < 2s+4
which gives r = s + 1 or s + 2 or s + 3. Since 2r is obtained from 2s by permuting
its digits, 2r − 2s is divisible by 9. If r = s + 1, we see that 2r − 2s = 2s and it is
clearly not divisible by 9. Similarly, 2s+2 − 2s = 3 × 2s and 2s+3 − 2s = 7 × 2s and
none of these is divisible by 9. We conclude that s < r is not possible. Hence r = s.
4. Is it possible to write the numbers
17, 18, 19, . . . , 32 in a 4 × 4 grid of unit
squares, with one number in each square,
such that the product of the numbers in each
2×2 sub-grids AM RG, GRN D, M BHR and
RHCN is divisible by 16?
Solution: NO! If the product in each 2 × 2 sub-square is divisible by 16, then the
product of all the numbers is divisible by 16 × 16 × 16 × 16 = 216 . But it is easy to
see that
17 × 18 × 19 × · · · × 32 = 215 k,
where k is an odd number. Hence the product of all the numbers in the grid is not
divisible by 216 .
5. Let ABC be an acute-angled triangle and let H be its ortho-centre. For any point
P on the circum-circle of triangle ABC, let Q be the point of intersection of the line
BH with the line AP . Show that there is a unique point X on the circum-circle of
ABC such that for every point P 6= A, B, the circum-circle of HQP pass through
X.
Solution: We consider two possibilities: Q lying between A and P ; and P lying
between A and Q. (See the figures.)
In the first case, we observe that
∠HXC = ∠HXP + ∠P XC = ∠AQB + ∠P AC,
since Q, H, X, P are concyclic and P, A, X, C are also concyclic. Thus we get
∠HXC = ∠AQE + ∠QAE = 90◦
because BE ⊥ AC.

2
In the second case, we have

∠HXC = ∠HXP + ∠P XC = ∠HQP + ∠P AC;

the first follows from H, X, Q, P are concyclic; the second follows from the concyclic-
ity of A, X, C, P . Again BE ⊥ AC shows that ∠HXC = 90◦ .
Thus for any point P 6= A, B on the circumcircle of ABC, the point X of intersection
of the circumcircles of ABC and HP Q is such that ∠HXC = 90◦ . This means X
is precisely the point of intersection of the circumcircles of HEC and ABC, which
is independent of P .

6. Let x1 , x2 , . . . , x2014 be positive real numbers such that 2014


P
j=1 xj = 1. Determine
with proof the smallest constant K such that
2014
X x2j
K ≥ 1.
1 − xj
j=1

Solution:PLet us take the general case: {x1 , x2 , . . . , xn } are positive real numbers
such that nk=1 xk = 1. Then
n n n n n
X x2k X x2k − 1 X 1 X X 1
= + = (−1 − xk ) + .
1 − xk 1 − xk 1 − xk 1 − xk
k=1 k=1 k=1 k=1 k=1

Now the first sum is −n − 1. We can estimate the second sum using AM-HM
inequality:
n
X 1 n2 n2
≥ Pn = .
1 − xk k=1 (1 − xk ) n−1
k=1
Thus we obtain
n
X x2k n2 1
≥ −(1 + n) + = .
1 − xk n−1 n−1
k=1
Here equality holds if and only if all xj ’s are equal. Thus we get the smallest
constant K such that
X x2j
2014
K ≥1
1 − xj
j=1
to be 2014 − 1 = 2013.
———-0———-

3
Regional Mathematical Olympiad-2014
Time: 3 hours December 07, 2014
Instructions:
• Calculators (in any form) and protractors are not allowed.
• Rulers and compasses are allowed.
• Answer all the questions.
• All questions carry equal marks. Maximum marks: 102.
• Answer to each question should start on a new page. Clearly
indicate the question number.

1. In an acute-angled triangle ABC, ∠ABC is the largest angle. The perpen-


dicular bisectors of BC and BA intersect AC at X and Y respectively. Prove
that circumcentre of triangle ABC is incentre of triangle BXY .
2. Let x, y, z be positive real numbers. Prove that
y2 + z2 z 2 + x2 x2 + y 2
+ + ≥ 2(x + y + z).
x y z

3. Find all pairs of (x, y) of positive integers such that 2x + 7y divides 7x + 2y.
4. For any positive integer n > 1, let P (n) denote the largest prime not ex-
ceeding n. Let N (n) denote the next prime larger than P (n). (For example
P (10) = 7 and N (10) = 11, while P (11) = 11 and N (11) = 13.) If n + 1 is
a prime number, prove that the value of the sum
1 1 1 1 n−1
+ + + ··· + = .
P (2)N (2) P (3)N (3) P (4)N (4) P (n)N (n) 2n + 2

5. Let ABC be a triangle with AB > AC. Let P be a point on the line AB
beyond A such that AP + P C = AB. Let M be the mid-point of BC and let
Q be the point on the side AB such that CQ ⊥ AM . Prove that BQ = 2AP .
6. Let n be an odd positive integer and suppose that each square of an n × n
grid is arbitrarily filled with either by 1 or by −1. Let rj and ck denote the
product of all numbers in j-th row and k-th column respectively, 1 ≤ j, k ≤ n.
Prove that
X n Xn
rj + ck 6= 0.
j=1 k=1

———-00———-
Solutions to RMO-2014 problems
1. In an acute-angled triangle ABC, ∠ABC is the largest angle. The perpendicu-
lar bisectors of BC and BA intersect AC at X and Y respectively. Prove that
circumcentre of triangle ABC is incentre of triangle BXY .

Solution: Let D and E be the mid-points of BC and AB respectively. Since X


lies on the perpendicular bisector of BC, we have XB = XC. Since XD ⊥ BC
and XB = XC, it follows that XD bisects ∠BXC. Similarly, Y E bisects ∠BY A.
Hence the point of intersection of XD and Y E is the incentre of 4BXY . But
this point of intersection is also the circumcentre of 4ABC, being the point of
intersection of perpendicular bisectors of BC and AB.

2. Let x, y, z be positive real numbers. Prove that

y 2 + z 2 z 2 + x2 x2 + y 2
+ + ≥ 2(x + y + z).
x y z

Solution: We write the inequality in the form

x2 y 2 y 2 z 2 z 2 x2
+ + + + + ≥ 2(x + y + z).
y x z y x z

We observe that x2 + y 2 ≥ 2xy. Hence x2 + y 2 − xy ≥ xy. Multiplying both sides


by (x + y), we get

x3 + y 3 = (x + y)(x2 − xy + y 2 ) ≥ (x + y)xy.

Thus
x2 y 2
+ ≥ x + y.
y x
Similarly, we obtain

y2 z2 z 2 x2
+ ≥ y + z, + ≥ x + y.
z y x z
Adding three inequalities, we get the required result.

3. Find all pairs of (x, y) of positive integers such that 2x + 7y divides 7x + 2y.

Solution: Let d = gcd(x, y). Then x = dx1 and y = dy1 . We observe that 2x + 7y
divides 7x + 2y if and only if 2x1 + 7y1 divides 7x1 + 2y1 . This means 2x1 + 7y1
should divide 49x1 + 14y1 . But 2x1 + 7y1 divides 4x1 + 14y1 . Hence 2x1 + 7y1
divides 45x1 . Similarly, we can show that 2x1 + 7y1 divides 45y1 . Hence 2x1 + 7y1
divides gcd(45x1 , 45y1 ) = 45 gcd(x1 , y1 ) = 45. Hence

2x1 + 7y1 = 9, 15 or 45.


If 2x1 + 7y1 = 9, then x1 = 1, y1 = 1. Similarly, 2x1 + 7y1 = 15 gives x1 = 4,
y1 = 1. If 2x1 + 7y1 = 45, then we get
(x1 , y1 ) = (19, 1), (12, 3), (5, 5).
Thus all solutions are of the form
(x, y) = (d, d), (4d, d), (19d, d), (12d, 3d), (5d, 5d).

4. For any positive integer n > 1, let P (n) denote the largest prime not exceeding n.
Let N (n) denote the next prime larger than P (n). (For example P (10) = 7 and
N (10) = 11, while P (11) = 11 and N (11) = 13.) If n + 1 is a prime number, prove
that the value of the sum
1 1 1 1 n−1
+ + + ··· + = .
P (2)N (2) P (3)N (3) P (4)N (4) P (n)N (n) 2n + 2

Solution: Let p and q be two consecutive primes, p < q. If we take any n such that
p ≤ n < q, we see that P (n) = p and N (n) = q. Hence the term pq 1 occurs in the

sum q − p times. The contribution from such terms is q−p 1 1


pq = p − q . Since n + 1 is
prime, we obtain
1 1 1 1
+ + + ··· +
P (2)N (2) P (3)N (3) P (4)N (4) P (n)N (n)
     
1 1 1 1 1 1 1 1 n−1
= − + − + ··· + − = − = .
2 3 3 5 p n+1 2 n+1 2n + 2
Here p is used for the prime preceeding n + 1.
5. Let ABC be a triangale with AB > AC. Let P be a point on the line AB beyond
A such that AP + P C = AB. Let M be the mid-point of BC and let Q be the
point on the side AB such that CQ ⊥ AM . Prove that BQ = 2AP .
Solution: Extend BP to F such P F =
P C. Then AF = AP + P F = AP + P C =
AB. Hence A is the mid-point of BF .
Since M is the mid-point of BC, it fol-
lows that AM k F C. But AM ⊥ CQ.
Hence F C ⊥ CQ at C. Therefore QCF
is a rigt-angled triangle. Since P C = P F ,
it follows that ∠P CF = ∠P F C. Hence
∠P QC = ∠P CQ which gives P Q = P C =
P F . This implies that P is the mid-point
of QF .
Thus we have AP + AQ = P F and BQ + QA = AP + P F . This gives
2AP + AQ = P F + AP = BQ + QA.
We conclude that BQ = 2AP .

2
6. Suppose n is odd and each square of an n × n grid is arbitrarily filled with either by
1 or by −1. Let rj and ck denote the product of all numbers in j-th row and k-th
column respectively, 1 ≤ j, k ≤ n. Prove that
n
X n
X
rj + ck 6= 0.
j=1 k=1

Solution: Suppose we change +1 to −1 in a square. Then the product of the


numbers in that row changes sign. Similarly, the product of numbers in the column
also changes sign. Hence the sum
n
X n
X
S= rj + ck
j=1 k=1

decreases by 4 or increases by 4 or remains same. Hence the new sum is congruent


to the old sum modulo 4. Let us consider the situation when all the squares have
+1. Then S = n + n = 2n = 2(2m + 1) = 4m + 2. This means the sum S is is
always of the form 4l + 2 for any configuration. Therefore the sum is not equal to 0.

———-00———-

3
Regional Mathematical Olympiad-2014
Time: 3 hours December 07, 2014
Instructions:
• Calculators (in any form) and protractors are not allowed.

• Rulers and compasses are allowed.


• Answer all the questions.
• All questions carry equal marks. Maximum marks: 102.
• Answer to each question should start on a new page. Clearly
indicate the question number.

1. Let ABC be an acute-angled triangle and suppose ∠ABC is the largest


angle of the triangle. Let R be its circumcentre. Suppose the circumcircle of
triangle ARB cuts AC again in X. Prove that RX is pependicular to BC.
2. Find all real numbers x and y such that
1
x2 + 2y 2 + ≤ x(2y + 1).
2

3. Prove that there does not exist any positive integer n < 2310 such that
n(2310 − n) is a multiple of 2310.

4. Find all positive real numbers x, y, z such that


1 1 1 1 1 1
2x − 2y + = , 2y − 2z + = , 2z − 2x + = .
z 2014 x 2014 y 2014

5. Let ABC be a triangle. Let X be on the segment BC such that AB = AX.


Let AX meet the circumcircle Γ of triangle ABC again at D. Show that the
circumcentre of 4BDX lies on Γ.

6. For any natural number n, let S(n) denote the sum of the digits of n. Find
the number of all 3-digit numbers n such that S(S(n)) = 2.

———-000———-
Solutions to RMO-2014 problems
1. Let ABC be an acute-angled triangle and suppose ∠ABC is the largest angle of the
triangle. Let R be its circumcentre. Suppose the circumcircle of triangle ARB cuts
AC again in X. Prove that RX is pependicular to BC.

Solution: Extend RX to meet BC in E.


We show that ∠XEC = 90◦ . Join RA,
RB and BX. Observe that ∠AXB =
∠ARB = 2∠C and ∠BXR = ∠BAR =
90◦ − ∠C. Hence ∠EXC = 180◦ − 2∠C −
(90◦ − ∠C) = 90◦ − ∠C. This shows that
∠CEX = 90◦ .

2. Find all real numbers x and y such that


1
x2 + 2y 2 + ≤ x(2y + 1).
2
Solution: We write the inequality in the form

2x2 + 4y 2 + 1 − 4xy − 2x ≤ 0.

Thus (x2 − 4xy + 4y 2 ) + (x2 − 2x + 1) ≤ 0. Hence

(x − 2y)2 + (x − 1)2 ≤ 0.

Since x, y are real, we know that (x − 2y)2 ≥ 0 and (x − 1)2 ≥ 0. Hence it follows
that (x − 2y)2 = 0 and (x − 1)2 = 0. Therefore x = 1 and y = 1/2.

3. Prove that there does not exist any positive integer n < 2310 such that n(2310 − n)
is a multiple of 2310.
Solution: Suppose there exists n such that 0 < n < 2310 and n(2310 − n) = 2310k.
Then n2 = 2310(n − k). But 2310 = 2 × 3 × 5 × 7 × 11, the product of primes.
Hence n − k = 2310l2 for some l. But n < 2310 and hence n − k < 2310. Hence
l = 0. This forces n = k and hence n2 = 2310(n − k) = 0. Thus n = 0 and we have
a contradiction.

4. Find all positive real numbers x, y, z such that


1 1 1 1 1 1
2x − 2y + = , 2y − 2z + = , 2z − 2x + = .
z 2014 x 2014 y 2014

Solution: Adding the three equations, we get


1 1 1 3
+ + = .
x y z 2014
We can also write the equations in the form
z x y
2zx − 2zy + 1 = , 2xy − 2xz + 1 = , 2yz − 2yx + 1 = .
2014 2014 2014
Adding these, we also get
2014 × 3 = x + y + z.
Therefore  
1 1 1 3
+ + (x + y + z) = × (2014 × 3) = 9.
x y z 2014
Using AM-GM inequality, we therefore obtain

1 1/3
   
1 1 1
9= + + (x + y + z) ≥ 9 × (xyz)1/3 = 9.
x y z xyz

Hence equality holds in AM-GM inequality and we conclude x = y = z. Thus


1 1
=
x 2014
which gives x = 2014. We conclude

x = 2014, y = 2014, z = 2014.

5. Let ABC be a triangle. Let X be on the segment BC such that AB = AX. Let AX
meet the circumcircle Γ of triangle ABC again at D. Show that the circumcentre
of 4BDX lies on Γ.

Solution: Draw perpendicular from A to BC


and extend it to meet Γ in F . We show
that F is the circumcentre of 4BDX. Since
AB = AX, we observe that F lies on the
perpendicular bisector of BX. Join CF and
CD. We observe that ∠ABX = ∠CDX and
∠AXB = ∠CXD. Hence 4ABX is similar
to 4CDX. In particular 4CDX is isosceles.

Moreover, ∠BCF = ∠BAF and ∠DCF = ∠DAF . Since AF is the perpendicular


bisector of BX, it also bisects ∠BAX. It follows that CF bisects ∠DCX and
hence F lies on the perpendicular bisector of DX. Together F is the circumcentre
of 4BXD.

6. For any natural number n, let S(n) denote the sum of the digits of n. Find the
number of all 3-digit numbers n such that S(S(n)) = 2.

Solution: Observe that S(S(n)) = 2 implies that S(n) = 2, 11 or 20. Hence we


have to find the number of all all 3 digit numbers abc such that a + b + c = 2, 11

2
and 20. In fact we can enumerate all these:
a + b + c = 2: abc = 101, 110, 200;
a + b + c = 11; abc = 902, 920, 290, 209, 911, 191, 119, 803, 830, 308, 380,
812, 821, 182, 128, 218, 281, 731, 713, 317, 371, 137, 173, 722, 272, 227, 740, 704,
407, 470, 650, 605, 560, 506, 641, 614, 416, 461, 164, 146, 623, 632, 362, 326, 263, 236;
a + b + c = 20; abc = 992, 929, 299, 983, 938, 398, 389, 839, 893, 974, 947, 794, 749,
479, 497, 965, 956, 659, 695, 596, 569, 884, 848, 488,
875, 875, 785, 758, 578, 587, 866, 686, 668, 776, 767, 677.
There are totally 85 three digit numbers having second digital sum equal to 2.

———-00———-

3
Regional Mathematical Olympiad-2014
Time: 3 hours December 07, 2014
Instructions:
• Calculators (in any form) and protractors are not allowed.

• Rulers and compasses are allowed.


• Answer all the questions.
• All questions carry equal marks. Maximum marks: 102.
• Answer to each question should start on a new page. Clearly
indicate the question number.

1. Let ABCD be an isosceles trapezium having an incircle; let AB and CD be


the parallel sides and let CE be the perpendicular from C on to AB. Prove
that CE is equal to the geometric mean of AB and CD.
2. If x and y are positive real numbers, prove that

4x4 + 4y 3 + 5x2 + y + 1 ≥ 12xy.

3. Determine all pairs m > n of positive integers such that

1 = gcd(n + 1, m + 1) = gcd(n + 2, m + 2) = · · · = gcd(m, 2m − n).

4. What is the minimal area of a right-angled triangle whose inradius is 1 unit?

5. Let ABC be an acute-angled triangle and let I be its incentre. Let the
incircle of triangle ABC touch BC in D. The incircle of the triangle ABD
touches AB in E; the incircle of the triangle ACD touches BC in F . Prove
that B, E, I, F are concyclic.

6. In the adjacent figure, can the numbers


1, 2, 3, 4, · · · , 18 be placed, one on each line seg-
ment, such that the sum of the numbers on the
three line segments meeting at each point is di-
visible by 3?

———-0000———-
Solutions to RMO-2014 problems
1. Let ABCD be an isosceles trapezium having an incircle; let AB and CD be the
parallel sides and let CE be the perpendicular from C on to AB. Prove that CE is
equal to the geometric mean of AB and CD.

Solution: Since ABCD has incircle, we have AB + CD = AD + BC. We alos


know that AD = BC and ∠A = ∠B. Draw DF ⊥ AB. Then 4DF C ∼ = 4CEB.
Hence F E = CD, and AF = EB. Now

CE 2 = BC 2 − BE 2 .

Observe 2BC = BC + AD = AB + CD = 2F E + 2EB. Hence BC = F E + EB.


Thus
CE 2 = (F E + EB)2 − BE 2 = (F E + 2EB)F E = AB · CD.
This shows that CE is the geometric mean of AB and CD.

2. If x and y are positive real numbers, prove that

4x4 + 4y 3 + 5x2 + y + 1 ≥ 12xy.

Solution: We have from AM-GM inequality,

4x4 + 1 ≥ 4x2 , 4y 3 + y = y(4y 2 + 1) ≥ 4y 2 .

Hence

4x4 + 4y 3 + 5x2 + y + 1 ≥ 4x2 + 4y 2 + 5x2


= 9x2 + 4y 2

≥ 2( 9 × 4)xy
= 12xy.

3. Determine all pairs m > n of positive integers such that

1 = gcd(n + 1, m + 1) = gcd(n + 2, m + 2) = · · · = gcd(m, 2m − n).

Solution: Observe that 1 = gcd(n + r, m + r) = gcd(n + r, m − n). Thus each of the


m − n consecutive positive integers n + 1, n + 2, . . . , m is coprime to m − n. Since
one of these is necessarily a multiple of m − n, this is possible only when m − n = 1.
Hence each pair is of the form (n, n + 1), where n ∈ N.

4. What is the minimal area of a right-angled triangle whose inradius is 1 unit?


Solution: Let ABC be the right-angled tri-
angle with ∠B = 90◦ . Let I be its incentre
and D be the point where the incircle touches
AB. Then s − b = AD = r = 1. We also
know that [ABC] = rs = r(a + b + c)/2 and
[ABC] = ac/2. Thus

ac a+b+c a+c−b
= = (a+c)− = (a+c)−1.
2 2 2
Using AM-GM inequality, we get
ac √
= a + c − 1 ≥ 2 ac − 1.
2

Taking ac = x, we get x2 − 4x + 2 ≥ 0. Hence

4+2 2 √
x≥ = 2 + 2.
2
Finally, √
ac (2 + 2)2 √
[ABC] = ≥ = 3 + 2 2.
2 2

Thus the least area of such a triangle is 3 + 2 2.

5. Let ABC be an acute-angled triangle and let I be its incentre. Let the incircle of
triangle ABC touch BC in D. The incircle of the triangle ABD touches AB in
E; the incircle of the triangle ACD touches BC in F . Prove that B, E, I, F are
concyclic.
Solution: We know BD = s − b and DC =
s−c, where s is the semiperimeter of 4ABC.
Let the incircle of 4ABD touch BC in P and
let AD = l. Then
l + BD − c l+s−c−b
DP = = .
2 2
Similarly, we can compute DF =
l+s−c−b
. Therefore DP = DF .
2
But ID ⊥ BC. Hence I is on the per-
pendicular bisector of P F . This gives
IP = IF .
Draw IQ ⊥ AB. Then B, Q, I, D are concyclic so that ∠QID = 180◦ − ∠B.
Since DP = DF and IP = IF , the triangles IDP and IDF are congruent. But
IDP is congruent to IQE. It follows that 4IDF ∼ = 4IQE. This shows that
∠QIE = ∠DIF . Therefore ∠QID = ∠EIF . But ∠QID = 180◦ − ∠B. Hence
∠EIF = 180◦ − ∠B. Therefore B, E, I, F are concyclic.

2
6. In the adjacent figure, can the num-
bers 1, 2, 3, 4, · · · , 18 be placed, one on
each line segment, such that the sum of
the numbers on the three line segments
meeting at each point is divisible by 3?

Solution: We group the numbers 1 to 18 in to 3 groups: those leaving remainder 0


when divided by 3; those leaving remainder 1; and those leaving remainder 2. Thus
the groups are:
{3, 6, 9, 12, 15, 18}, {1, 4, 7, 10, 13, 16}, {2, 5, 8, 11, 14, 17}
Now we put the numbers in such a way that each of the three line segments con-
verging to a vertex gets one number from each set. For example, here is one such
arrangement:

———-00———-

3
Regional Mathematical Olympiad 2014 (Mumbai region)
• There are six questions in this question paper. Answer all questions.

• Each question carries 10 points.


• Use of protractors, calculators, mobile phone is forbidden.
• Time allotted: 3 hours

1. Three positive real numbers a, b, c are such that a2 + 5b2 + 4c2 − 4ab − 4bc = 0. Can a, b, c
be the lengths of the sides of a triangle? Justify your answer.

2. The roots of the equation

x3 − 3ax2 + bx + 18c = 0

form a non-constant arithmetic progression and the roots of the equation

x3 + bx2 + x − c3 = 0

form a non-constant geometric progression. Given that a, b, c are real numbers, find all
positive integral values a and b.
3. Let ABC be an acute-angled triangle in which ∠ABC is the largest angle. Let O be its
circumcentre. The perpendicular bisectors of BC and AB meet AC at X and Y respectively.
The internal bisectors of ∠AXB and ∠BY C meet AB and BC at D and E respectively.
Prove that BO is perpendicular to AC if DE is parallel to AC.
4. A person moves in the x − y plane moving along points with integer co-ordinates x and y
only. When she is at point (x, y), she takes a step based on the following rules:

(a) if x + y is even she moves to either (x + 1, y) or (x + 1, y + 1);


(b) if x + y is odd she moves to either (x, y + 1) or (x + 1, y + 1).
How many distinct paths can she take to go from (0, 0) to (8, 8) given that she took exactly
three steps to the right ((x, y) to (x + 1, y))?

5. Let a, b, c be positive numbers such that

1 1 1
+ + ≤ 1.
1+a 1+b 1+c

Prove that (1 + a2 )(1 + b2 )(1 + c2 ) ≥ 125. When does the equality hold?

6. Let D, E, F be the points of contact of the incircle of an acute-angled triangle ABC with
BC, CA, AB respectively. Let I1 , I2 , I3 be the incentres of the triangles AF E, BDF , CED,
respectively. Prove that the lines I1 D, I2 E, I3 F are concurrent.

GOOD LUCK

1
Solutions to problems of RMO 2014 (Mumbai region)

1. Three positive real numbers a, b, c are such that a2 + 5b2 + 4c2 − 4ab − 4bc = 0. Can
a, b, c be the lengths of the sides of a triangle? Justify your answer.

Solution

No. Note that a2 + 5b2 + 4c2 − 4ab − 4bc = (a − 2b)2 + (b − 2c)2 = 0 ⇒ a : b : c = 4 : 2 : 1 ⇒


b + c : a = 3 : 4. The triangle inequality is violated.

2. The roots of the equation

x3 − 3ax2 + bx + 18c = 0

form a non-constant arithmetic progression and the roots of the equation

x3 + bx2 + x − c3 = 0

form a non-constant geometric progression. Given that a, b, c are real numbers,


find all positive integral values a and b.

Solution

Let α − d, α, α + d (d 6= 0) be the roots of the first equation and let β/r, β, βr (r > 0 and
r 6= 1) be the roots of the second equation. It follows that α = a , β = c and

a3 − ad2 = −18c; 3a2 − d2 = b, (1)

c(1/r + 1 + r) = −b; c2 (1/r + 1 + r) = 1. (2)


Eliminating d, r and c yields
ab2 − 2a3 b − 18 = 0, (3)
2

whence b = a ± (1/a) a6 + 18a. For positive integral values of a and b it must be that
a6 + 18a is a perfect square. Let x2 = a6 + 18a. Then a3 < x2 < a3 + 1 for a > 2 and hence
no solution. For a = 1 there is no solution. For a = 2, x = 10 and b = 9. Thus the admissible
pair is (a, b) = (2, 9).
3. Let ABC be an acute-angled triangle in which ∠ABC is the largest angle. Let O
be its circumcentre. The perpendicular bisectors of BC and AB meet AC at X
and Y respectively. The internal bisectors of ∠AXB and ∠BY C meet AB and BC
at D and E respectively. Prove that BO is perpendicular to AC if DE is parallel
to AC.

Solution

Observe that triangles AY B and BXC are isosceles (AY = BY and BX = CX). This
implies ∠BY C = 2∠BAC and ∠AXB = 2∠ACB. Since XD and Y E are angle bisectors
we have ∠AXD = ∠ACB and ∠CY E = ∠CAB. Hence XD is parallel to BC and Y E is
parallel to AB. Therefore
CE CY
= (4)
EB AY
and
AD AX
= . (5)
DB CX

1
CE AD
Now, if DE is parallel to AC then = . Therefore we must have
EB DB
CY AX
= . (6)
AY CX
But then
CY AX AC AC
+1= +1⇒ = ⇒ AY = CX. (7)
AY CX AY CX
Hence BY = AY = CX = BX. Thus ∠BXY = ∠BY X i.e ∠AXB = ∠BY C or ∠ACB =
∠BAC i.e triangle ABC is isosceles with AB = CB. Hence BO is the perpendicular bisector
of AC.
4. A person moves in the x − y plane moving along points with integer co-ordinates
x and y only. When she is at point (x, y), she takes a step based on the following
rules:

(a) if x + y is even she moves to either (x + 1, y) or (x + 1, y + 1);


(b) if x + y is odd she moves to either (x, y + 1) or (x + 1, y + 1).
How many distinct paths can she take to go from (0, 0) to (8, 8) given that she
took exactly three steps to the right ((x, y) to (x + 1, y))?

Solution

We note that she must also take three up steps and five diagonal steps. Now, a step to the
right or an upstep changes the parity of the co-ordinate sum, and a diagonal step does not
change it. Therefore, between two right steps there must be an upstep and similarly between
two upsteps there must be a right step. We may, therefore write

HV HV HV

The diagonal steps may be distributed in any fashion before, in between and after the HV
sequence. The required number is nothing but the number of ways of distributing 5 identical
objects into 7 distinct boxes and is equal to 11
6 .

5. Let a, b, c be positive numbers such that

1 1 1
+ + ≤ 1.
1+a 1+b 1+c

Prove that (1 + a2 )(1 + b2 )(1 + c2 ) ≥ 125. When does the equality hold?

Solution
1 1 1 a 1 1
+ + ≤1⇒ ≥ + . (8)
1+a 1+b 1+c 1+a 1+b 1+c
Similarly,
b 1 1 c 1 1
≥ + , ≥ + . (9)
1+b 1+a 1+c 1+c 1+a 1+c
Apply AM-GM to get that
a 2 b 2 c 2
≥p , ≥p , ≥p . (10)
1+a (1 + b)(1 + c) 1+b (1 + a)(1 + c) 1+c (1 + a)(1 + b)

2
Multiplying these results we get
abc ≥ 8. (11)
Now take

F = (1 + a2 )(1 + b2 )(1 + c2 ) ≥ 1 + a2 + b2 + c2 + a2 b2 + b2 c2 + c2 a2 + a2 b2 c2 (12)

and apply AM-GM to a2 , b2 , c2 and to a2 b2 , b2 c2 , c2 a2 to get

F ≥ 1 + 3(a2 b2 c2 )1/3 + 3(a4 b4 c4 )1/3 + a2 b2 c2 = [1 + (a2 b2 c2 )1/3 ]3 ≥ [1 + 82/3 ]3 = 125. (13)

Wherein the equality holds when a = b = c = 2.


6. Let D, E, F be the points of contact of the incircle of an acute-angled triangle
ABC with BC, CA, AB respectively. Let I1 , I2 , I3 be the incentres of the triangles
AF E, BDF , CED, respectively. Prove that the lines I1 D, I2 E, I3 F are concurrent.

Solution

Observe that ∠AF E = ∠AEF = 90◦ − A/2 and ∠F DE = ∠AEF = 90◦ − A/2. Again
∠EI1 F = 90◦ + A/2. Thus

∠EI1 F + ∠F DE = 180◦ .

Hence I1 lies on the incircle. Also

∠I1 F E = (1/2)∠AF E = (1/2)∠AEF = ∠I1 EF. (14)

Thus I1 E = I1 F . But then they are equal chords of a circle and so they must subtend equal
angles at the circumference. Therefore ∠I1 DF = ∠I1 DE and so I1 D is the internal bisector
of ∠F DE. Similarly we can show that I2 E and I3 F are internal bisectors of ∠DEF and
∠DF E respectively. Thus the three lines I1 D, I2 E, I3 F are concurrent at the incentre of
triangle DEF .

3
Regional Mathematical Olympiad-2015
Time: 3 hours December 06, 2015
Instructions:
• Calculators (in any form) and protractors are not allowed.

• Rulers and compasses are allowed.


• Answer all the questions.
• All questions carry equal marks. Maximum marks: 102.
• Answer to each question should start on a new page. Clearly
indicate the question number.

1. In a cyclic quadrilateral ABCD, let the diagonals AC and BD intersect at


X. Let the circumcircles of triangles AXD and BXC intersect again at Y .
If X is the incentre of triangle ABY , show that ∠CAD = 90◦ .
2. Let P1 (x) = x2 + a1 x + b1 and P2 (x) = x2 + a2 x + b2 be two quadratic poly-
nomials with integer coefficients. Suppose a1 6= a2 and there exist integers
m 6= n such that P1 (m) = P2 (n), P2 (m) = P1 (n). Prove that a1 − a2 is even.
7k − 5
3. Find all fractions which can be written simultaneously in the forms
5k − 3
6l − 1
and , for some integers k, l.
4l − 3
4. Suppose 28 objects are placed along a circle at equal distances. In how many
ways can 3 objects be chosen from among them so that no two of the three
chosen objects are adjacent nor diametrically opposite?
5. Let ABC be a right triangle with ∠B = 90◦ . Let E and F be respectively
the mid-points of AB and AC. Suppose the incentre I of triangle ABC lies
on the circumcircle of triangle AEF . Find the ratio BC/AB.

6. Find all real numbers a such that 3 < a < 4 and a(a − 3{a}) is an integer.
(Here {a} denotes the fractional part of a. For example {1.5} = 0.5; {−3.4}
= 0.6.)

———-0———-
CRMO-2015 questions and solutions
1. In a cyclic quadrilateral ABCD, let the diagonals AC and BD intersect at X. Let the
circumcircles of triangles AXD and BXC intersect again at Y . If X is the incentre of
triangle ABY , show that ∠CAD = 90◦ .
Solution: Given that X is the incentre of tri-
angle ABY , we have ∠BAX = ∠XAY . There-
fore, ∠BDC = ∠BAC = ∠BAX = ∠XAY =
∠XDY = ∠BDY . This shows that C, D, Y are
collinear. Therefore, ∠CY X + ∠XY D = 180◦ .
But the left-hand side equals (180◦ − ∠CBD) +
(180◦ − ∠CAD). Since ∠CBD = ∠CAD, we ob-
tain 180◦ = 360◦ − 2∠CAD. This shows that
∠CAD = 90◦ .
2. Let P1 (x) = x2 + a1 x + b1 and P2 (x) = x2 + a2 x + b2 be two quadratic polynomials with
integer coefficients. Suppose a1 6= a2 and there exist integers m 6= n such that P1 (m) = P2 (n),
P2 (m) = P1 (n). Prove that a1 − a2 is even.

Solution: We have

m2 + a1 m + b1 = n2 + a2 n + b2
2
n + a1 n + b1 = m2 + a2 m + b2 .

Hence
(a1 − a2 )(m + n) = 2(b2 − b1 ), (a1 + a2 )(m − n) = 2(n2 − m2 ).
This shows that a1 + a2 = −2(n + m). Hence

4(b2 − b1 ) = a22 − a21 .

Since a1 + a2 and a1 − a2 have same parity, it follows that a1 − a2 is even.


7k − 5 6l − 1
3. Find all fractions which can be written simultaneously in the forms and , for
5k − 3 4l − 3
some integers k, l.
7k − 5 6l − 1
Solution: If a fraction is simultaneously in the forms and , we must have
5k − 3 4l − 3
7k − 5 6l − 1
= .
5k − 3 4l − 3
This simplifies to kl + 8k + l − 6 = 0. We can write this in the form

(k + 1)(l + 8) = 14.

Now 14 can be factored in 8 ways: 1 × 14, 2 × 7, 7 × 2, 14 × 1, (−1) × (−14), (−2) × (−7),


(−7) × (−2) and (−14) × (−1). Thus we get 8 pairs:

(k, l) = (13, −7), (6, −6), (1, −1), (0, 6), (−15, −9), (−8, −10), (−3, −15), (−2, −22).

These lead respectively to 8 fractions:


43 31 55 5 61 19 13
, , 1, , , , , .
31 27 39 3 43 13 9
4. Suppose 28 objects are placed along a circle at equal distances. In how many ways can 3
objects be chosen from among them so that no two of the three chosen objects are adjacent
nor diametrically opposite?

Solution: One can choose 3 objects out of 28 objects in 28



3 ways. Among these choices
all would be together in 28 cases; exactly two will be together in 28 × 24 cases. Thus three
objects can be chosen such that no two adjacent in 28 3 − 28 − (28 × 24) ways. Among these,
furthrer, two objects will be diametrically opposite in 14 ways and the third would be on
either semicircle in a non adjacent portion in 28 − 6 = 22 ways. Thus required number is
 
28
− 28 − (28 × 24) − (14 × 22) = 2268.
3

5. Let ABC be a right triangle with ∠B = 90◦ . Let E and F be respectively the mid-points
of AB and AC. Suppose the incentre I of triangle ABC lies on the circumcircle of triangle
AEF . Find the ratio BC/AB.

Solution: Draw ID ⊥ AC. Then ID = r, the inradius of 4ABC. Observe EF k BC and


hence ∠AEF = ∠ABC = 90◦ . Hence ∠AIF = 90◦ . Therefore ID2 = F D · DA. If a > c,
then F A > DA and we have
b
DA = s − a, and F D = F A − DA = − (s − a).
2
Thus we obtain
(b + c − a)(a − c)
r2 = .
4
But r = (c + a − b)/2. Thus we obtain

(c + a − b)2 = (b + c − a)(a − c).

Simplification gives 3b = 3a + c. Squaring both sides and using b2 = c2 + a2 , we obtian


4c = 3a. Hence BC/BA = a/c = 4/3.
(If a ≤ c, then I lies outside the circumcircle of AEF .)
6. Find all real numbers a such that 3 < a < 4 and a(a − 3{a}) is an integer. (Here {a} denotes
the fractional part of a. For example {1.5} = 0.5; {−3.4} = 0.6.)

Solution: Let a = 3 + f , where 0 < f < 1. We are given that (3 + f )(3 − 2f ) is an integer.
This implies that 2f 2 + 3f is an integer. Since 0 < f < 1, we have 0 < 2f 2 + 3f < 5.
Therefore 2f 2 + 3f can take 1, 2, 3 or 4. Equating 2f 2 + 3f to each one of them and using
f > 0, we get √ √ √
−3 + 17 1 −3 + 33 −3 + 41
f= , , , .
4 2 4 4
Therefore a takes the values:
√ √ √
−3 + 17 1 −3 + 33 −3 + 41
a=3+ , 3 , 3+ , 3+ .
4 2 4 4

———-0———-

2
Regional Mathematical Olympiad-2015
Time: 3 hours December 06, 2015
Instructions:
• Calculators (in any form) and protractors are not allowed.

• Rulers and compasses are allowed.


• Answer all the questions.
• All questions carry equal marks. Maximum marks: 102.
• Answer to each question should start on a new page. Clearly
indicate the question number.

1. Let ABC be a triangle. Let B 0 and C 0 denote respectively the reflection of


B and C in the internal angle bisector of ∠A. Show that the triangles ABC
and AB 0 C 0 have the same incentre.
2. Let P (x) = x2 + ax + b be a quadratic polynomial with real coefficients.
Suppose there are real numbers s 6= t such that P (s) = t and P (t) = s.
Prove that b − st is a root of the equation x2 + ax + b − st = 0.

3. Find all integers a, b, c such that

a2 = bc + 1, b2 = ca + 1.

4. Suppose 32 objects are placed along a circle at equal distances. In how many
ways can 3 objects be chosen from among them so that no two of the three
chosen objects are adjacent nor diametrically opposite?
5. Two circles Γ and Σ in the plane intersect at two distinct points A and B,
and the centre of Σ lies on Γ. Let points C and D be on Γ and Σ, respectively,
such that C, B and D are collinear. Let point E on Σ be such that DE is
parallel to AC. Show that AE = AB.

6. Find all real numbers a such that 4 < a < 5 and a(a − 3{a}) is an integer.
(Here {a} denotes the fractional part of a. For example {1.5} = 0.5; {−3.4}
= 0.6.)

———-00———-
CRMO-2015 questions and solutions
1. Let ABC be a triangle. Let B 0 and C 0 denote respectively the reflection of B and C in
the internal angle bisector of ∠A. Show that the triangles ABC and AB 0 C 0 have the same
incentre.
Solution: Join BB 0 and CC 0 . Let the internal angle
bisector ` of ∠A meet BB 0 in E and CC 0 in F . Since
B 0 is the reflection of B in `, we observe that BB 0 ⊥ `
and BE = EB 0 . Hence B 0 lies on AC. Similarly, C 0 lies
on the line AB.
Let D be the point of intersection of BC and B 0 C 0 .
Observe that BB 0 k C 0 C. Moreover the triangles ABC
is congruent to AB 0 C 0 : this follows from the observation
that AB = AB 0 and AC = AC 0 and the included angle
∠A is common. Hence BC 0 = B 0 C so that C 0 CB 0 B is
an isosceles trapezium. This means that the intesection
point D of its diagonal lies on the perpendicular bisector
of its parallel sides. Thus ` passes through D. We also
observe that CD = C 0 D.
Let I be the incentre of 4ABC. This means that CI bisects ∠C. Hence AI/ID = AC/CD.
But AC = AC 0 and CD = C 0 D. Hence we also get that AI/ID = AC 0 /C 0 D. This implies
that C 0 I bisects ∠AC 0 B 0 . Therefore the two angle bisectors of 4AC 0 B 0 meet at I. This
shows that I is also the incentre of 4AC 0 B 0 .

2. Let P (x) = x2 + ax + b be a quadratic polynomial with real coefficients. Suppose there are
real numbers s 6= t such that P (s) = t and P (t) = s. Prove that b − st is a root of the
equation x2 + ax + b − st = 0.

Solution: We have

s2 + as + b = t,
t2 + at + b = s.

This gives
(s2 − t2 ) + a(s − t) = (t − s).
Since s 6= t, we obtain s + t + a = −1. Adding the equations, we obtain

s2 + t2 + a(s + t) + 2b = (s + t).

Therefore
(s + t)2 − 2st + a(s + t) + 2b = (s + t).
Using s + t = −(1 + a), we obtain

(1 + a)2 − 2st − a(1 + a) + 2b = −1 − a.

Simplification gives st = 1+a+b = P (1). This shows that x = 1 is a root of x2 +ax+b−st = 0.


Since the product of roots is b − st, the other root is b − st.
3. Find all integers a, b, c such that
a2 = bc + 1, b2 = ca + 1.

Solution: Suppose a = b. Then we get one equation: a2 = ac + 1. This reduces to


a(a − c) = 1. Therefore a = 1, a − c = 1; and a = −1, a − c = −1. Thus we get
(a, b, c) = (1, 1, 0) and (−1, −1, 0).
If a 6= b, subtracting the second relation from the first we get
a2 − b2 = c(b − a).
This gives a + b = −c. Substituting this in the first equation, we get
a2 = b(−a − b) + 1.
Thus a2 + b2 + ab = 1. Multiplication by 2 gives
(a + b)2 + a2 + b2 = 2.
Thus (a, b) = (1, −1), (−1, 1), (1, 0), (−1, 0), (0, 1), (0, −1). We get respectively c =
0, 0, −1, 1, −1, 1. Thus we get the triples:
(a, b, c) = (1, 1, 0), (−1, −1, 0), (1, −1, 0), (−1, 1, 0), (1, 0, −1), (−1, 0, 1), (0, 1, −1), (0, −1, 1).

4. Suppose 32 objects are placed along a circle at equal distances. In how many ways can 3
objects be chosen from among them so that no two of the three chosen objects are adjacent
nor diametrically opposite?

Solution: One can choose 3 objects out of 32 objects in 32



3 ways. Among these choices
all would be together in 32 cases; exactly two will be together in 32 × 28 cases. Thus three
objects can be chosen such that no two adjacent in 32 3 − 32 − (32 × 28) ways. Among these,
furthrer, two objects will be diametrically opposite in 16 ways and the third would be on
either semicircle in a non adjacent portion in 32 − 6 = 26 ways. Thus required number is
 
32
− 32 − (32 × 28) − (16 × 26) = 3616.
3

5. Two circles Γ and Σ in the plane intersect at two distinct points A and B, and the centre
of Σ lies on Γ. Let points C and D be on Γ and Σ, respectively, such that C, B and D are
collinear. Let point E on Σ be such that DE is parallel to AC. Show that AE = AB.

Solution: If O is the centre of Σ, then we have

1 1
∠AEB = ∠AOB = (180◦ − ∠ACB)
2 2
1 1 1
= ∠EDB = 180 − ∠EAB = 90◦ − ∠EAB .


2 2 2
But we know that ∠AEB + ∠EAB + ∠EBA = 180◦ .
Therefore
1 1
∠EBA = 180◦ − ∠AEB − ∠EAB = 180◦ − 90◦ + ∠EAB − ∠EAB = 90◦ − ∠EAB.
2 2
This shows that ∠AEB = ∠EBA and hence AE = AB.

2
6. Find all real numbers a such that 4 < a < 5 and a(a − 3{a}) is an integer. (Here {a} denotes
the fractional part of a. For example {1.5} = 0.5; {−3.4} = 0.6.)

Solution: Let a = 4 + f , where 0 < f < 1. We are given that (4 + f )(4 − 2f ) is an integer.
This implies that 2f 2 + 4f is an integer. Since 0 < f < 1, we have 0 < 2f 2 + 4f < 6.
Therefore 2f 2 + 4f can take 1, 2, 3, 4 or 5. Equating 2f 2 + 4f to each one of them and using
f > 0, we get
√ √ √ √ √
−2 + 6 −2 + 8 −2 + 10 −2 + 12 −2 + 14
f= , , , , .
2 2 2 2 2
Therefore a takes the values:
√ √ √ √ √
6 8 10 12 14
a=3+ , 3+ , 3+ , 3+ , 3+ .
2 2 2 2 2

———-00———-

3
Regional Mathematical Olympiad-2015
Time: 3 hours December 06, 2015
Instructions:
• Calculators (in any form) and protractors are not allowed.

• Rulers and compasses are allowed.


• Answer all the questions.
• All questions carry equal marks. Maximum marks: 102.
• Answer to each question should start on a new page. Clearly
indicate the question number.

1. Two circles Γ and Σ, with centres O and O0 , respectively, are such that O0
lies on Γ. Let A be a point on Σ and M the midpoint of the segment AO0 .
If B is a point on Σ different from A such that AB is parallel to OM , show
that the midpoint of AB lies on Γ.
2. Let P (x) = x2 + ax + b be a quadratic polynomial where a and b are real
numbers. Suppose hP (−1)2 , P (0)2 , P (1)2 i is an arithmetic progression of
integers. Prove that a and b are integers.
3. Show that there are infinitely many triples (x, y, z) of integers such that
x3 + y 4 = z 31 .
4. Suppose 36 objects are placed along a circle at equal distances. In how many
ways can 3 objects be chosen from among them so that no two of the three
chosen objects are adjacent nor diametrically opposite?
5. Let ABC be a triangle with circumcircle Γ and incentre I. Let the internal
angle bisectors of ∠A, ∠B and ∠C meet Γ in A0 , B 0 and C 0 respectively.
Let B 0 C 0 intersect AA0 in P and AC in Q, and let BB 0 intersect AC in R.
Suppose the quadrilateral P IRQ is a kite; that is, IP = IR and QP = QR.
Prove that ABC is an equilateral triangle.
6. Show that there are infinitely many positive real numbers a which are not
integers such that a(a − 3{a}) is an integer. (Here {a} denotes the fractional
part of a. For example {1.5} = 0.5; {−3.4} = 0.6.)

———-000———-
CRMO-2015 questions and solutions
1. Two circles Γ and Σ, with centres O and O0 , respectively, are such that O0 lies on Γ. Let A
be a point on Σ and M the midpoint of the segment AO0 . If B is a point on Σ different from
A such that AB is parallel to OM , show that the midpoint of AB lies on Γ.
Solution: Let C be the reflection of O0 with re-
spect to O. Then in triangle O0 AC, the midpoints
of the segments O0 A and O0 C are M and O, re-
spectively. This implies AC is parallel to OM ,
and hence B lies on AC. Let the line AC inter-
sect Γ again at N . Since O0 C is a diameter of Γ
it follows that ∠O0 N C = 90◦ . Since O0 A = O0 B,
we can now conclude that N is the midpoint of
the segment AB.
2. Let P (x) = x2 + ax + b be a quadratic polynomial where a and b are real numbers. Suppose
hP (−1)2 , P (0)2 , P (1)2 i is an arithmetic progression of integers. Prove that a and b are
integers.
Solution: Observe that

P (−1) = 1 − a + b, P (0) = b, P (1) = 1 + a + b.

The given condition gives

2b2 = (1 − a + b)2 + (1 + a + b)2 = 2(1 + b)2 + 2a2 = 2 + 4b + 2b2 + 2a2 .

Hence a2 + 2b + 1 = 0. Observe

1 + a2 + b2 + 2a + 2b + 2ab = (1 + a + b)2 ∈ Z.

But 1, b2 , 2a2 + 4b are all integers. Hence 4a + 4ab ∈ Z. This gives 16a2 (1 + b)2 is an integer.
But a2 = −(2b + 1). Hence 16(2b + 1)(1 + b)2 is an integer. But

16(2b + 1)(1 + b)2 = 16(1 + 4b + 5b2 + 2b3 ).

Hence 16b(4 + 2b2 ) is an integer. If b = 0, then b is an integer. Otherwise, this shows that b
is a rational number. Because b2 ∈ Z, it follows that b is an integer. Since a2 = −(2b + 1),
we get that a2 is an integer. Now 4a(1 + b) ∈ Z. If b 6= −1, then a is rational and hence a is
an integer. If b = −1, then we see that P (−1) = −a, P (0) = b = −1 and P (1) = a. Hence
a2 , 1, a2 is an AP. This implies that a2 = 1 and hence a = ±1.
3. Show that there are infinitely many triples (x, y, z) of integers such that x3 + y 4 = z 31 .

Solution: Choose x = 24r and y = 23r . Then the left side is 212r+1 . If we take z = 2k ,
then we get 212r+1 = 231k . Thus it is sufficient to prove that the equation 12r + 1 = 31k
has infinitely many solutions in integers. Observe that (12 × 18) + 1 = 31 × 7. If we choose
r = 31l + 18 and k = 12l + 7, we get

12(31l + 18) + 1 = 31(12l + 7),

for all l. Choosing l ∈ N, we get infinitely many r = 31l + 18 and k = 12l + 7 such that
12r + 1 = 31k. Going back we have infinitely many (x, y, z) of integers satisfying the given
equation.
4. Suppose 36 objects are placed along a circle at equal distances. In how many ways can 3
objects be chosen from among them so that no two of the three chosen objects are adjacent
nor diametrically opposite?

Solution: One can choose 3 objects out of 36 objects in 36



3 ways. Among these choices
all would be together in 36 cases; exactly two will be together in 36 × 32 cases. Thus three
objects can be chosen such that no two adjacent in 36 3 − 36 − (36 × 32) ways. Among these,
furthrer, two objects will be diametrically opposite in 18 ways and the third would be on
either semicircle in a non adjacent portion in 36 − 6 = 30 ways. Thus required number is
 
36
− 36 − (36 × 32) − (18 × 30) = 5412.
3

5. Let ABC be a triangle with circumcircle Γ and incentre I. Let the internal angle bisectors
of ∠A, ∠B and ∠C meet Γ in A0 , B 0 and C 0 respectively. Let B 0 C 0 intersect AA0 in P and
AC in Q, and let BB 0 intersect AC in R. Suppose the quadrilateral P IRQ is a kite; that is,
IP = IR and QP = QR. Prove that ABC is an equilateral triangle.
Solution: We first show that AA0 is perpendicular to B 0 C 0 . Observe ∠C 0 A0 A = ∠C 0 CA =
∠C/2; ∠A0 C 0 C = ∠A0 AC = ∠A/2; and ∠CC 0 B 0 = ∠CBB 0 = ∠B/2. Hence

∠C ∠A ∠B
∠C 0 AP + ∠AC 0 P = ∠C 0 AB + ∠BAP + ∠AC 0 P = + + = 90◦ .
2 2 2
It follows that ∠AP C 0 = ∠A0 P C 0 = 90◦ . Thus ∠IP Q = 90◦ . Since P IRQ is a kite, we
observe that ∠IP R = ∠IRP and ∠QP R = ∠QRP . This implies that ∠IRQ = 90◦ . Hence
the kite IRQP is also a cyclic quadrilateral. Since ∠IRQ = 90◦ , we see that BB 0 ⊥ AC.
Since BB 0 is the bisector of ∠B, we conclude that ∠A = ∠C.

We also observe that the triangles IRC and IP B 0 are congruent triangles: they are similar,
since ∠IRC = ∠IP B 0 = 90◦ and ∠ICR = ∠C/2 = ∠IB 0 P (= ∠BCC 0 ); besides IR = IP .
Therefore IC = IB 0 . But B 0 I = B 0 C. Thus IB 0 C is an equilateral triangle. This means
∠B 0 IC = 60◦ and hence ∠ICR = 30◦ . Therefore ∠C/2 = 30◦ . Hence ∠A = ∠C = 60◦ . It
follows that ABC is equilateral.
6. Show that there are infinitely many positive real numbers a which are not integers such that
a(a − 3{a}) is an integer. (Here {a} denotes the fractional part of a. For example {1.5} =
0.5; {−3.4} = 0.6.)

2
Solution: We show that for each integer n ≥ 0, the interval (n, n + 1) contains a such that
a(a − 3{a}) is an integer. Put a = n + f , where 0 < f < 1. Then (n + f )(n − 2f ) must
be an integer. This means 2f 2 + nf must be an integer. Since 0 < f < 1, we must have
0 < 2f 2 + nf < 2 + n. Hence 2f 2 + nf ∈ {1, 2, 3, . . . , n + 1}. Taking 2f 2 + nf = 1, we get a
quadratic equation:
2f 2 + nf − 1 = 0.
Hence √ √
−n + n2 + 8 −n + n2 + 8
f= , and a = n + .
4 4
Thus we see that each a in the set
( √ )
−n + n2 + 8
n+ : n∈N
4

is a real number, which is not an integer, such that a(a − 3{a}) is an integer.

Remark: Each interval (n, n + 1) contains n + 1 such numbers, for n ≥ 0, n an integer.

———-000———-

3
Regional Mathematical Olympiad-2015
Time: 3 hours December 06, 2015
Instructions:
• Calculators (in any form) and protractors are not allowed.

• Rulers and compasses are allowed.


• Answer all the questions.
• All questions carry equal marks. Maximum marks: 102.
• Answer to each question should start on a new page. Clearly
indicate the question number.

1. Let ABC be a triangle. Let B 0 denote the reflection of B in the internal


angle bisector ` of ∠A. Show that the circumcentre of the triangle CB 0 I lies
on the line `, where I is the incentre of ABC.
2. Let P (x) = x2 + ax + b be a quadratic polynomial where a is real and b is
rational. Suppose P (0)2 , P (1)2 , P (2)2 are integers. Prove that a and b are
integers.

3. Find all integers a, b, c such that

a2 = bc + 4, b2 = ca + 4.

4. Suppose 40 objects are placed along a circle at equal distances. In how many
ways can 3 objects be chosen from among them so that no two of the three
chosen objects are adjacent nor diametrically opposite?
5. Two circles Γ and Σ intersect at two distinct points A and B. A line through
B intersects Γ and Σ again at C and D, respectively. Suppose that CA =
CD. Show that the centre of Σ lies on Γ.

6. How many integers m satisfy both the following


√  properties:
√ 
(i) 1 ≤ m ≤ 5000; (ii) m = m + 125 ?
(Here [x] denotes the largest integer not exceeding x, for any real number
x.)

———-0000———-
CRMO-2015 questions and solutions
1. Let ABC be a triangle. Let B 0 denote the reflection of B in the internal angle bisector `
of ∠A. Show that the circumcentre of the triangle CB 0 I lies on the line `, where I is the
incentre of ABC.
Solution: Let the line ` meet the circumcircle of
ABC in E. Then E is the midpoint of the minor
arc BC. Hence EB = EC.
Note that ∠EBC = ∠EAC = A/2 and ∠IBC =
B/2. Hence

∠BIE = ∠ABI + ∠BAI = B/2 + A/2.

We also have

∠IBE = ∠IBC + ∠CBE = B/2 + A/2.


Therefore ∠BIE = ∠IBE, so that EB = EI. Since AE is the perpendicular bisector of
BB 0 , we also have EB = EB 0 . Thus we get

EB 0 = EC = EI.

This implies that E is the circumcentre of 4CB 0 I


2. Let P (x) = x2 + ax + b be a quadratic polynomial where a is real and b 6= 2 is rational.
Suppose P (0)2 , P (1)2 , P (2)2 are integers. Prove that a and b are integers.

Solution: We have P (0) = b. Since b is rational and b2 = P (0)2 is an integer, we conclude


that b is an integer. Observe that

P (1)2 = (1 + a + b)2 = a2 + 2a(1 + b) + (1 + b)2 ∈ Z


P (2)2 = (4 + 2a + b)2 = 4a2 + 4a(4 + b) + (4 + b)2 ∈ Z

Eliminating a2 , we see that 4a(b − 2) + 4(1 + b)2 − (4 + b)2 ∈ Z. Since b 6= 2, it followsthat


a is rational. Hence the equation x2 + 2x(1 + b) + (1 + b)2 − a2 + 2a(1 + b) + (1 + b)2 = 0
is a quadratic equation with integer coefficients and has rational solution a. It follows that
a is an integer.
3. Find all integers a, b, c such that

a2 = bc + 4, b2 = ca + 4.

Solution: Suppose a = b. Then we get one equation: a2 = ac + 4. This reduces to


a(a − c) = 4. Therefore a = 1, a − c = 4; a = −1, a − c = −4; a = 4, a − c = 1;
a = −4, a − c = −1; a = 2, a − c = 2; a = −2, a − c = −2. Thus we get (a, b, c) = (1, 1, −3),
(−1, −1, 3), (4, 4, 3), (−4, −4, −3); (2, 2, 0), (−2, −2, 0).
If a 6= b, subtracting the second relation from the first we get

a2 − b2 = c(b − a).

This gives a + b = −c. Substituting this in the first equation, we get

a2 = b(−a − b) + 4.
Thus a2 + b2 + ab = 4. Multiplication by 2 gives

(a + b)2 + a2 + b2 = 8.

Thus (a, b) = (2, −2), (−2, 2), (2, 0), (−2, 0), (0, 2), (0, −2). We get respectively c =
0, 0, −2, 2, −2, 2. Thus we get the triples:

(a, b, c) = (1, 1, −3), (−1, −1, 3), (4, 4, 3), (−4, −4, −3), (2, 2, 0), (−2, −2, 0),
(2, −2, 0), (−2, 2, 0), (2, 0, −2), (−2, 0, 2), (0, 2, −2), (0, −2, 2).

4. Suppose 40 objects are placed along a circle at equal distances. In how many ways can 3
objects be chosen from among them so that no two of the three chosen objects are adjacent
nor diametrically opposite?

Solution: One can choose 3 objects out of 40 objects in 40



3 ways. Among theese choices
all would be together in 40 cases; exactly two will be together in 40 × 36 cases. Thus three
objects can be chosen such that no two adjacent in 40 3 − 40 − (40 × 36) ways. Among these,
furthrer, two objects will be diametrically opposite in 20 ways and the third would be on
either semicircle in a non adjacent portion in 40 − 6 = 34 ways. Thus required number is
 
40
− 40 − (40 × 36) − (20 × 34) = 7720.
3

5. Two circles Γ and Σ intersect at two distinct points A and B. A line through B intersects Γ
and Σ again at C and D, respectively. Suppose that CA = CD. Show that the centre of Σ
lies on Γ.

Solution: Let the perpendicular from C to AD


intersect Γ at O. Since CA = CD we have that
CO is the perpendicular bisector of AD and also
the angular bisector of ∠ACD. From the for-
mer, it follows that OA = OD, and from the
latter it follows that ∠OCB = ∠OCA and hence
OA = OB. Thus we get OA = OB = OD. This
means O is the circumcentre of triangle ADB.
This shows that O is the centre of Σ.

6. How many integers m satisfy both the following properties:


√  √ 
(i) 1 ≤ m ≤ 5000; (ii) m = m + 125 ?
(Here [x] denotes the largest integer not exceeding x, for any real number x.)
√  √ 
Solution: Let m = m + 125 = k. Then we know that

k 2 ≤ m < m + 125 < (k + 1)2 .

Thus
m + 125 < k 2 + 2k + 1 ≤ m + 2k + 1.
This shows that 2k + 1 > 125 or k > 62. Using k 2 ≤ 5000, we get k ≤ 70. Thus k ∈
{63, 64, 65, 66, 67, 68, 69, 70}. We observe that 632 = 3969 and 642 = 632 + 127. Hence
p  p 
632 + 125 = 632 + 1 + 125 = 63,

2
√  √  √ 
but 632 + 2 + 125 = 64. Thus we get two values of m such that m = m + 125 for
k = 63. Similarly, 652 = 642 + 129 so that
p  p  p  p 
642 + 125 = 642 + 1 + 125 = 642 + 2 + 125 = 642 + 3 + 125 = 64,
√  √  √ 
but 642 + 4 + 125 = 65. Thus we get four values of m such that m = m + 125
for k = 64. Continuing, we see that there are 6, 8, 10, 12, 14, 16 values of m respectively for
k = 65, 66, 67, 68, 69, 70. Together we get
8×9
2 + 4 + 6 + 8 + 10 + 12 + 14 + 16 = 2 × = 72
2
values of m satisfying the given requirement.

———-0000———-

3
Regional Mathematical Olympiad 2015 (Mumbai region)
06 December, 2015
• There are eight questions in this question paper. Answer all questions.
• Each of the questions 1,2,3 carries 10 points. Each of the questions 4,5,6,7,8
carries 14 points.
• Use of protractors, calculators, mobile phone is forbidden.
• Time allotted: 4 hours

1. Let ABCD be a convex quadrilateral with AB = a, BC = b, CD = c and DA = d. Suppose

a2 + b2 + c2 + d2 = ab + bc + cd + da,

and the area of ABCD is 60 square units. If the length of one of the diagonals is 30 units,
determine the length of the other diagonal.
2. Determine the number of 3-digit numbers in base 10 having at least one 5 and at most one
3.

3. Let P (x) be a non-constant polynomial whose coefficients are positive integers. If P (n)
divides P (P (n) − 2015) for every natural number n, prove that P (−2015) = 0.
4. Find all three digit natural numbers of the form (abc)10 such that (abc)10 , (bca)10 and (cab)10
are in geometric progression. (Here (abc)10 is representation in base 10.)
5. Let ABC be a right-angled triangle with ∠B = 90◦ and let BD be the altitude from B on
to AC. Draw DE ⊥ AB and DF ⊥ BC. Let P , Q, R and S be respectively the incentres of
triangle DF C, DBF , DEB and DAE. Suppose S, R, Q are collinear. Prove that P , Q, R,
D lie on a circle.
6. Let S = {1, 2, . . . , n} and let T be the set of all ordered triples of subsets of S, say (A1 , A2 , A3 ),
such that A1 ∪ A2 ∪ A3 = S. Determine, in terms of n,
P
|A1 ∩ A2 ∩ A3 |
(A1 ,A2 ,A3 )∈T

where |X| denotes the number of elements in the set X. (For example, if S = {1, 2, 3} and
A1 = {1, 2}, A2 = {2, 3}, A3 = {3} then one of the elements of T is ({1, 2}, {2, 3}, {3}).)
7. Let x, y, z be real numbers such that x2 + y 2 + z 2 − 2xyz = 1. Prove that

(1 + x)(1 + y)(1 + z) ≤ 4 + 4xyz.

8. The length of each side of a convex quadrilateral ABCD is a positive integer. If the sum of
the lengths of any three sides is divisible by the length of the remaining side then prove that
some two sides of the quadrilateral have the same length.

END OF QUESTION PAPER

1
Regional Mathematical Olympiad 2015 (Mumbai region)
06 December, 2015
Hints and Solutions

1. Let ABCD be a convex quadrilateral with AB = a, BC = b, CD = c and DA = d. Suppose

a2 + b2 + c2 + d2 = ab + bc + cd + da,

and the area of ABCD is 60 square units. If the length of one of the diagonals is 30 units,
determine the length of the other diagonal.

Solution

a2 +b2 +c2 +d2 = ab+bc+cd+da ⇒ (a−b)2 +(b−c)2 +(c−d)2 +(d−a)2 = 0 ⇒ a = b = c = d.


Thus ABCD is a rhombus and

[ABCD] = (1/2)(d1 d2 ) (1)


2[ABCD]
where d1 and d2 are the lengths of the diagonals. Hence d2 = = 4 units.
d1
2. Determine the number of 3-digit numbers in base 10 having at least one 5 and at most one
3.

Solution

We count the number of 3-digit numbers with (i) at least one 5 and having no 3 and (ii) at
least one 5 and having exactly one 3 separately.

(i) Here we first count the whole set and subtract the number of 3-digit numbers having
no 5 from it. Since 3 is not there and 0 cannot be the first digit, we can fill the first digit
in 8 ways. But we can fill the second and third digits in 9 ways(as 0 can be included).
Thus we get 8 × 9 × 9 such numbers. If no 5 is there, then the number of such numbers is
7 × 8 × 8. Thus the number of 3-digit numbers not containing 3 and having at least one 5 is
(8 × 9 × 9) − (7 × 8 × 8) = 8(81 − 56) = 200.

(ii) If 3 is there as a digit, then it can be the first digit or may be the second or third digit.
Consider those numbers in which 3 is the first digit. The number of such numbers having at
least one 5 is (9 × 9) − (8 × 8) = 81 − 64 = 17. The number of 3-digit numbers in which the
second digit is 3 and having at least one 5 is (8 × 9) − (7 × 8) = 16. Similarly, the number of
3-digit numbers in which the third digit is 3 and having at least one 5 is (8 × 9) − (7 × 8) = 16.
Thus we get 17 + 16 + 16 = 49 such numbers.
Therefore the number of 3-digit numbers having at most one 3 and at least one 5 is 200+49 =
249.
3. Let P (x) be a non-constatnt polynomial whose coefficients are positive integers. If P (n)
divides P (P (n) − 2015) for every natural number n, prove that P (−2015) = 0.

Solution

Note that P (n) − 2015 − (−2015) = P (n) divides P (P (n) − 2015) − P (−2015) for every
positive integer n. But P (n) divides P (P (n) − 2015) for every positive integer n. Therefore
P (n) divides P (−2015) for every positive integer n. Hence P (−2015) = 0.

1
Note

In the original version of the problem the word ‘non-constant’ was missing. The
falsity of the statement was brought to the attention of the examiners by a
contestant who mentioned it in a remark at the end of a perfect solution to the
problem assuming that the polynomial is non-constant. Many students assumed
that the polynomial is non-constant and completed the solution. They deserved
full credit for doing so.

4. Find all three digit natural numbers of the form (abc)10 such that (abc)10 , (bca)10 and (cab)10
are in geometric progression. (Here (abc)10 is representation in base 10.)

Solution

Let us write

x = (a × 102 ) + (b × 10) + c, y = (b × 102 ) + (c × 10) + a, z = (c × 102 ) + (a × 10) + b.

We are given that y 2 = xz. This means


2
(b × 102 ) + (c × 10) + a = (a × 102 ) + (b × 10) + c (c × 102 ) + (a × 10) + b .
 

We can solve for c and get


10b2 − a2
c= .
10a − b
If a, b, c are digits leading to a solution, and if d = gcd(a, b) then d|c. Consequently, we may
assume that gcd(a, b) = 1. Now

999a2
c= − (10b + 100a),
10a − b
showing that 10a − b divides 999a2 . Since a, b are relatively prime, this is possible only if
10a − b is a factor of 999. It follows that 10a − b takes the values 1,3,9,27,37. These values
lead to the pairs
(a, b) = (1, 9), (1, 7), (1, 1), (4, 3).
We can discard the first two pairs as they lead to a value of c > 10. The third gives the
trivial solution (111, 111, 111). Taking d = 2, 3, 4, 5, 6, 7, 8, 9, we get 9 solution:

(abc)10 = 111, 222, 333, 444, 555, 666, 777, 888, 999.

The last pair gives c = 2 and hence the solution (432, 324, 243). Another solution is obtained
on multiplying by 2: (864, 648, 486).
Thus we have

(abc)10 = 111, 222, 333, 444, 555, 666, 777, 888, 999, 432, 864.

5. Let ABC be a right-angled triangle with ∠B = 90◦ and let BD be the altitude from B on
to AC. Draw DE ⊥ AB and DF ⊥ BC. Let P , Q, R and S be respectively the incentres of
triangle DF C, DBF , DEB and DAE. Suppose S, R, Q are collinear. Prove that P , Q, R,
D lie on a circle.

Solution

2
We first show that SR is perpendicular to QP . Consider triangles P F Q and RES. Observe
that AE k DF and ED k F C. Since ES bisects ∠AED and F P bisects ∠DF C, it follows
that ES k F P . Since ER ⊥ ES and F Q ⊥ F P , we also have ER k F Q.

If r1 and r2 are inradii of triangles DEA and DEB, and if r0 is the inradius of 4DAB, we
know that
AD BD
r1 = r0 , r2 = r0 .
AB AB
Hence
ES r1 AD
= = .
ER r2 BD
Similarly, we can prove that
FQ BD
= .
FP DC
But we know that AD/BD = BD/DC. Hence we conclude that ES/ER = F Q/F P .
Therefore 4ESR ∼ 4QF P . But SE ⊥ ER and ER k QF imply SE ⊥ QF . It follows that
SR ⊥ QP .
Since S, R, Q are collinear, we get SQ ⊥ QP . Thus ∠RQP = 90◦ . Consider the circumcircle
of 4P RQ. Since ∠RDP = 90◦ , we conclude that D lies on this circle. Hence P, Q, R, D are
concyclic.
6. Let S = {1, 2, . . . , n} and let T be the set of all ordered triples of subsets of S, say (A1 , A2 , A3 ),
such that A1 ∪ A2 ∪ A3 = S. Determine, in terms of n,
P
|A1 ∩ A2 ∩ A3 |
(A1 ,A2 ,A3 )∈T

where |X| denotes the number of elements in the set X. (For example, if S = {1, 2, 3} and
A1 = {1, 2}, A2 = {2, 3}, A3 = {3} then one of the elements of T is ({1, 2}, {2, 3}, {3}).)

Solution 1

Let X = (A1 , A2 , A3 ) ∈ T and let i ∈ A1 ∩A2 ∩A3 . The number of times the element i occurs
in the required sum is equal to the number of ordered tuples (A1 − {i}, A2 − {i}, A3 − {i})
such that

A1 − {i} ∪ A2 − {i} ∪ A3 − {i} = S − {i} (∗)

3
For every element of S − {i}, there are eight possibilities - whether the element belongs to or
does not belong to Ai for i = 1, 2, 3. Out of these the case when the element does not belong
to any of the three subsets violates (∗). Therefore, each element can satisfy the requirement
in 7 ways. The number of tuples is, therefore, 7n−1 and the sum is n.7n−1 .

Solution 2

Consider all tuples (A1 , A2 , A3 ) such that A1 ∩A2 ∩A3 = B (∗) and A1 ∪A2 ∪A3 = S (∗∗).
Let |B| = r. (*) and (**) lead to

A1 − B ∩ A2 − B ∩ A3 − B = Φ (∗ ∗ ∗) and A1 − B ∪ A2 − B ∪ A3 − B = S − B (∗ ∗ ∗∗)

As before, for every element of S − B, there are eight possibilities. Out of these two cases -i.
the element belongs to each of the three subsets A1 , A2 , A3 violates (***) and ii. the element
does not belong to any of the three subsets violates (****). Therefore, there are 6 possibilities
for each element. Also, |S − B| = n − r, therefore, the number of tuples satisfying (*) and
(**) is 6n−r . The number of ways we can select r elements is nr , therefore the required
number is
n  
X n
r6n−r = n7n−1
r=0
r

Solution 3

This solution was given by a contestant of standard IX. We provide a sketch of


the same.
Write Sn and Tn in place of S and T respectively. Also, let
P
|A1 ∩ A2 ∩ A3 | = an .
(A1 ,A2 ,A3 )∈Tn

We shall show that an = n.7n−1 using recurrence and induction. Let an = n.7n−1 for some
positive integer n. Then, the elements of Tn+1 are made by adding n + 1 to either A1 or A2
or A3 or A1 ∩ A2 or A2 ∩ A3 or A3 ∩ A1 or A1 ∩ A2 ∩ A3 in Tn . Further, it is easily established
that |Tn | = 7n . Now, if n + 1 is added to A1 ∩ A2 ∩ A3 then |A1 ∩ A2 ∩ A3 | increases by 1.
Otherwise it remains the same. So, as there are seven choices of where to put n + 1,

an+1 = 7an + |Tn |

Thus an+1 = (n + 1).7n , which completes the inductive step, as the base case n = 1 is trivial.
Therefore

|A1 ∩ A2 ∩ A3 | = n.7n−1
P
(A1 ,A2 ,A3 )∈T

7. Let x, y, z be real numbers such that x2 + y 2 + z 2 − 2xyz = 1. Prove that

(1 + x)(1 + y)(1 + z) ≤ 4 + 4xyz.

Solution

4
Write 1 + 2xyz = x2 + y 2 + z 2 ⇔ 3 + 3xyz = 32 (x2 + y 2 + z 2 ) + 23
⇒ 3 + 3xyz = (x2 + y 2 + z 2 ) + 12 [(x2 + 1) + (y 2 + 1) + (z 2 + 1)] ≥ x2 + y 2 + z 2 + x + y + z
(Use x2 + y 2 + z 2 ≥ xy + yz + zx and AM-GM: x2 + 1 ≥ 2x etc.)
⇒ 3 + 3xyz ≥ xy + yz + zx + x + y + z.
By adding 1 + xyz in both sides,
we get 4 + 4xyz ≥ 1 + x + y + z + xy + yz + zx + xyz = (1 + x)(1 + y)(1 + z).
Equality holds when x = y = z = 1.
8. The length of each side of a convex quadrilateral ABCD is a positive integer. If the sum of
the lengths of any three sides is divisible by the length of the remaining side then prove that
some two sides of the quadrilateral have the same length.

Solution

Let us take ABCD to be a quadrilateral with AB = a, BC = b, CD = c and DA = d where


a, b, c, d are integers. We know AC < AB + BC and AD < AC + CD < AB + BC + CD.
Thus we get d < a + b + c. Similarly we can write down three more inequalities: a < b + c + d,
b < c + d + a and c < d + a + b. We can also take d to be the largest side. Using the given
conditions, we can write

a + b + c + d = la, b + c + d + a = mb, c + d + a + b = nc, a + b + c + d = kd,

for some positive integers l, m, n, k.


Suppose no two sides are equal. Then a < d, b < d, c < d. Hence a + b + c < 3d. Since d
divides a + b + c and d < a + b + c < 3d, we must have a + b + c = 2d. Thus we obtain

a + b + c + d = 3d = la = mb = nc.

Write this as
3d 3d 3d
a= , b= , c= .
l m n
Using 2d = a + b + c, we get the equation
3 3 3
+ + = 2.
l m n
Here l, m, n are necessarily distinct. Suppose l = m. Then la = 3d = lb. This implies a = b,
a contradiction. Similarly m = n and l = n can be discarded. We may assume l < m < n.
This means, we have to solve the equation for distinct positive integers.
If l ≥ 4, then m ≥ 5 and n ≥ 6. Hence
2 1 1 1 1 1 1 37 2
= + + ≤ + + = <
3 l m n 4 5 6 60 3
which is impossible. This means l = 1, l = 2 or l = 3. For l = 1, we get b + c + d = 0, Which
is impossible. If l = 2, we get b + c + d = a, which contradicts a < b + c + d. If l = 3, then
a = d and this contradicts again a < d. Therefore some two of a, b, c, d are equal.

THE END

NOTE

We do not claim that the solutions presented here are the most elegant solutions but
we thought they would be instructive. For some problems we found that solutions by
contestants were different and we have included them in this document.

5
Regional Mathematical Olympiad-2016
Time: 3 hours October 09, 2016
Instructions:
• Calculators (in any form) and protractors are not allowed.

• Rulers and compasses are allowed.


• Answer all the questions.
• All questions carry equal marks. Maximum marks: 102.
• Answer to each question should start on a new page. Clearly
indicate the question number.

1. Let ABC be a right-angled triangle with ∠B = 90◦ . Let I be the incentre of


ABC. Draw a line perpendicular to AI at I. Let it intersect the line
p CB at
D. Prove that CI is perpendicular to AD and prove that ID = b(b − a)
where BC = a and CA = b.
2. Let a, b, c be positive real numbers such that
a b c
+ + = 1.
1+a 1+b 1+c
Prove that abc ≤ 1/8.
3. For any natural number n, expressed in base 10, let S(n) denote the sum of
all digits of n. Find all natural numbers n such that n = 2S(n)2 .
4. Find the number of all 6-digit natural numbers having exactly three odd
digits and three even digits.

5. Let ABC be a triangle with centroid G. Let the circumcircle of triangle


AGB intersect the line BC in X different from B; and the circumcircle of
triangle AGC intersect the line BC in Y different from C. Prove that G is
the centroid of triangle AXY .

6. Let ha1 , a2 , a3 , . . .i be a strictly increasing sequence of positive integers in


an arithmetic progression. Prove that there is an infinite subsequence of the
given sequence whose terms are in a geometric progression.

———-0———-
1. Let ABC be a right-angled triangle with ∠B = 90◦ . Let I be the incentre of ABC. Draw a line
perpendicular to AI atpI. Let it intersect the line CB at D. Prove that CI is perpendicular to AD
and prove that ID = b(b − a) where BC = a and CA = b.

Solution: First observe that ADBI is a cyclic


quadrilateral since ∠AID = ∠ABD = 90◦ . Hence
∠ADI = ∠ABI = 45◦ . Hence ∠DAI = 45◦ . But
we also have

∠ADB = ∠ADI + ∠IDB = 45◦ + ∠IAB


= ∠DAI + ∠IAC = ∠DAC.

Therefore CDA is an isosceles triangle with CD =


CA. Since CI bisects ∠C it follows that CI ⊥ AD.

This shows that DB = CA − CB = b − a. Therefore

AD2 = c2 + (b − a)2 = c2 + b2 + a2 − 2ba = 2b(b − a).


p
But then 2ID2 = AD2 = 2b(b − a) and this gives ID = b(b − a).
2. Let a, b, c be positive real numbers such that
a b c
+ + = 1.
1+a 1+b 1+c
Prove that abc ≤ 1/8.

Solution: This is equivalent to


X
a(1 + b)(1 + c) = (1 + a)(1 + b)(1 + c).

This simplifies to
ab + bc + ca + 2abc = 1
Using AM-GM inequality, we have

1 = ab + bc + ca + 2abc ≥ 4(ab · bc · ca · 2abc)1/4 .

Simplificaton gives
1
abc ≤ .
8
3. For any natural number n, expressed in base 10, let S(n) denote the sum of all digits of n. Find
all natural numbers n such that n = 2S(n)2 .

Solution: We use the fact that 9 divides n−S(n) for every natural number n. Hence S(n)(2S(n)−1)
is divisible by 9. Since S(n) and 2S(n) − 1 are relatively prime, it follows that 9 divides either S(n)
or 2S(n) − 1, but not both. We also observe that the number of digits of n cannot exceed 4. If n
has k digits, then n ≥ 10k−1 and 2S(n)2 ≤ 2 × (9k)2 = 162k 2 . If k ≥ 6, we see that

2S(n)2 ≤ 162k 2 < 54 k 2 < 10k−1 ≤ n.

If k = 5, we have
2S(n)2 ≤ 162 × 25 = 4150 < 104 ≤ n.
Therefore n ≤ 4 and S(n) ≤ 36.
If 9|S(n), then S(n) = 9, 18, 27, 36. We see that 2S(n)2 is respectively equal to 162, 648, 1458,
2592. Only 162 and 648 satisfy n = 2S(n)2 .
If 9|(2S(n) − 1), then 2S(n) = 9k + 1. Only k = 1, 3, 5, 7 give integer values for S(n). In these cases
2S(n)2 = 50, 392, 1058, 2048. Here again 50 and 392 give n = 2S(n)2 .
Thus the only natural numbers wth the property n = 2S(n)2 are 50, 162, 392, 648.

4. Find the number of all 6-digit natural numbers having exactly three odd digits and three even
digits.

Solution: First we choose 3 places for even digits. This can be done in 63 = 20 ways. Observe


that the other places for odd digits get automatically fixed. There are 5 even digits and 5 odd digits.
Any of these can occur in their proper places. Hence there are 56 ways of selecting 3 even and 3
odd digits for a particular selection of place for even digits. Hence we get 20 × 56 such numbers.
But this includes all those numbers having the first digit equal to 0. Since we are looking for 6-digit
numbers, these numbers have to be removed from our counting. If we fix 0 as the first digit, we
have, 2 placesfor even numbers and 3 places for odd numbers. We can choose 2 places for even
numbers in 52 = 10 ways. As earlier, for any such choice of places for even digits, we can choose
even digits in 52 ways and odd digits in 53 ways. Hence the number of ways of choosing 3 even and
3 odd digits with 0 as the first digit is 10 × 55 . Therfore the number of 6-digit numbers with 3 even
digits and 3 odd digits is

20 × 56 − 10 × 55 = 10 × 55 (10 − 1) = 281250.

5. Let ABC be a triangle with centroid G. Let the circumcircles of 4AGB and 4AGC intersect the
line BC in X and Y respectively, which are distinct from B, C. Prove that G is the centroid of
4AXY .

Solution: Let D be the midpoint of AB. Ob-


serve that DX · DB = DG · DA = DY · DC.
But DB = DC. Hence DX = DY . This
means that D is the midpoint of XY as well.
Hence AD is also a median of 4AXY . Now
we know that AG : GD = 2 : 1. If G0 is the
median of 4AXY , then G0 must lie on AD
and AG0 : G0 D = 2 : 1. We conclude that
G = G0 .

6. Let ha1 , a2 , a3 , . . .i be a strictly increasing sequence of positive integers in an arithmetic progression.


Prove that there is an infinite subsequence of the given sequence whose terms are in a geometric
progression.

Solution: Let ha1 , a2 , . . . , an+1 . . .i = ha, a + d, . . . , a + nd, . . .i be a strictly increasing sequence of


positive integers in arithmetic progression. Here a and d are both positive integers. Consider the
following subsequence:
ha, a(1 + d), a(1 + d)2 , . . . , a(1 + d)n , . . .i.
This is a geometric progression. Here a > 0 and the common ratio 1 + d > 1. Hence the sequence is
strictly increasing. The first term is a which is in the given AP. The second term is a(1+d) = a+ad
which is the (a + 1)-th term of the AP. In general, we see that
      
n n n
a(1 + d)n = a + d a+ ad + · · · + adn−1 .
1 2 n

2
Here the coefficient of d in the braces is also a positive integer. Hence a(1 + d)n is also a term of
the given AP.

3
Regional Mathematical Olympiad-2016
Time: 3 hours October 09, 2016
Instructions:
• Calculators (in any form) and protractors are not allowed.

• Rulers and compasses are allowed.


• Answer all the questions.
• All questions carry equal marks. Maximum marks: 102.
• Answer to each question should start on a new page. Clearly
indicate the question number.

1. Let ABC be a right-angled triangle with ∠B = 90◦ . Let I be the incentre


of ABC. Let AI extended intersect BC at F . Draw a line perpendicular to
AI at I. Let it intersect AC at E. Prove that IE = IF .
2. Let a, b, c be positive real numbers such that
a b c
+ + = 1.
1+b 1+c 1+a
Prove that abc ≤ 1/8.

3. For any natural number n, expressed in base 10, let S(n) denote the sum of
all digits of n. Find all natural numbers n such that n3 = 8S(n)3 +6nS(n)+1.
4. How many 6-digit natural numbers containing only the digits 1,2,3 are there
in which 3 occurs exactly twice and the number is divisible by 9?

5. Let ABC be a right-angled triangle with ∠B = 90◦ . Let AD be the bisector


of ∠A with D on BC. Let the circumcircle of triangle ACD intersect AB
again in E; and let the circumcircle of triangle ABD intersect AC again in
F . Let K be the reflection of E in the line BC. Prove that F K = BC.
6. Show that the infinite arithmetic progression h1, 4, 7, 10, . . .i has infinitely
many 3-term subsequences in harmonic progression such that for any two
such triples ha1 , a2 , a3 i and hb1 , b2 , b3 i in harmonic progression, one has
a1 a2
6= .
b1 b2

———-00———-
1. Let ABC be a right-angled triangle with ∠B = 90◦ . Let I be the incentre of ABC. Let AI extended
intersect BC in F . Draw a line perpendicular to AI at I. Let it intersect AC in E. Prove that
IE = IF .

Solution: Extend EI to meet CB extended in D.


First observe that ADBI is a cyclic quadrilateral
since ∠AID = ∠ABD. Hence ∠ADI = ∠ABI =
45◦ . Hence ∠DAI = 45◦ . Therefore IA = ID.
Consider the triangles AIE and DIF . Both are
right triangles. Moreover ∠IAE = ∠IAB = ∠IDB.
Since IA = ID, the triangles are congruent. This
means IE = IF .

2. Let a, b, c be positive real numbers such that


a b c
+ + = 1.
1+b 1+c 1+a
Prove that abc ≤ 1/8.

Solution: This is equivalent to


X
a(1 + c)(1 + a) = (1 + a)(1 + b)(1 + c).

This simplifies to X X
a2 + a2 c = 1 + abc
Using AM-GM inequality, we have
X X
1 + abc = a2 + a2 c ≥ 3(abc)2/3 + 3abc.

Let x = (abc)1/3 . Then


3x2 + 2x3 ≤ 1.
This can be written as (x + 1)2 (2x − 1) ≤ 0. Hence x ≤ 1/2. Thus
1
abc ≤ .
8
3. For any natural number n, expressed in base 10, let S(n) denote the sum of all digits of n. Find
all natural numbers n such that n3 = 8S(n)3 + 6nS(n) + 1.

Solution: We write the given condition as


n3 + (−2S(n))3 + (−1)3 = 3 × n × (−2S(n)) × (−1).
This is in the form x3 + y 3 + z 3 = 3xyz. We know that this can happen if and only if x + y + z = 0.
Thus we obtain a simpler condition
n − 2S(n) − 1 = 0.
Again we know that n − S(n) is divisible by 9. Hence 9 should divide S(n) + 1. It is easy to see
that the number of digits in n cannot be more than 2. For a three digit number maximum value
of S(n) can be 27 and 2S(n) + 1 ≤ 55.. Hence n is either a 1-digit number or a two digit number.
Hence S(n) ≤ 18. Since 9 divides S(n) + 1, we can have S(n) = 8 or S(n) = 17. But then n = 17
or n = 35. Among these n = 17 works but not 35. (S(35) = 8 and 2S(n) + 1 = 17 6= 35.) Hence
the only solution is n = 17.
4. How many 6-digit natural numbers containing only the digits 1, 2, 3 are there in which 3 occurs
exactly twice and the number is divisible by 9?

Solution:
Let S(n) be the sum of the digits of n. Then n ≡ S(n) (mod 9). For any admissible n we observe
that 10 ≤ S(n) ≤ 14 and hence there is no value of S(n) that is a multiple of 9. Thus no such n
exists.
5. Let ABC be a right-angled triangle with ∠B = 90◦ . Let AD be the bisector of ∠A with D on BC.
Let the circumcircle of triangle ACD intersect AB again in E; and let the circumcircle of triangle
ABD intersect AC again in F . Let K be the reflection of E in the line BC. Prove that F K = BC.

Solution: First we show that EB = F C. Con-


sider the triangles EBD and CF D. Observe that
∠CF D = 90◦ since ∠AF D = 90◦ (angle in a semi-
circle). Hence ∠CF D = ∠EBD. Since ACDE is a
cyclic quadrilateral, we have ∠CDE = 180◦ − ∠A.
Similarly, we see that AF DB is a cyclic quadrilateral
and therefore ∠F DB = 180◦ − ∠A. Thus we obtain
∠CDE = ∠F DB. This gives ∠F DC = ∠BDE. It
follows 4EBD ∼ 4CF D.

Since AD bisects ∠A, we have DB = DF . Hence 4EBD ∼ = 4CF D. Hence F C = EB = BK. We


also observe that AF = AB since 4ABD ∼ = 4AF D. It follows that F B k CK. Since F C = BK,
we conclude that CKDF is an siosceles trapezium. This gives F K = BC.

Alternate solution: First we show that K, D, F are collinear. Observe that ∠F DB = 180◦ − ∠A
by the concyclicity of AF DB. Moreover ∠BDK = ∠BDE = ∠A. Therefore ∠KDF = 180◦ . This
proves that KDF is a line segment.
Consider the triangles AKF and ABC. Since both are right-angled triangles and ∠A is common,
they are similar. We also see that 4AF D ∼= 4ABD since ∠AF D = ∠ABD = 90◦ , ∠F AD =
∠BAD = ∠A/2 and AD common. Hence AF = AB. This implies now that 4AF K ∼ = 4ABC.
Hence KF = BC.
6. Show that the infinite arithmetic progression h1, 4, 7, 10, . . .i has infinitely many 3-term subsequences
in harmonic progression such that for any two such triples ha1 , a2 , a3 i and hb1 , b2 , b3 i in harmonic
progression, one has  
a1 a2 a2 a3
6= 6= .
b1 b2 b2 b3

Solution: Consider h4, 7, 28i. We observe that


1 1 2
+ = .
4 28 7
Thus we look for the terms of the form a, b, ab which give a HP. The condition is
1 1 2
+ = .
a ab b
This reduces to b(1 + b) = 2ab or 2a = 1 + b. The terms of the given AP are of the form 3k + 1. If
we take a = 3k + 1, then b = 2a − 1 = 6k + 1. We observe that b is also a term of the gven AP.

2
Besides, ab = (3k + 1)(6k + 1) = 3(6k 2 + 3k) + 1 is again a term of the given AP. Thus the triple
of the form h3k + 1, 6k + 1, (3k + 1)(6k + 1)i form a HP. We observe that

3k + 1 6k + 1 (3k + 1)(6k + 1)
6= 6= .
3l + 1 6l + 1 (3l + 1)(6l + 1)

———-0———-

3
Regional Mathematical Olympiad-2016
Time: 3 hours October 16, 2016
Instructions:
• Calculators (in any form) and protractors are not allowed.
• Rulers and compasses are allowed.
• Answer all the questions.
• All questions carry equal marks. Maximum marks: 102.
• Answer to each question should start on a new page. Clearly
indicate the question number.
1. Let ABC be a triangle and D be the mid-point of BC. Suppose the angle
bisector of ∠ADC is tangent to the circumcircle of triangle ABD at D.
Prove that ∠A = 90◦ .
2. Let a, b, c be three distinct positive real numbers such that abc = 1. Prove
that
a3 b3 c3
+ + ≥ 3.
(a − b)(a − c) (b − c)(b − a) (c − a)(c − b)
3. Let a, b, c, d, e, f be positive integers such that
a c e
< < .
b d f
Suppose af − be = −1. Show that d ≥ b + f .

4. There are 100 countries participating in an olympiad. Suppose n is a positive


integer such that each of the 100 countries is willing to communicate in
exactly n languages. If each set of 20 countries can communicate in at least
one common language, and no language is common to all 100 countries, what
is the minimum possible value of n?
5. Let ABC be a right-angled triangle with ∠B = 90◦ . Let I be the incentre
of ABC. Extend AI and CI; let them intersect BC in D and AB in E
respectively. Draw a line perpendicular to AI at I to meet AC in J; draw a
line perpendicular to CI at I to meet AC in K. Suppose DJ = EK. Prove
that BA = BC.
6. (a) Given any natural number N , prove that there exists a strictly increasing
sequence of N positive integers in harmonic progression.
(b) Prove that there cannot exist a strictly increasing infinite sequence of
positive integers which is in harmonic progression.
———-000———-
1. Let ABC be a triangle and D be the mid-point of BC. Suppose the angle bisector of ∠ADC is
tangent to the circumcircle of triangle ABD at D. Prove that ∠A = 90◦ .
Solution: Let P be the center of the circumcircle Γ
of 4ABC. Let the tangent at D to Γ intersect AC
in E. Then P D ⊥ DE. Since DE bisects ∠ADC,
this implies that DP bisects ∠ADB. Hence the cir-
cumcenter and the incenter of 4ABD lies on the
same line DP . This implies that DA = DB. Thus
DA = DB = DC and hence D is the circumcenter
of 4ABC. This gives ∠A = 90◦ .
2. Let a, b, c be positive real numbers such that abc = 1 Prove that

a3 b3 c3
+ + ≥ 3.
(a − b)(a − c) (b − c)(b − a) (c − a)(c − b)

Solution: Observe that

1 (b − c)
=
(a − b)(a − c) (a − b)(b − c)(a − c)
(a − c) − (a − b) 1 1
= = − .
(a − b)(b − c)(a − c) (a − b)(b − c) (b − c)(a − c)
Hence
a3 b3 c3 a3 − b3 c3 − a3
+ + = +
(a − b)(a − c) (b − c)(b − a) (c − a)(c − b) (a − b)(b − c) (c − a)(c − b)
a2 + ab + b2 c2 + ca + a2
= −
b−c b−c
ab + b2 − c2 − ca
=
b−c
(a + b + c)(b − c)
= = a + b + c.
b−c
Therefore
a3 b3 c3
+ + = a + b + c ≥ 3(abc)1/3 = 3.
(a − b)(a − c) (b − c)(b − a) (c − a)(c − b)

3. Let a, b, c, d, e, f be positive integers such that


a c e
< < .
b d f
Suppose af − be = −1. Show that d ≥ b + f .

Solution: Since bc − ad > 0, we have bc − ad ≥ 1. Similarly, we obtain de − f c ≥ 1. Therefore

d = d(be − af ) = dbe − daf = dbe − bf c + bf c − adf = b(de − f c) + f (bc − ad) ≥ b + f.

4. There are 100 countries participating in an olympiad. Suppose n is a positive integer such that each
of the 100 countries is willing to communicate in exactly n languages. If each set of 20 countries can
communicate in at least one common language, and no language is common to all 100 countries,
what is the minimum possible value of n?
Solution: We show that n = 20. We first show that n = 20 is possible. Call the countries
C1 , · · · , C100 . Let 1, 2, · · · , 21 be languages and suppose, the country Ci (1 ≤ i ≤ 20) communicates
exactly in the languages {j : 1 ≤ j ≤ 20, j 6= i}. Suppose each of the countries C21 , · · · , C100
communicates in the languages 1, 2, · · · , 20. Then, clearly every set of 20 countries have a common
language of communication.
Now, we show that n ≥ 20. If n < 20, look at any country A communicating in the languages
L1 , · · · , Ln . As no language is common to all 100 countries, for each Li , there is a country Ai not
communicating in Li . Then, the n + 1 ≤ 20 countries A, A1 , A2 , · · · , An have no common language
of communication. This contradiction shows n ≥ 20.
5. Let ABC be a right-angled triangle with ∠B = 90◦ . Let I be the incentre of ABC. Extend AI
and CI; let them intersect BC in D and AB in E respectively. Draw a line perpendicular to AI at
I to meet AC in J; draw a line perpendicular to CI at I to meet AC in K. Suppose DJ = EK.
Prove that BA = BC.
Solution: Extend JI to meet CB extended
at L. Then ALBI is a cyclic quadrilateral.
Therefore ∠BLI = ∠BAI = ∠IAC. There-
fore ∠LAD = ∠IBD = 45◦ . Since ∠AIL =
90◦ , it follows that ∠ALI = 45◦ . Therefore
AI = IL. This shows that the triangles AIJ
and LID are congruent giving IJ = ID. Simi-
larly, IK = IE. Since IJ ⊥ ID and IK ⊥ IE
and since DJ = EK, we see that IJ = ID =
IK = IE. Thus E, D, J, K are concyclic.
This implies together with DJ = EK that
EDJK is an isosceles trapezium. Therefore
DE k JK. Hence ∠EDA = ∠DAC = ∠A/2
and ∠DEC = ∠ECA = ∠C/2. Since IE =
ID, it follows that ∠A/2 = ∠C/2. This means
BA = BC.
6. (a) Given any natural number N ≥ 3, prove that there exists a strictly increasing sequence of N
positive integers in harmonic progression.
(b) Prove that there cannot exist a strictly increasing infinite sequence of positive integers which is
in harmonic progression.

Solution: (a) Let N ≥ 3 be a given positive integer. Consider the HP


1 1 1 1
1, , , , . . . , .
2 3 4 N
If we multiply this by N !, we get the HP
N! N! N! N!
N !, , , ,..., .
2 3 4 N
This is decreasing. We write this in reverse order to get a strictly increasing HP:
N! N! N! N! N!
, , ,..., , , N !.
N N −1 N −2 3 2

(b) Assume the contrary that there is an infinite strictly increasing sequence ha1 , a2 , a3 , . . . , i of pos-
itive integers which forms a harmonic progression. Define bk = 1/ak , for k ≥ 1. Then hb1 , b2 , b3 , . . .i
is a strictly decreasing sequence of positive rational numbers which is in an arithmetic progression.

2
Let d = b2 − b1 < 0 be its common difference. Then b1 − b2 > 0. Choose a positive integer n such
that
b1
n> .
b1 − b2
Then  
b1
bn+1 = b1 + (b2 − b1 )n = b1 − (b1 − b2 )n < b1 − × (b1 − b2 ) = 0.
b1 − b2
Thus for all large n, we see that bn is negative contradicting bn is positive for all n.

———-0———-

3
Regional Mathematical Olympiad-2016
Time: 3 hours October 23, 2016
Instructions:
• Calculators (in any form) and protractors are not allowed.
• Rulers and compasses are allowed.
• Answer all the questions.
• All questions carry equal marks. Maximum marks: 102.
• Answer to each question should start on a new page. Clearly indicate the
question number.
1. Let ABC be an isosceles triangle with AB = AC. Let Γ be its circumcircle and let O
be the centre of Γ. Let CO meet Γ in D. Draw a line parallel to AC through D. Let
it intersect AB at E. Suppose AE : EB = 2 : 1. Prove that ABC is an equilateral
triangle.
2. Let a, b, c be positive real numbers such that
ab bc ca
+ + = 1.
1 + bc 1 + ca 1 + ab
1 1 1 √
Prove that 3 + 3 + 3 ≥ 6 2.
a b c
3. The present ages in years of two brothers A and B, and their father C are three distinct
b−1 b+1
positive integers a, b, and c respectively. Suppose and are two consecutive
a−1 a+1
c−1 c+1
integers, and and are two consecutive integers. If a+b+c ≤ 150 determine
b−1 b+1
a, b and c.
4. A box contains 4032 answer scripts out of which exactly half have odd number of
marks. We choose 2 scripts randomly and, if the scores on both of them are odd
number, we add one mark to one of them, put the script back in the box and keep the
other script outside. If both scripts have even scores, we put back one of the scripts
and keep the other outside. If there is one script with even score and the other with
odd score, we put back the script with the odd score and keep the other script outside.
After following this procedure a number of times, there are 3 scripts left among which
there is at least one script each with odd and even scores. Find, with proof, the number
of scripts with odd scores among the three left.
5. Let ABC be a triangle, AD an altitude and AE a median. Assume B, D, E, C lie in
that order on the line BC. Suppose the incentre of triangle ABE lies on AD and the
incentre of ADC lies on AE. Find, with proof, the angles of triangle ABC.
6. (i) Prove that if an infinite sequence of strictly increasing positive integers in arith-
metic progression has one cube then it has infinitely many cubes.
(ii) Find, with justification, an infinite sequence of strictly increasing positive integers
in arithmetic progression which does not have any cube.
———-0000———-
1. Let ABC be an isosceles triangle with AB = AC. Let Γ be its circumcircle and let O be the centre
of Γ. Let CO meet Γ in D. Draw a line parallel to AC through D. Let it intersect AB at E.
Suppose AE : EB = 2 : 1. Prove that ABC is an equilateral triangle.
Solution: Extend DE to meet BC at F . Join
BD and DA. Since CD is a diameter, we
see that ∠DBC = 90◦ . Since DF is paral-
lel to AC, it follows that 4EBF ∼ 4ABC.
Hence EB = EF . Now ∠EBF = ∠EF B =
90◦ − ∠EDB. But ∠EBF + ∠EBD = 90◦ .
Hence we obtain ∠EBD = ∠EDB, which
gives EB = ED. Since AE : EB = 2 : 1
and EB = ED, we obtain AE = 2ED. Hence
∠DAB = 30◦ . This implies ∠DCB = 30◦
and hence ∠BDC = 60◦ . But then ∠BAC =
∠BDC = 60◦ and hence 4ABC is equilateral
2. Let a, b, c be positive real numbers such that
ab bc ca
+ + = 1.
1 + bc 1 + ca 1 + ab
1 1 1 √
Prove that 3
+ 3 + 3 ≥ 6 2.
a b c
Solution: The given condition is equivalent to
X
ab(1 + ca)(1 + ab) = (1 + ab)(1 + bc)(1 + ca).

This gives
X X X X X X
ab + a2 b2 + abc a + abc a2 b = 1 + ab + abc a + a2 b2 c2 .

Hence X X
a2 b2 c2 + 1 = a2 b2 + abc a2 b.
Using X X
a2 b2 ≥ 3(abc)4/3 , a2 b ≥ 3abc,
we get
a2 b2 c2 + 1 ≥ 3(abc)4/3 + 3(abc)2 .
Taking x = (abc)2/3 , this reduces 3 2 2
√ to 2x + 3x − 1 ≤ 0. This gives (x + 1) (2x − 1) ≤ 0. Hence
x ≤ 1/2. Therefore abc ≤ 1/2 2. Finally
1 1 1 3 √
+ + ≥ ≥ 6 2.
a3 b3 c3 abc

3. The present ages in years of two brothers A and B, and their father C are three distinct positive
b−1 b+1 c−1
integers a, b, and c respectively. Suppose and are two consecutive integers, and
a−1 a+1 b−1
c+1
and are two consecutive integers. If a + b + c ≤ 150 determine a, b and c.
b+1
Solution: We have
b−1 b+1 c−1 c+1
= l, = l − 1, = m, =m−1
a−1 a+1 b−1 b+1
b−1 b+1
(If we take a ≤ b, we see that a−1 ≥ a+1 .)
Consider the first two relations: b − 1 = l(a − 1), b + 1 = (l − 1)(a + 1). Solving for a, we get
a = 2l − 3 and hence b = 2l2 − 4l + 1. Using the second set of relations, we obtain b = 2m − 3 and
c = 2m2 − 4m + 1. Thius we have 2m − 3 = 2l2 − 4l + 1 or m = (l − 1)2 + 1. Obviously l > 1. If
l = 2, we get a = 1 which forces b = 1 and c = 1, which is impossible. If l = 3, we get a = 3, b = 7
and c = 31. If l ≥ 4, then m ≥ 10 and c ≥ 161. But then a + b + c > 150. Thus the only choice is
a = 3, b = 7 and c = 31.
4. A box contains answer 4032 scripts out of which exactly half have odd number of marks. We choose
2 scripts randomly and, if the scores on both of them are odd number, we add one mark to one of
them, put the script back in the box and keep the other script outside. If both scripts have even
scores, we put back one of the scripts and keep the other outside. If there is one script with even
score and the other with odd score, we put back the script with the odd score and keep the other
script outside. After following this procedure a number of times, there are 3 scripts left among
which there is at least one script each with odd and even scores. Find, with proof, the number of
scripts with odd scores among the three left.

Solution: There are three types of processes. In the first type, the scripts with odd scores decreases
by 2. In the second and third types, there is no change in the number of scripts with odd scores.
Hence at each step, the number of scripts with odd score decreases by 0 or 2. Since there are 2016
scripts with odd scores, the number of scripts with odd scores at the end is either 0 or 2. Since it
is given that there is at least one script with odd scores, two of the three must have odd scores.
5. Let ABC be a triangle, AD an altitude and AE a median. Assume B, D, E, C lie in that order on
the line BC. Suppose the incentre of triangle ABE lies on AD and the incentre of ADC lies on
AE. Find the angles of triangle ABC.

Solution: Since AD ⊥ BE and the incentre of 4ABE


lies on AD, it follows that ABE is isosceles. In particu-
lar ∠BAD = ∠DAE = α, say. Since AE is the bisector
of∠DAC, it follows that ∠EAC = ∠DAE = α. Moreover,
we have
AD DE
= .
AC CE
Since BE = EC = a2 , we also have DE = 12 BE = a4 . Thus
we get

AD DE a/4 1
= = = .
AC EC a/2 2
Since 4ADE is a right-angled triangle and AD = AC/2, it follows that ∠ACD = 30◦ . Hence
∠DAC = 60◦ . Since ∠DAC = 2α, we get α = 30◦ . Now ∠A = 3α and hence ∠A = 90◦ . This gives
∠B = 60◦ .
6. i) Prove that if an infinite sequence of strictly increasing positive integers in arithmetic progression
has one cube then it has infinitely many cubes.
(ii) Find, with justification, an infinite sequence of strictly increasing positive integers in arithmetic
progression which does not have any cube.

Solution:
i) Let a be the first term of the AP and d be the common difference. (Here a, d are positive integers.)
We can find an integer b such that b3 = a + (n − 1)d for some n ∈ N. Consider (b + d)3 . We observe
that
(b + d)3 = b3 + d(3b2 + 3bd + d2 ) = a + (n − 1 + 3b2 + 3bd + d2 )d = a + (m − 1)d,

2
where m = n + 3b2 + 3bd + d2 is a positive integer. Hence (b + d)3 is also in the given AP. More
generally, the same method shows that (b + kd)3 is in the AP for every k ∈ N. Hence the given AP
contains infinitely many cubes.
ii) Consider the AP h2, 6, 10, 14, . . .i. Here a = 2 and d = 4. The general term is 2 + 4k, where
k ≥ 0 is an integer. Suppose 2 + 4k = b3 for some integer b. Then 2 divides b. Hence b = 2c for
some c. Therefore 8c3 = 2 + 4k or 4c3 = 2k + 1. But this is impossible since LHS is even and RHS
is odd. We conclude that the AP h2, 6, 10, 14, . . .i does not contain any cube.

———-0———-

3
Regional Mathematical Olympiad-2000
Problems and Solutions

1. Let AC be a line segment in the plane and B a point between A and C. Construct isosceles
triangles P AB and QBC on one side of the segment AC such that ∠AP B = ∠BQC = 120◦
and an isosceles triangle RAC on the otherside of AC such that ∠ARC = 120◦ . Show
that P QR is an equilateral triangle.

Solution: We give here 2 different solutions.

1. Drop perpendiculars from P and Q to AC and extend them to meet AR, RC in K, L


respectively. Join KB, P B, QB, LB, KL.(Fig.1.)
P K

Q P
Q
B
A C
A B C
L

K
R R
Fig. 1 Fig. 2
Observe that K, B, Q are collinear and so are P, B, L. ( This is because ∠QBC =
∠P BA = ∠KBA and similarly ∠P BA = ∠CBL.) By symmetry we see that
∠KP Q = ∠P KL and ∠KP B = ∠P KB. It follows that ∠LP Q = ∠LKQ and hence
K, L, Q, P are concyclic. We also note that ∠KP L + ∠KRL = 60◦ + 120◦ = 180◦ .
This implies that P, K, R, L are concyclic. We conclude that P, K, R, L, Q are con-
cyclic. This gives

∠P RQ = ∠P KQ = 60◦ , ∠RP Q = ∠RKQ = ∠RAP = 60◦ .

2. Produce AP and CQ to meet at K. Observe that AKCR is a rhombus and BQKP


is a parallelogram.(See Fig.2.) Put AP = x, CQ = y. Then P K = BQ = y,
KQ = P B = x and AR = RC = CK = KA = x + y. Using cosine rule in triangle
P KQ, we get P Q2 = x2 + y 2 − 2xy cos 120◦ = x2 + y 2 + xy. Similarly cosine rule in
triangle QCR gives QR2 = y 2 + (x + y)2 − 2xy cos 60◦ = x2 + y 2 + xy and cosine rule
in triangle P AR gives RP 2 = x2 + (x + y)2 − 2xy cos 60◦ = x2 + y 2 + xy. It follows
that P Q = QR = RP .

2. Solve the equation y 3 = x3 + 8x2 − 6x + 8, for positive integers x and y.

1
Solution: We have

y 3 − (x + 1)3 = x3 + 8x2 − 6x + 8 − (x3 + 3x2 + 3x + 1) = 5x2 − 9x + 7.

Consider the quadratic equation 5x2 − 9x + 7 = 0. The discriminant of this equation is


D = 92 − 4 × 5 × 7 = −59 < 0 and hence the expression 5x2 − 9x + 7 is positive for all real
values of x. We conclude that (x + 1)3 < y 3 and hence x + 1 < y.
On the other hand we have

(x + 3)3 − y 3 = x3 + 9x2 + 27x + 27 − (x3 + 8x2 − 6x + 8) = x2 + 33x + 19 > 0

for all positive x. We conclude that y < x + 3. Thus we must have y = x + 2. Putting
this value of y, we get

0 = y 3 − (x + 2)3 = x3 + 8x2 − 6x + 8 − (x3 + 6x2 + 12x + 8) = 2x2 − 18x.

We conclude that x = 0 and y = 2 or x = 9 and y = 11.

3. Suppose hx1 , x2 , . . . , xn , . . .i is a sequence of positive real numbers such that x1 ≥ x2 ≥


x3 ≥ · · · ≥ xn · · · , and for all n
x1 x4 x9 x 2
+ + + · · · + n ≤ 1.
1 2 3 n
Show that for all k the following inequality is satisfied:
x1 x2 x3 xk
+ + + ··· + ≤ 3.
1 2 3 k

Solution: Let k be a natural number and n be the unique integer such that (n − 1)2 ≤
k < n2 . Then we see that
k
X xr x
1 x2 x3   x4 x5 x8 
≤ + + + + + ··· +
r=1
r 1 2 3 4 5 8
x(n−1)2
 
xk xn2 −1
+··· + + ··· + + ··· + 2
(n − 1)2 k n −1
x x1 x1   x4 x4 x4 
1
≤ + + + + + ··· +
1 1 1 4 4  4
x(n−1)2 x(n−1)2

+··· + + ··· +
(n − 1)2 (n − 1)2
3x1 5x2 (2n − 1)x(n−1)2
= + + ··· +
1 4 (n − 1)2
n−1
X (2r + 1)xr2
=
r2
r=1

2
n−1
X 3r
≤ x2
r=1
r2 r
n−1
X xr2
= 3 ≤ 3,
r
r=1
where the last inequality follows from the given hypothesis.
4. All the 7-digit numbers containing each of the digits 1, 2, 3, 4, 5, 6, 7 exactly once, and not
divisible by 5, are arranged in the increasing order. Find the 2000-th number in this list.
Solution: The number of 7-digit numbers with 1 in the left most place and containing
each of the digits 1, 2, 3, 4, 5, 6, 7 exactly once is 6! = 720. But 120 of these end in 5 and
hence are divisible by 5. Thus the number of 7-digit numbers with 1 in the left most place
and containing each of the digits 1, 2, 3, 4, 5, 6, 7 exactly once but not divisible by 5 is 600.
Similarly the number of 7-digit numbers with 2 and 3 in the left most place and containing
each of the digits 1, 2, 3, 4, 5, 6, 7 exactly once but not divisible by 5 is also 600 each. These
account for 1800 numbers. Hence 2000-th number must have 4 in the left most place.
Again the number of such 7-digit numbers beginning with 41,42 and not divisible by 5 is
120 − 24 = 96 each and these account for 192 numbers. This shows that 2000-th number
in the list must begin with 43.
The next 8 numbers in the list are: 4312567, 4312576, 4312657, 4312756, 4315267, 4315276,
4315627 and 4315672. Thus 2000-th number in the list is 4315672.
5. The internal bisector of angle A in a triangle ABC with AC > AB, meets the circumcircle
Γ of the triangle in D. Join D to the centre O of the circle Γ and suppose DO meets AC
in E, possibly when extended. Given that BE is perpendicular to AD, show that AO is
parallel to BD.
Solution: We consider here the case when ABC is an acute-angled triangle; the cases
when ∠A is obtuse or one of the angles ∠B and ∠C is obtuse may be handled similarly.

N O
B C
M

3
Let M be the point of intersection of DE and BC; let AD intersect BE in N . Since M E
is the perpendicular bisector of BC, we have BE = CE. Since AN is the internal bisector
of ∠A, and is perpendicular to BE, it must bisect BE; i.e., BN = N E. This in turn
implies that DN bisects ∠BDE. But ∠BDA = ∠BCA = ∠C. Thus ∠ODA = ∠C. Since
OD = OA, we get ∠OAD = ∠C. It follows that ∠BDA = ∠C = ∠OAD. This implies
that OA is parallel to BD.

6. (i) Consider two positive integers a and b which are such that aabb is divisible by 2000.
What is the least possible value of the product ab?
(ii) Consider two positive integers a and b which are such that abba is divisible by 2000.
What is the least possible value of the product ab?

Solution: We have 2000 = 24 53 .


(i) Since 2000 divides aa bb , it follows that 2 divides a or b and similarly 5 divides a or b.
In any case 10 divides ab. Thus the least possible value of ab for which 2000|aa bb must
be a multiple of 10. Since 2000 divides 1010 11 , we can take a = 10, b = 1 to get the least
value of ab equal to 10.
(ii) As in (i) we conclude that 10 divides ab. Thus the least value of ab for which
2000|ab ba is again a multiple of 10. If ab = 10, then the possibilities are (a, b) =
(1, 10), (2, 5), (5, 2), (10, 1). But in all these cases it is easy to verify that 2000 does not
divide ab ba . The next multiple of 10 is 20. In this case we can take (a, b) = (4, 5) and
verify that 2000 divides 45 54 . Thus the least value here is 20.

7. Find all real values of a for which the equation x4 − 2ax2 + x + a2 − a = 0 has all its roots
real.

Solution: Let us consider x4 − 2ax2 + x + a2 − a = 0 as a quadratic equation in a. We


see that thee roots are
a = x2 + x, a = x2 − x + 1.
Thus we get a factorisation

(a − x2 − x)(a − x2 + x − 1) = 0.

It follows that x2 + x = a or x2 − x + 1 = a. Solving these we get


√ √
−1 ± 1 + 4a −1 ± 4a − 3
x= , or x = .
2 2
Thus all the four roots are real if and only if a ≥ 3/4.

4


 

    !
" # $%&%'
()+*-,/.10-2436547!8 9;:<,>=?,@*BAC,>79EDEDGF
HIKJ4J4LNLNMM1yOQPSZ\LzRUTWM^]_VL|X1{\Y[f?boT}Z\MKLQfUM^hC]_boLT}MlRULN`am~M^dlbaLeM^kcEM^V_baf?LedgT&fUfUhihCRUOQTjP[RUkNRUcWTWMlVBLX1MlmnYboRUIT_p?€‚`aRLƒUqrLOsRUT…XQ„&t\`ouKboTWbaLzM^];†BPv‡ˆf?M^T]_mnqgf?c_X[p?]wRUTEy‰VwuKYSbaM^f?]T;†ŠqxO f?T I
qr†;O‹ RUboTWdKV-{6LN‡Œmn{6f?LeTTEVWq1bokNXQcWI`oRŽ@m–c_Mlf“{W{\OQf?dlP—LsRU TWVRU‡iTWV-‘Š‘’bodR{\m^LNL“m^{\`of>LeTWkNRVWMlb˜LekNV-cW`oRf?m–TMlOQfP”X1YsRUI–TWV™šm^Xf•UYSLm^M^Le]Wd^R{6MxLeY|kM^ bo•ULe`ab˜„Udt‚{EdnRcWmnknRU]`˜`aLeM^`\]WMlRf M
P|‘I
›>IKœboTEV<RU`o`\{WmnbožsLed‚ŸRUTWV& Qd^cWk~]<M^]WRM¡Ÿ6¢¡£ˆ¤nŸ/ K£  ¢gbodM^]_Ld^¥>cWRm^L1fUh¡RUT&b˜T}MlLNpULNmeI
¦>IKœboTEVM^]_LzT>cWžzZ\LNmfUh‚{\f?d^baM^ba•ULboT…MlLNpULNmnd§wuK]Wb˜kn]<d^RM^bod^h¨„ M^]_Lkf?TWVWboM^baf?T
© ªU§ ª6«z¬ © HN§­_H_«¯®
°²± LNm^L³ ´µ¶V_LeT_fUMlLedetWh¨fUmxRUT}„m^LeRU`G´Wt_M^]_Lz`oRm^pULed^MKboT…MlLNpULNmKT_fUML·_kLNLeVWboT_p“´_¸_LUI¹p_Ir³º¤U»¼µ ¬ HI¾½
¼_Ir¿¯f?TWd^boV_LNmRUT&ÀjÁ<ÀjRm^mnR„ fUhT>cWžzZ\LNmndeÂ
ÃÄÄÄ ÆEÇlÇ ÆEÇ ¢ ÆWÇÉÈ ÊNÊNÊËÆEÇÍÌ ÐNÑÑÑ
Æ ¢I Ç Æ ¢l¢ Æ ¢ È ÊNÊNÊËÆ ¢ I Ì
Å Æ}̅I ÇÎÆ…Ì Æ}ÌÈ ÊNÊNÊÏƅÌ!I Ì Ò
¢
ŽÚ_c_{W{\Hf?Öndl›>L Ö LeRUknÖl]ÓÀÓmnRUfTWuÔV&kÛ f?TWdnboHdlÖnM^dQ›>Ö fUhrM^Öl]_À¡LIÀÕÜÝhšT>À cWž|bodrZ\f>LNVEmnd<VÞtÞH{WÖn›>m^fÖn¦>•ULÖ ®NM^®N]W® RÖlÀ×M1M^b˜]_TLsd^T>f?cWžžzL<Z\fULNmnmnV_d LNmsRUÖTWV Æ…Ö ØºÙ ¬ Ö Æ!ÙnØÖ h+fUm
Rmn¬ LHÖn›>Ön¦>®NÖ ®N®N® ®N® ÖlÀjboT<dlf?ž¬ LfUm~V_LN®Nme®N®I ÆEÇlÇ Æ ¢l¢ Æ>ÈlÈ ®N®N® ƅÌ!Ì
ß>IÜàâTO R|Mlmnbo›?RUâTWXãp?`aLKRUqrTWVOXXá t…á[bodiqxRO {\ICf?™CboT…mnMCf•Uf?LzT“M^Os]ERXŒMrd^â¯cWq k~]sM^]W¤R›UM¯ä!qxI á[bodšM^]_Lxb˜T}MlLNmnTERU`WZEbod^LekMlfUmšfUh¶â¯q|IiŽ@c_{W{\f?dlL
¬ ¬ ¬
å>IÜÝh‚§4Ölæ6Ö^´Rm^LM^]WLzd^boV_Led¯fUhRMlmnb˜RUT_p?`aLUI¡M^]_LeT<{Em^f•ULM^]ERM
ç § ¢ ° æQèj´…½4£æ ¢ ° ´|è;§\½Þ£ ´ ¢ ° §“è;æW½ ç@é §/æ>´ ®
¤…IKêE™šmnm^dlfM•UL1HN­UM^­U]E­sRM–{\M^f?]Wd^boLxM^ba{W•UmnLgf>f>VWVWcEVkMiboT}fUhGMlLNM^pU]WLNLrm~dêEmnZ…dl„<MxRHN­Už­U­QcW`a{\M^baf?{/d^`aboLxM^bafU•UhCLx›LN­U•U­_LeHT<I b˜T}MlLNpULNmndVEbìë6LNm~dšh+m^f?žM^]_Lg{Wm^f@VWcWkMšfUh¶M^]_L

H
Problems and Solutions: CRMO-2011

1. Let ABC be a triangle. Let D, E, F be points respectively on the segments BC,


CA, AB such that AD, BE, CF concur at the point K. Suppose BD/DC =
BF/F A and ∠ADB = ∠AF C. Prove that ∠ABE = ∠CAD.
Solution: Since BD/DC = BF/F A,
the lines DF and CA are parallel. We
also have ∠BDK = ∠ADB = ∠AF C =
180◦ − ∠BF K, so that BDKF is a cyclic
quadrilateral. Hence ∠F BK = ∠F DK.
Finally, we get

∠ABE = ∠F BK = ∠F DK
= ∠F DA = ∠DAC,

since F D k AC.
2. Let (a1 , a2 , a3 , . . . , a2011 ) be a permutation (that is a rearrangement) of the num-
bers 1, 2, 3, . . . , 2011. Show there exist two numbers j, k such that 1 ≤ j <
that
k ≤ 2011 and aj − j = ak − k .
Solution: Observe that 2011
P 
j=1 aj − j = 0, since (a1 , a2 , a3 , . . . , a2011 ) is a permu-
P2011
tation of 1, 2, 3, . . . , 2011. Hence j=1 aj − j is even. Suppose aj − j 6= ak − k

for all j 6= k. This means the collection |aj − j| : 1 ≤ j ≤ 2011 is the same
as the collection {0, 1, 2, . . . , 2010} as the maximum difference is 2011-1=2010.
Hence
2011
aj − j = 1 + 2 + 3 + · · · + 2010 = 2010 × 2011 = 2011 × 1005,
X
2
j=1

which is odd. This shows that |aj − j| = |ak − k| for some j 6= k.

3. A natural number n is chosen strictly between two consecutive perfect squares.


The smaller of these two squares is obtained by subtracting k from n and the
larger one is obtained by adding l to n. Prove that n − kl is a perfect square.
Solution: Let u be a natural number such that u2 < n < (u + 1)2 . Then
n − k = u2 and n + l = (u + 1)2 . Thus

n − kl = n − n − u2 (u + 1)2 − n
 

= n − n(u + 1)2 + n2 + u2 (u + 1)2 − nu2


 
= n2 + n 1 − (u + 1)2 − u2 + u2 (u + 1)2
= n2 + n 1 − 2u2 − 2u − 1 + u2 (u + 1)2

2
= n2 − 2nu(u + 1) + u(u + 1)
2
= n − u(u + 1) .
4. Consider a 20-sided convex polygon K, with vertices A1 , A2 , . . . , A20 in that
order. Find the number of ways in which three sides of K can be chosen so
that every pair among them has at least two sides of K between them. (For
example (A1 A2 , A4 A5 , A11 A12 ) is an admissible triple while (A1 A2 , A4 A5 , A19 A20 )
is not.)
Solution: First let us count all
A 18 the admissible triples having A1 A2
as one of the sides. Having cho-
K A4 sen A1 A2 , we cannot choose A2 A3 ,
A3 A4 , A20 A1 nor A19 A20 . Thus we
A 19
have to choose two sides sepa-
rated by 2 sides among 15 sides
A 20 A3
A4 A5 , A5 A6 , . . ., A18 A19 . If A4 A5 is
A1 A2 one of them, the choice for the re-
maining side is only from 12 sides

A7 A8 , A8 A9 , . . ., A18 A19 . If we choose A5 A6 after A1 A2 , the choice for the third


side is now only from A8 A9 , A9 A10 , . . ., A18 A19 (11 sides). Thus the number
of choices progressively decreases and finally for the side A15 A16 there is only
one choice, namely, A18 A19 . Hence the number of triples with A1 A2 as one of
the sides is
12 × 13
12 + 11 + 10 + · · · + 1 = = 78.
2
Hence the number of triples then would be (78 × 20)/3 = 520.
n(n − 7)(n − 8)
Remark: For an n-sided polygon, the number of such triples is ,
6
for n ≥ 9. We may check that for n = 20, this gives (20 × 13 × 12)/6 = 520.

5. Let ABC be a triangle and let BB1 , CC1 be respectively the bisectors of ∠B,
∠C with B1 on AC and C1 on AB. Let E, F be the feet of perpendiculars
drawn from A onto BB1 , CC1 respectively. Suppose D is the point at which
the incircle of ABC touches AB. Prove that AD = EF .
Solution: Observe that ∠ADI =
∠AF I = ∠AEI = 90◦ . Hence
A, F, D, I, E all lie on the circle
with AI as diameter. We also
know
∠A
∠BIC = 90◦ + = ∠F IE.
2
This gives
 
∠A
∠F AE = 180◦ − 90◦ +
2
∠A
= 90◦ − .
2
∠A
We also have ∠AID = 90◦ − . Thus ∠F AE = ∠AID. This shows the chords
2
F E and AD subtend equal angles at the circumference of the same circle.
Hence they have equal lengths, i.e., F E = AD.
6. Find all pairs (x, y) of real numbers such that
2 +y 2
16x + 16x+y = 1.

Solution: Observe that

1 2 1 2
   
2 1 2
x +y+x+y + = x+ + y+ ≥ 0.
2 2 2

This shows that x2 + y + x + y 2 ≥ (−1/2). Hence we have

2 1/2
2 +y 2
 2 
1 = 16x + 16x+y ≥ 2 16x +y · 16x+y , (by AM-GM inequality)
2 1/2
 2 
= 2 16x +y+x+y
≥ 2(16)−1/4 = 1.

Thus equality holds every where. We conclude that


 2  2
1 1
x+ + y+ = 0.
2 2

This shows that (x, y) = (−1/2, −1/2) is the only solution, as can easily be
verified.

———-00000———-
Problems and Solutions. . . CRMO-2002
1. In an acute triangle ABC, points D, E, F are located on the sides BC, CA, AB respectively
such that
CD CA AE AB BF BC
= , = , = .
CE CB AF AC BD BA
Prove that AD, BE, CF are the altitudes of ABC.

Solution: Put CD = x. Then with usual notations we get


CD · CB ax
CE = = .
CA b
Since AE = AC − CE = b − CE, we obtain

b2 − ax AE · AC b2 − ax
AE = , AF = = .
b AB c

A A

α E E
β

α β
α
F F

B D C B D C

Fig. 1 Fig. 2

This in turn gives


c2 − b2 + ax
BF = AB − AF = .
c
Finally we obtain
c2 − b2 + ax
BD = .
a
Using BD = a − x, we get
a 2 − c2 + b 2
x= .
2a
However, if L is the foot of perpendicular from A on to BC then, using Pythagoras theorem
in triangles ALB and ALC we get

b2 − LC 2 = c2 − (a − LC)2

which reduces to LC = (a2 − c2 + b2 )/2a. We conclude that LC = DC proving L = D. Or, we


can also infer that x = b cos C from cosine rule in triangle ABC. This implies that CD = CL,
since CL = b cos C from right triangle ALC. Thus AD is altitude on to BC. Similar proof
works for the remaining altitudes.
Alternately, we see that CD ·CB = CE ·CA, so that ABDE is a cyclic quadrilateral. Similarly
we infer that BCEF and CAF D are also cyclic quadrilaterals. (See Fig. 2.) Thus ∠AEF =
∠B = ∠CED. Moreover ∠BED = ∠DAF = ∠DCF = ∠BCF = ∠BEF . It follows that
∠BEA = ∠BEC and hence each is a right angle thus proving that BE is an altitude. Similarly
we prove that CF and AD are altitudes. (Note that the concurrence of the lines AD, BE,
CF are not required.)
2. Solve the following equation for real x:

(x2 + x − 2)3 + (2x2 − x − 1)3 = 27(x2 − 1)3 .

Solution: By setting u = x2 + x − 2 and v = 2x2 − x − 1, we observe that the equation reduces


to u3 + v 3 = (u + v)3 . Since (u + v)3 = u3 + v 3 + 3uv(u + v), it follows that uv(u + v) = 0.
Hence u = 0 or v = 0 or u + v = 0. Thus we obtain x2 + x − 2 = 0 or 2x2 − x − 1 = 0 or
x2 − 1 = 0. Solving each of them we get x = 1, −2 or x = 1, −1/2 or x = 1, −1. Thus x = 1 is
a root of multiplicity 3 and the other roots are −1, −2, −1/2.

(Alternately, it can be seen that x − 1 is a factor of x2 + x − 2, 2x2 − x − 1 and x2 − 1. Thus


we can write the equation in the form

(x − 1)3 (x + 2)3 + (x − 1)3 (2x + 1)3 = 27(x − 1)3 (x + 1)3 .

Thus it is sufficient to solve the cubic equation

(x + 2)3 + (2x + 1)3 = 27(x + 1)3 .

This can be solved as earlier or expanding every thing and simplifying the relation.)
3. Let a, b, c be positive integers such that a divides b2 , b divides c2 and c divides a2 . Prove that
abc divides (a + b + c)7 .

Solution: Consider the expansion of (a + b + c)7 . We show that each term here is divisible
by abc. It contains terms of the form rklm ak bl cm , where rklm is a constant( some binomial
coefficient) and k, l, m are nonnegative integers such that k + l + m = 7. If k ≥ 1, l ≥ 1, m ≥ 1,
then abc divides ak bl cm . Hence we have to consider terms in which one or two of k, l, m are
zero. Suppose for example k = l = 0 and consider c7 . Since b divides c2 and a divides c4 ,
it follows that abc divides c7 . A similar argument gives the result for a7 or b7 . Consider the
case in which two indices are nonzero, say for example, bc6 . Since a divides c4 , here again abc
divides bc6 . If we take b2 c5 , then also using a divides c4 we obtain the result. For b3 c4 , we use
the fact that a divides b2 . Similar argument works for b4 c3 , b5 c2 and b6 c. Thus each of the
terms in the expansion of (a + b + c)7 is divisible by abc.
4. Suppose the integers 1, 2, 3, . . . , 10 are split into two disjoint collections a1 , a2 , a3 , a4 , a5 and
b1 , b2 , b3 , b4 , b5 such that
a1 < a2 < a3 < a4 < a5 ,
b1 > b2 > b3 > b4 > b5 .
(i) Show that the larger number in any pair {aj , bj }, 1 ≤ j ≤ 5, is at least 6.
(ii) Show that |a1 − b1 | + |a2 − b2 | + |a3 − b3 | + |a4 − b4 | + |a5 − b5 | = 25 for every such partition.

2
Solution:
(i) Fix any pair {aj , bj }. We have a1 < a2 < · · · < aj−1 < aj and bj > bj+1 > · · · > b5 . Thus
there are j − 1 numbers smaller than aj and 5 − j numbers smaller than bj . Together
they account for j − 1 + 5 − j = 4 distinct numbers smaller than aj as well as bj . Hence
the larger of aj and bj is at least 6.
(ii) The first part shows that the larger numbers in the pairs {aj , bj }, 1 ≤ j ≤ 5, are
6, 7, 8, 9, 10 and the smaller numbers are1, 2, 3, 4, 5. This implies that

|a1 − b1 | + |a2 − b2 | + |a3 − b3 | + |a4 − b4 | + |a5 − b5 |


= 10 + 9 + 8 + 7 + 6 − (1 + 2 + 3 + 4 + 5) = 25.

5. The circumference of a circle is divided into eight arcs by a convex quadrilateral ABCD, with
four arcs lying inside the quadrilateral and the remaining four lying outside it. The lengths of
the arcs lying inside the quadrilateral are denoted by p, q, r, s in counter-clockwise direction
starting from some arc. Suppose p + r = q + s. Prove that ABCD is a cyclic quadrilateral.

Solution: Let the lengths of the arcs XY , U V , EF , GH be respectively p, q, r, s. We also


the following notations: (See figure)

D G
δ_1
F
C
δ_4
γ_2 γ_1

δ_2 γ_4 E
H
δ_3 γ_3

P β_3
α_3

β_2 V
β_4
β_1
α_4
X U B
α_2
α_1

A Y
Fig. 3

∠XAY = α1 , ∠AY P = α2 , ∠Y P X = α3 , ∠P XA = α4 , ∠U BY = β1 , ∠BV P = β2 , ∠V P U =


β3 , ∠P U B = β4 , ∠ECF = γ1 , ∠CF P = γ2 , ∠F P E = γ3 , ∠P EC = γ4 , ∠GDH = δ1 ,
∠DHP = δ2 , ∠HP G = δ3 , ∠P GD = δ4 .
We observe that X X X X
αj = βj = γj = δj = 2π.
It follows that X X
(αj + γj ) = (βj + δj ).
On the other hand, we also have α2 = β4 since P Y = P U . Similarly we have other relations:
β2 = γ4 , γ2 = δ4 and δ2 = α4 . It follows that

α1 + α3 + γ1 + γ3 = β1 + β3 + δ1 + δ3 .

3
But p + r = q + s implies that α3 + γ3 = β3 + δ3 . We thus obtain

α1 + γ1 = β1 + δ1 .

Since α1 + γ1 + β1 + δ1 = 360◦ , it follows that ABCD is a cyclic quadrilateral.


6. For any natural number n > 1, prove the inequality:
1 1 2 3 n 1 1
< 2 + + + ··· + 2 < + .
2 n + 1 n2 + 2 n2 + 3 n +n 2 2n

Solution: We have n2 < n2 + 1 < n2 + 2 < n2 + 3 · · · < n2 + n. Hence we see that


1 2 n 1 2 n
+ + ··· + 2 > + + ···+ 2
n2 + 1 n2 + 2 n +n n2 + n n2 + n n +n
1 1
= 2
(1 + 2 + 3 + · · · n) = .
n +n 2
Similarly, we see that
1 2 n 1 2 n
+ + ···+ 2 < + 2 + ···+ 2
n2 + 1 n2 + 2 n +n n2 n n
1 1 1
= 2
(1 + 2 + 3 + · · · n) = + .
n 2 2n

7. Find all integers a, b, c, d satisfying the following relations:


(i) 1 ≤ a ≤ b ≤ c ≤ d;
(ii) ab + cd = a + b + c + d + 3.

Solution: We may write (ii) in the form

ab − a − b + 1 + cd − c − d + 1 = 5.

Thus we obtain the equation (a − 1)(b − 1) + (c − 1)(d − 1) = 5. If a − 1 ≥ 2, then (i) shows


that b − 1 ≥ 2, c − 1 ≥ 2 and d − 1 ≥ 2 so that (a − 1)(b − 1) + (c − 1)(d − 1) ≥ 8. It follows
that a − 1 = 0 or 1.
If a − 1 = 0, then the contribution from (a − 1)(b − 1) to the sum is zero for any choice of b.
But then (c − 1)(d − 1) = 5 implies that c − 1 = 1 and d − 1 = 5 by (i). Again (i) shows that
b − 1 = 0 or 1 since b ≤ c. Taking b − 1 = 0, c − 1 = 1 and d − 1 = 5 we get the solution
(a, b, c, d) = (1, 1, 2, 6). Similarly, b−1 = 1, c−1 = 1 and d−1 = 5 gives (a, b, c, d) = (1, 2, 2, 6).
In the other case a − 1 = 1, we see that b − 1 = 2 is not possible for then c − 1 ≥ 2 and
d − 1 ≥ 2. Thus b − 1 = 1 and this gives (c − 1)(d − 1) = 4. It follows that c − 1 = 1,
d − 1 = 4 or c − 1 = 2, d − 1 = 2. Considering each of these, we get two more solutions:
(a, b, c, d) = (2, 2, 2, 5), (2, 2, 3, 3).
It is easy to verify all these four quadruples are indeed solutions to our problem.

4
Solutions to CRMO-2003

1. Let ABC be a triangle in which AB = AC and ∠CAB = 90◦ . Suppose M and


N are points on the hypotenuse BC such that BM 2 + CN 2 = M N 2 . Prove that
∠M AN = 45◦ .

Solution:
Draw CP perpendicular to CB and BQ perpendicular to CB such that CP = BM ,
BQ = CN . Join P A, P M , P N , QA, QM , QN . (See Fig. 1.)

C

P N

α
α

M
α 2β

A B

α
Q
Fig. 1.

In triangles CP A and BM A, we have ∠P CA = 45◦ = ∠M BA; P C = M B, CA =


BA. So △CP A ≡ △BM A. Hence ∠P AC = ∠BAM = α, say. Consequently,
∠M AP = ∠BAC = 90◦ , whence P AM C is a cyclic quadrilateral. Therefore ∠P M C =
∠P AC = α. Again P N 2 = P C 2 + CN 2 = BM 2 + CN 2 = M N 2 . So P N = M N , giv-
ing ∠N P M = ∠N M P = α, in △P M N . Hence ∠P N C = 2α. Likewise ∠QM B = 2β,
where β = ∠CAN . Also △N CP ≡ △QBM , as CP = BM, N C = BQ and
∠N CP = 90◦ = ∠QBM . Therefore, ∠CP N = ∠BM Q = 2β, whence 2α + 2β = 90◦ ;
α + β = 45◦ ; finally ∠M AN = 90◦ − (α + β) = 45◦ .

Aliter: Let AB = AC = a, so that BC = 2a; and ∠M AB = α, ∠CAN = β.(See
Fig. 2.)
By the Sine Law, we have from △ABM that
BM AB
= .
sin α sin(α + 45◦ )
√ √
a 2 sin α a 2u
So BM = = , where u = tan α.
cos α + sin α 1+u

N
π/4

a
β
M

π/4
α

A a B

Fig. 2.

a 2v
Similarly CN = , where v = tan β. But
1+v

BM 2 + CN 2 = M N 2 = (BC − M B − N C)2
= BC 2 + BM 2 + CN 2
− 2BC · M B − 2BC · N C + M B · N C.

So
BC 2 − 2BC · M B − 2BC · N C + 2M B · N C = 0.
This reduces to
√ √
2
√ a 2u √ a 2v 4a2 uv
2a − 2 2a − 2 2a + = 0.
1+u 1+v (1 + u)(1 + v)

Multiplying by (1 + u)(1 + v)/2a2 , we obtain

(1 + u)(1 + v) − 2u(1 + v) − 2v(1 + u) + 2uv = 0.

Simplification gives 1 − u − v − uv = 0. So
u+v
tan(α + β) = = 1.
1 − uv
This gives α + β = 45◦ ,whence ∠M AN = 45◦ , as well.

2
  h i  
n n  n
2. If n is an integer greater than 7, prove that − is divisible by 7. Here
7 7 7
denotes the number of ways of choosing 7 objects from among nobjects; also, for any
real number x, [x] denotes the greatest integer not exceeding x.

Solution: We have  
n n(n − 1)(n − 2) . . . (n − 6)
= .
7 7!
In the numerator, there is a factor divisible by 7, and the other six factors leave the
remainders 1,2,3,4,5,6 in some order when divided by 7.
Hence the numerator may be written as

7k · (7k1 + 1) · (7k2 + 2) · · · (7k6 + 6).


hni 
Also we conclude that = k, as in the set n, n − 1, . . . n − 6 , 7k is the only
p
number which is a multiple of 7. If the given number is called Q, then
(7k1 + 1)(7k2 + 2) . . . (7k6 + 6)
Q = 7k · −k
 7! 
(7k1 + 1) . . . (7k6 + 6) − 6!
= k
6!
k[7t + 6! − 6!]
=
6!
7tk
= .
6!
We know that Q is an integer, and so 6! divides 7tk. Since gcd(7, 6!) = 1, even after
cancellation there is a factor of 7 still left in the numerator. Hence 7 divides Q, as
desired.

3. Let a, b, c be three positive real numbers such that a + b + c = 1. Prove that among
the three numbers a − ab, b − bc, c − ca there is one which is at most 1/4 and there
is one which is at least 2/9.
Solution: By AM-GM inequality, we have

a+1−a 2 1
 
a(1 − a) ≤ = .
2 4
Similarly we also have
1 1
b(1 − b) ≤ and c(1 − c) ≤ .
4 4
Multiplying these we obtain
1
abc(1 − a)(1 − b)(1 − c) ≤ .
43

3
We may rewrite this in the form
1
a(1 − b) · b(1 − c) · c(1 − a) ≤ .
43

Hence one factor at least among a(1 − b), b(1 − c), c(1 − a) has to be less than or
1 1
equal to ; otherwise lhs would exceed 3 .
4 4
Again consider the sum a(1−b)+b(1−c)+c(1−a). This is equal to a+b+c−ab−bc−ca.
We observe that  2
3 ab + bc + ca ≤ a + b + c ,
which, in fact, is equivalent to (a − b)2 + (b − c)2 + (c − a)2 ≥ 0. This leads to the
inequality
1 1 2
a + b + c − ab − bc − ca ≥ (a + b + c) − (a + b + c)2 = 1 − = .
3 3 3

Hence one summand at least among a(1 − b), b(1 − c), c(1 − a) has to be greater
2 2
than or equal to ; otherwise lhs would be less than .
9 3
4. Find the number of ordered triples (x, y, z) of nonnegative integers satisfying the
conditions:

(i) x ≤ y ≤ z;
(ii) x + y + z ≤ 100.

Solution: We count by brute force considering the cases x = 0, x = 1, . . . , x = 33.


Observe that the least value x can take is zero, and its largest value is 33.
 
x = 0 If y = 0, then z ∈ 0, 1, 2, . . . , 100 ; if y=1, then z ∈ 1, 2, . . . , 99 ; if y = 2,
then z ∈ 2, 3, . . . , 98 ; and so on. Finally if y = 50, then z ∈ {50}. Thus there are
altogether 101 + 99 + 97 + · · · + 1 = 512 possibilities.

x
 = 1. Observe that y ≥ 1. If y =
 1, then z ∈ 1, 2, . . . , 98 ; if y = 2, then z ∈
2, 3,
 . . . , 97
; if y = 3, then z ∈ 3, 4, . . . , 96 ; and so on. Finally if y = 49, then
z ∈ 49, 50 . Thus there are altogether 98 + 96 + 94 + · · · + 2 = 49 · 50 possibilities.
General
 case. Let x be even, say, x = 2k, 0 ≤ k ≤ 16. If y = 2k, then z ∈
2k, 2k + 1, . . . , 100 − 4k ; if y = 2k + 1, then z ∈ 2k + 1, 2k + 2, . . . , 99 − 4k ; if
y = 2k + 2, then z ∈ 2k + 2, 2k + 3, . . . , 99 − 4k ; and so on.

Finally, if y = 50 − k, then z ∈ 50 − k . There are altogether

(101 − 6k) + (99 − 6k) + (97 − 6k) + · · · + 1 = (51 − 3k)2

possibilities.

4

Let x be odd, say, x = 2k + 1, 0 ≤ k ≤16. If y = 2k + 1, then z ∈ 2k + 1, 2k +
2, . . . , 98 −
 4k ; if y = 2k + 2, then z ∈ 2k + 2, 2k + 3, . . . , 97 − 4k ; if y = 2k + 3,
then z ∈ 2k + 3, 2k + 4, . . . , 96 − 4k ; and so on.

Finally, if y = 49 − k, then z ∈ 49 − k, 50 − k . There are altogether

(98 − 6k) + (96 − 6k) + (94 − 6k) + · · · + 2 = (49 − 3k)(50 − 3k)

possibilities.
The last two cases would be as follows:
 
x = 32: if y = 32, then z ∈ 32, 33, 34, 35, 36 ; if y = 33, then z ∈ 33, 34, 35 ; if
2

y = 34, then z ∈ 34 ; altogether 5 + 3 + 1 = 9 = 3 possibilities.

x = 33: if y = 33, then z ∈ 33, 34 ; only 2=1.2 possibilities.
Thus the total number of triples, say T, is given by,
16
X 16
X
2
T = (51 − 3k) + (49 − 3k)(50 − 3k).
k=0 k=0

Writing this in the reverse order, we obtain


17
X 17
X
T = (3k)2 + (3k − 2)(3k − 1)
k=1 k=0
17
X X17
= 18 k2 − 9 k + 34
k=1 k=1
   
17 · 18 · 35 17 · 18
= 18 −9 + 34
6 2
= 30, 787.

Thus the answer is 30787.

Aliter
It is known that the number of ways in which a given positive integer n ≥ 3 can be
expressed as a sum of three positive
 2  integers x, y, z (that is, x + y + z = n), subject
n
to the condition x ≤ y ≤ z is , where {a} represents the integer closest to a. If
12
(n + 3)2
 
zero values are allowed for x, y, z then the corresponding count is , where
12
now n ≥ 0.

Since in our problem n = x + y + z ∈ 0, 1, 2, . . . , 100 , the desired answer is
100 
(n + 3)2
X 
.
n=0
12

5
For n = 0, 1, 2, 3, . . . , 11, the corrections for {} to get the nearest integers are
3 −4 −1 −1 −4 3 −4 −1 −1 −4
, , , 0, , , , , , 0, , .
12 12 12 12 12 12 12 12 12 12
So, for 12 consecutive integer values of n, the sum of the corrections is equal to
 
3−4−1−0−1−4−3 −7
×2= .
12 6

101 5 
Since = 8 + , there are 8 sets of 12 consecutive integers in 3,4,5, ... ,103
12 12
with 99,100,101,102,103 still remaining. Hence the total correction is
 
−7 3−4−1−0−1 −28 1 −115
×8+ = − = .
6 12 3 4 12

So the desired number T of triples (x, y, z) is equal to


100
X (n + 3)2 115
T = −
12 12
n=0
12
+ 22 + 32 + · · · + 1032 − 12 + 22
 
115
= −
12 12
103 · 104 · 207 5 115
= − −
6 · 12 12 12
= 30787.

5. Suppose P is an interior point of a triangle ABC such that the ratios

d(A, BC) d(B, CA)d(C, AB)


, ,
d(P, BC) d(P, CA)d(P, AB)

are all equal. Find the common value of these ratios. Here d(X, Y Z) denotes the
perpendicular distance from a point X to the line Y Z.
Solution: Let AP, BP, CP when extended, meet the sides BC, CA, AB in D, E, F
respectively. Draw AK, P L perpendicular to BC with K, L on BC.(See Fig. 3.)

6
y

A A (h,k)

F E
P
P(u,v)

x
B (0,0) C (a,0)
B K L D C

Fig. 3. Fig. 4.

Now
d(A, BC) AK AD
= = .
d(P, BC) PL PD
Similarly,
d(B, CA) BE d(C, AB) CF
= and = .
d(P, CA) PE d(P, AB) PF
So, we obtain
AD BE CF AP BP CP
= = , and hence = = .
PD PE PF PD PE PF
AP BP
From = and ∠AP B = ∠DP E, it follows that triangles AP B and DP E are
PD PE
similar. So ∠ABP = ∠DEP and hence AB is parallel to DE.
Similarly, BC is parallel to EF and CA is parallel to DF . Using these we obtain
BD AE AF DC
= = = ,
DC EC FB BD
whence BD2 = CD2 or which is same as BD = CD. Thus D is the midpoint of BC.
Similarly E, F are the midpoints of CA and AB respectively.
We infer that AD, BE, CF are indeed the medians of the triangle ABC and hence P
is the centroid of the triangle. So
AD BE CF
= = = 3,
PD PE PF
and consequently each of the given ratios is also equal to 3.

Aliter

7
Let ABC, the given triangle be placed in the xy-plane so that B = (0, 0), C = (a, 0)
(on the x- axis). (See Fig. 4.)
Let A = (h, k) and P = (u, v). Clearly d(A, BC) = k and d(P, BC) = v, so that
d(A, BC) k
= .
d(P, BC) v
The equation to CA is kx − (h − a)y − ka = 0. So
,
d(B, CA) −ka (ku − (h − a)v − ka)
= p p
d(P, CA) 2
k + (h − a)2 k2 + (h − a)2
−ka
= .
ku − (h − a)v − ka
Again the equation to AB is kx − hy = 0. Therefore
,
d(C, AB) ka (ku − hv)
= √ √
d(P, AB) 2
h +k 2 h2 + k2
ka
= .
ku − hv
From the equality of these ratios, we get
k −ka ka
= = .
v ku − (h − a)v − ka ku − hv
The equality of the first and third ratios gives ku−(h+a)v = 0. Similarly the equality
of second and third ratios gives 2ku − (2h − a)v = ka. Solving for u and v, we get
h+a k
u= , v= .
3 3
k
Thus P is the centroid of the triangle and each of the ratios is equal to = 3.
v
6. Find all real numbers a for which the equation
x2 + (a − 2)x + 1 = 3|x|
has exactly three distinct real solutions in x.

Solution: If x ≥ 0, then the given equation assumes the form,


x2 + (a − 5)x + 1 = 0. · · ·(1)
If x < 0, then it takes the form
x2 + (a + 1)x + 1 = 0. · · ·(2)
For these two equations to have exactly three distinct real solutions we should have

8
(I) either (a − 5)2 > 4 and (a + 1)2 = 4;
(II) or (a − 5)2 = 4 and (a + 1)2 > 4.
Case (I) From (a+1)2 = 4, we have a = 1 or -3. But only a = 1 satisfies (a−5)2 > 4.

Thus a = 1. Also when a = 1,√equation (1) has solutions x = 2 + 3; and (2) has
solutions x = −1, −1. As 2 ± 3 > 0 and −1 < 0, we see that a = 1 is indeed a
solution.
Case (II) From (a − 5)2 = 4, we have a=3 or 7. Both these values of a satisfy the
inequality (a + 1)2 > 4.√When a = 3, equation (1)
√ has solutions x = 1, 1 and (2) has
the solutions x = −2 ± 3. As 1 > 0 and −2 ± 3 < 0, we see that a = 3 is in fact a
solution.
When a = 7, equation (1) has solutions x = −1, −1, which are negative contradicting
x ≥ 0.
Thus a = 1, a = 3 are the two desired values.

7. Consider the set X = 1, 2, 3, . . . , 9, 10 . Find two disjoint nonempty subsets A and
B of X such that
(a) A ∪ B = X;
(b) prod(A) is divisible by prod(B), where for any finite set of numbers C, prod(C)
denotes the product of all numbers in C ;
(c) the quotient prod(A)/prod(B) is as small as possible.

Solution: The prime factors of the numbers in set 1,2,3, . . . , 9,10 are 2,3,5,7. Also
only 7 ∈ X has the prime factor 7. Hence it cannot appear in B. For otherwise, 7 in
the denominator would not get canceled. Thus 7 ∈ A.
Hence
prod(A)/prod(B) ≥ 7.
The numbers having prime factor 3 are 3,6,9. So 3 and 6 should belong to one of A
and B, and 9 belongs to the other. We may take 3, 6 ∈ A, 9 ∈ B.
Also 5 divides 5 and 10. We take 5 ∈ A, 10 ∈ B. Finally we take 1, 2, 4 ∈ A, 8 ∈ B.
Thus  
A = 1, 2, 3, 4, 5, 6, 7 , B = 8, 9, 10 ,
so that
prod(A) 1·2·3·4·5·6·7
= = 7.
prod(B) 8 · 9 · 10
prod(A)
Thus 7 is the minimum value of . There are other possibilities for A and
prod(B) 
B:
 e.g., 1 may belong to either A or B. We may take A = 3, 5, 6, 7, 8 , B =
1, 2, 4, 9, 10 .

9

 
!#"%$!&('*),+.-0/1)2&43
56-768:9;&6-<9>=?)@/
=?82-ACB)@3
5D=?-?&E3
-0/FG9H&6+49>82)2&6-<IJ&6$H3K)2&L3
-M/
'*-?=03
)2&6N>=?)@/
=?82-OAJ1PQ/
$;R!-3
569;3K3
5(-0/
-):'
9S6&():TLS(-7$!)2&E3VUW$!&X3
5(-17-0/
7-?&Y+.)2=?S68:9;/V+L/9ZB&\[]/
$!^_F`3
$O3
56-K8:)2&(-<Iba6'*S(=M5X3
569;3[]$H/K9H&EcX7$H)2&L3Vd`$H&X3
56-182)2&6-
Ife
d1U`/
-?7./
-?'*-?&E3
'3
56-82-?&(NH3
5g$;[h3
56-K39;&6NH-?&E3[,/
$!^id`3
$<3
5(-1=?)@/
=?82-A
jlkmonqpZr]ksut

Y
O

Q y

X z
u
x
u

v -03wyx-K3
56-K[]$E$H3$H[z3
56-7-0/
7-0&Y+()2=?S(8:9;/[]/
$H^iF`3
$3
56-V8:)2&(-IfaY9;&Y+>{gx-K3
56-K8:-0&6NH3
5\$H[h3
5(-K39H&(N!-?&E3wK|}[]/
$!^
l R t P

w`3
$\=?)@/
=?82-A ~K&DF1wW39H!-9€7$!)2&E3U'*S6=5D3
569;3Uwƒ‚ƒ{zK„…-<'*5($ZB`3
5Y9†31U)2'K3
5(-+.-?'*)@/
-Z+47$!)2&E3Z1‡z$#3
5():'
-?&Y+aL39Hˆ-19;&Ec\7$H)2&L3Vd`$H&g82):&(-IQ9H&6+\82-03‰€x-3
56-82-?&(NH3
5g$;[h3
56-K39;&6NH-?&E3KdŠ‹[]/
$H^id`3
$#A
ΠS(/*3
5(-0/V8:-M3Ox-3
56-/9!+.):S('K$;[u3
5(-=0)2/
=08:-AŽ9H&Y+482-03K‰Xx-3
56-8:-0&6NH3
5D$;[ 3
5(-39;&6NH-?&E31dŠ[,/
$!^dW3
$>A K!$!)2&
F1d‘e
Ud v -03<Udi‚G’ze“F1d_‚i”.e
wdi‚•0 Œ /
$H^/
):NH5E3O9;&6N!82-Z+–3*/
),9;&6NH8:-0'Od1FŠ%e
F|gdeMF1wd‘e
d1UwB%-€5Y9ZRˆ-
/
-?'*7-?=03
)2Rˆ-082c—”E˜€‚i!˜™‰(˜;e
F1w1˜>‚Gˆ˜™{˜He
”E˜€‚iF1w1˜™š•
˜€‚i!˜™{˜™š•
˜;e
’˜€‚G{˜™š•
˜;œ›L$gB‘-€$!x.39H)2&
‰ ˜ ‚`” ˜‘  ˜ ‚ƒ ˜ ™{ ˜ ™Ž• ˜  ˜ ‚ƒ{ ˜ ™C• ˜ ‚y’ ˜ KžV-?&6=0-‰>‚Ÿ’zV‡‘56)2'KNH):R!-?'dŠ‚`d| B56)2=5–)2'B5Y9;3VB‘-
&6-?-?+(-Z+€3
$O'*56$ZB1
¡LOPh$!'*)@3
)2Rˆ-1)2&E3
-?N!-0/
'V9;/
-1B/
)@3*3
-?&g$H&49H828l3
56-[¢9;=?-?'$H[ 9=0S6x-!a($!&(-1$!&g-Z9;=M5l‘£‘3-Z9;=M54=0$H/
&6-M/<¤¢R!-0/*3
-0¥6¦%$H[Q3
5(-=?S6x-Ha
3
56-#7./
$L+(S6=M3$;[3
56-€&LS6^x-M/
'1$!&—3
56-#[o9H=?-?'3
5Y9†3^#-?-M39†3<3
5(-#=?$H/
&(-0/O)2'B/
)@3*3
-?&q–‡‘5(->'*S(^§$H[3
56-€&LS6^<x-0/
'
B/
)@3*3
-?&#9;3u9;8:8.3
56-=?$;/
&6-0/
'J)2'%¡H¨!¨;©(Jªb[Y«+.-?&6$;3
-?'Q3
56-‘'*S(^W$H[3
5(-‘&LS6^x-M/
'Q$!&#9;8:8L3
56-‘[o9H=?-0'?aE¬Y&Y+<9H828L3
56-7$!'*'*)2x(8:-
R!9;82S6-?'$;[Q«
jlkmonqpZr]ksut
v -03>­V®¯°#d1Uw1±ƒx-X9…=?S(x-!a9;&Y+²3
5(-X&LS6^<x-0/
'O³e“´;e
µHe*¶Ye*·Ee
¸šx-XB/
)@3*3
-?&š$H&3
56-\[¢9H=0-?'>­K®<¯°Xa®<Uw1¯<a
d1Uw1±%a­Kd±Q°Xal­K®Uda¯w1±Q°}/
-0'*7-?=03
)2Rˆ-?8@cE1‡‘5(-?&43
56-7(/
$L+.S6=03
'B/
)23*3
-0&œ9†3K3
5(-<=?$;/
&6-0/
'­al®#aq¯<al°Xalda
UOa6wa±C9;/
-1/
-?'*7-0=03
)2Rˆ-?8@c>³.¶L·ˆa(³L´?·ˆa.³.´?¸qa6³L¶¹¸qa6µM¶L·ˆa(´Mµ?·ˆa(´0µZ¸qa6µ0¶¹¸qu‡‘56-1'*S6^$H[Q3
56-0'*-1º&LS6^<x-0/
'‘)2' t
S R

D C

P Q

A B

»
‚ ¤¼·u™¸¦0¤¢³L´ ™C´0µ ™Cµ0¶K™Ž³L¶.¦Q‚`¤¼·‘™²¸¦0¤¢³1™—µZ¦0¤¢´ ™C¶.¦M½
‡‘56)2'1)2'N!)2Rˆ-?&–3
$Xx-O-ZTES69H8Q3
$D¡H¨H¨H©g‚¡ ˜V¾!¿¾ ?ÀˆÁL\~Kx('*-0/
Rˆ-3
569;3&($!&6-$H[%3
5(-[¢9;=03
$H/
'³™šµHaz´™š¶Yaz·™¸—):'
-ZTESY9;8l3
$gH‡‘5LS6'¤¢³1™²µ?¦0¤¢´ ™²¶.¦0¤¼·%™¸¦)2'V-ZTES69H8q3
$#© ¾†¿¾ ?ÀˆÁEaq¡ ¾ À ¾ ZÀˆÁEaq¡ ¾†¿¾†¿!¿ ©>$;/1¡ ¾ ¡ ¾H ¨.!‘žK-?&(=?-13
56-
7$!'*'*)2x682-RH9H82S6-?'$;[Q«Ã‚³™C´ ™Cµu™—¶™Ž·‘™²¸49;/
-1©V™ ¿ ™ZÀ!Á1‚WZÁ;©.a¡™ŽÀV™š?ÀˆÁ‚ƒ†Á  a¡™ ¿ ™ ¿H¿ ©<‚ ¿!¿!Ä a
$H/K¡™²¡™  ¨.K‚  ¨ Â
‡‘5LS6'‘3
56-M/
-9;/
-1©O7$H'*'*):x(82-RH9H82S(-?'$H[Q«y9H&6+>3
5(-0c\9;/
-O†Á†©(a2†Á  a ¿!¿HÄ a  ¨ Â
¿ v -03ÅÆ9;Ë &Y+DÇË x-3
5(-O/
$E$H3
'$H[3
5(-#TESY9!+L/9;3
)2=#-ZTESY9†3
)2$!&…’ ˜ ™šÈX’  #‚G¨.aqB56-0/
-#ÈÉ):'<9H&œ$L+6+…)2&E3
-?NH-0/Z v -03
ÊYË ‚Å ™—Ç aL[o$H/V̜ͨL%PQ/
$†Rˆ-K3
569;3[o$;/V̖ͨ.a
¤¼9ˆ¦ ÊYË )2'V9H&\):&E3
-?NH-0/ZÎ9;&Y+
¤¢xY¦…N!=Z+–¤ ÊYË e ÊYË!ÏQÐ ¦ ‚ƒH
jlkmonqpZr]ksut ›.)2&6=0-gÅÑ9H&Y+ŽÇW9†/
-g3
56-g/
$E$H3
'>$H[K3
56-4-ZTESY9†3
)2$!&’ ˜ ™ŸÈ\’  —‚ɨLaB‘-4569†R!-gÅ ˜ ™yÈ\Å  C‚
¨.e“Ë Ç ˜ ™yÈ>ËLÇ ÔzÐ  ŽË.‚ÒÔ ¨.½ÓXS(823
)27(82c.)2&6NŽxEcÅ Ë.Ô ˜ e“Ç ËLÔ ˜ /
-?'*7-?=03
)2Rˆ-?8@cB‘-D5Y9ZRˆ-DÅ Ë ™yÈXÅ ËLÔzÐ  Å Ë.Ô ˜ ‚Ò¨Ž9H&6+
Ç ™ŽÈ>Ç  Ç ˜ ‚èL
£K+(+()2&6N€B‘-#$Hx(39H)2&œÅ Ë ™Ç Ë ‚  ȅ¤¢Å ËLÔzÐ ™Ç Ë.ÔqÐ ¦Q™ƒ¤¢Å Ë.Ô ˜™Ç ËLÔ ˜Z¦M€‡‘5()2'NH)2Rˆ-?'9\/
-?=0S(/*/
-?&(=?-/
-?8:9;3
)2$H&…[o$;/
̖͚¡ t
ÊYË ‚  ÊYËLÔzÐ ™ ÊYËLÔ ˜ e*ÌDÕ¡ ¤]Ö.¦
¤¼9ˆ¦—×K$ZB ÊYØ ‚W™1‚y¡O9H&6+ ÊqÐ ‚ÆÅ4™—Ç—‚  Ȝ‘‡‘5.S(' ÊYØ 9;&Y+ ÊqÐ 9;/
-)2&E3
-?N!-0/
'?Ù cg)2&Y+.S6=03
)2$!&laY)@3[o$!8282$ZB'
[]/
$H^ ¤]Ö.¦%3
5Y9†3 ÊYË )2'9H&g)2&E3
-?N!-M/[]$H/V-Z9H=5X̖ͨ.
¤¢xY¦œ„—-9HN!9H)2&–S6'*-€¤]Ö.¦3
$>7./
$†Rˆ-xLc\)2&Y+.S6=03
)2$!&43
569;3VN!=Z+C¤ Ê Ë e Ê ËˆÏhÐ ¦‚ÑH‡‘5():'V)2'K=?82-Z9†/
82c43*/
S(-1[o$H/̅‚y¨.al9H'
N!=Z+–¤b¡Le  Èg¦Q‚`HaYxEc#3
56-1NH)2Rˆ-?&X=0$!&Y+.)@3
):$H&X3
5Y9†3ÈÚ)2'$L+6+l
v -03VN!=?+—¤ ÊYË.Ô ˜ e ÊYË.ÔzÐ ‚Ñ!e“Ì—ÍŸ¡Lªb[u)@3B‘-0/
-3
$#5Y9;767-?&g3
569;3N!=?+Ž¤ ÊYËLÔzÐ e ÊYË ¦ۃ!aY39;ˆ-9€7./
)2^#-ÜD3
569;3
+()2RL):+(-?'x$;3
5 Ê ËLÔzÐ 9H&Y+ Ê Ë ‡‘5(-?&œ[,/
$!^Ò¤]ÖL¦MaB‘-ONH-033
5Y9†3ܗ+.)2R.):+.-?' Ê Ë.Ô 9H82'*$.O‡‘5LS6'%܅)2'9€[o9H=03
$;/$H[
N!=Z+Ž¤ Ê Ë.Ô ˜ e Ê ËLÔzÐ ¦Maq9>=0$!&E3*/9!+.):=M3
):$H&q›.$€NH=Z+²¤ Ê ËLÔzÐ e Ê Ë ¦‚GHžK-?&(=?-B%-56˜ 9†R!-N!=?+²¤ Ê Ë e Ê ËˆÏhÐ ¦‚!a[o$;/
9H828l̜ͨ.

©(OPJ/
$†Rˆ-u3
5Y9;3Q3
5(-‘&LS6^<x-0/Q$H[63*/
):7(Ë 82-?'¤¼­Ëe
®#e¯1Ë ¦qB56-M/
-­e
®#e¯9;/
-'*S(x6'*-03
'J$;[qÝEHeM¡Le ¾?¾?¾ e*ÌzÞ'*S(=M53
5Y9;3 ­€ß®gßV¯y‚
à e
­Žß>®_‚ á à e®ßX¯‚Ãá ¨)2'Á  ¡E½ À ™ Â
jlkmonqpZr]ksut
A
B
2 3
1

5
4

6 C
7
X

v -03>|â‚ÒÝE!eM¡Ee ¿ e ¾0¾?¾ e“Ìhވ`„—-4S('*-4ãu-0&6&+.):9HNH/9;^ä[]$H/\'*-03


'\­e
®#e¯Ú3
$C'*$!82Rˆ-43
56-47./
$!x(8:-0^g`‡‘5(-D/
-0N!)2$!&('
$H3
5(-0/K3
569H&–­šßg®Ÿß4¯i¤uB56)2=5œ)2'K3
$>x--?^#7(3fcY¦9†/
-&LS6^<x-0/
-Z+…!a ¡Ea ¿ a ©(a  a À.a Á49H'K'*56$ZB&œ)2&43
56-¬YNHS(/
-!Îl-! N.:a 
=?$H/*/
-0'*7$!&Y+.'3
$g­å.¤¼®y斯1¦  ­šß–®ç斯Kç;aJ¡\=?$H/*/
-0'*7$!&Y+.'3
$g­Ãߖ®<å!¯i‚G­šß–®ƒßœ¯Kç†eJÁ\=0$H/*/
-?'*7$H&Y+('3
$
|Då.¤¼­Žæ>®ÃæX¯1¦ ‚íVçhßX®çhßg¯Kç†a6'*)2&6=0-­Žß\®Ãß\¯Ÿ‚ à
è
Œ Ë )@/
'“3
82c43
5(-&LS6^<x-0/V$H[%B‘9?c('K$H[9;'*'*)2N!&6)2&(Ng-?82-?^#-?&E3
'$;[%|é3
$€3
56-<&LS6^x-M/
'/
-?NH)2$!&6'B)@3
5($!S(39;&Ec4=?$!&6+()@3
)2$!&œ):'
Á Ž£V^#$H&6Ng3
5(-?'*->3
5(-0/
-\9;/
-X=?9H'*-?'#)2&CB56)2=5š¡4$H/  $;/Ox$;3
59†/
-X-?^#7.3¹cEœ‡‘56->&LS6^<x-0/$;[+.)2'“3*/
):x(S(3
)2$!&(')2&
B56)2=5C¡>)2'1-?^#7.3¹c4)2'À Ë v ):!-0B)2'*-O3
56-&LS(^x-0/K$H[+.):'“3*/
)2x(S(3
)2$!&('1):&–B56)2=5  )2'-?^#7.3¹cg)2'9H82'*$4À Ë OÙ%S(313
5(-?&
B‘-O569ZRˆ-<'*S(x(3*/9;=03
-Z+–3fB)2=?-3
56-<&LS6^x-M/$H[‘+()2'“3*/
)2x6S.3
):$H&6'1)2&DB56)2=M5œx$H3
5D3
56-/
-?N!)2$!&('<¡\9H&6+ Â 9;/
-O-0^#7(3¹cE›.Ë $
3
$O=?$H^#7-?&6'
9†3
-B%-5Y9ZRˆ-3
$€9!+(+\3
56-1&LS(^x-0/$;[ +()2'“3*/
Ë )2x6S(3
)2Ë$H&6'):&gË B5()2Ë=M54x$;Ë3
5D¡O9H&6Ë + Â 9;Ë /
--?^#7.3¹cE%‡‘56)2'V)2' Â
žK-?&(=?-K3
5(-1+(-?'*)@/
-Z+\&.S(^x-0/‘$H[h3*/
)27682-?'¤¼­e
®€e
¯1¦)2&gÁ  À  À ™  ‚ÆÁ  ¡E½ À ™ Â
 v -03 ­V®¯°`x-‘9KTESY9H+./
)28:9;3
-0/9;8¼Îˆ’#9H&6+ŠŸx-%3
56-%^#),+.7$!)2&E3
'h$H[­¯š9;&Y+®°ƒ/
-?'*7-?=M3
):R!-?8@cO9;&Y+13
56-‘82)2&6-?'Q3
5./
$!S6NH5
|_9H&Y+OŠy/
-?'*7-?=03
)2Rˆ-?8@c7Y9;/9;828:-08Y3
$<®°Xe
­K¯Æ^#-?-M3%)2&€F v -M3‘d‘eUOe*w<e
±–x-3
56-^#):+(7$!)2&E3
'J$H[q­K®#e
®<¯Ke¯°Xe
°#­
/
-?'*7-?=03
)2Rˆ-082cE PQ/
$†Rˆ-3
5Y9†3
¤¼9ˆ¦CTESY9!+L/
)28,9†3
-0/9H82'V­Kd1F±C9H&6+g­Kd|4±C569ZRˆ-K3
56-K'
9H^#-9;/
-?9.Î
¤¢xY¦œ3
56-19†/
-Z9H'$H[z3
56-TESY9H+./
)28:9;3
-0/9;8z­Vd1F±Qe
®UF1d‘e¯wF1UOe
°€±QF1w‹9;/
-9H828q-ZTESY9H8¢
jlkmonqpZr]ksut
„—-1S('*-3
5(-1[¢9H=M3
' t ¤¢),¦3
5(-82)2&6-ê¹$!)2&6)2&6N#3
5(-^#):+.7$!)2&E3
'$;[J3
56-1'*):+(-?'V$H[u93*/
):9H&(N!82-<)2'V7Y9;/9;828:-08z3
$O3
56-13
56)@/+g'*):+(-HÎ
¤¢)2)]¦9;&Lc\^#-Z+.),9;&X$;[ 93*/
),9;&6NH8:-x():'*-0=03
'V)@3
'9;/
-?9.ÎQ¤¢)2)2),¦3fB‘$O3*/
):9H&(N!82-?'5Y9ZRL):&(NO-ZTESY9;8qx69H'*-?'V9H&6+Xx$!S(&Y+.-Z+\xEc>'
9H^#-
7Y9†/9H8282-?8z82)2&6-?'569†R!--ZTESY9;8z9†/
-Z9.
C

D
X
Q

O
S
Y

A P B

¼¤ 9ˆ¦—×K$ZBš®<°‹)2'%7Y9†/9H8282-?8Y3
$d±œ9;'uB%-?8289H'%F|…u›.$F|_)2'%7Y9†/9H8282-?863
$d±%QžK-0&6=?-1ë d|D±zìl‚`ë d1F±zìb £V+6+.):&(N
ë d1­±zì3
$Ox$H3
5\'*),+.-?'B‘-1N!-M3<ë ­Vd|D±zì‚`ë ­Kd1F±zì¼%‡‘5()2'7./
$†Rˆ-?'7Y9†/*3¤¼9ˆ¦M
¤¢xY¦…×K$ZB
ë ­Kd|D±zìí‚ ë  ­Kd|XìE™Æë ­ ±h|Xì
‚ ¡ ë ­K®|Xì.™ ¡ ë ­V°O|Xì‚ © ë ­K®<¯VìL™ © ë ­V°€¯Vì
‚ © ë ­K®<¯°ì¼½
žK-?&(=?->xLc¤¼9ˆ¦Maë ­Vd1F±hÐ ì‚ îÐ ë ­K®<¯°ì¼›L)2^#):8:9†/
82cŽxEcC'“c.^#^#-03*/*cœ-Z9;=M5²$H[V3
56-g9†/
-Z9H'4ë ­VUF1dKì¼eZë ¯wF1UìV9H&6+
ë °€±JF1wì)2'-ZTESY9;8u3
$ î ë ­K®<¯°켅‡‘5LS('3
5(-€[]$!S./ON!)2Rˆ-0&Ž9;/
-Z9;'O9;/
->-ZTESY9;8¢–‡‘56)2'7./
$;R!-?'<769;/*3\¤¢x¦MŽë ×K$;3
- t ëqì
+(-?&($H3
-?'9†/
-Z9;ì¼
À. v -03Oï@Ü Ð e¢Ü ˜ e¢ÜðHe ¾0¾Z¾ e¢Ü Ë e ¾0¾?¾ñ x-9>'*-ZTES(-?&6=?-<$H[%7(/
)2^#-?'1+.-0¬Y&(-Z+4xLcOÜ Ð ‚¡€9H&6+D[]$H/ÌÍÑ!eoÜ ËˆÏhÐ )2'3
56-<8:9;/
N!-0'“3
7(/
)2^#-V[o9H=03
$;/K$;[Ü Ð Ü ˜ ¾?¾?¾ Ü Ë ™!½q¤¼‡‘5.S('JÜ ˜ ‚ ¿ e¼Üð‚ÆÁ!¦MuPQ/
$†Rˆ-3
5Y9†3JÜ Ë ‚ á  [o$;/V9H&Ec>ÌJ
jlkmonqpZr]ksut ÙucÃ+(9;39gÜ Ð ‚ò¡Ee¢Ü ‚ ¿ e¢Ü ð ‚éÁL‹ªf3€[]$!8282$†B'XxLc)2&Y+.S6=03
)2$!&3
5Y9;3<Ü Ë e*ÌóÍô¡—)2'\$L+(+lGë Œ $;/g)@[
Ü ˜ e¢ÜYðHe ¾?¾?¾ e¢Ü ËLÔzÐ 9†/
-$L+6+a.3
5(-?&Ü ˜ Ð Ü ˜ ¾Z¾?¾ Ü ËLÔzÐ ™K):'9;82'*$#$L+6+>9H&Y+>&6$;/ ¿ ‡‘56)2'9H82'*$[]$!8282$ZB'VxEc€)2&Y+.S6=03
)2$!&l Π$H/
)@[ÜðV‚yÁ9;&Y+X)@[Üð;e¢Ü e ¾0¾?¾ Ü Ë.ÔqÐ 9;/
-&6-0)23
5(-0/K¡<&6$;/ ¿ a63
5(-?&OÜ Ð Ü ˜ Üð ¾?¾?¾ Ü Ë.ÔzÐ ™9;/
-1&6-0)23
5(-0/xEcg¡&($H/xEc ¿ ‘›.$
Ü Ë ):'&(-?)@3
56-0/K¡<&($H/ ¿ î
õ
ÁL v -03’g9H&Y+\‰>x-K7$!'*)@3
)2Rˆ-/
-Z9H8q&LS6^x-M/
'‘'*S6=5\3
569;3‰ ð ™C‰#Õ²’  ’ ð PQ/
$†Rˆ-3
5Y9†3
¤¼9ˆ¦—‰#ö’göÆHÎ9H&6+
¤¢xY¦…’˜J™Ž‰(˜VöÆ!
jlkmonqpZr]ksut
¤¼9ˆ¦Ž›.)2&6=0-’>9;&Y+O‰O9†/
-V7$H'*)@3
):R!-!aEB‘-5Y9ZRˆ-‰€Õ’  ’ ð  ‰ ð ö²’zJ£82'*$<’  ’ ð Ͳ‰u™g‰ ð Û¨L ›.$’h¤“  ’ ˜ ¦Û¨L
žK-?&(=?-’göÆ!u‡‘5LS6'‰€ö²’göÆ!a(7(/
$†RL):&(NO7Y9†/*3¤¼9ˆ¦M
¤¢xY¦—£VN!9H)2&g’ ð ™C‰ ð Õ²’  ‰u›.$
’ ˜  ’Y‰K™Ž‰ ˜ Õ ’’<™C ‰‰ ½
‡‘5Y9†3V)2'
’ ˜ ™Ž‰ ˜ Õ ’’<™Ž ‰‰ ™C’Y‰<‚ ’  ‰™C’<™Ž’Y‰‰ ¤¢’™—‰(¦ ½
žK-0/
-K’Y‰¤¢’™C‰(¦%öÆ ¾ ‰ ¾ ¤“u™ÃZ¦u‚Æ¡†‰u›L$O’ ˜ ™C‰ ˜ ö ’  ’‰™Ž™²‰ ¡;‰ ‚ ’’™C™C‰‰ ‚ƒ!u‡‘56)2'7(/
$†Rˆ-?'1¤¢xY¦M

÷

 
 !"$#&%')(**+
,.-/ 021346587:9;0=<?>2@BADC.02EGFIHJ<BKLNMPOP<)1N0QLR<BOTSVUXWZYXWZ[6WJ\]9;081N^0`_XMaKVb;@BMPA.1dce@)f34GWZ4g5gWZ587!WJ7h3iLN0kj
cNb;0Q>l1RMmC.0kOPn6cNHJ>d^61N^J<)11NLRMP<BAoBOP0Qc3Y`[p<)AZKX5`\Uq<BLN0r0QFIHMPOP<)1N0QLR<BOT-tsLR@uCB0v1N^J<)1346587Mac<8LR^@B_69HJcQ-
w 021R0kLR_XMmA0`Mm1Rc<)Ao.Om0xck-
y z;{}|~Q€}z;v‚ƒ 0!^J<„CB0!Y`[†…‡4ˆ7Š‰B‹Œ…U=\-ŽDMPAJ>k03Y`[<BAJK$5`\U‘<BLN0’9;@)1R^“0xFIHMmOa<u1R0kLd<)O<BAJK
Y=[”…$U=\eW.1R^0knX_6HJc
19;0>k@BAo.LNH0QAI11NLRMP<BAoBOP0QcQ-–•^MPcvMP_gbJOmMP0Qcv1N^Z<u1e3Y—…˜Y`[i…”[83™…˜5`\Œ…˜\U
…—U=5g-šOacN@h›58œˆ˜…ŽžBŸI …q›r[`Y`3ˆ-¡¢D0k0=£tMPoJ-e,.- ¤
D R C

V F
L
W Q
S
M O
U

A P B

Fig. 1.
¥ 0kAJ>k05`\¦MPcGbJ<)Ld<)OPOP0kO1N@ŒY=3=-˜§@u¨©5`\ª…«Y`3:MP_XbOPMm0xc?1N^Z<u1Š5`\Y`3MPch<bZ<)Ld<)OPOm0QOm@.oBLd<)_’-$¬­A
bZ<)LN1NMa>2HOa<)L=\3Mac8bJ<)Ld<)OPOP0kOt1N@5`Yª<)AJK®\3…¯5`YX-?•^Mac`c^@u¨c1R^J<u1ˆ37‡Mac8bJ<BLR<BOmOP0kOt1N@!4g5°<BAJK
37°…$4g5g- ¥ 0QAJ>20`34g587:MPce<gbJ<BLR<BOmOP0kOP@Bo.LR<B_’-
/ 0211R^0KVMa<)o.@BAJ<BO35”<)AZK?4ˆ7±9Mac0x>l10x<B>d^X@)1R^0kL<)1v²±-•^0kA?7G²³…$7G4g‰)‹`…qY`[q…q5`\®…p3[6-
•^DHJc=MmA1RLNMa<)Ao.Om0?37G5gW 1R^0h_X0QKMP<BAJc=3[gWt7G²±W–5`\”<BLN0?<)OPO0xF.HZ<)O¢-G•^DHJc=37Š5°Mac`0xF.HJMmOa<u1R0kLd<)O¢-
•^JMPceMP_XbOPMm0xc34g587ªMac<gLR^@B_69HJcQ-–´@BLR0k@uCB0QL1R^0=<)AJoBOP0Qc<)LR0`žBŸ. 8<BAJK,x‹)ŸI u-
‹-r¬µf·¶¸¹?<)LR0eMPA.1R0ko.0kLdckWI<)AJK’,uº8KVMPCIMaKV0xc9;@)1N^h1N^002EVbLR0QcRcMP@BAJc–¶;»–¼½‹)¶J¹v¾’¹V»t¼½¿)¶¾Àº„¹g<)AJKX¶;»–¼ŠÁB¶J¹v¾
‹)¹V»¾®¶h¼Â¹ÃWV1N^J0kA’bLR@uCB01N^J<)18,uº6KVMmCDMaKV0Qc¶J¹ˆ¼“,x‹)¶6¾$,„¿u¹Ã-
y z;{}|~Q€}z;v‚Ä 9JcN0kLRCB01N^J<)1¶;»¼ÀÁ)¶Z¹¾Å‹)¹D»¾Â¶X¼¹?…i¡}¶g¼¹¤2¡Æ¶X¼‹)¹¾Ç,„¤l-•^DHJc,„ºˆKVMmCDMaKV0Qc0QMÈ1R^0kL
¶G¼¹Š@BL¶h¼®‹u¹8¾p,B-DHJbbZ@Ic081R^J<u1=,uº6KVMmCDMaKV0Qc¶G¼¹Ã-¬­AŠ1N^Mac>k<.c08¶½É$¹Ž¡Æ_X@VKÂ,„ºB¤v<BAJKŠ^0kAJ>k0
¶ » ¼‹)¶J¹`¾®¹ » ¼®¿u¶g¾“º„¹gÉp¹ » ¼‹)¹ » ¾®¹ » ¼®¿u¹8¾“º„¹XɎ‹)¹ ¡Æ_X@VKÂ,„º)¤2Ê
•^DHJcr1R^0`oBMPCB0QAŠ>k@BAJKVMm1NMP@BAŠ1N^Z<u1=,„ºˆKVMPCDMPKV0xce¶Z»¼®‹u¶Z¹¾¹V»¼®¿u¶ˆ¾“º„¹?MP_gbJOmMP0Qc1R^J<u1`,uºˆ<BOPcN@XKVMmCDMaKV0Qc
‹)¹Š<BAJK’^0kAZ>20`¹½MÈ1dc0QOÈf
-ËrHV11N^J0kA!¶Ép¹q¡Æ_X@VK,uº)¤rMP_XbOmMP0Qc1N^J<)1=,„ºgKVMPCDMPK0Qc¶<)Oac@Z- ¥ 0QAJ>20ˆMmA’1R^MPc
>Q<BcN0X,„º6KVMPCIMaKV0xc¶J¹g¼],„‹u¶6¾p,x¿u¹Ã-
VHbb;@.cN0`@BAŠ1N^0`@B1N^0QLe^J<)AJKŠ1R^J<u1=,uº=KVMPCDMPK0Qce¶h¼®‹u¹8¾$,.-•^IHZc¶½Ép‹)¹ˆ¼“,¡}_g@VK,uº)¤r<)AJKŠ^0QAJ>20
¶ » ¼®‹u¶Z¹8¾Å¹ » ¼¿)¶g¾]ºu¹gÉǹ » ¼®¿u¹`¾Åž ¡}_g@VKÂ,„ºB¤lÊ
•^DHJc=,„ºˆKVMmCDMaKV0Qc¹D»¼®¿u¹`¾ÅžJ-ËvH1¶½Ép‹)¹ˆ¼“,¡}_g@VK,uº)¤r<)Oac@gMP_gbJOmMP0Qc1R^J<u1
¶J¹g¼],„‹u¶6¾p,x¿u¹XÉp‹·ÌT¹ » ¼¿)¹8¾Œž.ÍΡ}_g@VKÂ,„ºB¤lÊ
VMmAJ>k0?,„º6KVMPCDMPKV0xc¹V»e¼¿)¹¾ŒžWMm1efÆ@BOPOm@u¨cv1R^J<u1=,uºˆKMmCDMaKV0Qc¶Z¹6¼“,x‹)¶6¾$,„¿u¹Ã-
ÁJ-r¬µfrÏZ¸dЄ¸Nш<)LR0=1N^JLN0Q0ˆLR0Q<)OtADH_ˆ9;0kLdccHJ>d^1N^J<)1?Ò ÏX¼ŒÐ)Ò;Ó¯Ò ÑBÒÈWÒ Ð¼ÅÑBÒÃÓÒ ÏÔÒÈWÒ Ñe¼ÅÏÔÒÃÓ¯Ò Ð)ÒÕW;1R^0kAbLN@uC.0
1R^J<u1@.A08@)fÏ;¸RЄ¸RÑMacr1N^0=cNH_ª@)ft1N^0`@B1N^0QL1
¨v@Z-
y z;{}|~Q€}z;v‚tÖ cMPAogÒ Ï·¼`Ð)Ò.Ó™Ò ÑBÒÈW„¨v0v@B9V1d<)MPAh¡}Ï ¼8Ð2¤»ӓÑ2»¨e^MP>d^ˆMPc0QFIHMPCu<)OP0kAI1·1N@g¡}Ï ¼8Ð.¼8Ñk¤2¡TÏ·¼`Є¾ˆÑk¤Ó
ŸJ-tDMP_XMmOa<)LROPnBW¡TÐV¼=ÑV¼=ÏV¤2¡TÐV¼=Ñ.¾gÏV¤tӓŸ<)AJKh¡TÑV¼=Ït¼ˆÐ2¤2¡TÑV¼=Ï·¾XÐ2¤tÓǟ-´HOÈ1RMmbJOmnDMPAo1R^0QcN0rMmA0xFIHJ<)OPMÈ1RMm0xckW
¨r0`oB0k1
¼=¡}Ͼ“Ðv¼Ñk¤ » ¡TоÅѼÏV¤ » ¡}ѾŒÏ=¼®Ð2¤ » ӓŸÊ
•^JMPc8fÆ@.LR>k0Qc1N^Z<u1 {}×tØ MPc80xFIHJ<)O–1R@½ÙQ0kLR@J- ¥ 0QAJ>20?Mm18fÆ@BOPOm@u¨c1R^J<u1`0kMm1N^J0kL=Ï6¾$Ðg…±Ñˆ@.LˆÐv¾ÇÑX…±Ï@.L
Ñ…ŽÏg…ŽÐ„-
Ú -£tMPAJK’1R^0ˆADH_69Z0QL@Bf<BOmO¿„j­KVMPoBMm1ADH_ˆ9;0kLdcˆ¡}MmAÀ9J<BcN0h,QŸI¤0Q<.>d^!@)f¨e^Ma>d^À>2@.AI1R<)MPAJc1R^0ˆ9JOm@V>dÛÂ,x¿?<BAJK
MacKVMPCDMPcNMP9Om089DnÀ,„¿V-e¡}£J@BLe0kEV<B_XbOm0.WÁ,„¿ Ú ¿WIÁ Ú ,„¿B¿6<)LR01
¨r@?cHJ>d^½ADH_ˆ9;0kLdcQ- ¤
y z;{}|~Q€}z;v‚ šADnŠcNHJ>d^’AIHJ_ˆ9;0kLcN^@BHJOPKŠ9Z0ˆ9Z@B1N^KVMPCDMPcNMm9JOm0`9Dn½¿X<BAJK’Á-•^J0`OP<.c
1KVMPoBMm1@)f<XADH_ˆ9;0kL
KMmCDMacMP9OP0?9In¿½_ˆHJc1=9;0?0kMm1N^0QL6¿½@BL=ŸJ- ¥ 0kAZ>20G<BAIncHJ>d^ADH_69Z0QL8f}<)OPOPc`MPAI1N@!@.A0?@)fv1N^0?fÆ@.OmOP@u¨eMPAo
cN0kC.0kA’>k<)1N0ko.@BLRMm0xckÜ
¡}Ma¤ÏDÐ2Ñu,x¿S ¡ÆMPMݤÏDÐu,x¿)ŸJS ¡}MmMPMa¤ÏVЄ,„¿B¿S ¡ÆMPC¤Ï;,x¿BÐ2ŸS ¡}C¤ÏZ,„¿BÐ2¿SÔ¡ÆCDMa¤,„¿)ÏVÐlŸJS ¡ÆCDMPMa¤,„¿)ÏVÐ2¿-
¥ 0kLR0=Ï;¸RЄ¸RÑ<)LR0`KVMPoBMm1RcQ-/·021HZce>2@BHJA.1e^@u¨q_?<)ADnhADH_ˆ9;0kLdc@)f–0Q<.>d^½>Q<u1R0koB@.LNn?<BLN01N^J0kLR0B-
¡}Ma¤g¬­A]1N^JMPc?>Q<BcN0’Ïi…‡ Þ ŸWv<BAJK®1R^0Áuj­KVMPoBMm1gADH_69Z0QLhÏVÐlÑhMac?KVMPCIMacNMm9OP0’9DnŒÁWr<)AJK]^0kAZ>20’@BA0½@)f1N^0
ADH_69Z0QLRcMPA½1N^0=cN021`ßI,QŸI‹V¸k,xŸ.¿¸kÊkÊQÊJ¸RàBàBàá.-–•^MacoBMPCB0QceÁ.ŸBŸ6ADH_ˆ9;0kLdck-
¡}MmMݤršoI<)MPAŠ<6AIHJ_ˆ9;0kL@)f·1R^0fÆ@BLR_âÏVЄ,x¿BŸ`MPcKVMPCDMPcNMP9Om09Dn!,x¿ˆMÈft<)AJKG@BAOPn?MÈf1N^0=‹ujµKVMPoBMm1vADH_69Z0QLeÏDÐMPc
KMmCDMacMP9OP0`9InŠÁ- ¥ 0QAJ>208Mm1_ˆHJc19;08fÆLN@._¯1N^0=cN021`ßI,x‹¸k,x¿¸kÊQÊkÊ;¸NàBàá.-–•^0kLR0`<)LR0`ÁBŸXcHZ>d^ŠADH_69Z0QLRcQ-
¡}MmMPMݤešcMPA®¡ÆMPMa¤2WJ^0QLN0`<)LR08<)o.<BMmA’ÁBŸgADH_69Z0QLRcQ-
¡}MmC¤eDMP_XMmOa<)L1N@!¡ÆMPMa¤2SZÁ.ŸˆADH_69Z0QLRcQ-
¡}C¤eDMm_XMPOP<BLe1N@!¡}MmMݤlWJÁ.ŸgAIHJ_ˆ9;0kLdck-
¡}CDMa¤ ƒ 0ˆ>Q<)A’9Z0QoBMPA½1N^0ˆ<BAJ<)OPnVcMac@)ft1N^J0=ADH_ˆ9;0kL@Bf1R^0`fÆ@.LN_‡,x¿)ÏVÐlŸg<BcMmAÅ¡}MmMݤl- ¥ 0QLN0=<Bo.<)MPA’ÏDÐ<.ce<
‹ujµKMmo.MÈ1AIHJ_ˆ9;0kL8_ˆHJc189Z0XKVMPCIMacNMm9OP0g9In!ÁJW;9JHV18Ï!…™ŸŠMac<BOPcN@Gb;0kLR_XMPcRcMP9OP0B- ¥ 0QAJ>20gMm1_6HJc
189;0ˆfÆLR@B_
1R^0=cN021`ßxŸ.Ÿ¸RŸBÁ¸RŸBžJ¸kÊkÊQÊJ¸RàBàVáI-t•^J0kLR08<BLN0`Á Ú cNHJ>d^’ADH_ˆ9;0kLdck-
¡}CDMmMݤ ¥ 0QLN0=<Bo.<)MPAh1R^0kLR0=<)LR08ÁBÁgADH_ˆ9;0kLdckSÏVÐ_ˆHJc19;08fÆLN@._¯1N^0=cN021`߄Ÿ,.¸NŸ Ú ¸NŸDºD¸kÊQÊkÊZ¸NàIºDá.-
šKKVMPAoh<)OPOÃ1N^J0QcN0`¨v08oB0k1ÁBŸ.Ÿe¾ŒÁBŸ¾ŒÁBŸ¾ŒÁBŸ¾ŒÁBŸ¾ŒÁ Ú ¾ÅÁ.Á=… ÚIã º=AIHJ_ˆ9;0kLdck-
¥ @u¨v0QCB0QL`1N^Mac6MPcˆA@)161N^0½>2@BLRLR0Q>l1`äZoBHLR0G<Bc=1N^0QLN0GMPc6@uCB0QL=>2@BHJA.1RMmAJoJ-À/ 0216HZcˆcN0k0h^@u¨_6HJ>d^Å@uCB0QL
>k@BHAI1RMmAoMPcgKV@.A0G9DnOm@D@.ÛIMPAoÀ<)1ˆ1R^0GMPAI1N0kLdcN0Q>l1RMm@.AÅ@)f0x<B>d^®bJ<BMmLg@)f>Q<u1R0koB@.LNMP0QcQ-šªAIHJ_ˆ9;0kLgMmAŽ¡}Ma¤
@.9DCIMP@BHZcOPn>k<BAA@)1`OPMm0?MPA$¡ÆMPMa¤2Wv¡}MmC¤@BLh¡}CDMa¤8<.c8MPc80QCIMaKV0QA.18fÆLR@B_‘1N^0?Oa<Bc1=KVMmo.MÈ1x-?•^0kLR0?>k<)AJA@)1`9;0h<
>k@B_X_X@BA½ADH_ˆ9;0kLMPA®¡ÆMݤ<)AJK¡}MmMPMa¤<.c<)ADnX1
¨r@?cHJ>d^’ADH_69Z0QLRceKVMmåÃ0kLMPA½1N^0 Ú jT1R^½KVMPoBMm1Q-¬µf–<gADH_ˆ9;0kL
9;0kOP@BAJo.cv1N@g9Z@B1N^¡ÆMݤv<BAJKÀ¡}C¤lWD1N^J0kAŠcNHJ>d^½<ˆADH_69Z0QLv@Bf·1N^0fÆ@.LN_¯Ï;,x¿,x¿V-–•^MacrMPcKVMPCDMPcNMm9JOm09DnhÁ=@BAJOmn
fÆ@.LˆÏ…âÁ¸NžJ¸NàJ-h•^IHZc81N^0QLN0h<BLN0?Á’>2@._X_g@.AAIHJ_ˆ9;0kLdc`MmA$¡}Ma¤`<BAJKÇ¡ÆMPMa¤2-ŠšADH_69Z0QL=¨e^Ma>d^ÂMac`9;@)1R^
MPAp¡ÆMݤ8<)AZK“¡}CDMmMݤ8Mac`@)fr1N^0hfÆ@BLR_†,x¿BÑu,x¿Š<)AJKKVMPCDMPcNMP9MmOPMm1
nÀ9DnÁ’o.MmC.0Qc`Ñ?…¯Ÿ¸RÁ¸Rž¸NàJSÔ1N^DHJcˆ¨v0X^J<„C.0 Ú
ADH_69Z0QLRce>k@B_X_X@BA½MmA®¡}Ma¤e<BAJK¡ÆCDMmMݤl-v•^J<u1e02EV^J<BHJc1e<BOmO bZ@IcNcNMm9JMmOPMÈ1RMm0xc¨eMÈ1R^®¡ÆMݤl-
§@u¨—¡ÆMPMݤ>k<)AŠ^J<„CB0>2@._g_X@.AŠADH_69Z0QLRcr¨eMm1N^½@BAOPnh>k<)1N0ko.@BLRMm0xc¡}MmC¤r<)AZKÀ¡ÆCDMݤl-•^0QLN08<)LR0A@6ADH_69Z0QLRc
>k@B_X_X@BA!9;021
¨r0k0QAÅ¡ÆMPMa¤<)AJK®¡}CIMݤe<.c0kCDMaKV0kAI1fÆLR@B_:ÁujµLdK½KMmo.MÈ1x-•^J0kLR0=MPc@BAOPn½@BA0`ADH_69Z0QL>2@._X_g@.A
1R@!¡ÆMPMa¤r<)AZKÀ¡ÆCDMݤlWVAJ<B_X0kOPn!,x¿V,„¿)Ÿ=<BAJKG1N^MacrMPceKVMPCDMPcNMP9Om09DnhÁ-•^0QLN0MPcrA@B1N^MPAo?>2@._X_g@.A?1N@!¡}MmMPMa¤r<BAJK
¡}C¤8<Bc8>k<BAÀ9;0XcN0k0QAÀfÆLR@B_:1R^0?ÁujµLRKKVMPoBMm1Q-6•^J0g@.AOPnADH_ˆ9;0kL`>2@._X_g@.A1R@¡}MmMPMݤ8<)AJK]¡ÆCDMmMݤMac6,„¿V,„¿B¿
<BAJKŠ1N^MacMaceA@)1KVMPCDMPcNMP9Om0`9DnGÁJ-–¬µ1>k<)A½0Q<.cMPOPnh9Z08MPAVfÆ0kLRLR0QKG1N^J<)1A@XAIHJ_ˆ9;0kLeMac>2@._g_X@.AG1R@’¡}MmC¤<BAJK
¡}CDMa¤v9DnXOm@D@BÛDMPAo6<)1v1R^0`‹„jµAJKGKVMPoBMm1Q-DMP_XMmOa<)LROPngAJ@6ADH_69Z0QLrMacr>2@B_X_X@BAG1N@½¡ÆC¤r<)AJK¡ÆCDMmMݤl-•^DHJcv1N^J0kLR0
<BLN0`ÁD¾ Ú ¾6,„… ã AIHJ_ˆ9;0kLdc¨e^MP>d^!<BLN0`>2@.HAI1N0xKh1
¨eMa>20.-
‹
ƒ ˆ0 >k@BAJ>kOmHJK081N^J<)11N^0ˆAIHJ_ˆ9;0kL@Bf¿ujµKVMPoBMm1ADH_ˆ9;0kLdce¨e^Ma>d^!>2@.A.1d<)MPA½1N^0=9JOm@V>dÛÀ,„¿g<)AJK!KVMPCDMPcNMP9Om0`9Dn
,„¿6MPc ÚIã º¼ ã … Ú º)à-
¿-r¬­Ag1RLNMa<)Ao.Om034g5gW.OP0217¦9;01N^0e_XMaKVbZ@.MmAI1@Bf;465g-–¬µf ›r37h4—… Ú ¿   <)AJK?›r3587°…pÁBŸ   W.KV0k1N0kLR_XMmAJ0
›r4637!-
y z;{}|~Q€}z;v‚ w LR<„¨”4ˆæÇb;0kLRbZ0QAJKVMa>2HOa<)L1R@X35±<)AJKˆç
@BMPA!æŒ1R@?7!-¡TD0Q0=£tMmoZ-‹V-è¤
A

15

60 45 30

B D C

Fig. 2.

VMmAJ>k0›r4658挅ŽÁBŸ. )W)¨r0vo.021v›584ˆæŒ…ŽžBŸI u-DMmAZ>20e4ˆær5ÇMPc<LRMmo.^.1NjT1RLNMa<)AJoBOP0¨eMm1N^h›r4658挅ŽÁBŸ. )W)¨r0
^Z<„CB046殅Ž4658‰)‹`…Ž4ˆ7!-t•^DHJc MPA81NLRMa<)Ao.Om046æ7!W„¨r0@B9Zc0QLNC.0–1N^J<)1t4ˆæŅŽ4ˆ7i<BAJKˆ›r7h46挅$žBŸI )-
•^JMPcMP_gbJOmMP0Qc1R^J<u1`4ˆæv7Mac<BAÀ0xF.HJMmOa<u1R0kLd<)O1NLRMP<BAoBOP06<BAJK^0QAJ>20Xæ4é…iæv7!- Ö cNMmAJo!›ræv7h4ª…¦žBŸI 
<BAJK›r37h4±… Ú ¿B )W¨v0`o.021›r37hæ]…i,x¿B )-vËrHV1›r7h3æ]…i,x¿B )-v•^DHJceæv7°…pæv3=- ƒ 0=^0QAJ>20`^J<„C.0
æv7â…pæv3$…Žæ4G-•^MacMm_XbOPMm0xc1N^Z<u1æ“MPce1N^0`>2MPLd>2H_?>20QAI1NLR0@Bf1R^081NLRMa<)Ao.Om084ˆ7h3ˆ-•^DHJc
›r4ˆ37°… ‹, ›r4ˆæ7¯… ‹Â, ê ž.Ÿ   …ŽÁBŸ   Ê
žJ- w 021R0kLR_XMmA0=<BOmO·1NLRMmbOP0Qc`¡TÏ;¸RЄ¸RÑk¤v@)f–bZ@IcMm1NMPCB08MPA.1R0ko.0kLdcecHJ>d^Š1N^Z<u1ÏhëÇÐë“Ñ<)AJK
Ï8¾ŒÐ¾ŒÑ¾ŒÏVоŒÐlѾŒÑ2Ïg…ŽÏVÐlѾ$,.Ê
y z;{}|~Q€}z;v‚ sHV1N1NMPAoˆÏ¼,…®ì WVÐÔ¼,…Ží8<)AJK?Ñ ¼,…ÇîDWI1N^J00QFIHJ<)1NMP@BAX_?<„nˆ9;0¨eLRMÈ1N1N0QA?MmA?1R^0efÆ@.LN_
ìZí„î=…p‹J¡Õì6¾Œíe¾Åî)¤ ¾ Ú ¸
¨e^J0kLR0vì¸NíD¸î6<)LR0MPAI1N0ko.0kLdcvcNHJ>d^G1N^J<)1eŸXëì!ë“í6ëŒîD- Ä 9Zc0QLNC.01N^Z<u1–슅pŸ6MPcrA@B1ebZ@IcNcNMm9JOm0.W.fÆ@.Lr1N^0QA
Ÿh…¦‹¡ÈìX¾]í)¤t¾ Ú ¨e^Ma>d^MPcMP_gb;@.cRcNMm9OP0gMmAAJ@BAA0Qo.<)1NMPCB0=MPAI1N0ko.0kLdck-•^DHJc¨r0ˆ_?<„n’¨eLRMÈ1R0=1N^JMPcMPA!1N^0
fÆ@.LN_
‹ï ì;, í ¾ í„, î ¾ îd, ìvð ¾ ì;Úí„î …i,BÊ
¬µfÔìÓ$ÁJWV1N^0QAíXÓ$Á?<)AJK½îXÓ$Á-v•^J0kA’Om0kfÝ1cNMPKV0ˆMPce9;@BHJAJKV0QK’9Dn½žI‰uà¾ Ú ‰B‹.º=¨e^Ma>d^½MacOP0QcRc1R^J<)A,B-
ƒ 0=>2@.AJ>2OPHJKV081R^J<u1슅i,8@BL‹V-
ñ6òØxó“ôZõ VHbb;@.cN0ìÀ…,.-•^0kA¨v0ˆ^J<„CB0ˆí„î?…i‹J¡}í¾Œî)¤t¾“žG@BL6¡Tí8¼Œ‹B¤k¡Æî`¼Å‹B¤e…,xŸ-•^JMPcoBMPCB0xc
ív¼!‹=…˜,BWIîr¼’‹`…i,QŸ`@.Lvív¼’‹`…q‹=<)AZKXîv¼!‹`…Ž¿J¡ÆLR0Q>k<BOmO;íˆë]î)¤l-•^MacMP_XbOPMm0xc¡Èì ¸RíD¸î)¤…™¡,B¸RÁ¸k,„‹B¤2W
¡,B¸ Ú ¸dºB¤l-
ñ6òØxó“ö·õ ¬µf–ìŅ°‹VW 1N^0?0xF.HZ<u1NMP@BALR0QKVHZ>20Qc1R@À‹)í„î½…°‹¡¢‹¾pí¾Çî)¤¾ Ú @.L=í„î’…âí¾Çî8¾ Ú -Š•^MPc
LR0QKHJ>20xc1N@À¡Tí¼“,„¤2¡}î¼Ç,„¤…˜¿V- ¥ 0QAJ>20=í8¼“,8…—,ˆ<BAJK½î¼Ç,…˜¿gMac1R^0=@BAJOmn’c@.OmH1NMP@BA·-r•^MPcoBMPCB0xc
¡Èì ¸RíD¸î)¤…—¡T‹V¸d‹V¸Rž.¤l-
÷ 0kCB0QL1RMmAJog9Z<B>dÛ?1R@XÏ;¸dЄ¸NÑuWV¨r08oB0k1e1N^LR0k01NLRMPbOm0xckÜ¡TÏ;¸RЄ¸RÑk¤…™¡¢‹V¸ Ú ¸k,xÁ.¤lWÔ¡¢‹V¸R¿¸ ã ¤lW·¡}ÁJ¸NÁJ¸dº)¤2-
Á
ºV-/ 021Ï;¸RÐu¸NÑ9Z081R^LN0Q08bZ@IcMm1NMPCB0`LR0Q<BO;ADH_69Z0QLRcecNHJ>d^½1N^Z<u1Ï8¾]оÅÑ…i,B-/·0k1
ø …p_gMPAˆù.ÏIúr¾ÅÏ » Ð2Ñu¸Ðlú¾ÅÏVÐ » Ñu¸Ñ2ú¾ŒÏVÐlÑ ».û Ê
sLN@uC.01R^J<u11R^0`LN@D@B1Rc@)ft1N^J0`0QFIHJ<u1RMm@.AŠ¶;»v¾Å¶g¾ Ú ø …$ŸX<BLN08LR0Q<)O¢-
y z;{}|~Q€}z;v‚ DHbb;@.cN0`1N^J0ˆ0QFIHJ<)1NMP@BA’¶Z»¾Œ¶X¾ Ú ø …”ŸX^J<.cA@hLR0Q<BO·LR@I@B1RcQ-r•^J0kA®,¼Ç,Qž øü ŸJ-e•^MPc
MP_XbOPMm0xc1N^J<)1
,e¼“,QžÌ}Ï ú ¾ÅÏ » ÐlÑkÍ ü Ÿ¸ý,e¼“,Qž ̢Рú ¾ŒÏVÐ » ÑkÍ ü Ÿ¸þ,e¼“,QžÌ}Ñ ú ¾ŒÏVÐlÑ » Í ü ŸJÊ
Ä 9Zc0QLNC.01R^J<u1
,e¼“,Qž Ì ÏDú¾ŒÏ » ÐlÑ Í ü Ÿ …·ÿ ,e¼“,Qž.Ï » Ì Ï8¾]Ð2Ñ Í ü Ÿ
…·ÿ ,e¼“,Qž.Ï » Ì ,e¼®Ðv¼ÂѾ]ÐlÑ Í ü Ÿ
…·ÿ ,e¼“,Qž.Ï » ¡
,e¼®Ð2¤2¡,¼ÂÑQ¤ ü Ÿ
…·ÿ ,x, ž ü Ï » ¡,e¼Ð2¤k¡
,e¼Ñk¤lÊ
VMm_XMPOP<BLNOPnh¨r0`_X<„n?@.9V1R<BMmA
, ü Ð » ¡,e¼ÂÑQ¤2¡
,¼ÂÏV¤l¸ , ü Ñ » ¡
,e¼ÏV¤2¡
,e¼®Ð2¤2Ê
,xž ,xž
´HOÈ1RMmbJOmnDMPAoX1N^0xc081R^LR0k08MmAJ0QFIHJ<)OPMm1NMP0QcQW¨v08o.021
Ï » Ð » Ñ » ¡
,e¼ÏV¤ » ¡
,e¼®Ð2¤ » ¡
,¼Ñk¤ » ,xž, ú Ê
¥ @u¨v0QCB0QLQWVŸ ü Ï ü ,8MP_XbOmMP0Qce1N^J<)1Ï;¡,e¼ÂÏV¤rë”,„‰ Ú - ¥ 0kAZ>20
Ï » Ð » Ñ » ¡
,e¼ÏD¤ » ¡,e¼Ðk¤ » ¡,e¼ÂÑQ¤ » … Ì Ïá,e¼ÂÏV¤ Í » Ì Ðu¡,e¼Ð2¤ Í » Ì ÑB¡
,¼Ñk¤ Í » ë ,Qž, ú ¸
<X>k@BAI1NLd<BKVMa>l1RMm@.A·- ƒ 0=>k@BAJ>kOmHZKV01N^J<)1e1N^0`o.MmC.0kA½0QFIHJ<u1RMm@.AŠ^Z<BcLR0Q<)OÔLR@D@)1dck-

Ú

  "!#$&%' ()! +*-,. ,0/12(4356+75982$&82":!)%')'$<;=5>$&7?
A@4 B@< C$4";8"4% D" E 7 (,:
F,G  BH943)I8$48")!:%')'$&;J56$47?1/K7L + B,: + A,M I*-,G (.
?N43"?O%'QP=@<R@&ST@4UV$4";482"4%'D" EXWH$&7YDF43:NP=@<R(@&SZ@4U[$4]\<^_>_a`6b2c4def

gihkjmlnpo6hkqXrs :;$4D+43:I I@<R(@4/t@4P$4


7:EM'%uXvN":

w FR
PZx w +/FPCxzy{|~} w / +PCx w B€

 %u? %''$&'E43)F7)EG'%'% ‚E]75ƒ/=@&SZ@<R(@4 „% D";

w …R
U†xzy{|ˆ‡ w /AR
S‰xzy{|X‡ w /
…SŠxzy{|ˆ‡‹y{|~} w ŒxK"{|=} w ]€

Žt3M);~9
7)4%' w FR
P‡ w +R+U‘xK"{ | H’“=5>7''7YN;=43:IR(@4U-@4P”'%'I7•43F;4?A'%'

A Q

F M

B P D C

– #  Z;4%'? %'u$& E—8)$47pD˜43) w


PARŒ‡ w (PFS xŠ"{ | ™Žt3)%';š%'? 8:'% ";]43)•P=@&SZ@<R›$&
7'u%'"$"ƒŽt3M);tSZ@4Uœ743•u% …7Q43F'%uH7%')%uPž)!-R

ŸM J%')!•43L_'c4d¢¡£¤87;4;4%': …D¢u
75H¥
‡¤¦p,2N3$4F¥k@&¦+$4A87;4% 4% DF%u§$&;I;4)<3¨43:A!)% DG%'!";
¥+‡©"ª¦…)!˜"ª !)% DG%'!";~¥…‡z"¦Y

g hkjmlnpo6hkqXr  %')]"ª]!)% DG%'!";¥F‡z"¦p,i9;43)("ªB!)% DG%'!";t¥}—Ÿ¦


)!«3")(% I';7š!)% DG%'!";
i
¬¥(}"Ÿ¦pNŽN3)%';t%'¨4$&-%'?8:'%'";~43)A"ª)­¯®°¬¥A‡¦±<  %'? %u'$& E¨§­¯®m¥‡©"ª¦±9x²³§­¯®m¥F‡Ÿ¦±9x²
§­¯®°¬¥(‡T"Ÿ¦±tx´²Š§­¯®°¬¥‡¤¦±<  %u)F!µ®¶@""ª±x·,i97)')!A43)¸§ª)­¯®°¬¥(‡¹¦±<+Žt3º:;
9
? "Ešt$<% F¬¥
‡‹¦=xK¸§ª»Q567$N;7?A)4$&iM)?A2$N»2ˆvN")

´
‡‹¬¦txK¸§ª»F‡¤¼¦txK¸¸§»A‡‹¬¦9}®°»A‡‹¦±<€

Žt3:%';~;437Y;=43)¬!:% DM%u!";=»
‡‹¦…)!Q3")»
‡‹¦…½¬M – 
43)$45>7$&
7)4%'

¬®m¥
‡‹¦±ƒxK¸§ª»A‡‹¼¦txŒ"ª»F‡‹¼®°»F‡‹¦±~½Z"ªN‡‹¼¾¨¬AxK"¬M€

’“A567' 7pN;A43)¥(‡z¦¨½™ŸM˜Ž0¿G%'«¥ÀxÁŸª-:!—¦BxÁ¼M,ƒˆQ;B43)(43š7)!)% 4% 7:;F75t43)


8)$&7: "?1$4F;44%u;Â)"!iHŽt3M);~43A? %')%'?)?D¢u+75ƒ¥…‡‹¦I%';ŸM

ªMN’“5µ¥k@&¦p@4Ã+$4
43$&+827;&% 4% D+$4"´M)?A$&;,G8)$47pD
43)

¥ºÄH‡© ¦<ÄH‡© ÃÄX‡©


‡ ‡ ½ªM€
¦ƒ‡—à ÃX‡—¥ ¥
‡‹¦

gihkjmlnpo6hkqXr – ();F43)F$&% DG%'´%u"ź:'% 4% ";=¥MÄX‡z½zŸ¥:,¦<Ä9‡z½ÆŸ¦+:!-ÃĈ‡©½zŸâtvN")


9+7)4%'
¥ Ä ‡© ¦ Ä ‡© Ã Ä ‡Æ Ÿ¥ Ÿ¦ ŸÃ
‡ ‡ ½ ‡ ‡ €
¦ƒ‡—à ÃH‡—¥ ¥+‡‹¦ ¦ƒ‡—à ÃH‡—¥ ¥…‡¤¦
Ÿ ¥
 Ÿ ¦
 Ÿ Ã

‡ ‡ ½ªM€
¦ƒ‡—à ÃX‡‹¥ ¥+‡‹¦
Ç !)!)%')(¬(743•;&%'!";,G43)%';=%u;="ź:% D¢'"§=7

®°Ÿ¥
‡‹Ÿ¦ƒ‡‹ŸÃ"±~È ‡ ‡ ½yM€
¦H‡Éà Nj¥ ¥
‡‹¦ºÊ

Žµ¿M%u˚xz¦ƒ‡—â,)Ì x©ÃH‡—¥k,:ÍAx©¥
‡‹¦p,43)%u;=%';="ÅM)% D "§=7

®6Ë(‡ÉÌ
‡—ͺ±~È ‡ ‡ ½yM€
Ë Ì Í Ê

Žt3:%';~%';t7);"ÅM")F75 DžΠ“Ï Î %'"ÅM)'% ‚E

Кj>nYÑMÒ¢q0ÓnYÑGj>ԃr Žt3)
;4:;4%'44% 7);=¦ƒ‡—ÃNxÆË0,)ÃX‡—¥(x©Ìk,:¥+‡‹¦=x©Í '"!);=7

Õ Ÿ¥ Õ Ì
‡ÉÍF}Ë Õ Ë Ì
x x È ‡ }#ª½¹¬+}#ª
xzªM€
¦H‡—Ã Ë Ì Ë9Ê

¸ Ç ¬F¾]¬;4ÅM)$4…%';9!)%';4;""!š%']7LyF$4"4 ";=MEL'%u";X8k$&' ":7(%'4;9;&%'!";9;4)&3B43)='243";4


$4"4) ";N3)YDF%'º$;4%u!";ƒWH$47pDF43)N43$4F$4
'9YEM; n<Ö(h 7$<"§N$4"4 ";

gihkjmlnpo6hkqXrt× 7);4%'!$~43
!:%';4;"4% 7B75µ43+% D"•¬¾•¬(;4ÅM)$4+%'B7L77$&"º=$4"4 ";
N%'43«'";…827;&;4% : $4";Žt3(7)'E¨$4"4'N% 43Ø$4"%';…¤¾—($4"4   %'? %u'$& E,
9…
¾šŸM,´¾šªA$4"4 ";t5>7$=$4";NŸ],Ùª():% 4;J’“B43+;+750¸))% 4;99…?L"Eš3)"D+"% 43)$
BN¾¸($4"4) …7$~Ÿ+¾BŸ;4ź:$4  %'? %'u$& E,43$&Iš…]¾L¼A$4"4 …567$~$&"(¼):% 4;
)!¹¾#¬Q7$ Ÿš¾˜ª¨$4"4 B567$(¬¨))% 4; Ç ºE$4"4 BN% 43À$4"ØÚ¨))%'4;F?);F¨¾ÉÚ
$4"4) ,iN3)%'<3-%u;7…87;4;4%': F;4%u)A43)(u$4";…;4%'!(7)'!-(¬B))%'4; Ç )!غE•$4"4 
N%'43$4"F+):% 4;ƒ?A:;ƒtFŸ~¾+¸
$4"4) t’“5k43$&N%';ƒºE(!:%';4;"4% 7(7543N% D"L¬~¾F¬F;4ź:$4
%'¨7šy]7G7)$&"º$4"4) ";IN% 43¨$&";+¥)Û+Üz¥ Üz¥MÝAÜz¥ºÞÜz¥MßÜz¥ºàÜz¥MáAÜz¥MâAÜz¥Mã¢,
Ä
43"•ˆ
7k;$4D+43)

¥ Û ½T@0¥ ½ŸM@0¥ Ý ½ªM@¥ Þ ½‹¸@¥ ß ½‹¸@¥ à ½¼M@¥ á ½¤¬M@0¥ â ½¬M@¥ ã ½M@


Ä
)!Q3)")
43+74µ$4"75J'i43+$&"4 ";N%u;

¥ Û ‡—¥ ‡Æää䢇—¥ ã ½TX‡‹ŸN‡¤ªN‡À¸I‡É¸I‡¤¼t‡‹¬N‡‹¬t‡‹Axzªy吪¬M@


Ä
N3:%'&3š%';943+$4"75043…% D"•;4ÅM)$4Hv")
% 50 ¬F¾Q¬(;4ÅM)$4+%';~!)%';&;""!Q%'B7Ly$4"4 ";
;t;44% 8:)'"!Q%uš43
8)$47k "?Q,G43$4
?);=+97]7)$&"ºt$4"4 ";"

¼M I +*[]«Åº:!$&%''$<X%'#N3)%'<3À I æ%';A8:$&''"H7# +*ç)!À8$48")!)%':'$+7« I*-è


I žxzª +*-è)!B43+$4"7543+ÅM)!$&%'u$&k%u;ˆ¸ƒ’“50(% $< Iš…!$&YNB7)&3)%uuk43)
;4%u!";~75043)Fź:!$&%''$<°,GÂ:)!š% 4;=$&!:%');

gihkjmlnpo6hkqXr 9P,)R(,é(,)ê4387%'º4;H7507§4=75´%'% $& t% 43 43…;4%'!)";ˆ I B,M (, +*-,


*B „$4";8"4% D" E  %':F *†%';N8$48")!)%')u$ˆ7B  O:!•  ë%';=8:$&u "i7]*Q (,:ˆ(;F43:
IPÁxO …êCxëêJ*›x™*Béëx1ìG,µ43B$&!)%');
75=43]%':;4$&% "!Ø% $& •A (PÁxO Rëx1̘:!
ARTxT +éZx©ËHíI;4%' I žxzª +*-,ˆFtìN‡ÉÌxTª®6ì‡À˱<

Ÿ
D r R x C
x
Q
r

S y

A r P K y-x B

 %u)…43F$4"75J I 
*î%u;~¸,)ˆA';7

¸Ax I*«®m I T‡¤ +*Q±ƒx ®°Ÿ¢ì§±ï°¸:®6ì‡ÉËi±¶ð€


Ÿ Ÿ
Žt3M);Nˆ7)4%'«ì)®6쇗Ëi±=x·IíI;4%'BWHEM43)7$&;…437$&"?Q,ˆ7)4%'« Ä x„ ñ Ä ‡¹ +ñ Ä 
v7Y9D$F ŒxCÌA‡‹Ë0,2 (ñçxCÌ }#˘:!Ø
ñ[xKŸ¢ìG  :;4%'44%'43";4(:!-;4%u?8:'% 56EG%',9
HËkÌx©ì Ä ƒò~Hì‡BÌxzª®6ì‡B˱µ%'D";HÌ xzŸ¢ì‡Qª¢ËƒŽt3M);Jì Ä xÆ˵®°Ÿ¢ì‡Qª¢Ëi±ƒ)!(43)%u;H;4%'?8k'% Â)";
7#®6ì}—ª¢Ëi±®6ì+‡¤Ë±Ix”{G – ]7)')!(43)+ìšxª¢Ë0óN7pž43$4"'4%'7Øì®6셇¤Ëi±xë%'?8k'% ";
43)N¸ìÄtxzªM,:% DM%u(ì
xZô ªõŸM

¬MWX$47YDI43)~43$4…$4…%'Â:)%'" EA? ºE 87;4% 4%'D%'º$&;9ö-;4:&3]43)~öX®6öA‡—Y±ƒBN÷G8)$4";4;4"!


;9A;&)?”75i‚ˆ7(87;4% 4%'D;4ÅM)$4";9%'-d£ƒ_ c&d¢¡f£k97!)%¯ø$4"ºX~"EG;®mv$4I¥MÄ´‡Ø¦fÄN)!š¦fÄi‡«¥MÄN$&
7);&%'!$4"!Q;=43)F;4?
$&8)$4";"º44% 7iù±

g hkjmlnpo6hkqXr R1xžöX®6öQ‡CY±<’‚
%';+7ºD")% "º
7•<37º7;4 ö—x”ú Ä ,´567$+43"ÀR™%u;I'$4"!EØ
i
;4:?ë75ˆ97•;4ź:$4";û
R”xKú Ä ïüú Ä ‡C"ðx‘ïüú Ä ð Ä ‡¤ú Ä 
’“5H56)$443$…ú Ä %'4;" 5H%';…š;4)?Á75X‚ˆ7
;4ÅM)$4";",);4"Ešú•ÄtxÀýkÄX‡—þÄ¢,43"

RCx™ï¯ý Ä ‡‹þ Ä ð:ï°ú Ä ‡©"ð…x™ïaý:ú‡—þð Ä ‡žï¯ý]}˜þ¢ú¨ð Ä €

ó7Ø43)]43)-‚ˆ7‹$48:$4";"º44% 7);B5>7$•RV$4Ø!)%';4%u)CŽt3M);,=567$š÷G? 8: ,t9«?L"E4¿


úîxz¼»2,§ýšxzª»2,2þ+x©¸§»2,:N3$&F»šD¢$<% ";=7YD$…)4$<iº)?A$&;ƒ’¶Q43)%';=;4Fö-xÆú Ä xZŸ¼» Ä ,
)!
RZxO» Ä ð Ä ‡žïü¼»ð Ä x™ï&"¼» Ä ‡É¸§»ð Ä ‡KïŸ{» Ä }#ª»ð Ä €
Ç ;…ˆ(D$4E»¨7pD$A)4)$&Jº)?A$&;,9
%'Â:)%'" EQ? §E«º:?F$&;I75943(5>$47?†öX®6öQ‡CY±
"<3•75JN3)%'<3QQ+÷M8:$4";4;"!•;;4)?™75µ97];&ź)$&";t%'š97]!:%';4%')~~"EG;

Úºtÿœ+43A;t75ƒ'i87;4% 4%'D
%'§$<;=$4"$I43)•7$"ź)´7] )!¨ «û§ÿ[ÿV

5m))4% 7Q;&)&3Q43:
 ƒ®6ˇÉÌ)±Hx ƒ®6Ëḵˆ567$N'Ë«½‹¸,)ÌQ½‹¸H’“ 5 ƒ®°±ƒxTyM,:!$&? %u
 ƒ®°y±<

gihkjmlnpo6hkqXr – 
7:;$4DF43)

ƒ®°y±ƒxƒ®6¸I‡—¼±ˆxƒ®6¸+äY¼±Hxƒ®°Ÿ{§±Hxƒ®¶"¬N‡É¸º±Hxƒ®¶"¬+为±Hxƒ®°¬¢¸º±
xƒ®°…äY±Hxƒ®°N‡—±Hxƒ®¶"¬±ˆxƒ®6¸+为±Hxƒ®6¸I‡—¸º±Hxƒ®°±<€

v ")% 5 ƒ®°±NxžyM,´43"
ƒ®°y±xžyMš®mŽt3)%';…%';I7;$&%uFŽt3$&? YE-(743$
!)% ø2$4"ºI9YEM;

75N8:8)$47&3:%' ƒ®°±F56$47? ƒ®°y±<˜Žt3]%u?87$44§A43)%'«7«2B7:;$&D"!É%u;F43B5m43:43)
$&:  ƒ®6Ë(‡—Ì)±H x ƒ®6Ë:Ì)±98)8:'% ";~7: EšN3"šËØ)!•Ìš$4FI ";t¸ s 
? YEQ;$<%';t);&%'
M)?A2$<;~Ë-)!QÌBN3)%u&3Q$4F;&? ' $~43)¨¸,):~43)N%u;t7=D'%'!iƒ 7$÷G?8k 

ƒ®°y±Hxƒ®°ªIäpª±Hxƒ®°ªt‡‹ª±Xxƒ®°¬±Hxƒ®6¸‡‹Ÿ±Xxƒ®6¸+äpŸ±Hxƒ®°±<@

%';=)7N(D¢u%'!Q;$&%')ù±

ª

     !"# $&%

')(+*,.-0/&12435,7698:6); <=->,.?@6)8BADCE, F-HGJIK6)8=ADCL,9MN/PO35,&->QB,73BILRJ, ;.-HSDGTS9U


VW1X/P2ZYWIE->Q:OSD8[12M
698BF[1\]35,7-HQ=,^69CE-HI_-H<BF=,PU`G>SDab1cS)8[/P2(edNQ=SfYg-HQh6i-&VW2j\XOlknmpo)qr(
sutwvyxzf{`tw|~}
 G>6Y  *n€5,rG>€w, 8BF=IL; <=CK6)G7-HS/&1M"O‚ €5,rGƒ?
€w, 8=FBIL;r<BCK6)G-HS„/P2Mj6)8BF…O† €w, GJ€5,r8BFBIL;r<N?
CL6)G-HS‡1\ˆ( ‰0QB,r8Š\† ‹ O‚Œ( dNIE8B; , A

/POc3=ILR>,r;r->RTV0/XpYŽ,SD3BRJ, GƒD,P->Qh69-ŽV1^/&O‘‹
VW‚’/&O“( ‰0Qp<BRZIL8”-HGJIK6)8=ADCL,rR‡/&•"O 6)8BF
/P‚O“Ye,–RJ, ,‡->Qh69-—VW•˜/&O ‹ V‚’/POM
E

V0/&‚O™‹›šDœDqP‹]VW/P•~O“M698BFž/&O”ILRP; SDaŸ?
aSD8 ( ¡, 8B;r,—->G>IL6)8BADCE, R¢/P•"O 6)8BFŠ/&‚O
K
M

69G>,Œ;rSD8BADGJ<B, 8£-XADIE¤IL8=A¥O• ‹ O‚Œ(§¦Ž<=- L

O• k O†4¨R>IL8=; ,41\ CLIE, R¥IL8=R>ILF=,–-HQB,


->G>IL6)8BADCE,£©ª3£«”6); <=->, 8B,rR>R4€BG>SD€w, Gƒ-@«=¬.( ‰0Qp<BR
\X† k‡O†4(W‰0QBILR+ILa€BCLIE, RŽ->Qh69-­VW\XO† k
B D C

VWO\X† ‹ šDœ q7® VW\XO†4(4¯¢,;rSD8B; CE<BFB,


->Qh69-°VW\XO† k mpo9q ( dNIE8B; ,…VW2j\XO ‹
VW\XO†4B-HQB,­GJ, R><=CE-0U`SDCECLS±YWR (
²v³z±´pµi|·¶Nz±´žsutwvyxzf{`tw|~}
*,r-V2j\XO¸‹c¹N(¯¢,Qh6D,2^O§‹cº¤».¼B©ª»Ž½—¾r¬
6)8BF2^\¿‹gºe; S)RÀ2(˜ÁR>IE8BA^RJIL8B,WGJ<BCL,WIL8-HGJIK6)8=ADCL,
A 2j\XOBYe,­Qh6D,
2jO ‹ 2^\
RJIL8¹ RJIL8©ª2¥½Ã¹)¬¤Ä
E

c θ b

‰0QBIERGJ, FB<=; , RW-HS


©y»¤½¾r¬=R>IL8¹
;rSDRÀ2¿‹‡»5RJIL8W2¢;rSDRÀ¹ ½»w; SDRw2¢RJIL8W¹ Ä
d¤ILa€BCLI_Åh;69->ILSD8ADI_D, R¾uRJIL8W¹
;rSDRÀ2¿‹‡»5RJIL8W2¢;rSDRÀ¹F[R>S-HQh69-
B D ab/(b+c) C
a

-H6)8W¹j‹ ¾u»5; RJILS)8Rh22 ‹ ; RJILS)8WRÀ1 2 ‹ R>IE8©`ÆÂR>IL¼D8Ç 1 ® 2X¬ Ä


d¤IL8B;r,0/&12–IER"6); <=->,.?@6)8BADCE, F DYe,WQh6D,/ZÈÃÆ·¼DÇN(¡+, 8=; ,+1ž½“2›kÃƼDÇ&SDG"1ckÉ©`Æ·¼DÇ)¬ ® 2(
0‰ Q=, G>,.U`SDG>,­RJIL81ckÃR>IE8©`Æ·¼)Ç ® 2X¬Ê(˜‰0QBILRWILa€BCEIL, R~-HQh6i--H6)8W¹Ÿk‡'^6)8BF:QB,r8B; ,^¹ŸkËÆ·¼9mB(
Ǥ(+*,.-jºÀÌH»fÌH¾X35,Ÿ->QBG>,r,8B69-H<BG>6)C
8p<Ba3w, GJR­R><B;ÍQž-HQh6i-jº¨ÈZ»È!¾698BFžAD; F·©ª¾ ® ºÀÌH¾ ® »r¬P‹l')(
d¤<B€B€wSDR>,-HQ=, G>,:,ÊÎ=IERJ->Rˆ698°IE8£-H, A), GϞR><B;ÍQ°-HQh69-º½gÏÀÌH»W½‡ÏhÌ;W½–ÏžU`S)G>aÐ->QB,:R>IEFB, RŸS)UP6
GJILADQ£-ƒ?Ñ6)8BA)CL, Fˆ-HG>IL6)8BADCE,)(˜Ò˜GJSf),j->Qh69-0->QB, GJ,^,ÊÎ=IERJ-WIE8p->, AD,rG>RÓ@ÌHԙRJ<B;ÍQ“->Qh69-+¾˜½°Ï‹‡Ó³Õ½°ÔÕ±(
sutwvyxzf{`tw|~}
¯Ö,­Qh6D,
©y¾
½ÃϤ¬ Õ ‹!©ªº7½ÃϤ¬ Õ ½g©y»
½°Ï¤¬ Õ Ä
‰0Q=ILR0G>,rFB<B;r, R+->S
Ï Õ ½ËÇ)Ïw©ªº&½Ã» ® ¾ ¬½°º Õ ½°» Õ ® ¾ Õ ‹gœ Ä
¤d SDCE¤IL8=A-HQB,­×p<h6)FBG>69-HIE;7, ×p<h69->ILSD8U`SDGWÏhBYe,­SD3=-Í69IL8
ÏX‹ ® ©yº0½¢» ® ¾ ¬À؅٠©yºP½Ã» ® ¾ ¬ Õ ® ©ªº Õ ½°» Õ ® ¾ Õ ¬"‹ ® ©ªº0½¢» ® ¾r¬À؅٠ÇB©y¾ ® º¤¬ ©y¾ ® »r¬ Ä
d¤IL8B;r,+ÏjILR"6)8IL8£->, AD,rG ¤ÇB©ª¾ ® º¤¬ ©ª¾ ® ».¬~a<=RJ-~35,67€w, GJU`,r;r-~R>×p<h69G>,)MNRH6«ˆÇB©ª¾ ® º¤¬ ©ª¾ ® ».¬e‹—ÚÀÕ±
¦e<=-WAD; F·©ª¾ ® ºhÌ; ® ».¬"‹4'D("¡, 8B;r,jYe,­Qh6D,
¾ ® ºŸ‹–Ç9Û Õ Ì ¾ ® »W‹—Ü Õ SDG ¾ ® ºŸ‹…Û Õ Ì ¾ ® »0‹‡Ç9Ü Õ Ì
YWQ=, G>,0ۓknœ76)8=FÜknœ7698BFAD;rF·©ªÛÂÌHÜN¬
‹4'D(Ý@8Ÿ, I_-HQB,rGS)Uh->QB,T;69R>, R"Ï‹ ® ©yº½» ® ¾ ¬fØÇ9ÛÀÜÀ(
Ý@8:-HQ=,­ÅBG>RJ-; 6)R>,
¾"½ÃÏ‹gÇ)¾ ® º ® »˜ØnÇ9ÛÀܟ‹gÇ9Û Õ ½°Ü Õ ØËÇ9ÛÀܟ‹›©`ÛØËÜN¬ Õ ½ÞÛ Õ Ä
¯Ö,7SD3BRJ, GƒD,­-HQh69-WÛ‹…܁IEaˆ€=CLIL,rRe->Qh69-WÛ‹…ÜŸ‹4'j6)8BF:QB,r8B; ,^¾ ® ºŸ‹–ÇNÌ; ® »0‹¿'D("¡+,r8B; ,
ºhÌÍ»±Ì;X6)G>,Ÿ-HQ=G>, ,; S)8BR>,r; <=->IED,IL8£->, AD,rG>R (Ÿ¯Ö,6)CLRJS[RJ, ,-HQh6i-j¾T½—Ï‹l'ŸU`SDG>;rIL8BA[»e½…Ï“‹cœN
;rSD8£-HG>6)FBIL;.-HIE8BAj->Qh69-Ž»w½žÏIERŽ6jR>IEFB,+S9UÂ6j->G>IK698BADCL,9(‰0Qp<BRŽÛŒ‹g ß Ü6)8=FQB,r8B; ,P¾u½ϟILR˜-HQ=,PR><=a
S9U-@YeSˆ8=SD8N?@àr, GJSˆIE8p->, AD,rG0R>×p<h6)GJ, Rr(
d¤ILaILCK69G>CE«)NIE8“->QB,XRJ, ; S)8BF’;69R>,^Ye,AD,.-&¾"½nÏ‹–Üp՘½–©`ÛØnÜN¬ƒÕ(0‰0Qp<BR7¾"½nψILRW-HQB,^R><BaS)U
-@YeSR>×p<h6)GJ, R (
²v³z±´pµi|·¶Nz±´žsutwvyxzf{`tw|~}
á 8B,Waˆ6«<BR>,;ÍQh6)GJ, ;r-H6)G>IERH69->ILSD8S)Uu€BG>IEaˆI_-HIE),0Ò
«p-HQh6)A)SDG>, 6)8ˆ->G>IL€=CL, Rr( á 3BRJ, GJ),+->Qh69-~AD; F·©ª¾ ®
ºhÌ; ® ».¬&‹l'ILa€BCLIE, R-HQh69-j¾Ž½¥ÏÀÌHº^½¥ÏhÌÍ»e½¥Ï6)GJ,ŸG>, CL69-HI_D,rCE«’€=G>ILa,)(¡+, 8=; ,Ÿ-HQB,rG>,,.ÎNIERJ-
IE8£-H, A), G>RWÔâkËãžR><B;ÍQ[-HQh6i-
º&½°ÏŸ‹…Ô Õ ® ã Õ Ì »
½°ÏŸ‹‡ÇiÔãÌ ¾˜½ÃÏX‹gÔ Õ ½Þã Õ Ä
äN(+åIL8BF6)CECh€h6)ILGJR+©yºhÌÍ».¬
S)UG>, 6)Ch8p<Ba3w, GJRŽR><=;ÍQ-HQh6i-~YWQ=, 8B,.D, GTæžIER~6^G>S¤S)-"S)U5Ú Õ ½’ºDڏ½’»0‹‡œN
æ·Õ ® ǟIERW6)CLRJS6GJSNS9-WS)U->QB,­, ×p<h69->ILSD8u(
sutwvyxzf{`tw|~}
ç S)8BR>IEFB, GW-HQB,j, ×p<h6i-HILS)8“ÚÀ՘½Ãº£Ú½Ë»‹¿œN(0Ýè-PQh69R-@YeSG>S¤S)->R±©ª8=S)-8B, ;r, R>R>6)G>IECE«[G>, 6)C³¬ÊhRH6«“æ
698BF:é~("ê˜I_-HQB,rG+枋…é¢SDGW楋g ß é"(
ë^¶hìr´„íh}
d¤<B€B€wSDR>,:æZ‹âé""R>S-HQB69-æ–ILR6’FBSD<B3BCE,G>S¤S)-(¢dNIE8B; ,:æ·Õ ® ÇILR69CLR>S’6žGJSNS9-
-HQ=,SD8BCE«
€wSDRJR>IL3=ILCLI_-@«XIER"樋gæÕ ® ÇN(
‰0QBIEReGJ, FB<=; , R~-HSˆ©yæj½Ö'±¬ ©yæ ® ÇD¬"‹‡œN(
¡+,r8B; ,P枋 ® '+SDG~枋–Ǥ(
á 3BRJ, GƒD,­-HQh69-+ºX‹ ® Ç9æ„6)8BF“»0‹‡æÕ (˜‰0Q¤<=R©ªºÀÌH»r¬"‹›©yÇNÌ'f¬ŽS)Gj© ® mBÌHmp¬Ê(
ë^¶hìr´žî }
d¤<B€B€wSDR>,¨æ ‹ ß é"(Š‰0QB, GJ,Ö69G>,U`SD<BG€wSDR>RJIL3BIECLI_-HIL,rR M©`݃¬æ⋠æÕ ® ÇÞ698BFgé‹ éÂÕ ® ǤM
©`Ý>݃¬æ⋠é·Õ ® ÇÞ6)8BF‡él‹ æ·Õ ® ǤM©ªÝJÝ>݃¬æ⋠æÕ ® LJ‹ éÂÕ ® ÇÞ6)8BFgæ ‹ ß é"M^SDG
©`Ýèï­¬Ñ鋇æ Õ~® Ç­‹gé Õ"® Ç6)8BF[旋… ß é
©`݃¬T¡+,rG>,©yæeÌ>鬋›©ðÇNÌ ® 'f¬ŽS)Gj© ® 'DÌÊÇD¬Ê("¡+, 8=; ,©yºhÌÍ».¬"‹bñ ® ©ªæ:½°é¬.ÌHæé·ò‹›© ® 'DÌ ® ÇD¬.(
©`Ý>݃¬0dN<=€B€5S)R>,^枋géÂÕ ® Ç6)8=F[é‹gæÕ ® ÇN(˜‰0Q=, 8
æ ® é‹gé Õ ® æ Õ ‹›©`é ® æ¬ ©`é½°æ
¬ Ä
d¤IL8B;r,­æ…‹— ß é"=YŽ,jAD,.-+遽Ã樋 ® 'D(~¡SfYe,r), GrwYe,X69CLR>SŸQB6±),
æ:½°é’‹gé Õ ½°æ Õ ® m^‹›©yæ:½Þ靬 Õ ® Ç)æ·é ® m Ä
Ç
‰0Qp<BR ® 'Ÿ‹b' ® Ç)æ·é ® mB YWQ=IL;ÍQžILa€BCLIE, RW-HQh6i-­æé°‹ ® 'D(j‰0QB, GJ,rU`SDGJ,©ªºhÌÍ».¬P‹ ñ® ©y撽
靬ÊÌÍæé ò ‹!©ƒ')Ì ® '±¬.(
©`Ý>ÝJ݃¬­ÝèUæ‡‹æ Õ ® ǒ‹é Õ ® Ç698BF„æb‹ ß é"->QB, 8Þ懋 ® é"(‰0Qp<BR斋ÇN
ég‹ ® Ç[SDG
枋 ® 'D=鞋Z'D(˜Ý@8:-HQ=ILRW;6)RJ,ˆ©yºhÌÍ».¬"‹›©yœ=Ì ® mp¬T698BFÖ©yœ=Ì ® 'f¬Ê(
©`Ýèï­¬PóS)-H,^-HQh69-&鰋›æÕ ® Lj‹!éÂÕ ® Ç6)8BFžæZ‹! ß é°ILRPIEFB, 8£->IL;69C-HS¨©`Ý>ÝJ݃¬.uRJS-HQh6i-&YŽ,AD,.-
,ÊÎ=6);r->CE«R>6)aˆ,7€B6)ILGJRj©yºhÌÍ».¬.(
‰0Qp<BRWYe,jA),r-+ôŸ€h6)ILGJR M©ªºhÌÍ».¬~‹!© ® mBÌHmp¬ÊÂ©ðÇNÌ 'f¬.Â© ® ')Ì ® Ç)¬.·©ƒ')Ì ® '±¬.Â©ªœ=Ì ® mp¬.Â©ªœ=Ì ® 'f¬.(
m=(+¡S±Y–aˆ6)8£«ôi?èFBILADI_-T8p<Ba3w, GJRP69G>,&-HQ=, G>,jR><=;ÍQ“-HQB69-õ
©ª6£¬-HQB,­FBIEADIE->RTS)U,6);ÍQ“8¤<=a35,rG6)G>,j6)CEC5U`G>S)a‘-HQ=,jRJ,r-jöp'DÌÊÇNÌHä=ÌHmBÌÊop÷pM
©`3À¬:6)8£«:FBILADI_-Ž->Qh69-+6)€B€w,6)GJR0IL8-HQ=,j8p<BaX35,rG+69€B€5, 6)G>R69-WCE,6)Rƒ-W-@YWIL; ,ø
©ªêÎB69aˆ€BCE,)õ~ÇDÇ)oDÇDoDÇXIERW6)8[6)FBaILR>RJIL3BCE,&8p<Ba3w, GrBYWQBILCE,jÇDÇDÇN'±ä)äILR08BS)- (ù¬
sutwvyxzf{`tw|~}
d¤IL8B;r,7,6);ÍQ“FBILADI_-TS¤; ;r<BG>R+69-WCL, 6)RJ-0-@YWIE; ,9BYŽ,jQh6D,­U`SDCLCES±YWIL8BA€wSDRJR>IL3=ILCLI_-HIE, R õ
íhú ‰0QBG>,r,ˆFBIEADIE->RPS¤; ;r<BG7-@YWIL; ,Ÿ, 6);ÍQ (Ÿ¯Ö,Ÿa6«’;ÍQ=SNSDRJ,Ÿ-HQBGJ, ,FBILA)IE-HR+U`G>SDa§öp'DÌÊÇNÌÍäNÌHmBÌÊo¤÷IL8
û o ‹›'±œYT6«¤R (ÝèU˜,69;ÍQ’SN;r; <BGJRP,ÊÎB69;r-HC_«-@YWIL;r,)5->QB,j8¤<=a35,rGPS)U
R><B;ÍQ69FBaˆIER>RJIL3BCE,­ôi?@FBIEADIE-
äDü
8p<BaX35,rG>RWILR ô=ý
ÇNý¤ÇNýpÇNý
þ '±œj‹‡šDœDœ Ä
î ú ‰TYeSFBIEADIE->RjS¤; ;r<BG­-HQBGJ, ,ˆ->ILa, Rj,6);ÍQu(:¯Ö,; 6)8Œ;ÍQBS¤SDRJ,Ç[F=ILADI_-HR­IL8 û oÇ ü ‹™'±œ[YŽ6«¤R (
¡, 8B;r,j->QB,­8p<Ba3w, GWS)U6)FBaILR>RJIL3BCE,&ôi?èFBILA)IE-T8p<BaX35,rG>RILR
ô=ý '±œj‹–Ç)œ)œ
ä=ýpäNý˜þ Ä
ÿ ú á 8=,F=ILADI_-
S¤; ; <=G>R"U`SD<BG
-HIEaˆ,rR"6)8BFŸ-HQ=,+S)->QB, G
-@YWIE; ,9(
¯¢,6)GJ,0;ÍQBSNS)R>IL8=Aj-@YeS^F=ILADI_-HR˜6)AD6)IL8 
YWQ=IL;ÍQ:;6)8[3w,7FBSD8B,­IL8ž'±œXYŽ6«¤R ( ‰0QB,^-@YeSˆF=ILADI_-HR06)GJ,7IL8£-H,rG>;ÍQh698BAD,693BCL,9(e¡, 8B;r,7-HQB,­FB,rR>ILGJ, F
8p<BaX35,rGS9U
6)F=aˆIER>R>IE3BCL,Pôi?@F=ILADI_-T8p<Ba3w, GJR+IER
Ç þ mBô=ý¤ýǤý˜þ '±œ^‹gäDœ)œ Ä
 ú åIL8h6)CECE«ˆ6)CEC FBILA)IE-HRT6)G>,&->QB,­RH6)a,)("‰0QB,rG>,j6)G>,^oR><=;ÍQ8p<Ba3w, GJR (
‰0Qp<BRW-HQ=,­->S)-Í69C 8p<Ba3w, GWS)U6)FBaILR>RJIL3BCE,P8p<Ba3w, GJR+IERWšDœDœW½ÃÇ)œDœ½ÃäDœ)œW½Ëoj‹Z'm£œDoN(
o¤( ‘-HGH69€5,rà IL<=a /&1X2^O˜IE8ŒYWQBIL;ÍQÞ/P1¸IER€h6)G>6)CLCE, C"-HSž2jO"ILRIE8BR>;rG>IE35,rF„IL8Þ6’; IEG>;rCL,YWI_-HQ
;r, 8£-HGJ, ˆ(+dN<=€B€5S)R>,X->QB,XFBIL6)ADSD8B6)CLRW/&2Š6)8=F1O™S)U-HQB,j-HGH69€5,rà IL<=a IE8£-H, GJR>,r;r-76i-­†4u6)8BF
X† ‹–ÇN(
©ª6£¬ÝèUVW/&†‡1âIERXô)œDq ·FB,r->, G>aIL8=,)ÂYWIE-HQ¨€BGJSNS)Uэ->QB,ˆF=I 5, G>,r8B; ,ˆ35,.-@YŽ,r, 8Œ->QB,ˆCE, 8BA9-HQBR­S)U
-HQB,­€h6)G>6)CLCE, C5R>ILF=, R (
©`3À¬ÝèU"V0/7†‡OILRWôDœ)q BÅh8BF:-HQB,­FBI u,rG>,r8B; ,­3w,r-@Ye, , 8“-HQB,­CL,r8BA)->QBR0S)U·-HQB,­€h6)G>6)CLCE, C5R>ILF=, R (
sutwvyxzf{`tw|~}
d¤<B€B€wSDR>,XVW/&†‡1›‹–ô)œDq (~‰0QB, 8“/7†‡1‘6)8BF“2X†‡Oâ6)G>,­,rפ<=ILCK6i-H, G>6)Cu->G>IK698BADCL,rR (  G>6± Y X‚
€w, GJ€5,r8BFBIE; <BCL6)G&-HS“1O(L©ªRJ, ,ˆåILAB(_'f¬PóS)-H,-HQB69
- † 3BILRJ, ;.-HRXVW/&†‡1âRJS“-HQB69-XV X†‡‚ ‹
ä
)ä œ q (e¡+, 8=; , X‚”‹ †n¼DÇj‹!'D("Ýè-U`SDCLCESfYWRe-HQh69-+‚ž† ‹ † Õ ® ^‚ Õ ‹ ä=("¯¢,^6)CLRJS
S)3BR>,rGJD,­-HQB69-
/&1 ® 2jO‘‹…1Ÿ† ® †‡Ob‹g1‚b½°‚ž† ® ©ªO‚ ® ‚ž– † ¬~‹–Ç)‚ž†gÌ
RJIL8B;r,j‚§IERT-HQB,­aILFN?è€5SDIE8£-TS)U1O(~¡+,r8B; ,j/&1 ® 2jOc‹‡Ç ä=(
D C
D C
M
M

K K P
A B
Q
O
O

A B

Fig. 2
d¤<B€B€wSDR>,[V0/7†‡O ‹™ôDœDqŸRJSž->Qh69-V0/7†g1 ‹Ð'fÇ)œ)q (  G>6Y
 ->QBG>SD<=ADQ™€h6)G>6)CLCE, C"-HS
Fig. 1

/P2”©³YWIE-HQ‘SD8/P1 698BFcS)8ž1O¬.(ˆ©`R>, ,åILAB( ÇD¬PA£6)IE8† 3=ILR>,r;r->RjVW/&†‡1lR>S-HQB69-


V ^† ‹ V X † ”‹ ôDœ q (‰0Qp<BRX†âIER6)8„,rפ<=ILCK6i-H, G>6)C˜-HGJIK6)8BA)CL,)(’¡+, 8=; ,FBIL6)aˆ,.-H,rG
€w, GJ€5,r8BFBIE; <BCL6)Gˆ->S„1O 6)CER>SŒ3=ILR>,r;r->R:†(ɉ0QBILRA)IED,rRO† ‹j1(–Ý@8n->QB,-HGJIK6)8=ADCL,rR
O†‡26)8BFŒ1  ·Ye,Qh6),1 ‹bO†4˜VWO†‡2 ‹™'fÇ)œ)qX‹lVW1 
6)8BFŒVO2^† ‹
V j 1 ©`€BG>S)€5,rGJ-@«žS)U0;.«¤; CLIE;פ<B6)FBG>IECK69->, G>6)C³¬Ê(¡+,r8B; ,O†‡2 6)8BF¢1  6)GJ,ˆ;rSD8BADGJ<B, 8£-XR>S
->Qh69-XO2‹1 (:‰0Q¤<=RX/&1 ® O2â‹b/ ([óS)-H,-HQh6i-X/ â‹ ‚ lRJIL8B;r,‚’/  lILR^6
€B6)GH6)CECL,rCLSDADG>6)a[(W¦Ž<=-­ ‚ cILR+-@YWIE; ,X->QB,Ÿ6)C_-HI_-H<BFB,^S)U˜-HGJIK6)8=ADCL , ^†4(­dNIE8B; , † ‹›ÇN5->QB,
69CE-HI_-H<BF=,&S)U ^† IERÇ þ 䣼)Çj‹ ä=(˜‰0QBIERWADIE), RW!/ ¿‹–Ç ä=(
²v³z±´pµi|·¶Nz±´žsutwvyxzf{`tw|~}
ÁR>IE8BA­R>SDa,0-HG>IEADSD8BS)aˆ,.-HGJ«))Ye,+; 6)8ˆA),r-"R>S)CL<=->ILSD8BR
U`S)G˜35S)->Q->QB,W€h6)Gƒ-HR"R>IEa<BCE-H6)8B,rSD<BR>C_«D(*Â,.-
‚Œu•…3w,X->QB,aILFN?è€5SDIE8£-HRWS)U~/&1698BFž2^OâG>, RJ€5,r;r->IED,rCE«)(j‰0Q=, 8¨•0Ìʆg"Ì Ì͂ 6)GJ,; SDCECLIE8B,6)G
©`R>,r,ŸåILA=( ä6)8BFåILAB( mp¬.(*,r-7VW/&†‡‚ ‹¿¹B©y‹!VWO†‡•~¬.u6)8B F X† ‹ZÏh(7d¤IL8B;r,X/7†g1l6)8BF
2^†‡O 6)G>,:RJILaILCK69Gj->G>IK698BADCL,rR "IEU&†‡O ‹¸†‡2 ‹âÚn-HQ=, 8ˆg/¸‹¸†‡1 $ ‹ #¤ÚnU`S)GŸR>SDa,
€wSDRJIE-HI_D,7;rSD8BRJ-H6)8£- #5(
óS±Yn†g‚” ‹ #¤Ú+; S)RÀ¹Np†‡•Þ‹—ÚP;rSDRÀ¹NDRJS&-HQh6i%- X‚â' ‹ &(#¤ÚP;rSDRÀ¹ ® )Ï &f6)8BF X•„‹¥ÚP;rSDRÀ¹À½Ïh(
+CLRJS/P‚” ‹ #¤ÚPRJIL8¹Ÿ6)8BF“O•„‹¥ÚPRJIL8¹N("ÁR>IE8BA
/P‚ Õ  ½ X‚ Õ ‹g!/  Õ ‹‡*O  Õ ‹‡O• Õ  ½ X• Õ Ì
Ye,­AD,r-
# Õ Ú Õ RJIL8 Õ ¹W½g+© #¤Ú+; SDRÀ¹ ® Ϥ¬ Õ ‹¥Ú Õ RJIL8 Õ ¹W½‡©³ÚP;rSDRÀ¹W½ÃϤ¬ Õ Ä

m
L
D C
L
x x D C
x x

M kx θ M kx

A K
kx θ kx B
O
O

A B
K

d¤ILa€BCLI_Åh;69->ILSD8ADI_D, R
Fig. 3 Fig. 4

©+# Õ ® 'f¬@Ú Õ ‹gÇiÚwÏw©+#­½…'f¬=;rSDRÀ¹ Ä


d¤IL8B;r,
#^½—'jknœ==Ye,^AD,.-^©+# ® 'f¬@Ú:‹‡Ç9Ïe; S)RÀ¹N(˜‰0Qp<BR
/&1 ® 2jOc‹–ÇB©ª/P‚ ® •"O¬¸‹ Ç=©+#¤Ú+R>IE8W¹ ® ÚPRJIL8W¹D¬
‹ Ç=+© # ® 'f¬@ÚPRJIL8W¹
‹ mDÏe; S)Rh¹
R>IE8W¹
‹ Ç9ÏeR>IE8WÇ)¹ Ä
ÝèUTV0/7†‡1â‹cô)œDq -HQB,r8„Ç)¹‹‘ôDœDqr(ÝèUTV0/7†‡OЋcô)œDq -HQB,r8ŒÇ9¹:‹'fÇ9œDq (Ý@8Ö,rIE->QB, G­;69R>,
RJIL8Ç)¹j‹ 䣼DǤ(˜ÝèU˜Ï‹gÇNB-HQ=, 8[/&1 ® 2^Oc‹–Ç äNhIE8:35S)->Q-HQB,­;69R>, Rr(
ôN(+ҘGJSf),j->Qh69- õ
©ª6£¬:oÈ oW- ½ , oW- ½ . o¤M
©`3À0 ¬ /k /½ , /½ . /NM
©`;±¬ã¨k ã½ , ã½ . ãU`SDGW69CLCuIE8£-H, A), G>RW ã 1—š=(
sutwvyxzf{`tw|~}
¯Ö,Qh6),:©ðÇ Ä ÇD¬ Õ ‹¿m Ä /9m[ÈÉoNuRJS-HQh69- ok¿Ç Ä ÇN(7¡, 8B;r, . o:k Ç Ä Çk›' Ä mB 69RŸ©ƒ' Ä mp¬ Õ ‹
' Ä šDôŸÈnÇ Ä Ç¤(˜‰0QB, GJ,rU`SDGJ, , ok . ok–' Ä mB(%PFBF=IL8BABNYŽ,jAD,r-
o½ , o0½ . oknÇ Ç½—' mP½—' mX‹‡o
Ä Ä Ä Ä
¯Ö,­SD3BRJ, GJ),j->Qh69- äÈËä=2 , /^‹‡Ç698B F . /È , /^‹‡ÇN(˜‰0Qp<BR
/½ , /½ . /Èn䏽ËÇW½ËÇ­ ‹ 34 È /Ä
d¤<B€B€wSDR>,Ÿ5 ã 1Zš=(7‰0QB,r8¨ã5Õ 1¿š)㝍 R>S->Qh69-75 ã 1Zä ã(7‰0QBIER&ADIE), R 7
ã 6¿ã ¼)ä=(7‰0QB,rG>,rU`S)G>,
ã. žÈ , ãžÈ  ã 6¥ã¼)ä=(
¯Ö,7-HQp<BRSD3=-H6)IL8
ぽ , ã½ . ãžÈÉ©ªã ¼)䣬½g©ªã¼9䣬½‡©`ã¼)䣬˜‹…ã Ä

879 œ 879

o

     !"# $&%

')(+*,.-0/&132546,&7)897): ;<-=,.>?7)8A@CBD, E-=F=GH7)8A@CBD,)IJBH,.-0KLNMO46,+-=PA,RQGHES>?T6UCGH8V-NW"U)X12YZ/&1!F[, W[T\,]:.>


-=GD^), BD_)(`*,]-0-=PA,aT6, F=T6, 8<EAGH:];ABb7)FcXdF=UCQeMO-NU/R2!7)8AE9-=PA,aT6, F=T6, 8<EAGH:];ABb7)Ff7g-h1!-=U12iQ, ,.-
GD8kjl(cm`F=Un^C,o-NPA7g-+jKGHWh, pJ;q7gBr-NUs-NPA,o:]GHF=:];AQs>?FN7gEAGH;AWfU)X/R12( tu'wvwx
y{z6|~}w€dz6c‚ *,]-„ƒ 46,…-NPA,†:]GHF[: ;AQs>
:], 8V-NF[,3U)X/&12(ˆ‡)UCGD8ƒ‰KYŠƒ3j‹7)8AE
ƒ‰M(ŒŽ,…W[PAUn-=Pq7g-‘13K9ƒ3j GHW7 A
F[, :.-’7)8A@)BH,)(“”-•XdUCBHBDUw–W-=Pq7g-K—j ˜
13ƒ ˜ ™Yš-=PA,!:]GHF[: ;AQs>?F=7)EAGH;<W›U)X
K

/R12(
œ 4AW[, F^C,-NPq7g-‰ž–j132Ÿ˜ ž–j¡2˜
¢)£C¤ ( ¥ˆ, 8A:],¦12§¡j GHW¨7e:]_S: BDGH:
F

pJ;q7gEAF=GDBb7g-=, FN7gB©( ªhPJ;AW¦ž–¡j1 ˜ N O

'«C£ ¤k¬ ž–12o/3( ­f;Z-®ž–1ƒ‰/ ˜


¯)žˆ12o/ 7)8<E ƒ‰M 4AGDW=,]:]-NW°ž–1ƒ‰/‰( B
¥ˆ, 8A:],ž–1ƒ3M˜žˆ132§/‰(—ŒŽ,-NPJ;AW D C
U)4<-’7)GD8
ž–M§j1›±²ž–1ƒ3M³˜´žˆ¡j•1³±°žˆ132‰¡µ˜5'n«C£ ¤ ¶
ªhP<GHWoGHQTABHGD, Wˆ-NPq7g-‰1—LNƒL’MhL’j·7gF=,: UC8S>?:]_S:]BHGH:g(¥+, 8<: ,—žˆ1M§ƒŸ˜ žˆ1j•ƒ(­f;<-§UC4<W=, F^C,
-=Pq7g-0žˆ13M‰ƒO˜¨¢C£ ¤ W[GH8A:],&ƒ3M´GDW¸T6, F[T\,]8AEAGD: ;ABH7)Fc-=U3/R1—(`ªhPJ;AW–ž–1jƒO˜³¢)£ ¤ ("¹ZGD8A: ,Rj1
7g8AEºƒ3K7)F=,aT6, F[T\,]8AEAGD: ;ABH7)F0-NU12YqGD-hXdUCBDBHUn–Wf-=Pq7g-ˆ1K9ƒ‰j»GHWˆ7sF=,]:]-’7g8A@CBH,g(
¼|½n¾J¿ÀÂÁZn¾…yrz6|Ã}Äw€dz6c‚ Œl,s:7)8„7)BDW=U@C,.-&-NP<,s: UC8A:]BH;AW[GHUC8;<W=GH8<@9-NF=GD@CUC8AU)Q,.-NF[_)( œ 4<W=, F^C,
-=Pq7g-žˆj•M‰1‹˜žh/&M§¡ ˜Ÿ¢C£ ¤h¬ žh/3Ic7)8AEšž–1j•Mµ˜»'n«C£ ¤–¬ ž–1µW[GH8A:],912§¡j GHW‰7
:._S: BHGD:apJ;q7)EAF[GHBb7À-N, F=7)B©(Å+W[GH8A@s-NP<,§W[GH8A,Æ>?F[;ABH,aGD8k-=PA,a-NF[Gb7)8A@)BH,&13M‰j„Y
j1 ˜ 1M ¶
W[GH8ˆž–jM§1 W[GH8ˆž–1M§j
ªhP<GHWhF=,]EA;A:], W+-=U
j1i˜È¯Ç :]W[GHUC8–Wq2 / ˜³™: UCWŠ/ ¶
­¸;<-ˆ1K ˜›ÉSÊC¯§˜›™‘W=GH8h/‰("ªhPS;<W
j•K—Ëf˜³j13˱°13K—Ë0˜›™&Ë ¶
ªhP<GHWh@CGÌ^C, Wˆj•K͘³™(
¯S(+m`F[Uw^),§-=Pq7g-h-=PA, F[,‰,ÆÎ<GDW[-–-?¸UGD8<Ïq8AGÌ-N,aW=,]pJ;A, 8A:], WЩÉCÑVҝÑÀÓqÔh7g8AE…Ð~ՒÑCҝÑÀÓqÔfUgXT6UCW=GÌ-NGÌ^C,aGH8V-=, @C,]F=W
W[;A:’P•-=Pq7g-h-=PA,aXdUCBHBDUw–GD8A@:]UC8AEAGÌ-NGHU)8AWhPAUCBDEW=GHQ3;ABD-N7)8A, U);AW=BÌ_\Ö
× GbØ'‰Ù²ÉAÔ0ÙÚÉ Ë ÙÛÉVÜ+Ù´ÝnÝnÝ]I
× GHGbØ9ÉVÑÙÚՒÑÙ²É ÑË Y<XdUCF–7gBHBrޑ߮'CI
× GDGHG½ØÉVÑ ¬ '‰EAGÌ^SGHEA,]W–Õ’Ñ ¬ 'CYAXdUCF–7)BDB{ޑ߮'CI
× GD^<ØÉ ÑË ¬ '‰EAGÌ^SGHEA,]W–Õ ÑË ¬ 'CYAXdUCF–7)BDB{ޑ߮'C(
ut 'n¢Àx
y{z6|~}w€dz6c‚ *,]-+;AW&BHUSUCà7À-+-=PA,3TAF[UC4ABD, QŸU)X
ÏA8AEAGH8<@-?¸U9T\U)W=GD-=GD^),‰GD8J-=, @C,]F=WRɊLNÕoW[;A:’Pá-NPA7g-
'oÙÛÉÙÚÕRÙÛÉ Ë YŠÉ ¬ 'oEAGD^SGHE<, WˆÕ ¬ '§7)8AE•É Ë ¬ '‰EAGÌ^SGHEA,]W–Õ Ë ¬ 'C(`ªhPJ;AW–f,§Pq7^C,
Õ ¬ 'R˜³â × É ¬ 'wØ.L 7)8AE Õ Ë¸¬ '&˜¨ã × É Ë¸¬ 'wØ ¶
ä BDGHQGH8q7g-=GH8A@‰Õ+XdF[UCQ -=PA, W[,‰,]pS;A7g-NGDUC8AW YA¸,§@C,]-
× â ˸¬ ãÃ؝És˜´â Ë"¬ ¯Câo±°ã ¶
ªhPJ;AWˆGD-fXdUCBHBDUw–W0-=Pq7g-
ɘ â Ë â ¬ Ë ¯C¬ âoã ±°ã ˜i' ¬ ¯ â × â Ë ¬¬ ããÃØ ¶
Œl,R8<, , EkÉ-=U4\,R7)89GD8JÜ -=, @C,]F (cåfPAUSUCW[,oâ Ë ¬ 㘳¯§W=U3-=Pq7g-hɘi'±lã ¬ ☛â Ë ¬ â ¬ '&7)8AE
Õ&˜¦â × É ¬ 'wر®'3˜æâ ¬ â Ë ¬ ¯C≱³'C(&Œl,07)8V-§Éç!'–PAGH:’PGDWa7)W=W[;AF=,]E‘GDX`¸,s:’PAUZU)W=,
â‘ß!èZ(3éˆUw!ÉÙOՉGDW&, pJ;AGÌ^)7gBH, 8V-a-=U × â Ë ¬ 'wØ × â ¬ ¯CØaçi£—Ü –P<GH:’P„7)@V7)GD8áGHW&7)W=W[;AF=,]ElUC8<: ,
âßOè<(a“”-aGDWR, 7)W[_•-NUW=,],3-NPA7g-oÕÙ®É Ë GHW+, pJ;AGÌ^)7gBH, 8V-&-=Ukâ × â ¬ èVâ Ë ±ÚêJØRç´£7)8AEá-NP<GHWRGDW
7gBHW=U-=F=;A,aXdUCF–7gBHBÄâßëè<(`ªhPJ;AW–f,§EA,]ÏA8A,
ÉVÑ ˜ × Þ±²¯)Ø Ë"¬ × Þ±°¯CØ ¬ '+˜›Þ Ë ±°è)Þ±ë'CL
ÕNÑ ˜ × Þ±²¯)Ø Ü ¬ × Þ±°¯CØ Ëc¬ ¯ × Þ±²¯Cرë'&˜ëÞ Ü ±²vgÞ Ë ±íì)Þ±¨'CL
XdU)Fhބ߮')(`ªhPA, 8ºf,oW=,],§-=Pq7g-
'§ÙÚÉCÑÙÛÕNÑÙ²Õ ÑË L
XdU)F–7)BHB\ބ߮')(cî•UCF=,]Un^C, F
ÉVÑ ¬ '&˜ëÞ × Þ±íèVØ.L ÕNÑ ¬ '+˜›Þ × Þ±°èVØ × Þ±°¯CØ
7g8AE
É ÑË ¬ 'R˜¨Þ × Þ±°èVØ × Þ±ë'wØ × Þ±²¯CØÆL Õ ÑË ¬ 'R˜¨Þ × Þ±°èCØ × Þ±°¯CØ × Þ±¨'wØ × Þ Ë ±šêCÞ±²¯)Ø ¶
ªhPJ;AW–¸,§PA7n^),37Tq7)GDF0U)XïEA, W[GHF[, EºW=, pJ;A,]8A: ,]WЩÉCÑCҖ7g8AE…Ð~ՒÑCÒ](
èZ(R¹S;AÜTAT6UCW=,§É97)8<EÕ&7gF=,aF=, 7)B8J;AQ46, F[WˆW[;A:’P•-=Pq7g-–-NP<,§F[UZUg-NWhU)XÂ-NPA,o:];A4AGH:o,]pS;A7g-NGDUC8
ÉCð ¬ ð Ë ±íÕNð ¬ 'R˜›£7)F[,‰7gBHBrT6UCW=GÌ-NGÌ^C,&F[,7)B8J;AQ46, F[W (¸m`F=Un^C,o-NPq7À-Ö
× Gbب£sÙÛè)ÉSÕ+ñ®' 7)8<E × GHGbØëÕ+ß®ò èZ(
tu'n¢Àx
y{z6|~}w€dz6c‚ *Ä,.-+ó¸YAô"Y<õ46,&-NP<,§F[UZUg-NW–U)XÂ-NPA,o@CGÌ^C, 8k,]pS;A7g-NGDUC8r(`ŒŽ,oPq7^),
ó±šô±†õº˜ É' L óïô—±íôõ±†õ{ó‘˜ ÉÕ L óÂôÄõ˜ É' ¶
“”-hXdU)BHBHUn–W0-=Pq7g-ˆÉqL’Õ&7)F[,aT\U)W=GD-=GD^),)(
Œl,o-=PJ;AW+U)4<-’7)GD8
èCÕ ˜¨è × óÂô9±šôõ±šõ\óØhñ × ó±íô9±šõ{؝Ëf˜ ' L
É ÉË

¯
–P<GH:’Pº@CGD^), Wˆ£sÙÛèCÉJÕRñ®'C(cî•U)F=, Un^), F
Õ Ë ˜ × óïô9±íôõ±šõ\ó
ØöË
ÉË
˜ ó Ë ô Ë ±íô Ë õ Ë ±šõ Ë ó Ë ±²¯góïôõ × ó±šô±šõ{Ø
˜ ó Ë ô Ë ±íô Ë õ Ë ±šõ Ë ó Ë ± É ¯ Ë ¶
ªhPJ;AW
Õ Ë ¬ ¯ ˜³ó Ë ô Ë ±íô Ë õ Ë ±íõ Ë ó Ë ß ' × óÂô9±šôõ±šõ\óØ Ë ˜ Õ Ë ¶
ÉË è èCÉ Ë
ªhP<GHWhGHQTABDGH, W¸-NPq7g-ˆè × Õ Ë ¬ ¯CØ0ßëÕ Ë UCF–Õ Ë ßëèZ("¥+, 8<: ,‰Õ&ß³ò èZYA-=PA,o: UC8<: BH;<W=GHU)8kXdU)BHBHUn–W](
ê<(+÷ïGH8AEá-=PA,8S;<Q4\,]FaU)Xf7)BHBìÀ>?EAGD@CGD-R8q7g-=;AFN7)B8S;<Q4\,]F=WoW=;<:’P„-=Pq7g-a-NP<,W[;AQ»U)Xc-NP<, GHFREAGH@CGÌ-NW&GDW
'£7g8AEk,7):’P•U)Xï-=PA,oEAGH@)GD-NW0£<YD')Yø¯ZYùèsUZ:]: ;AF[Wˆ7g-ˆBH,7gW[-–UC8A:],oGH8-NPA,]Q•( tu'êgx
y{z6|~}w€dz6c‚ Œl,§UC4AW=,]F[^),§-=Pq7g-+£R±ë'¸±Ú¯–±°è˜›ì<(f¥ˆ, 8A:],‰-=PA,oF=, Q7)GD8AGH8A@-?¸U—E<GH@CGÌ-NWhQ;AW-
7g: : U);A8V-ˆXdU)F–XdU)F0-NPA,oW=;<QŸêA("ªhPAGHW–GHW–T6UCW=W[GH4ABD,&–GD-=P•ê3˜³£ˆ±íê3˜i'¸±²è‰˜´¯–±²¯Z("ªhPJ;AW+¸,
W[, ,-NPq7À-§-=PA,EAGH@)GD-NWaGD8„7)8V_‘W=;A:’PŽìÀ>?EAGD@CGD-R8J;AQ46, F§Q;AW[-§46,sXdF=UCQ»U)8A,U)Xf-=PA,: UCBDBH, :.-NGDUC8AW Ö
ú £ZL'CLƯZLNè<L’£<LNêZûJY ú £ZL'CLƯZLNè<L'CL’èSûaUCF ú £<L'CL’¯ZL’è<LƯSLƯSûJ(
åfU)8AW=GDEA, F3-=PA,9: 7)W=,GD8Ž–PAGH:’P…-=PA,9EAGD@CGD-=W37gF=,—XdF[UCQ-NPA,:]UCBHBD, :]-=GHUC8 ú £<L')LƯZL’è<LN£<LNê<ûJ(•¥+,]F=,
£lUZ:]: ;AF[W—-?–GH:],‘7)8AE²-=PA,áEAGD@CGD-=W'CYü¯ZYùè<YùêlUZ:]: ;AF—UC8A: ,, 7):’Pr(O­f;Z-k£Ž:7g8A8AU)-46,-=PA,ÏqF[W[-
E<GH@CGÌ-(
¥ˆ, 8A:],&-NPA,&ÏAF=W[-0EAGH@CGÌ-fQ3;AW[-h46,aUC8A,RU)X"'CYü¯ZYüèZYùêA(c¹S;ATAT6UCW=,&¸,&Ï<Î'&7)W0-NPA,+ÏqF=W-–EAGH@CGÌ-(
ªhP<, 8-=PA,38J;AQ46, F+U)XcìÀ>”EAGH@CGÌ-–8J;AQ46, F=W+GH8º–PAGH:’P-=PA,3F[, Q7)GH8<GH8A@vEAGD@CGD-=Wˆ7)F=,‰£<Yù£<Yø¯SYüè<Yýê—GDW
vSþøÊC¯Sþ˜ŸìC£Z(á¹<7gQ,GHWo-NPA,:7)W[,—–GD-=PŽU)-=PA, F‰EAGH@)GD-NW]Ö¯ZYùè<YýêA(kªhPS;<W-NPA,8S;<Q4\,]F3U)XhìÀ>?EAGD@CGD-
8J;AQ34\,]F=W–GH8–PAGD:’Pº-=PA,oEAGH@CGÌ-NWh£ZYD'CYü¯ZYùè<Yù£<YùêUS: :];AF–GHW–ìC£sÿáꉘ´¯gêC£<(
¹S;ATAT6UCW=,-=PA,E<GH@CGÌ-NW7)F=,ºXdF=UCQ -=PA,:]UCBHBD, :]-=GHUC8 ú £<L')LƯZL’è<L 'CL’èZûJ(ÛªhPA,8S;<Q4\,]F—U)XaìÀ>?EAGD@CGD-
8J;AQ34\,]F=W§4\,]@CGH8A8<GH8A@k–GD-=P°'GHW‰vZþüÊC¯Zþ˜ìC£<(—ªhPA,8J;AQ34\,]F3U)X¸-NPAUCW[,—46, @CGD8A8AGD8A@–GD-NP…¯kGDW
vSþøÊ × ¯ZþüØ × ¯Sþø؈˜eè)£º7g8AE-NPA,8J;AQ46, FaU)Xc-NP<UCW=,46, @CGD8A8AGH8<@9–GD-NP‘èGDW§vZþøÊ)¯Zþr˜ ìC£<(3ªhPS;<Wo-=PA,
-=U)-’7gBˆ8J;AQ34\,]F—GH8š-NP<GHW:7)W[,áGHWìC£s±®èC£s±®ìC£í˜ 'nv)£<(  BÌ-N, F[8q7g-=, BD_)Y¸¸,á:7g8Ú7)BDW=UŽ:]UC;A8V-
GÌ-º7)WkXdUCBHBDUw–W]ֆ-NPA,„8J;AQ46, FºU)XsìÀ>”EAGH@)GD-8J;AQ46, F[W•U)8A,l: 7)8³U)4<-’7)GD8ÛXdF=UCQ -=PA,l:]UCBHBD, :.-NGHU)8
ú £ZL'CLƯZLNè<L'CL’èSûs–GD-NPŽ£7)BDW=U•7)W‰7•T6UCW=W[GH4ABD,ÏAF=W[-3EAGH@)GD-oGHW‰ì<þüÊ × ¯SþøØ × ¯ZþüØo˜'n«C£<Iï-NPA,8J;AQ46, F
UgXˆìÀ>?EAGD@CGD-§8J;AQ46, F[WsUC8A,9: 7)8†UC4<-N7)GH8„XdF=UCQ‹-NP<,9: UCBDBH,]:]-NGDUC8 ú £ZL'CLƯZLNè<L'CL’èSû—GD8Ž–PAGH:’Pí£GDW
-=PA,&ÏqF[W[-–EAGD@CGD-fGHWˆvZþüÊ × ¯SþøØ × ¯ZþüØ`˜›èC£Z(`ªhPS;<W–-=PA,o8J;AQ46, FhU)XïìÀ>”EAGH@CGÌ-08J;AQ46, F[W0XdUCF=Q, E4V_9-=PA,
:]UCBHBD, :.-NGHU)8 ú £<L'CL’¯ZL’è<L')L’èZûoW=;<:’P•-NPA7g-–8AU8J;AQ46, F–Pq7)W–GD-=W0ÏqF=W-ˆEAGH@CGÌ-–£sGHWa'n«)£ ¬ è)£‰˜O'wv)£<(
÷ïGH8q7)BDBD_BHUSUCà—7g-h-NPA,§: U)BHBH,]:]-=GHUC8 ú £<L 'CLƯZL’èZLƯZLƯSûJ(¥+,]F=,o-NPA,o8J;AQ46, FˆU)XïU)X`ìÀ>”EAGH@)GD-08J;AQ34\,]F=W
GD8ᏖPAGH:’Pí'GHW&-=PA,sÏqF=W-oEAGH@CGÌ-aGHW§vSþøÊ)èZþr˜ ¯)£<IÄ-NP<,s8S;<Q4\,]FoU)X¸-=PAUCW[,PA7n^SGD8A@•¯k7gWo-=PA,ÏqF[W[-
E<GH@CGÌ-–GHWRvZþøÊ)¯ZþA˜®ìC£ZI\7g8AE-NP<,‰8J;AQ46, F+U)X
-=PAUCW=,§Pq7^SGH8A@—è—7)Wˆ-NPA,oÏqF[W[-&E<GH@CGÌ-–GHWRvZþøÊgè<þA˜O¯)£Z(
ªhPJ;AW¸-NPA,+8S;<Q4\,]FfU)XÂ7gEAQGDW=W[GH4ABD,–ìÀ>?EAGD@CGD-"8J;AQ34\,]F=W0PA,]F=,&GDW0¯g£"±„ìC£"±„¯)£‰˜5'n£)£<(
ªhPAGDWfQ7n_
7gBHW=U‰4\,RUC4<-’7gGH8A,]E—;AW[GH8A@3-=PA,&U)-=PA, F¸Q,.-NPAUSE9U)XÄ:]UC;A8V-NGD8A@AÖ`ìZþøÊ)èZþ ¬ vZþüÊ)è<þJ˜i'n¯)£ ¬ ¯)£§˜5'n£C£Z(
÷ïGH8q7)BDBD_+-NPA,f-=U)-’7gBZ8J;AQ46, FU)X6ìÀ>?E<GH@CGÌ-ï8J;AQ34\,]F=W
GD83–PAGH:’Ps, 7):’PsU)Xq-=PA,0EAGD@CGD-=W£<YÌ'CYü¯ZYùèR7)TAT6,7gF=W
7À-–BH,7gW[-–UC8A:],oGHWˆ¯gêV£ˆ±ë'wv)£–±¨'£C£§˜›êV¢C£<(
vS(ˆªhP<F=, ,f8AUC8 ], F=URF=,7gBZ8J;AQ46, F[W`ÉqL’ÕnL Ç 7)F=,¸WN7)GDE‰-NUR4\,¸GH83PA7)F=QUC8AGD:¸TAF[UC@CF[, W=W[GHUC8‰GDX É' ± ' ˜ ¯Õ (
÷ïGH8AE37)BHBV-NP<F=, ,Æ>”-=, F=Q5Pq7)F[QU)8AGH:"TAF[UC@CF=,]W=W[GHUC8AW
ɊLNÕwL Ç U)XAW[-NF[GH:.-NBD_§GD8A: F[,7)W[GH8A@+T\UCW[GD-=GD^C,cGH8VÇ -=, @C,]F=W
GD89–PAGH:’Pɘ´¯g£7g8AEÕ+EAGD^SGHE<, W Ç ( tu'wx
y{z6|~}w€dz6c‚ ¹ZGH8A:],‰¯)£<L’ÕnL Ç 7gF=,aGH8kPq7gF=QUC8AGH:RTAF=UC@)F=, W[W=GDUC8rYA¸,§PA7n^),
' ± ' ˜ ¯L
¯)£ Ç Õ
è
–P<GH:’PºF=, E<;A: ,]Wˆ-=UÕ Ç ±°¯)£CÕ ¬ Cê £ Ç ³ ˜ £<(`ªhP<GHWhQ7n_7)BHW[Us4\,a–F=GÌ-=-=, 8ºGH8-NPA,aXdUCF[Q
× êV£ ¬ Õ.Ø × Ç ±²¯)£CØ`˜³«C£C£ ¶
ªhPJ;AWa¸,Q;AW[-&Pq7^C,¯)£ÙOÕsÙOêV£9UCF]Yr, pJ;AGÌ^)7gBH, 8V-=BD_CYr£Ù®êV£ ¬ ÕsÙ!¯g£<(3*,.-o;AWa: UC8<W=GHE<, F
-=PA,aXÃ7):]-=UCF=GDWN7g-=GHUC8—U)X
«)£C£sGH8–PAGH:’PºUC8A,a-=, F=QÍGHWhBH,]W=Wh-NPA7)8•¯)£<Ö
× êV£ ¬ Õ.Ø × Ç ±°¯)£VØ"˜›«C£)£‰˜5'‰ÿ«C£C£§˜´¯sÿêV£C£§˜›êÿ¯g£C£
˜´vÿ…'ìC£§˜³«ÿŽ'n£C£§˜i'n£sÿ«)£‰˜5'nìsÿ‘v)£ ¶
Œl,a-NPJ;AW–@C,.-ˆ-=PA,oTq7)GDF=W
× ÕnL Ç Ø"˜ × èC¢<L )«)£VØ.L × èC«<L’è)«C£VØ.L × èCì<L 'n«C£VØÆL × èVvSL'êC£CØ.L × èV¯SL’«)£VØ.L × èC£<L=ìC£VØÆL × ¯wêAL’èC£)Ø ¶
 QU)8A@§-=PA, W[ , aTq7)GDF=W YV¸,RW[, ,+-=Pq7g-cUC8ABÌ_voTA7)GHF[W × èC¢<L g«C£VØ.Y × èC«<L’è)«C£VØ.Y × èCì<L'«C£VØ.Y × èVvZL' êV£VØ.Y
× èC£<L’ì)£VØXd;ABÌÏqBHBR-=PA,„: UC8AE<GD-NGDUC8ëU)X‰EAGD^SGHW[GH4AGDBHGÌ-?_\օÕEAGD^SGDEA, W Ç (ͪhPJ;AW•-=PA, F[,Ž7gF=,lv†-=F=GDTABH,]W
W=7g-NGDW[X½_SGH8<@-NPA,oF=,]pJ;AGHF[, Q, 8V-ˆU)Xï-=PA,oTAF=U)4ABH,]Q•(
ìZ(+÷ïGH8AE-NP<,38J;AQ46, FˆU)X`7gBHBrGD8J-=, @C,]F>”W=GHE<, E 

 
 
  "!o-=F=Gb7g8A@CBH,]W0–GD-=PT6, F=GDQ,.-N,]F
¯g£C£C«<( tu'nìÀx
y{z6|~}w€dz6c‚ *,]--NP<,W=GDEA, W46,ºðÄL=ðÄ"L #\Yc–PA, F[,•ðÄ$L #š7)F[,•T6UCW[GD-NGÌ^C,kGH8V-=, @C,]F=W (¨¹SGH8A:],º¸,á7)F[,
BDUZU)àSGD8A@9XdUCF§UC4<-=;AW=,Æ>ö7)8A@)BH, E‘-NF[Gb7)8<@CBH,]W %Y #†ç!ðÄ(ºî•UCF[, Un^C,]F Y"¯wð& ± #‘˜Ÿ¯)£C£)«ºW[PAUn–W‰-NPA7g- #
GDWo,]^C,]8r(k­f;<'- #…Ùið±ÚðÄYï4V_‘-NF[Gb7)8<@CBH,GD8A, pJ;q7gBHGD-?_)(ªhPJ;A(W #…Ù'n£)£)êA(ªhPS;<W‰-NP<,T6UCW=W[GH4ABD,
-=F=GDTABH,]Wï7gF=, × #6L=ðÄL=ð\Ø`˜ × 'n£C£V¯ZL’v)£Cè<LÆvg£CèVØ.Y × 'n£C£C£ZLÆv)£)êAL’v)£)êJØÆY × ¢C¢C«ZLÆv)£VvZL’v)£VvCØÆYw7)8AEW[URUC8r(ïªhPA,
@), 8A,]FN7)B
XdUCF[QµGDW × #6L=ðÄL=ð\Ø&˜ × 'n£)£)ê ¬ ¯)â\LÆv)£V¯+±›â6LÆv)£V¯&±ëâAØÆY–PA, F[,9â˜'CLƯZL’èZL ¶¶¶ LÆv)£<'C(
­¸;<-h-NPA,o:]UC8AEAGÌ-NGHU)8k-=Pq7g-h-=PA,a-NF=GH7)8A@CBD,&GHWhU)4<-N;AW[,oBH,7gEAWh-NU
× 'n£C£gê ¬ ¯CâAØ Ë çë¯ × v)£V¯h±°âAØ Ë ¶
ªhP<GHWhW=GDQT<BHGDÏA, Wf-=U
v)£C¯ Ë ±²â Ëc¬ ì × v)£V¯CØ[âçÚ£ ¶
¹SUCBD^SGH8<@-NPAGDWhpS;A7)EAFN7À-NGH:aGD8A, pJ;q7gBHGD-?_XdUCFˆâ\YA¸,oW=, ,o-NPA7g-
âÙÛv)£C¯ × è ¬ ¯ ò ¯CØ.L UCF âçëv)£C¯ × è–±°¯ ò ¯gØ ¶
¹SGH8A:],aâñ¨v)£<'CYJf,a: 7)89F[;ABH,+UC;<-f-=PA,&W[, : U)8AEkT\UCW[W=GD4AGHBDGD-?_C(ªhPJ;AWˆâÙÛv)£V¯ × è ¬ ¯ ò ¯gØ.YS–PAGH:’P
GDW–7)TATAF[UÎ<GDQ—7À-N, BÌ_9«)ì<(D' êVèV¯Z(Œl,o: UC8<: BH;<EA,a-NPq7g-Râ9ñë«Cì<(`ªhPJ;AWˆf,o@C,.-+«Cì-NF=GH7)8A@CBD, W
× #6L=ðÄL=ð\Ø`˜ × 'n£C£gê ¬ ¯Câ\LÆv)£C¯h±²â\LÆvg£V¯h±²âAØÆL â˜i')LƯZL’è<L ¶¶¶ LN«Cì ¶
ªhP<,&Bb7)W-0UC4Z-N;AW[,R-NF[Gb7)8A@)BH,ˆGH8-=PAGHW¸BHGDW[-¸GHW Ö × «CèV¯SYøv)«)«<Yøvg«C«VØ.( × “”-fGHWf, 7)W[_-NU:’P<, :’à—-=Pq7g-h«)èV¯ Ë ¬
vg«C« Ë ¬ v)«C« Ë ) ˜ )èCìsçÛ£<Y<–P<, F=,‰7)Wˆ«CèC£ Ë ¬ v)«C¢ Ë ¬ v)«C¢ Ë ˜ ¬ êV¢gêJ¯ÙÛ£<(üØ

*,+ £ ,
* +

ê
Regional Mathematical Olympiad-2009
Problems and Solutions

1. Let ABC be a triangle in which AB = AC and let I be its in-centre. Suppose


BC = AB + AI. Find ∠BAC.
Solution:

A B

We observe that ∠AIB = 90◦ + (C/2). Extend CA to D such that AD = AI.


Then CD = CB by the hypothesis. Hence ∠CDB = ∠CBD = 90◦ − (C/2). Thus

∠AIB + ∠ADB = 90◦ + (C/2) + 90◦ − (C/2) = 180◦ .

Hence ADBI is a cyclic quadrilateral. This implies that


B
∠ADI = ∠ABI = .
2
But ADI is isosceles, since AD = AI. This gives

∠DAI = 180◦ − 2(∠ADI) = 180◦ − B.

Thus ∠CAI = B and this gives A = 2B. Since C = B, we obtain 4B = 180◦ and
hence B = 45◦ . We thus get A = 2B = 90◦ .

2. Show that there is no integer a such that a2 − 3a − 19 is divisible by 289.


Solution: We write

a2 − 3a − 19 = a2 − 3a − 70 + 51 = (a − 10)(a + 7) + 51.

Suppose 289 divides a2 − 3a − 19 for some integer a. Then 17 divides it and hence
17 divides (a − 10)(a + 7). Since 17 is a prime, it must divide (a − 10) or (a + 7).
But (a + 7) − (a − 10) = 17. Hence whenever 17 divides one of (a − 10) and (a + 7),
it must divide the other also. Thus 172 = 289 divides (a − 10)(a + 7). It follows
that 289 divides 51, which is impossible. Thus, there is no integer a for which 289
divides a2 − 3a − 19.
3. Show that 32008 + 42009 can be wriiten as product of two positive integers each of
which is larger than 2009182 .
Solution: We use the standard factorisation:

x4 + 4y 4 = (x2 + 2xy + 2y 2)(x2 − 2xy + 2y 2 ).

We observe that for any integers x, y,

x2 + 2xy + 2y 2 = (x + y)2 + y 2 ≥ y 2 ,

and
x2 − 2xy + 2y 2 = (x − y)2 + y 2 ≥ y 2.
We write 4 4
32008 + 42009 = 32008 + 4 42008 ) = 3502 + 4 4502 .
Taking x = 3502 and y = 4502 , we se that 32008 + 42009 = ab, where
2 2
a ≥ 4502 , b ≥ 4502 .

But we have 2 182


4502 = 22008 > 22002 = 211 > (2009)182 ,
since 211 = 2048 > 2009.

4. Find the sum of all 3-digit natural numbers which contain at least one odd digit
and at least one even digit.
Solution: Let X denote the set of all 3-digit natural numbers; let O be those
numbers in X having only odd digits; and E be those numbers in X having only
even digits. Then X \ (O ∪ E) is the set of all 3-digit natural numbers having at
least one odd digit and at least one even digit.The desired sum is therefore
X X X
x− y− z.
x∈X y∈O z∈E

It is easy to compute the first sum;

X 999
X 99
X
x = j− k
x∈X j=1 k=1
999 × 1000 99 × 100
= −
2 2
= 50 × 9891 = 494550.

Consider the set O. Each number in O has its digits from the set {1, 3, 5, 7, 9}.
Suppose the digit in unit’s place is 1. We can fill the digit in ten’s place in 5 ways
and the digit in hundred’s place in 5 ways. Thus there are 25 numbers in the set
O each of which has 1 in its unit’s place. Similarly, there are 25 numbers whose
digit in unit’s place is 3; 25 having its digit in unit’s place as 5; 25 with 7 and 25
with 9. Thus the sum of the digits in unit’s place of all the numbers in O is

25(1 + 3 + 5 + 7 + 9) = 25 × 25 = 625.

2
A similar argument shows that the sum of digits in ten’s place of all the numbers in
O is 625 and that in hundred’s place is also 625. Thus the sum of all the numbers
in O is
625(102 + 10 + 1) = 625 × 111 = 69375.
Consider the set E. The digits of numbers in E are from the set {0, 2, 4, 6, 8}, but
the digit in hundred’s place is never 0. Suppose the digit in unit’s place is 0. There
are 4 × 5 = 20 such numbers. Similarly, 20 numbers each having digits 2,4,6,8 in
their unit’s place. Thus the sum of the digits in unit’s place of all the numbers in
E is
20(0 + 2 + 4 + 6 + 8) = 20 × 20 = 400.
A similar reasoning shows that the sum of the digits in ten’s place of all the numbers
in E is 400, but the sum of the digits in hundred’s place of all the numbers in E
is 25 × 20 = 500. Thus the sum of all the numbers in E is

500 × 102 + 400 × 10 + 400 = 54400.

The required sum is

494550 − 69375 − 54400 = 370775.

5. A convex polygon Γ is such that the distance between any two vertices of Γ does
not exceed 1.

(i) Prove that the distance between any two points on the boundary of Γ does
not exceed 1.
(ii) If X and Y are two distinct points inside Γ, prove that there exists a point
Z on the boundary of Γ such that XZ + Y Z ≤ 1.

Solution:

(i) Let S and T be two points on the boundary of Γ, with S lying on the side
AB and T lying on the side P Q of Γ. (See Fig. 1.) Join T A, T B, T S. Now
ST lies between T A and T B in triangle T AB. One of ∠AST and ∠BST is
at least 90◦ , say ∠AST ≥ 90◦ . Hence AT ≥ T S. But AT lies inside triangle
AP Q and one of ∠AT P and ∠AT Q is at least 90◦ , say ∠AT P ≥ 90◦ . Then
AP ≥ AT . Thus we get T S ≤ AT ≤ AP ≤ 1.
Q F
T Z2
P E

A S B C Z1 D

Fig. 2
Fig. 1

3
(ii) Let X and Y be points in the interior Γ. Join XY and produce them on
either side to meet the sides CD and EF of Γ at Z1 and Z2 respectively. WE
have

(XZ1 + Y Z1 ) + (XZ2 + Y Z2 ) = (XZ1 + XZ2 ) + (Y Z1 + Y Z2 )


= 2Z1 Z2 ≤ 2,

by the first part. Therefore one of the sums XZ1 + Y Z1 and XZ2 + Y Z2 is
at most 1. We may choose Z accordingly as Z1 or Z2 .

6. In a book with page numbers from 1 to 100, some pages are torn off. The sum of
the numbers on the remaining pages is 4949. How many pages are torn off?
Solution: Suppose r pages of the book are torn off. Note that the page numbers
on both the sides of a page are of the form 2k − 1 and 2k, and their sum is 4k − 1.
The sum of the numbers on the torn pages must be of the form

4k1 − 1 + 4k2 − 1 + · · · + 4kr − 1 = 4(k1 + k2 + · · · + kr ) − r.

The sum of the numbers of all the pages in the untorn book is

1 + 2 + 3 + · · · + 100 = 5050.

Hence the sum of the numbers on the torn pages is

5050 − 4949 = 101.

We therefore have
4(k1 + k2 + · · · + kr ) − r = 101.
This shows that r ≡ 3 (mod 4). Thus r = 4l + 3 for some l ≥ 0.
Suppose r ≥ 7, and suppose k1 < k2 < k3 < · · · < kr . Then we see that

4(k1 + k2 + · · · + kr ) − r ≥ 4(k1 + k2 + · · · + k7 ) − 7
≥ 4(1 + 2 + · · · + 7) − 7
= 4 × 28 − 7 = 105 > 101.

Hence r = 3. This leads to k1 + k2 + k3 = 26 and one can choose distinct positive


integers k1 , k2 , k3 in several ways.

———-00———-

4
Regional Mathematical Olympiad-2010
Problems and Solutions

1. Let ABCDEF be a convex hexagon in which the diagonals AD, BE, CF are concurrent
at O. Suppose the area of traingle OAF is the geometric mean of those of OAB and
OEF ; and the area of triangle OBC is the geometric mean of those of OAB and OCD.
Prove that the area of triangle OED is the geometric mean of those of OCD and OEF .

D Solution: Let OA = a, OB = b, OC = c,
E OD = d, OE = e, OF = f , [OAB] =
u x, [OCD] = y, [OEF ] = z, [ODE] = u,
d [OF A] = v and [OBC] = w. We are given
e
z y that v 2 = zx, w2 = xy and we have to
f O prove that u2 = yz.
F c C Since ∠AOB = ∠DOE, we have
v a w
b 1
u de sin ∠DOE de
x = 2 = .
x 1 ab
A B ab sin ∠AOB
2
Similarly, we obtain
v fa w bc
= , = .
y cd z ef
Multiplying, these three equalities, we get uvw = xyz. Hence

x2 y 2 z 2 = u2 v 2 w2 = u2 (zx)(xy).

This gives u2 = yz, as desired.

2. Let P1 (x) = ax2 − bx − c, P2 (x) = bx2 − cx − a, P3 (x) = cx2 − ax − b be three quadratic


polynomials where a, b, c are non-zero real numbers. Suppose there exists a real number
α such that P1 (α) = P2 (α) = P3 (α). Prove that a = b = c.

Solution: We have three relations:

aα2 − bα − c = λ,
bα2 − cα − a = λ,
cα2 − aα − b = λ,

where λ is the common value. Eliminating α2 from these, taking these equations pair-
wise,we get three relations:

(ca − b2 )α − (bc − a2 ) = λ(b − a), (ab − c2 )α − (ca − b2 ) = λ(c − b),


(bc − a2 ) − (ab − c2 ) = λ(a − c).

Adding these three, we get

(ab + bc + ca − a2 − b2 − c2 )(α − 1) = 0.

(Alternatively, multiplying above relations respectively by b − c, c − a and a − b, and


adding also leads to this.) Thus either ab + bc + ca − a2 − b2 − c2 = 0 or α = 1. In the
first case
1 
0 = ab + bc + ca − a2 − b2 − c2 = (a − b)2 + (b − c)2 + (c − a)2
2

1
shows that a = b = c. If α = 1, then we obtain

a − b − c = b − c − a = c − a − b,

and once again we obtain a = b = c.

3. Find the number of 4-digit numbers(in base 10) having non-zero digits and which are
divisible by 4 but not by 8.

Solution: We divide the even 4-digit numbers having non-zero digits into 4 classes:
those ending in 2,4,6,8.

(A) Suppose a 4-digit number ends in 2. Then the second right digit must be odd in
order to be divisible by 4. Thus the last 2 digits must be of the form 12, 32,52,72
or 92. If a number ends in 12, 52 or 92, then the previous digit must be even in
order not to be divisible by 8 and we have 4 admissible even digits. Now the left
most digit of such a 4-digit number can be any non-zero digit and there are 9 such
ways, and we get 9 × 4 × 3 = 108 such numbers. If a number ends in 32 or 72,
then the previous digit must be odd in order not to be divisible by 8 and we have
5 admissible odd digits. Here again the left most digit of such a 4-digit number can
be any non-zero digit and there are 9 such ways, and we get 9 × 5 × 2 = 90 such
numbers. Thus the number of 4-digit numbers having non-zero digits, ending in 2,
divisible by 4 but not by 8 is 108 + 90 = 198.
(B) If the number ends in 4, then the previous digit must be even for divisibility by 4.
Thus the last two digits must be of the form 24, 44, 54, 84. If we take numbers
ending with 24 and 64, then the previous digit must be odd for non-divisibility by
8 and the left most digit can be any non-zero digit. Here we get 9 × 5 × 2 = 90 such
numbers. If the last two digits are of the form 44 and 84, then previous digit must
be even for non-divisibility by 8. And the left most digit can take 9 possible values.
We thus get 9 × 4 × 2 = 72 numbers. Thus the admissible numbers ending in 4 is
90 + 72 = 162.
(C) If a number ends with 6, then the last two digits must be of the form 16,36,56,76,96.
For numbers ending with 16, 56,76, the previous digit must be odd. For numbers
ending with 36, 76, the previous digit must be even. Thus we get here (9 × 5 × 3) +
(9 × 4 × 2) = 135 + 72 = 207 numbers.
(D) If a number ends with 8, then the last two digits must be of the form 28,48,68,88. For
numbers ending with 28, 68, the previous digit must be even. For numbers ending
with 48, 88, the previous digit must be odd. Thus we get (9 × 4 × 2) + (9 × 5 × 2) =
72 + 90 = 162 numbers.

Thus the number of 4-digit numbers, having non-zero digits, and divisible by 4 but not
by 8 is
198 + 162 + 207 + 162 = 729.

Alternative Solution:. If we take any four consecutive even numbers and divide them
by 8, we get remainders 0,2,4,6 in some order. Thus there is only one number of the
form 8k + 4 among them which is divisible by 4 but not by 8. Hence if we take four even
consecutive numbers

1000a + 100b + 10c + 2, 1000a + 100b + 10c + 4,


1000a + 100b + 10c + 6, 1000a + 100b + 10c + 8,

there is exactly one among these four which is divisible by 4 but not by 8. Now we
can divide the set of all 4-digit even numbers with non-zero digits into groups of 4 such

2
consecutive even numbers with a, b, c nonzero. And in each group, there is exactly one
number which is divisible by 4 but not by 8. The number of such groups is precisely
equal to 9 × 9 × 9 = 729, since we can vary a, b.c in the set {1, 2, 3, 4, 5, 6, 7, 8, 9}.

4. Find three distinct positive integers with the least possible sum such that the sum of the
reciprocals of any two integers among them is an integral multiple of the reciprocal of
the third integer.

Solution: Let x, y, z be three distinct positive integers satisfying the given conditions.
We may assume that x < y < z. Thus we have three relations:
1 1 a 1 1 b 1 1 c
+ = , + = , + = ,
y z x z x y x y z
for some positive integers a, b, c. Thus
1 1 1 a+1 b+1 c+1
+ + = = = = r,
x y z x y z
say. Since x < y < z, we observe that a < b < c. We also get
1 r 1 r 1 r
= , = , = .
x a+1 y b+1 z c+1
Adding these, we obtain
1 1 1 r r r
r= + + = + + ,
x y z a+1 b+1 c+1
or
1 1 1
+ + = 1. (1)
a+1 b+1 c+1
Using a < b < c, we get
1 1 1 3
1= + + < .
a+1 b+1 c+1 a+1
Thus a < 2. We conclude that a = 1. Putting this in the relation (1), we get
1 1 1 1
+ =1− = .
b+1 c+1 2 2
Hence b < c gives
1 2
< .
2 b+1
Thus b + 1 < 4 or b < 3. Since b > a = 1, we must have b = 2. This gives
1 1 1 1
= − = ,
c+1 2 3 6
or c = 5. Thus x : y : z = a + 1 : b + 1 : c + 1 = 2 : 3 : 6. Thus the required numbers
with the least sum are 2,3,6.

Alternative Solution: We first observe that (1, a, b) is not a solution whenever 1 <
1 1 l
a < b. Otherwise we should have + = = l for some integer l. Hence we obtain
a b 1
a+b
= l showing that a b and b a. Thus a = b contradicting a 6= b. Thus the least

ab
number should be 2. It is easy to verify that (2, 3, 4) and (2, 3, 5) are not solutions and
(2, 3, 6) satisfies all the conditions.(We may observe (2, 4, 5) is also not a solution.) Since
3 + 4 + 5 = 12 > 11 = 2 + 3 + 6, it follows that (2, 3, 6) has the required minimality.

3
5. Let ABC be a triangle in which ∠A = 60◦ . Let BE and CF be the bisectors of the
angles ∠B and ∠C with E on AC and F on AB. Let M be the reflection of A in the
line EF . Prove that M lies on BC.

A Solution: Draw AL ⊥ EF and extend


it to meet AB in M . We show that AL =
LM . First we show that A, F, I, E are con-
cyclic. We have
L E ∠A
F ∠BIC = 90◦ + = 90◦ + 30◦ = 120◦ .
2
I
Hence ∠F IE = ∠BIC = 120◦ . Since
∠A = 60◦ , it follows that A, F, I, E are
concyclic. Hence ∠BEF = ∠IEF =
∠IAF = ∠A/2. This gives
B M C
∠B ∠A
∠AF E = ∠ABE + ∠BEF = + .
2 2
Since ∠ALF = 90◦ , we see that
∠B ∠A ∠C
∠F AM = 90◦ − ∠AF E = 90◦ − − = = ∠F CM.
2 2 2
This implies that F, M, C, A are concyclic. It follows that
∠C
∠F M A = ∠F CA = = ∠F AM.
2
Hence F M A is an isosceles triangle. But F L ⊥ AM . Hence L is the mid-point of AM
or AL = LM .
" #
n
6. For each integer n ≥ 1, define an = √  , where [x] denotes the largest integer not
n
exceeding x, for any real number x. Find the number of all n in the set {1, 2, 3, . . . , 2010}
for which an > an+1 .

Solution: Let us examine the first few natural numbers: 1,2,3,4,5,6,7,8,9. Here we see
that an = 1, 2, 3, 2, 2, 3, 3, 4, 3. We observe that an ≤ an+1 for all n except when n + 1
is a square in which case an > an+1 . We prove that this observation is valid in general.
Consider the range
m2 , m2 + 1, m2 + 2, . . . , m2 + m, m2 + m + 1, . . . , m2 + 2m.
Let n take values in this range so that n = m2 + r, where 0 ≤ r ≤ 2m. Then we see that

[ n] = m and hence " #  
n m2 + r hri
√  = =m+ .
n m m
Thus an takes the values m, m, m, . . . , m, m + 1, m + 1, m + 1, . . . , m + 1, m + 2, in this
| {z } | {z }
m times m times
range. But when n = (m + 1)2 , we see that an = m + 1. This shows that an−1 > an
whenever n = (m + 1)2 . When we take n in the set {1, 2, 3, . . . , 2010}, we see that the
only squares are 12 , 22 , . . . , 442 (since 442 = 1936 and 452 = 2025) and n = (m + 1)2 is
possible for only 43 values of m. Thus an > an+1 for 43 values of n. (These are 22 − 1,
32 − 1, . . ., 442 − 1.)

———-00———-

4
Model solutions of RMO 2012 (Mumbai region)

1. Let α be the common zero of the three given polynomials. Then

α2 + aα + b = 0; (1)

α2 + α + ab = 0; (2)
2
aα + α + b = 0. (3)
(1)-(2) yields
(a − 1)(α − b) = 0. (4)
(3)-(2) yields
(a − 1)(α2 − b) = 0. (5)
From (4) we conclude that a = 1 or α = b and from (5) we see that a = 1 or α2 = b. But
if a = 1 then the three polynomials are same as x2 + x + b, contradicting the fact that they
are different. Therefore we must have α = b and α2 = b. Thus α = α2 i.e α = 0 or α = 1. If
α = 0 then b = 0 and this is not a feasible solution since we need to find b 6= 0. Hence α = 1,
which yields b = 1 and from (1), a = −2.
Thus a = −2, b = 1 and the polynomials are x2 − 2x + 1, x2 + x − 2 and −2x2 + x + 1.
2. Observe that n2 + 3n + 51 = (n − 5)(n + 8) + 91. If 13|n2 + 3n + 51, then 13|(n − 5)(n + 8).
Therefore 13|n − 5 or 13|n + 8. Observe that

13|n − 5 ⇔ 13|(n + 8) − 13 ⇔ 13|n + 8;

Now, 21n2 + 89n + 44 = (7n + 4)(3n + 11) = {(3(n + 8) + 4(n − 5)}{2(n + 8) + (n − 5)}.
Writing n + 8 = 13m1 and n − 5 = 13m2 , where m1 and m2 are positive integers we get

21n2 + 89n + 44 = 169(3m1 + 4m2 )(2m1 + m2 ).

Therefore 169 divides 21n2 + 89n + 44.

Comments:

There were different solutions to this problem by the students. We present two such solutions.

First solution:

Observe that n2 + 3n + 51 ≡ 0 ≡ {(n − 5)2 + 26}(mod 13) ⇒ n ≡ 5(mod 13). Now


21n2 + 89n + 44 = (7n + 4)(3n + 11) ≡ 0(mod169) because 7n + 4 ≡ 39 ≡ 0(mod13) and
3n + 11 ≡ 26 ≡ 0(mod13).

Second solution:

13|n2 + 3n + 51 ⇒ 13|(n + 8)(n − 5) ⇒ n ≡ 5(mod13). Let nk = 13k + 5, k an integer, and


f (nk ) = 21n2k + 89nk + 44. Then

f (nk+1 ) − f (nk ) = 169(44 + 42k) ≡ 0(mod169) ⇒ f (nk ) ≡ f (n0 )(mod 169).

As f (n0 ) = 1014 ≡ 0(mod169) we conclude that f (nk ) ≡ 0( mod169).


5.22[x]
3. Writing x = [x] + {x} in the given equation and simplifying we obtain 25{x} = .
22[x] − 11
5{x}
As 0 ≤ {x} < 1, 1 ≤ 2 < 32 and hence the right hand side is positive. Therefore
5.22[x]
22[x] − 11 > 0, i.e [x] ≥ 2. Also, 5 < 2[x] ≤ 16. Hence there is a solution for every real
2 − 11
x with [x] ≥ 2 of which only one is rational, namely x = 14/5.
4. Since AEB = 90◦ and ED is perpendicular to AB, ED2 = AD.DB. Now

[ABE]2 = [ABC][ABH] ⇔ ED2 = HD.CD ⇔ AD.DB = HD.CD.

But triangles ADH and CDB are similar because 6 DAH = 6 BCD, 6 ADH = 6 CDB.

A B
D

Therefore, AD/CD = HD/DB, i.e AD.DB = HD.CD. Thus [ABE]2 = [ABC][ABH].


1 2 3
5. We have + + = 1 and 1 ≤ a ≤ b ≤ c. Therefore 1/a ≥ 1/b ≥ 1/c and
a b c
1 2 3 1 2 3 6
1= + + ≤ + + = ⇒ a ≤ 6.
a b c a a a a
Since a is a prime and 1 ≤ a ≤ 6, the possible values of a are 2, 3 and 5.

Case 1: a = 2.
2 3 1 2 1 1 2 3 5
a=2⇒ + = . Now < ⇒ b ≥ 5 and 1/b ≥ 1/c ⇒ = + ≤ ⇒ b ≤ 10.
b c 2 b 2 2 b c b
2 3 1
Hence 5 ≤ b ≤ 10. Substituting the possible values of b in the equation + = we obtain
b c 2
(b, c) = (5, 30), (6, 18), (7, 14), (8, 12), (10, 10) as the admissible pairs. Therefore the solutions
in this case are (a, b, c) = (2, 5, 30), (2, 6, 18), (2, 7, 14), (2, 8, 12), (2, 10, 10).

Case 2: a = 3.

Emulating the method outlined in the analysis of Case 1 we find that 4 ≤ b ≤ 7 and the
solutions in this case are (a, b, c) = (3, 4, 18), (3, 6, 9).

Case 3: a = 5.

There is no solution in this case.


In summary, the solutions are (a, b, c) = (2, 5, 30), (2, 6, 18), (2, 7, 14), (2, 8, 12), (2, 10, 10), (3, 4, 18), (3, 6, 9).
6. WSUM = 70656. Partition the set of all subsets of S into two sets -T1- consisting of those
subsets which contain the element 1 and T2 those which do not contain 1. For every subset
S2 belonging to T2, there is a unique subset S1 belonging to T1 which is {1} ∪ S2. Consider
any element a ≥ 2 of S. Let S2 be such that the element a occupies an even numbered position
in it. It will occupy an odd numbered position in S1 . The total contribution to WSUM by a
from both these subsets is 5a. The same is true if a occpies an odd numbered position in S2 .
Therefore, the total contribution of the element a to WSUM is 5a multiplied by the number
of subsets of T2 that contain a. The number of these subsets is 28 . The contribution of the
element 1 to WSUM is clearly 3 × 29 . Therefore the sum of all WSUMs is
P10
T = 3 × 29 + 5 × 28 × j=2 j = 276 × 28 = 70656.
Pa−1
Aliter: The total contribution of the element a ≥ 2 over all subsets is t = j=0 a−1

j (2 +
ξ(j))210−a a = 0. Here ξ(j) = 0 if j is odd and ξ(j) = 1 if j is even. a−1

j is the number of
10−a
j-element subsets of {1, 2, ..., a − 1} and 2 is the total number of subsets of {a + 1, a +
2, ..., 10}. Simplifying, we get t = 2.29 a + 28 a = 5.28 a.

7. O, E, and X are collinear. Join O with A, B and C. Triangles OCX and CEX are similar.

O
E
B
A
X
D

Therefore XC/XO = XE/XC, i.e XC 2 = XO.XE. But XC 2 = XB.XA. Hence


XB.XA = XE.XO implying B, A, O, E are concyclic. Therefore 6 OAB = 180◦ − 6 OEB =
40◦ . So, 6 AOB = 180◦ − 26 OAB = 100◦ .
8. Put x = 2a, y = 2b and z = 2c. The problem reduces to showing

1 8 1
+ ≤
(a − 1)(b − 1)(c − 1) (a + 1)(b + 1)(c + 1) 4
 
1 1 2a
subject to 1/a+1/b+1/c = 1. Observe that a > 1, b > 1, c > 1 and a−1 = a + ≥√
b c bc
2b 2c
(by A.M-G.M inequality). Similarly we get b − 1 ≥ √ and c − 1 ≥ √ . Multiplying these
ca ab
and taking the reciprocal we obtain

1 1
≤ . . . . (I)
(a − 1)(b − 1)(c − 1) 8

a+1 2 2 2 p
Next observe that = 1+ ≥ √ whence a + 1 ≥ 2 2(a − 1). Similarly we
p a−1 a − 1 pa − 1
obtain b + 1 ≥ 2 2(b − 1) and c + 1 ≥ 2 2(c − 1). Multiplying these yields
p √
(a + 1)(b + 1)(c + 1) ≥ 16 2(a − 1)(b − 1)(c − 1) ≥ 16 2.8 = 64.
Therefore
8 1
≤ . . . . (II)
(a + 1)(b + 1)(c + 1) 8

By adding (I) and (II) we get

1 8 1
+ ≤ .
(a − 1)(b − 1)(c − 1) (a + 1)(b + 1)(c + 1) 4

Comments:

We present another method which many students had adopted. By the A.M-G.M-H.M
inequality,

a+b+c √ 3
≥ 3 abc ≥ = 3.
3 1/a + 1/b + 1/c

Thus a + b + c ≥ 9 and abc ≥ 27. Using these two inequalities we obtain

(a − 1)(b − 1)(c − 1) = abc − (ab + bc + ca) + (a + b + c) − 1 ≥ 8,


(a+1)(b+1)(c+1) = abc+(ab+bc+ca)+(a+b+c)+1 = 2abc+(a+b+c)+1 ≥ 2(27)+9+1 = 64.

From these two inequalities we get

1 8 1
+ ≤ .
(a − 1)(b − 1)(c − 1) (a + 1)(b + 1)(c + 1) 4
Mumbai region Regional Mathematical Olympiad 2013 December 1, 2013

1. Let ABC be an isosceles triangle with AB = AC and let Γ denote its circumcircle. A point
D is on arc AB of Γ not containing C. A point E is on arc AC of Γ not containing B. If
AD = CE prove that BE is parallel to AD.

2. Find all triples (p, q, r) of primes such that pq = r + 1 and 2(p2 + q 2 ) = r2 + 1.

3. A finite non-empty set of integers is called 3-good if the sum of its elements is divisible by 3.
Find the number of non-empty 3-good subsets of {0, 1, 2, . . . , 9}.

4. In a triangle ABC, points D and E are on segments BC and AC such that BD = 3DC and
AE = 4EC. Point P is on line ED such that D is the midpoint of segment EP . Lines AP
and BC intersect at point S. Find the ratio BS/SD.

5. Let a1 , b1 , c1 be natural numbers. We define

a2 = gcd(b1 , c1 ) , b2 = gcd(c1 , a1 ) , c2 = gcd(a1 , b1 ) ,

and
a3 = lcm(b2 , c2 ) , b3 = lcm(c2 , a2 ) , c3 = lcm(a2 , b2 ) .
Show that gcd(b3 , c3 ) = a2 .

6. Let P (x) = x3 + ax2 + b and Q(x) = x3 + bx + a, where a, b are non-zero real numbers.
Suppose that the roots of the equation P (x) = 0 are the reciprocals of the roots of the
equation Q(x) = 0. Prove that a and b are integers. Find the greatest common divisor of
P (2013! + 1) and Q(2013! + 1).

————— ◦ —————
Mumbai region Regional Mathematical Olympiad 2013 December 1, 2013

1. Let ABC be an isosceles triangle with AB = AC and let Γ denote its circumcircle. A point
D is on the arc AB of Γ not containing C and a point E is on the arc AC of Γ not containing
B such that AD = CE. Prove that BE is parallel to AD.

Solution. We note that triangle AEC and triangle BDA are congruent. Therefore AE =
BD and hence ∠ABE = ∠DAB. This proves that AD is parallel to BE.

2. Find all triples (p, q, r) of primes such that pq = r + 1 and 2(p2 + q 2 ) = r2 + 1.

Solution. If p and q are both odd, then r = pq − 1 is even so r = 2. But in this case
pq ≥ 3 × 3 = 9 and hence there are no solutions. This proves that either p = 2 or q = 2. If
p = 2 then we have 2q = r + 1 and 8 + 2q 2 = r2 + 1. Multiplying the second equation by 2 we
get 2r2 + 2 = 16 + (2q)2 = 16 + (r + 1)2 . Rearranging the terms, we have r2 − 2r − 15 = 0,
or equivalently (r + 3)(r − 5) = 0. This proves that r = 5 and hence q = 3. Similarly,
if q = 2 then r = 5 and p = 3. Thus the only two solutions are (p, q, r) = (2, 3, 5) and
(p, q, r) = (3, 2, 5).

3. A finite non-empty set S of integers is called 3-good if the the sum of the elements of S is
divisble by 3. Find the number of 3-good non-empty subsets of {0, 1, 2, . . . , 9}.

Solution. Let A be a 3-good subset of {0, 1, . . . , 9}. Let A1 = A∩{0, 3, 6, 9}, A2 = A∩{1, 4, 7}
and A3 = A ∩ {2, 5, 8}. Then there are three possibilities:
• |A2 | = 3, |A3 | = 0;
• |A2 | = 0, |A3 | = 3;
• |A2 | = |A3 |.
Note that there are 16 possibilities for A1 . Therefore the first two cases correspond to a total of
32 subsets that are 3-good. The number of subsets in the last case is 16(12 +32 +32 +12 ) = 320.
Note that this also includes the empty set. Therefore there are a total of 351 non-empty 3-
good subsets of {0, 1, 2, . . . , 9}.

4. In a triangle ABC, points D and E are on segments BC and AC such that BD = 3DC and
AE = 4EC. Point P is on line ED such that D is the midpoint of segment EP . Lines AP
and BC intersect at point S. Find the ratio BS/SD.

Solution. Let F denote the midpoint of the segment AE. Then it follows that DF is parallel
to AP . Therefore, in triangle ASC we have CD/SD = CF/F A = 3/2. But DC = BD/3 =
(BS + SD)/3. Therefore BS/SD = 7/2.

5. Let a1 , b1 , c1 be natural numbers. We define

a2 = gcd(b1 , c1 ) , b2 = gcd(c1 , a1 ) , c2 = gcd(a1 , b1 ) ,

and
a3 = lcm(b2 , c2 ) , b3 = lcm(c2 , a2 ) , c3 = lcm(a2 , b2 ) .
Show that gcd(b3 , c3 ) = a2 .

1
Mumbai region Regional Mathematical Olympiad 2013 December 1, 2013

Solution. For a prime p and a natural number n we shall denote by vp (n) the power of p
dividing n. Then it is enough to show that vp (a2 ) = vp (gcd(b3 , c3 )) for all primes p. Let p
be a prime and let α = vp (a1 ), β = vp (b1 ) and γ = vp (c1 ). Because of symmetry, we may
assume that α ≤ β ≤ γ. Therefore, vp (a2 ) = min{β, γ} = β and similarly vp (b2 ) = vp (c2 ) =
α. Therefore vp (b3 ) = max{α, β} = β and similarly vp (c3 ) = max{α, β} = β. Therefore
vp (gcd(b3 , c3 )) = vp (a2 ) = β. This completes the solution.

6. Let a, b be real numbers and, let P (x) = x3 + ax2 + b and Q(x) = x3 + bx + a. Suppose that
the roots of the equation P (x) = 0 are the reciprocals of the roots of the equation Q(x) = 0.
Find the greatest common divisor of P (2013! + 1) and Q(2013! + 1).

Solution. Note that P (0) 6= 0. Let R(x) = x3 P (1/x) = bx3 + ax + 1. Then the equations
Q(x) = 0 and R(x) = 0 have the same roots. This implies that R(x) = bQ(x) and equating
the coefficients we get a = b2 and ab = 1. This implies that b3 = 1, so a = b = 1. Thus
P (x) = x3 + x2 + 1 and Q(x) = x3 + x + 1. For any integer n we have

(P (n), Q(n)) = (P (n), P (n) − Q(n)) = (n3 + n2 + 1, n2 − n) = (n3 + n2 + 1, n − 1) = (3, n − 1) .

Thus (P (n), Q(n)) = 3 if n − 1 is divisible by 3. In particular, since 3 divides 2013! it follows


that (P (2013! + 1), Q(2013! + 1)) = 3.

2
INMO-2000
Problems and Solutions

1. The in-circle of triangle ABC touches the sides BC, CA and AB in K, L and M
respectively. The line through A and parallel to LK meets M K in P and the line
through A and parallel to M K meets LK in Q. Show that the line P Q bisects the
sides AB and AC of triangle ABC.
Solution. : Let AP, AQ produced meet BC in D, E respectively.

M
P Q

B D K E C

Since M K is parallel to AE, we have ∠AEK = ∠M KB. Since BK = BM , both


being tangents to the circle from B, ∠M KB = ∠BM K. This with the fact that M K
is parallel to AE gives us ∠AEK = ∠M AE. This shows that M AEK is an isosceles
trapezoid. We conclude that M A = KE. Similarly, we can prove that AL = DK.
But AM = AL. We get that DK = KE. Since KP is parallel to AE, we get
DP = P A and similarly EQ = QA. This implies that P Q is parallel to DE and
hence bisects AB, AC when produced.
[The same argument holds even if one or both of P and Q lie outside triangle ABC.]

2. Solve for integers x, y, z:

x + y = 1 − z, x3 + y 3 = 1 − z 2 .

Sol. : Eliminating z from the given set of equations, we get

x3 + y 3 + {1 − (x + y)}2 = 1.
This factors to
(x + y)(x2 − xy + y 2 + x + y − 2) = 0.

Case 1. Suppose x + y = 0. Then z = 1 and (x, y, z) = (m, −m, 1), where m is an


integer give one family of solutions.
Case 2. Suppose x + y 6= 0. Then we must have

x2 − xy + y 2 + x + y − 2 = 0.

This can be written in the form

(2x − y + 1)2 + 3(y + 1)2 = 12.

Here there are two possibilities:

2x − y + 1 = 0, y + 1 = ±2; 2x − y + 1 = ±3, y + 1 = ±1.

Analysing all these cases we get

(x, y, z) = (0, 1, 0), (−2, −3, 6), (1, 0, 0), (0, −2, 3), (−2, 0, 3), (−3, −2, 6).

3. If a, b, c, x are real numbers such that abc 6= 0 and

xb + (1 − x)c xc + (1 − x)a xa + (1 − x)b


= = ,
a b c
then prove that either a + b + c = 0 or a = b = c.
Sol. : Suppose a + b + c 6= 0 and let the common value be λ. Then

xb + (1 − x)c + xc + (1 − x)a + xa + (1 − x)b


λ= = 1.
a+b+c
We get two equations:

−a + xb + (1 − x)c = 0, (1 − x)a − b + xc = 0.

(The other equation is a linear combination of these two.) Using these two equations,
we get the relations
a b c
2
= 2 = .
1−x+x x −x+1 (1 − x)2 + x

Since 1 − x + x2 6= 0, we get a = b = c.

2

4. In a convex quadrilateral P QRS, P Q = RS, ( 3+1)QR = SP and ∠RSP −∠SP Q =
30◦ . Prove that
∠P QR − ∠QRS = 90◦ .

Sol. : Let [Fig] denote the area of Fig. We have

[P QRS] = [P QR] + [RSP ] = [QRS] + [SP Q].

Let us write P Q = p, QR = q, RS = r, SP = s. The above relations reduce to

pq sin ∠P QR + rs sin ∠RSP = qr sin ∠QRS + sp sin ∠SP Q.



Using p = r and ( 3 + 1)q = s and dividing by pq, we get
√ √
sin ∠P QR + ( 3 + 1) sin ∠RSP = sin ∠QRS + ( 3 + 1) sin ∠SP Q.

Therefore, sin ∠P QR − sin ∠QRS = ( 3 + 1)(sin ∠SP Q − sin ∠RSP ).

Q R

Fig. 2.

This can be written in the form


∠P QR − ∠QRS ∠P QR + ∠QRS
2 sin cos
2 2
√ ∠SP Q − ∠RSP ∠SP Q + ∠RSP
= ( 3 + 1)2 sin cos .
2 2
Using the relations
∠P QR + ∠QRS ∠SP Q + ∠RSP
cos = − cos
2 2
and √
∠SP Q − ∠RSP ( 3 − 1)
sin ◦
= − sin 15 = − √ ,
2 2 2

3
we obtain √
∠P QR − ∠QRS √ ( 3 − 1) 1
sin = ( 3 + 1)[− √ ]= √ .
2 2 2 2
This shows that
∠P QR − ∠QRS π 3π
= or .
2 4 4
Using the convexity of P QRS, we can rule out the latter alternative. We obtain
π
∠P QR − ∠QRS = .
2

5. Let a, b, c be three real numbers such that 1 ≥ a ≥ b ≥ c ≥ 0. Prove that if λ is a


root of the cubic equation x3 + ax2 + bx + c = 0 (real or complex),then |λ| ≤ 1.
Sol. : Since λ is a root of the equation x3 + ax2 + bx + c = 0, we have

λ3 = −aλ2 − bλ − c.

This implies that

λ4 = −aλ3 − bλ2 − cλ
= (1 − a)λ3 + (a − b)λ2 + (b − c)λ + c

where we have used again

−λ3 − aλ2 − bλ − c = 0.

Suppose |λ| ≥ 1. Then we obtain

|λ|4 ≤ (1 − a)|λ|3 + (a − b)|λ|2 + (b − c)|λ| + c


≤ (1 − a)|λ|3 + (a − b)|λ|3 + (b − c)|λ|3 + c|λ|3
≤ |λ|3 .

This shows that |λ| ≤ 1. Hence the only possibility in this case is |λ| = 1. We conclude
that |λ| ≤ 1 is always true.

6. For any natural number n, (n ≥ 3), let f (n) denote the number of non-congruent
integer-sided triangles with perimeter n (e.g., f (3) = 1, f (4) = 0, f (7) = 2). Show
that
(a) f (1999) > f (1996);
(b) f (2000) = f (1997).
Sol. :
(a) Let a, b, c be the sides of a triangle with a + b + c = 1996, and each being a positive
integer. Then a + 1, b + 1, c + 1 are also sides of a triangle with perimeter 1999 because

a<b+c =⇒ a + 1 < (b + 1) + (c + 1),

4
and so on. Moreover (999, 999, 1) form the sides of a triangle with perimeter 1999,
which is not obtainable in the form (a + 1, b + 1, c + 1) where a, b, c are the integers and
the sides of a triangle with a + b + c = 1996. We conclude that f (1999) > f (1996).
(b) As in the case (a) we conclude that f (2000) ≥ f (1997). On the other hand, if
x, y, z are the integer sides of a triangle with x + y + z = 2000, and say x ≥ y ≥ z ≥ 1,
then we cannot have z = 1; for otherwise we would get x + y = 1999 forcing x, y to
have opposite parity so that x − y ≥ 1 = z violating triangle inequality for x, y, z.
Hence x ≥ y ≥ z > 1. This implies that x − 1 ≥ y − 1 ≥ z − 1 > 0. We already
have x < y + z. If x ≥ y + z − 1, then we see that y + z − 1 ≤ x < y + z, showing
that y + z − 1 = x. Hence we obtain 2000 = x + y + z = 2x + 1 which is impossible.
We conclude that x < y + z − 1. This shows that x − 1 < (y − 1) + (z − 1) and
hence x − 1, y − 1, z − 1 are the sides of a triangle with perimeter 1997. This gives
f (2000) ≤ f (1997). Thus we obtain the desired result.

5
INMO-2001
Problems and Solutions

1. Let ABC be a triangle in which no angle is 90◦ . For any point P in the plane
of the triangle, let A1 , B1 , C1 denote the reflections of P in the sides BC, CA, AB
respectively. Prove the following statements:

(a) If P is the incentre or an excentre of ABC, then P is the circumcentre of A1 B1 C1 ;


(b) If P is the circumcentre of ABC, then P is the orthocentre of A1 B1 C1 ;
(c) If P is the orthocentre of ABC, then P is either the incentre or an excentre of
A1 B1 C1 .

Solution:
(a)

A
B1
C1

B C

A1

If P = I is the incentre of triangle ABC, and r its inradius, then it is clear that
A1 I = B1 I = C1 I = 2r. It follows that I is the circumcentre of A1 B1 C1 . On the
otherhand if P = I1 is the excentre of ABC opposite A and r1 the corresponding ex-
radius, then again we see that A1 I1 = B1 I1 = C1 I1 = 2r1 . Thus I1 is the circumcentre
of A1 B1 C1 .
A1
A

B1

B C

C1

I1

(b)
Let P = O be the circumcentre of ABC. By definition, it follows that OA1 bisects
and is bisected by BC and so on. Let D, E, F be the mid-points of BC, CA, AB
respectively. Then F E is parallel to BC. But E, F are also mid-points of OB1 , OC1
and hence F E is parallel to B1 C1 as well. We conclude that BC is parallel to B1 C1 .
Since OA1 is perpendicular to BC, it follows that OA1 is perpendicular to B1 C1 .
Similarly OB1 is perpendicular to C1 A1 and OC1 is perpendicular to A1 B1 . These
imply that O is the orthocentre of A1 B1 C1 . (This applies whether O is inside or
outside ABC.)
(c)
let P = H, the orthocentre of ABC. We consider two possibilities; H falls inside
ABC and H falls outside ABC.
Suppose H is inside ABC; this happens if ABC is an acute triangle. It is known
that A1 , B1 , C1 lie on the circumcircle of ABC. Thus ∠C1 A1 A = ∠C1 CA = 90◦ − A.
Similarly ∠B1 A1 A = ∠B1 BA = 90◦ − A. These show that ∠C1 A1 A = ∠B1 A1 A.
Thus A1 A is an internal bisector of ∠C1 A1 B1 . Similarly we can show that B1 bisects
∠A1 B1 C1 and C1 C bisects ∠B1 C1 A1 . Since A1 A, B1 B, C1 C concur at H, we conclude
that H is the incentre of A1 B1 C1 .
OR If D, E, F are the feet of perpendiculars of A, B, C to the sides BC, CA, AB
respectively, then we see that EF, F D, DE are respectively parallel to B1 C1 , C1 A1 ,
A1 B1 . This implies that ∠C1 A1 H = ∠F DH = ∠ABE = 90◦ − A, as BDHF is a
cyclic quadrilateral. Similarly, we can show that ∠B1 A1 H = 90◦ − A. It follows that
A1 H is the internal bisector of ∠C1 A1 B1 . We can proceed as in the earlier case.
If H is outside ABC, the same proofs go through again, except that two of A1 H,
B1 H, C1 H are external angle bisectors and one of these is an internal angle bisector.
Thus H becomes an excentre of triangle A1 B1 C1 .

2
2. Show that the equation

x2 + y 2 + z 2 = (x − y)(y − z)(z − x)

has infinitely many solutions in integers x, y, z.

Solution: We seek solutions (x, y, z) which are in arithmetic progression. Let us put
y − x = z − y = d > 0 so that the equation reduces to the form

3y 2 + 2d2 = 2d3 .

Thus we get 3y 2 = 2(d − 1)d2 . We conclude that 2(d − 1) is 3 times a square. This
is satisfied if d − 1 = 6n2 for some n. Thus d = 6n2 + 1 and 3y 2 = d2 · 2(6n2 ) giving
us y 2 = 4d2 n2 . Thus we can take y = 2dn = 2n(6n2 + 1). From this we obtain
x = y − d = (2n − 1)(6n2 + 1), z = y + d = (2n + 1)(6n2 + 1). It is easily verified that

(x, y, z) = ((2n − 1)(6n2 + 1), 2n(6n2 + 1), (2n + 1)(6n2 + 1)),

is indeed a solution for a fixed n and this gives an infinite set of solutions as n varies
over natural numbers.

3. If a, b, c are positive real numbers such that abc = 1, prove that

ab+c bc+a ca+b ≤ 1.

Solution: Note that the inequality is symmetric in a, b, c so that we may assume that
a ≥ b ≥ c. Since abc = 1, it follows that a ≥ 1 and c ≤ 1. Using b = 1/ac, we get

ab+c ca+b cb−c


ab+c bc+a ca+b = = ≤ 1,
ac+a cc+a aa−b
because c ≤ 1, b ≥ c, a ≥ 1 and a ≥ b.

4. Given any nine integers show that it is possible to choose, from among them, four
integers a, b, c, d such that a + b − c − d is divisible by 20. Further show that such a
selection is not possible if we start with eight integers instead of nine.

Solution:
Suppose there are four numbers a, b, c, d among the given nine numbers which leave
the same remainder modulo 20. Then a + b ≡ c + d (mod 20) and we are done.
If not, there are two possibilities:
(1) We may have two disjoint pairs {a, c} and {b, d} obtained from the given nine
numbers such that a ≡ c (mod 20) and b ≡ d (mod 20). In this case we get a+b ≡ c+d
(mod 20).

3
(2) Or else there are at most three numbers having the same remainder modulo 20 and
the remaining six numbers leave distinct remainders which are also different from the
first remainder (i.e., the remainder of the three numbers). Thus there are at least 7
disinct remainders modulo 20 that can be obtained from the given set of nine numbers.
These 7 remainders give rise to 72 = 21 pairs of numbers. By pigeonhole principle,
there must be two pairs (r1 , r2 ), (r3 , r4 ) such that r1 + r2 ≡ r3 + r4 (mod 20). Going
back we get four numbers a, b, c, d such that a + b ≡ c + d (mod 20).
If we take the numbers 0, 0, 0, 1, 2, 4, 7, 12, we check that the result is not true for these
eight numbers.

5. Let ABC be a triangle and D be the mid-point of side BC. Suppose ∠DAB = ∠BCA
and ∠DAC = 15◦ . Show that ∠ADC is obtuse. Further, if O is the circumcentre of
ADC, prove that triangle AOD is equilateral.

Solution:

A
15
α

E
α
B C
D

Let α denote the equal angles ∠BAD = ∠DCA. Using sine rule in triangles DAB
and DAC, we get
AD BD CD AD
= , = .
sin B sin α sin 15◦ sin α
Eliminating α (using BD = DC and 2α + B + 15◦ = π), we obtain 1 + cos(B + 15◦ ) =
2 sin B sin 15◦ . But we know that 2 sin B sin 15◦ = cos(B −15◦ )−cos(B +15◦ ). Putting
β = B − 15◦ , we get a relation 1 + 2 cos(β + 30) = cos β. We write this in the form

(1 − 3) cos β + sin β = 1.

Since sin β ≤ 1, it follows that (1 − 3) cos β ≥ 0. We conclude that cos β ≤ 0 and
hence that β is obtuse. So is angle B and hence ∠ADC.

4

We have the relation (1 − 3) cos β + sin β = 1. If we set x = tan(β/2), then we get,
using cos β = (1 − x2 )/(1 + x2 ), sin β = 2x/(1 + x2 ),
√ √
( 3 − 2)x2 + 2x − 3 = 0.
√ √ √ √
Solving for x, we√ obtain x = 1 or x = 3(2 + 3). If x = 3(2 + 3), then
tan(β/2) > 2 + 3 = tan 75◦ giving us β > 150◦ . This forces that B > 165◦ and
hence B + A > 165◦ + 15◦ = 180◦ , a contradiction. thus x = 1 giving us β = π/2.
This gives B = 105◦ and hence α = 30◦ . Thus ∠DAO = 60◦ . Since OA = OD, the
result follows.
OR
Let ma denote the median AD. Then we can compute

c2 + m2a − (a2 /4) 2∆


cos α = , sin α = ,
2cma cma
where ∆ denotes the area of triangle ABC. These two expressions give

c2 + m2a − (a2 /4)


cot α = .
4∆
Similarly, we obtain
b2 + m2a − (a2 /4)
cot ∠CAD = .
4∆
Thus we get
c2 − a2
cot α − cot 15◦ = .
4∆
Similarly we can also obtain

c2 − a2
cot B − cot α = ,
4∆
giving us the relation
cot B = 2 cot α − cot 15◦ .
√ √ √
If B is acute then 2 cot α > cot 15◦ = 2 + 3 > 2 3. It follows that cot α > 3. This
implies that α < 30◦ and hence

B = 180◦ − 2α − 15◦ > 105◦ .

This contradiction forces that angle B is obtuse and consequently ∠ADC is obtuse.
Since ∠BAD = α = ∠ACD, the line AB is tangent to the circumcircle Γ of ADC at
A. Hence OA is perpendicular to AB. Draw DE and BF perpendicular to AC, and
join OD. Since ∠DAC = 15◦ , we see that ∠DOC = 30◦ and hence DE = OD/2.
But DE is parallel to BF and BD = DC shows that BF = 2DE. We conclude that

5
BF = DO. But DO = AO, both being radii of Γ. Thus BF = AO. Using right
triangles BF O and BAO, we infer that AB = OF . We conclude that ABF O is a
rectangle. In particular ∠AOF = 90◦ . It follows that
∠AOD = 90◦ − ∠DOC = 90◦ − 30◦ = 60◦ .
Since OA = OD, we conclude that AOD is equilateral.
OR
Note that triangles ABD and CBA are similar. Thus we have the ratios
AB CB
= .
BD BA

This reduces to a2 = 2c2 giving us a = 2c. This is equivalent to sin2 (α + 15◦ ) =
2 sin2 α. We write this in the form

cos 15◦ + cot α sin 15◦ = 2.

Solving for cot α, we get cot α = 3. We conclude that α = 30◦ , and the result
follows.
6. Let R denote the set of all real numbers. Find all functions f : R → R satisfying the
condition
f (x + y) = f (x)f (y)f (xy)
for all x, y in R.
Solution: Putting x = 0, y = 0, we get f (0) = f (0)3 so that f (0) = 0, 1 or −1. If
f (0) = 0, then taking y = 0 in the given equation, we obtain f (x) = f (x)f (0)2 = 0
for all x.
Suppose f (0) = 1. Taking y = −x, we obtain
1 = f (0) = f (x − x) = f (x)f (−x)f (−x2 ).
This shows that f (x) 6= 0 for any x ∈ R. Taking x = 1, y = x − 1, we obtain
f (x) = f (1)f (x − 1)2 = f (1) [f (x)f (−x)f (−x)]2 .
Using f (x) 6= 0, we conclude that 1 = kf (x)(f (−x))2 , where k =
f (1)(f (−1))2 . Changing x to −x here, we also infer that 1 = kf (−x)(f (x))2 . Com-
paring these expressions we see that f (−x) = f (x). It follows that 1 = kf (x)3 . Thus
f (x) is constant for all x. Since f (0) = 1, we conclude that f (x) = 1 for all real x.
If f (0) = −1, a similar analysis shows that f (x) = −1 for all x ∈ R. We can verify
that each of these functions satisfies the given functional equation. Thus there are
three solutions, all of them being constant functions.

6

   "!
#$ %'&)(

*,+.-0/132546/
798:6;=<0:6;02,>/
7=?.@BADC"EGFHI7KJL<'HM4N<O:P24N<=Q'2HM1G/RS/Q'QT/
UNHWVX:"UYHIZ0:PUGHIU
[ 7\:P] [ 2^ _`46/
7'UYHMZ0:1aVY<0:bRc/
^M^M/dJLHM7\>eUYHW;UXVY2,VX:PfB:P7gVYUh
i 2kjmla?.@nHMUoQ'2,1p2^M^W:P^qVX/rCsEBt i 2udl$?.Ewv)@xCyt
i{z jpl$@BAwHIUoQ'2,1N2^M^W:P^qVX/"EbF3t i{z uPl$@eF|vA}Ert
i 4~jmlaADCHMUoQ€2,1N2^M^W:P^qVX/rFb?bt i 4udlaA}?v)C"FB+
i 2gly‚`<\/dJƒVY<'2,V„HWR32^M^bVY<0:OUYHW;UYVY2,VX:Pf3:P7gVYU…2,1N:KVX1 [ :_bVY<0:P7†VY<\:O<0:6;02,>/
7‡HIU
46ˆ`4^MHM4 i H‰+Š:+W_\HMVS427 z :DHM7\UY461pH z :PZ"HM7‹2r4HW1N4^W:lp+
i{z lŒ1Y/d8:sVY<\2,V_ŽHM7KR{246V_Ž279ˆ\8:s/RaVY<0:PUX:UYH‘;UXVY2,VX:Pf3:P79VYU32^MUX/5HMf3Q€^Wˆ„VY<'2,V
VY<0:D<\:6;`2,>/
7„HMUS46ˆ`4^MHM4,+
’“€”–•—˜{“€™š
i 2gl$‚ [ Q\Qq/
UX:.2^M^'VY<\:›UYHW;3UYVY2,VX:Pf3:P7gVYUL2,1Y:.VX1 [ :+œS<0:P7s?.@xCsEB_`@3ADEbF3_\A}CsFD?
2,1Y:›HMUX/
UY46:P^M:PUŽVX1N2,Qq:PH [ frUt9HWRž„ŸY SŸN¡†ŸY¢aŸp£xŸY¤¥2,1Y:.VY<0:.frHMZ`¦ Qq/
HM79VYU/R?.@s_k@3A3_
ADCy_CsEB_EbF3_FD?§1Y:PUXQq:P46VYHW8:P^Wˆ_ VY<0:P7…J$:rUX::BVY<\2,VGž¨¢ª©§?›@"ŸYEGCyt« $£¬©
@BA›ŸYEGF­27\Z®¡‡¤‡©A}C„ŸYFD?G+

E P D
Q

F M
Z

X Y C
R
L

A K B

¯ R?›C„_'@xEB_TADF°VY<0:PfBUY:P^W8:PUL46/
7\4 [ 1.2,VL2rQT/
HI7gV.±3_\VY<0:P7„±D?†v†±}@°v‡±bA|v
±DCnv±DE¥v±DF3+ i ±‡HMUŽ/
7"VY<0:LQq:1YQq:P7\Z'HM4 [ ^M2,1 z HMUY:P46VX/1²/R:P24p<"/RVY<\:.UYHMZ\:PU+³l
´ :P7\46:L?GŸY@sŸpA›ŸYCyŸYErŸYF‡2,1N:L46/
7\46ˆ`4^MHM4S27\Z3^IHW:o/
7r2}4HW1N4^M:aJLHWVY<346:P79VX1Y:µ±3+¶›VY<0¦
:1YJ.HMUX:$VY<0:PUX:S^MHM7\:PU ?›Cy_9@eEr_9A}F)R{/1Nf¥2µVX1NHM270>
^M:_
UY2Pˆe·„¸3¹b+ i ‚k::S-«HW>0+³lGœS<\:P7
ž¨·yŸN¡‡¸²Ÿp£b¹b_JL<\:P75:6;kVX:P7\Z\:PZ_ z :P46/
f3:rVY<0:BHM79VX:1N7\2^Ž270>
^M: z HMUX:P46VX/1NUD/R$VY<\:
VX1NHI270>
^W:µ·„¸B¹‡27\Z®<0:P7'46:D46/
7\4 [ 1.2,VLVY<0:bHM7\46:P79VX1Y:G±}ºT/R·„¸B¹b+Ž»›US:P2,1N^MHM:1.±µº
^MHM:PUS/
75VY<0:eQq:1YQq:P7\Z\HM4 [ ^I2,1 z HMUX:P46VX/1µ/RŽ:P24p<…/RŽVY<0:xUYHMZ0:PUP+ ´ :P7\46:r±µºW?§vn±µº‘@
v°± º A§v°± º C¼v°± º Env­± º F3_0>
HW8`HM70>3VY<0:D46/
7\46ˆ`4^MHM4HMV½ˆ/R?GŸY@sŸpA›ŸYC„ŸYEBŸYF3+
i{z lL‚ [ Q\Qq/
UX: i 2 j lp_ i 2 u lp_ i{z j lp_ i{z u lo2,1N:bVX1 [ :+$œS<0:P7‹J$:bUX::GVY<\2,VL?›C¬v@xEwv
A F3+Ž».UNU [ f3:}VY<\2,V i 4~jml$HMUaVX1 [ :+ŽœL<0:P7„A}CHMUoQ'2,1N2^I^W:P^qVX/r?›F3+ ¯ VaRc/
^M^M/dJLUaVY<'2,V
D
VX1NHI270>
^W:PUa¸3A}C27\Z‹¸GFb?|2,1N:bUYHMfBHI^M2,1P+«œS<\HMU$>
HW8:PU
Fb¸ ¸3A FD¸¾=¸3A FGA
v v v v¥*,¿
?.¸ ¸GC ?.¸¾=¸eC ?.C
À :D/ z VY2HM7®FD¸wv)?›¸§27\Zy¸xA‡v)¸GC„+ŽœS<\HIU²R{/1N46:PULVY<\2,VSVX1NHM27\>
^W:PULA}¸e?‡27'Z
Cs¸bF¼2,1Y:‹46/
70>1 [ :P7gV+ ¯ 7Q'2,1NVYHM4 [ ^M2,1x?µA¼vCsF¼UX/5VY<\2,V i 4udlGHMUeVX1 [ :+KœL<0:
46/
7\4^ [ UYHW/
7R{/
^M^W/JLUoRc1N/
f i 2glp+$Á./dJ|2UYU [ fB:}VY<\2,V i 4 u loHIUoVX1 [ :t'H‰+Š:+W_€?µA‡v)FDC„+
À :r<'2P8:rUX::P7KVY<\2,VD?›CÂv§@eE¬vnADF3+ ¯ V}R{/
^M^W/JLUµVY<\2,VDVX1pHM270>
^W:PU}FDC‹AÃ27'Z
?}A}CÄ2,1N:46/
7\>1 [ :P79V+ ¯ 7OQ€2,1YVYHM4 [ ^M2,1BÅo?›CAƒv¼ÅoFGA}C„+O‚`HMfrHM^M2,1N^Wˆ_J$:427
UY<\/dJ¼VY<\2,V„ÅSADFD?ÆvÇÅoCs?.F3+ À :K46/
7\4^ [ Z0:„VY<'2,V„A}CÈHMUQ€2,1N2^M^W:P^.VX/É?.F
>
HW8`HM70> i 4~jmlp+
Êk+.˛:VX:1NfBHI70:²VY<\:$^W:P2UYVQq/
UYHMVYHW8:8,2^ [ :$VY2,Ì:P7 z ˆbVY<0:o:6;kQ\1N:PUYUYHW/
7eÍ9Îg¾rÏmÎg¾3Ð6Î\ÑGÒ,Í`ÏmÐ
2U Í'ŸNÏŸYЎ8,2,1Yˆ}/8:1«2^I^Qq/
UYHWVYHW8:HI7gVX:>:1NUP+-«HM7\Zb2^MUX/L2^M^gVX1NHWQ'^M:PU i Í'ŸNÏŸYÐPlRc/1JL<\HM4p<
VY<\HIUa^W:P2UXVa8,2^ [ :DHMUS2,VXVY2HI70:PZ+
’“€”–•—˜{“€™š À :D/ z UX:1Y8:bVY<\2,V
*
£†vÍ Î ¾ÓÏ Î ¾=Ð Î ÑKÒ,Í`ÏmÐLv Í}¾ÉÏ«¾ÓÐlÖÕ i ÍxÑKÏl u ¾ i ÏoѐÐPl u ¾ i ÐaѐÍ0l u Õ¿
Ê.Ô Ô
‚`HI7\46:LJ$:}2,1Y:}^W/9/Ì`HM70>GRc/1aVY<\:µ^M:P2UXV²Qq/
UYHMVYHW8:.8,2^ [ :›VY2,Ì:P7 z ˆs£x_0HWV$R{/
^M^W/dJ.U²VY<'2,V
̀ŸNÏŸYÐ}2,1Y:D70/VS2^M^q:P] [ 2^‰+ŽœS< [ Ua͵¾=Ͼе×*¾*¾ÊbvØB27'Z i ÍbѐÏl u ¾ i Ï$Ñ
ÐPl u ¾ i Ð.Ñ=Í0l u ×n*S¾|*S¾ÚًvnÊk+"œS< [ U}J²:"UX::BVY<\2,Ve£×¥Ø0+"œ 2,Ì`HM70>®Í5v*,_
Ïav¥*µ27\ZÐLvÊk_`J$:›>:V.£‡vØ0+œS<\:1Y:Rc/1N:.VY<0:}^W:P2UYV$8,2^ [ :›/R«£|HIU²Ø327\ZsVY<\HIU
HMU$24N<'HW:8:PZs/
7\^Wˆ z ˆrÍa¾…Ͼ5ÐLvØx27\Z i ͛ѨÏl u ¾ i Ï«Ñ®ÐPl u ¾ i ÐÑ®Í0l u vÊk+œL<0:
VX1NHMQ'^W:PU$Rc/1LJL<\HM4p<y£†vØr2,1N:DVY<0:1Y:R{/1Y:D>
HW8:P7 z ˆ
i ̀ŸNÏŸYÐPl²v i *,ŸP*,ŸNÊ
lpŸ i *,ŸNÊkŸP*dlpŸ i ÊkŸP*,ŸP*dlp¿
Òk+.Û:VaÜŸXÝ z :}QT/
UNHWVYHW8:µ1Y:P2^MUSU [ N4 <‹VY<'2,VSÜx¾ÝBvÊk+Œ1Y/d8:GVY<\2,V
Ü Î Ý Î i Ü Î ¾=Ý Î loÞ Êk¿
’“€”–•—˜{“€™š À :b<\2P8:GRc1N/
fÃVY<0:D»µß‹¦Öà}ßáHI70:P] [ 2^MHWV½ˆ_`VY<\2,V
Ü3¾Ý u
Ü'ÝÞ¬â v¥*,¿
Ê ã
œS< [ UoJ$:D/ z VY2HM7yÙ3äÓÜ'ÝÞ*,+ À :GJS1NHWVX:
Ü Î Ý Î0å Ü Î ¾Ý Î6æ v å Ü€Ý æ Î å Üx¾=Ý æ€å Ü u х܀Ý}¾=Ý u æ
u
v Ê å Ü'Ý æ Î å Üx¾=Ý æ ÑOÒ~Ü'Ý Õ
Ô
v Ê å Ü'Ý æ Î å ØbÑKÒ~Ü€Ý æ ¿
œS< [ o
U J$:b70::PZ‹VX/rQ\1Y/d8:bVY<\2,V
å Ü'Ý æ Î å ØbÑOÒ~Ü'Ý æ Þ*,¿
Œ [ XV VYHM70>3çev)Ü'ÝT_0VY<\HMUaHM70:P] [ 2^MHMV½ˆr1N:PZ [ 46:PUaVX/
ç Î\å ØDÑKÒ,ç æ Þ*,Ÿ
R{/1sهäƒç|ÞÆ*,+ À :…427Q\1Y/d8:5VY<\HIU"HM7ÚZ\H‘èq:1Y:P79V"Jo2Pˆ`U+ À :…427)Q [ VrVY<\:
HM7\:P] [ 2^IHWV½ˆrHI7VY<0:DR{/1Nf
Ò,çéSѐØ
ç Î ¾)*}×ÉÙ`¿
´ :1Y:5VY<0:5:6;kQ'1Y:PUYUYHW/
7)HM7VY<0:…ê$ë ’ R–246VX/1NU‹VX/ i çyÑw*dl u å Ò,ç u ¾¥Ê,ç"¾ì* æ 27'Z
å Ò,ç u ¾ÉÊ,ç}¾* æ HMUSQq/
UYHMVYHW8:DUYHM7\46:GHWVYULZ\HIUY461NHMfBHI7\27gVaC¬v­ÑLíBäÙ`+.¶›1L2,Q'Q'^Wˆ`HM70>
VY<0:D»µß®¦Öà}ßáHM70:P] [ 2^MHWVˆrVX/BVY<\:}QT/
UNHWVYHW8:}1Y:P2^MUSØDÑKÒ,ç0ŸYç0ŸYç0ŸYç\_€J$:D/ z VY2HM7
ØbÑKÒ,盾ÓÒ,ç é
ç Î å ØGѐÒ,ç æ Þ â Þ*,¿
Ø ã
Ø0+.˛/5VY<0:1Y:s:6;`HIUXV‹*ÙÙ¨^IHM70:PUeHM7KVY<\:"Q'^I270:_Ž70/¨VY<01Y::/RSVY<0:Pfî46/
7\4 [ 1Y1Y:P79V_U [ 4p<
VY<\2,VLVY<0:ˆsHI7gVX:1NUY:P46VS:6;`246VY^MˆHI7®Ê,ÙÙ
Ê3Qq/
HM79VYUpï
’“€”–•—˜{“€™š »›7gˆ"UX:V²/R²*ÙÙe^IHM70:PUHI7BVY<0:›Q'^M270:›427 z :.Q'2,1YVYHWVYHM/
70:PZ"HI7gVX/e2b'7'HWVX:
7 [ f z :1a/R«Z\HMU–ðñ/
HM79VaUX:VYU_'UN2Pˆ‹?DjPŸY?›u,ŸY? Î Ÿ¿¿¿ŸY?.òk_\U [ 4N<‹VY<\2,V
i Hcl»›7gˆKV½J$/K^MHM70:PUeHM7O:P24N<É?oó‹2,1Y:Q'2,1p2^M^W:P^²VX/¨:P24p<Ó/VY<0:1P_²Rc/1‹*…Þ°ô Þªõ
i Q\1N/d8`HMZ0:PZ_`/R46/ [ 1NUX:_«ö ?$ógö0× Ê
lpt
i HMHcl"R{/1²ô„v) ÷ øÖ_0VY<0:b^MHM7\:PU$HI7‹? ó 27\Z®?›ù2,1Y:D70/VLQ'2,1N2^M^W:P^ +
¯ RLö ?$ókö9v†úró,_«*xÞô‹ÞõT_TVY<0:P7®VY<\:GVX/VY2^7 [ f z :1L/RQq/
HM79VYUS/RŽHI7gVX:1NUY:P46VYHW/
7„HIU
>
HW8:P7 z ˆ û úróPú®ù‰_T2Uµ70/"VY<01Y::x^MHM70:PU.2,1Y:x46/
7\4 [ 1Y1Y:P79V+.œL< [ U.J$:e<\2d8:xVX/
jXü9óü ù ü ò
'7'Z"Qq/
UYHMVYHW8:DHM79VX:>:1NUaú5jPŸXú®u~Ÿ¿¿¿ŸXú®òeU [ 4p<‹VY<\2,V
ò
û úró.v¥*ÙÙ`Ÿ û úró6ú®ùTvÊ,ÙÙ
ÊkŸ
ómý j
R{/1L27®2,þB1pfB2,VYHW8:b27\UYJ²:1SVX/rVY<0:D>
HW8:P7®] [ :PUXVYHM/
7+
À :D/ z UX:1Y8:DVY<\2,V
ò ò u
u
û ú ó v â û úró ÑOÊâ û úróú®ù
ópý j ómý j ã ã
v *ÙÙ u ÑOÊ i Ê,ÙÙ
Ê
l²vÿk¿
U J$:b<\2d8:DVX/r4N<\/9/
UX:Gú j ŸXú u Ÿ¿¿¿ŸXú®òeU [ N4 <®VY<\2,V
œS< [ o
ò ò
û ú ó vw*ÙÙ`Ÿ û ú ó u  v ÿk¿
ópý j ómý j
À :D/ z UX:1Y8:DVY<\2,V ÿav
 9+o‚k/3J$:bfB2dˆsVY2,Ì:Gú5j²v  9_€UX/3VY<'2,V
ò
û úróLvÊÒkŸ û ôevÊ ò ú ó u v  9¿
ómýqu
Á./dJ§J²:BfB2dˆ…4N<0/k/
UX:rú®u3vìÿk_ú Î v­ú é v­Ø0_ú3v­úxv v­ú5j é v*,+
-«HM7\2^I^Wˆ_`J²:D427yVY2,Ì:
õ"v¥*6Ø0Ÿ i ú j ŸXú u Ÿ¿¿¿ŸXú j é lv i  9ŸNÿkŸXØ0ŸXØ0ŸP*,ŸP*,ŸP*,ŸP*ŸP*ŸP*ŸP*,Ÿ*,Ÿd*,ŸP*lpŸ
Q\1N/d8`HM70>xVY<0:D:6;0HMUXVX:P7\46:b/R$*ÙÙ"^MHM70:PU$JLHWVY<:6;0246VY^WˆyÊ,ÙÙ
ÊBQT/
HI7gVYUo/R«HM79VX:1NUX:P46VYHW/
7+
ÿk+.˛/ÓVY<0:1Y:5:6;0HMUXVVY<01Y::…Z\HIUXVYHM7\46VsQq/
UYHWVYHW8:¨1Y:P2^}7 [ f z :1NÙŸNÏŸYШU [ 4N<)VY<\2,V‹VY<\:
7 [ f z :1pU$̀_€Ï_\Ð~_€Ï¾=Ðoѐ̀_'Ў¾=ÍGÑOÏd_\Í}¾ÓÏ$ѐе27\Z®Í}¾ÓϾеR{/1Nfª2 d¦ VX:1Nf
2,1NHMVY<\f3:VYHM4›Q\1Y/>1N:PUYUYHW/
7®HI7‹UX/
f3:}/1NZ0:16ï
’“€”–•—˜{“€™š À :GUY<0/dJVY<'2,VaVY<0:D27\UXJ$:1LHM U ‹+'‚ [ Q'QT/
UY:_`HMRQq/
UYUYH z ^M:_k^W:VàŸNÏŸYÐ
z :SVY<01N::›Z\HIUXVYHM7\46VQq/
UYHWVYHM8:S1Y:P2^T7 [ f z :1NUU [ 4N<"VY<\2,VoÍ'_0Ï_kÐ~_0ÏT¾¨ÐыÍ'_`о¨ÍLÑ®Ï_
ÍS¾ÏŽÑyЛ27\ZÍa¾KϾ…Ð.R{/1Nfn 2 d¦ VX:1pfÃ2,1NHWVY<\f3:VYHI4LQ\1N/>1Y:PUYUYHW/
7sHM7UX/
f3:›/1NZ0:1d+
À :bfr2Pˆ®2UYU [ f3:}VY<\2,VLÍä ϵäÉÐ~+$œS<0:P7‹VY<0:1N:b2,1Y:D/
7\^Mˆ"VJ²/r42UY:PULJ$:b70::PZyVX/
4p<0:P4YÌTh i ¯ lÍ›¾ÏѨÐ}äÉÍsäÉÐ ¾KÍ}Ѩϛä ϵäÉеä Ï ¾ÐŽÑ¨ÍsäÉÍ.¾Ï ¾Ð›27\Z i ¯Y¯ l
Í}¾ÓÏ$ѐеäÉÍä ϛäÉЎ¾=ÍxÑKϛäÉеä ώ¾ÐaѐÍsäÉÍ}¾Óώ¾Ð~+
 !#" ‚ [ Q\Qq/
UX:}VY<0:b4p<\2HM7y/RŽHM70:P] [ 2^MHWVYHW:PUaÍD¾ÓÏoÑOÐDä͋äÚЍ¾=ÍxÑÏ}äÏDä
еä ϾÐŽÑyÍsäÉÍS¾Ï¾…Ð.<0/
^IZ\U>/k/`Z+Ž^W: V $ z :.VY<0:µ46/
fBf3/
7‹Z'H‘èT:1N:P7\46:+œS< [ U
J$:DUX::DVY<\2,V
ÐLvÍD¾ÓϾ=ÐoÑKÊ#$\Ÿ«Ïav)͵¾ÉÏ«¾ÓÐaÑKÒ#$\ŸÍ3v)Í}¾Óώ¾=ÐoÑKÿ#$'¿
»›Z\Z\HM7\>DVY<0:PUX:_`J$:›UX::›VY<\2,VaÍa¾…Ͼ5ÐSvÿ#$\+&% [ V$VY<0:P7ÍxvÙG46/
79VX1N2Z\HM46VYHI70>GVY<\:
Qq/
UYHWVYHM8kHWVˆB/RÍ'+
 '!(!#" ‚ [ Q\Qq/
UX:3VY<0:HM70:P] [ 2^MHWVYHW:PU}Íx¾Ï.ÑÓЮä­Í=ä§Ïä­Ða¾ÍrÑÉÏä­Ðyä
Ͼ=ÐoѐÍsäÉÍD¾ÓϾ=е2,1Y:DVX1 [ :+»L>
2HI7‹J²:GUX::DVY<\2,V
ÐSv)ÍD¾ÓϾ=ÐoÑKÊ#$\ŸoÏav)͵¾ÉÏ«¾ÓÐaÑ…Ø $\ŸÍ3v)ÍD¾=ώ¾=ÐoÑKÿ#$'¿
À :DVY< [ Ua/ z VY2HM7®Í}¾ÓϾ=ÐLv i ***),Ê
l+$\+oœS<\HIUo>
HW8:PU
* Ò
Í3v $\Ÿ Ïav $\ŸÐSv '$ ¿
Ê Ê Ê
Á./VX:®VY<\2,VBÍB¾ÏDÑ Ðyv Ír¾†ÏS¾ÐDÑ,#$ÓvîÑ i **),Ê
l+$\+ ´ /dJ$:8:1rJ$:y2^MUY/¨>:V
Í3¾Ï›ÑÉÐs- v  i ** ),Ê
l¾ i Ò ),Ê
lSÑ i ),Ê
l. $KvÂÑ i Ò ),Ê
l+$\+ ¯ VGR{/
^M^W/JLUGVY<\2,VxÒ/„v/
>
HW8`HM70 > $xv)Ù`0 + % [ VSVY<\HIUaHMUSHMf3Qq/
UYUNH z ^W:+
œS< [ UµVY<0:1N:B2,1Y:r70/‹VY<01N::BZ\HMUXVYHM7'46V›Qq/
UYHMVYHW8:x1Y:P2^7 [ f z :1NU}̀ŸNÏŸYÐ3U [ 4N<VY<\2,Vb̀_
Ï_€Ð~_qώ¾ÓÐaÑK̀_€Ð$¾=ÍxÑOÏ_€ÍD¾ÉÏoÑOÐD27\Z„Í}¾ ώ¾=Ð}R{/1Nf21 d¦ VX:1Nf2,1NHWVY<\fB:VYHM4
Q\1N/>1Y:PUYUYHW/
7‹HI7‹UX/
f3:D/1NZ\:1P+
u u
k+µ‚ [ Q\Qq/
UX:ŽVY<0:32 7 [ f z :1pU*,ŸNÊkŸNÒkŸ¿¿¿«Ÿ42 2,1Y:a2,1Y1p270>:PZbVX/µR{/1Nf|272 z ˆ52s2,1Y1p2Pˆ
46/
7\UNHMUXVYHM70>./6R 2s1Y/JLU27\7 Z 246/
^ [ fB7\U«U [ 4N<xVY<\2,V«VY<0:S7 [ f z :1pU HM7e:P24p<31Y/dJ i Rc1Y/
f
^W:R{VVX/e1NHW>
<9Vpl27'Zr:P24p<s46/
^ [ fB7 i R{1Y/
f°VX/QVX/ z /VXVX/
f"lŽ2,1N:›HM7"HM7\461Y:P2UNHM70>}/1NZ0:1d+
˛:P70/VX: z ˆÍ ó òDVY<0:G7 [ f z :1SHM73ô,¦ VY<y1Y/dJ|27\Z®õ9¦ VY<®46/
^ [ fr7+²‚ [ Q\Qq/
UX:}ÏñóDHMUoVY<\:
fB2~;0HMf [ f Qq/
UYUYH z ^M:"7 [ f z :13/R›:P7gVX1pHW:PUxVY<\2,VB427Ó/`44 [ 1r2UBÍ óÖó _.*Þìô  Þ 2+
Œ1Y/d8:DVY<'2,V
Ïdj¾ÓÏmuŽ¾ÓÏ Î ¾,pÏ98"Þ
2
å 2 u ÑKÒ:2s¾Óÿ æ ¿
Ò
<i ; ;`2f3Q€^W:h ¯ 7VY<0:}42UX5: 2„vÒk_0VY<0:µ/
7\^Wˆr7 [ f z :1pU²J.<\HM4N<427/`44 [ 1S2UaÍkuXu›2,1N:
Ø0_€ÿe/= 1 xUX/BVY<\2,V›ÏpuLvÒk+³l
’“€”–•—˜{“€™š ‚`HM7\46:Í,óó<\2UeVX/…:6;046::PZÓ2^M^$VY<0:‹7 [ f z :1NUeHM7OVY<0:VX/QÉ^W:RcVD ô >„ô
U [ z fB2,VX1NH‘; i :6;04^ [ Z\HI70>„HWVYUY:P^WRplp_27\Z=UYHM7\46:"VY<0:1Y:‹2,1Y:3ô u Ñ*s:P79VX1NHW:PU_J$:f [ UYV
<\2d8:sÍ,óÖó‹×‡ô u +„‚`HMfBHM^I2,1N^Wˆ_Í,óó"f [ UXVG70/VG:6;`46::PZO:P24"/RaVY<0:"7 [ f z :1pUbHM7…VY<\:
z /VXVX/
f 1pHW>
<gV i 2…Ñ…ôb¾|*d? l > i 2…Ñ…ôb¾|*dlµU [ z fB2,VX1NH‘; i /VY<0:1bVY<\27OHWVYUX:P^WRpl›27'Z
VY<0:1N:G2,1Y: i 2„Ѩôµ¾†*dl u ÑÚ*GU [ 4N<„:P7gVX1pHW:PUL>
HM8kHM7\>"Í,óóe@ Þ 2 u Ñ i 2yѨô}¾*dl u ¾†*,+
œS< [ UoJ$:bUX::DVY<'2,V
Í~óÖó5A'B9ô u Ÿ½ô u ¾)*,Ÿ½ô u ¾ÓÊkŸ¿¿¿ Ÿ42 u Ñ i 2®Ñyôµ¾)*dl u ¾)*DC¿
œS<\:S7 [ f z :1/Rq:P^W:Pf3:P79VYUHM7xVY<\HIUŽUX:VHMU&2 u Ñ i 2bÑGô¾…*dl u Ñeô u ¾yÊk+«œL<\HMUHMf3Q€^MHW:PU
VY<\2,V
Ï ó ÞE2 u Ñ i 2®Ñyôµ¾)*dl u Ñyô u ¾ÉÊbv i Ê:2"¾ÓÊ
l–ôeÑKÊô u Ñ i Ê:2yÑÉ*dlp¿
¯ VaRc/
^I^W/dJLUSVY<\2,V
8 8 8
û Ïñó Þ Ê:2"¾ÉÊ,Õ û ôbÑOÊ û ô u  Ñ 2 å Ê:2yÑÉ* æ
pó ý j Ô ómý j ópý j
2 i 2"¾)*dl 2 i 2r¾)*dl i Ê:2‹¾)*dl
v â :Ê 2s¾=Ê â ÑKÊ â Ñ2 i Ê:2yÑÓ*dl
ã Ê ã  ã
v
2 u
2 ÑK:Ò 2s¾=ÿ,Õ«Ÿ
Ò Ô
JL<'HM4N<‹HMUoVY<\:D1Y:P] [ HW1Y:PZ z / [ 7\Z+

    "!#$&%')(+*,
-/. 0214365)798:<;>=?3@=?A<BCD:ECD; 79=?3F4G9:EHIEJ=?368KGL:<CNMPO
:E=?3K7Q3RCD:<; 791?;NS
14793RC 1?TH&I'J'.
U BSS
145D:VC)W:XG9793:Y5ZIM[=?38\JNM']4^W:Y3S6;)1_86BA`:Y8$]?a':<:<C+H>J=?38'H&Ib793'cd=?368
e ;):Y5)S:YA`C)79f?:YGLg?.hi:<CNj"O
:C)W:'S147Q3RC&^W:<;):'H&M793kCD:<; 5D:YA`C)5>C)W:'G97Q3:l5):<F4a':Y3RC
c e =?368nm,O
:C)W: To1k1?Cp1?TqS
:<;)S
:Y38679A<BG9=/;rTo;)14asj143tCD1'IEJ. U W1u^vC)W6=/Cpjnm
O67Q5D:YA`C)5wpcm e .
x$y{z}|i~uoy{€+ƒ‚ ; 1_8B6A`:„H&M…CD1†a':<:<C‡IEJˆ793[‰.‹Š414793Œm„c=?38Œm e .‹Ž&; =Y^
S
:<;)S
:Y387QA<BG9=/; 5$To;)14a e =?38Ec[143ECD1IEJvCD1a':<:<C+79C793E‘=?368'’K;):Y5DS
:YA`C)7Lf?:YG9g?.
hi:<ClB58:Y31?CD:“wIlm e Okg•”=?38†wXJ>m–c—O_gK˜.š™š:‡5)W1u^›C)W6=/C'”œž˜vO_g
S; 1Ÿf 793F¡C)=?3&”œžC)=?3X˜+.K¢XW795l79a'S6GQ7L:Y5>C)W=/C\wpjnm e œ£wj¤m–cE.Q¥ U :<:t¦Z7LF4B;):
O
:YGL1u^N.¨§
A

F
D

E
P

B M K Q L C

U 7Q3A`: C)W:A`:<f 79=?35XH>‰]Ic©=?368ªJ e A`1436A<B;Y]^2:NaE=Ygt^X;7LCD:


IE‰ œ« J>c œ›® H e œ ¬
‰J ¬2­ cH « ­ e I ®¯
™š:N1?O{5D:<;)f?:NC)W=/C
e j œ"° H e j\± œ"° M e j\± œ"° H e M ±
jnc ° HcjE± ° MNcj\± ° H&cM ± ¯
² 1Ÿ^2:<f?:<;&5)C)=?38=/; 8“A`14aES6BC)=/C)7L1435793Rf?14G9f_793FlO{=?5D:Y5F47Lf?:
¬
° H e M ±
œ ¬N³ ® ° H&IM ± ­ ° H&cM ±$œ « ³ « ® ° H JNM ± ­
=?38
¬
° HIM ±
œ ® ³•«¬N³ « ° H&IEJ&± ­ ° H>J>M ±œ ® ³•¬N³ « ° H&IEJ&± ¯
¢XW_B5p^2:N1?OC)=?793 e j œ›® ³ «
j¤c ® ³´¬ ¯
µ 3tC)W:N1?C)W:<;W=?368
C)=?3”šœ mš e  œ e I´5)793pI C)=?3X˜–œ c’ œ cJš5)793J
 mš ­ m–’ m–’ ¯
¶ 5)793F e I·œ¹¸šºŸ»º ¼_½ HIn]pclJ¾œ¿¸šºŸ»
º À4½ H>J£=?368ÁH&IÂ5)7Q3IÜ,H JK)5 793J]2^2:
1?OC)=?7Q3
C)=?3” œ Ä+® ³ « Ä m–’
C)=?3˜ ® ³•¬ZÅ mš  Å
œ Ä+® ³ « Ä j¤ c
® ³•¬ZÅ e j Å
œ Ä+® ³ « ď® ³•¬ œÆ-
® ³•¬ Å ® ³ « Å ¯
™š:A`1436A<G9B8:>C)W=/C&”–œŒ˜.
Ç_.¦Z7938‡=?GQG
S; 79a':Y5ÉȪ=?368“Ê_]6=?38t:<f?:Y3¡3_BaO
:<; 5pËKÌÁÇ_]65)=/C)7Q5DTÍg 793FC)W::YÎkB=/C)79143
È6Ï ³ È{ÏRÐ$Ñ ³ƒÒ<Ò<Ò/³ È ³ -œŒÊ/Ó ³ Ê ³ -
¯
x$y{z}|i~uoy{€+ µ Okf 7L14B5 GLg>ÈœŒ Ô Ê_.r™š:^X; 7LCD: C)W795793tC)W:>To1?; a
È ¸ È6ÏkÐ$Ñ ³ È6ÏRÐ6Ó ³vÒ<Ò<Ò/³ - ½ œŒÊ¥}Ê ³ -Ÿ§
¯
Õ TʄÖÈ Ï/×ØÓ Ù -/]ZC)W:Y3ÂʄÚ[È Ï?×ØÓ =?38@W:Y36A`:\^2:¤5D:<:¤C)W=/CÊ Ó ÚŒÈ Ï .‡¢XWkB65N^2:
1?OC)=?7Q3
Ê/Ó ³ Ê'ږÈ6Ï ³ È6Ï/×ØÓ>ښÈ6Ï ³ È{ÏRÐ$Ñ ³vÒ<Ò<ÒÛ³ È
­
5)7Q3A`:NËÂÌÇ_. Õ CTo14G9GL1u^5XC)W=/C ÊEÜÂÈ Ï?×ØÓ . U 793A`:N˕ÌÇE=?38ª795=?3ª:<f?:Y3ª3_BaO
:<;Y]
ËÉÝ/Ǥ795 =¤36=/C)B; =?GZ3kB6aO
:<; G9=/;)F?:<; C)W=?3´-/.N¢XW795&79a'S{G97L:Y5C)W=/C>ʖœƒ Ô È Ï?×ØÓ Okg“C)W:
F47Lf?:Y3‡A`143687LC)7L143nC)W=/CXÊl795p=S; 79aE:?.əš:A`1436A<G9B8: C)W=/CÊ'Ü„È Ï?×ØÓ ³ -/.™š:a\=Yg
=?G95)1'^X; 7LCD: C)W:=/O
1uf?:;):YG9=/C)79143‡793tC)W:NTo1?; a
È ¸ È Ï?×ØÓ Ù - ½ ¸ È Ï?×ØÓ ³ - ½ œÆ¥¨È Ù -Ÿ§ÞÊ¥}Ê ³ -Ÿ§
¯
¢XW6795X5)W1u^5XC)W6=/CÊE879f_798:Y5E¸ßÈ Ï?×ØÓXÙ - ½ ¸àÈ Ï?×ØÓ ³ - ½ .á2BCÊ\Ü@È Ï?×ØÓ ³ -=?38ªÊ'7Q5
=S;79a':?. ² :Y3A`:NC)W:>143G9g¤S
145)5)79O679G97LCâgE795ÊNœÂÈ Ï?×ØÓ ³ -/.V¢XW67952F47Lf?:Y5
È2¸àÈ Ï?×ØÓ Ù - ½ œÆ¥¨È Ù -Ÿ§`¥}Ê ³ -Ÿ§+œÆ¥¨È Ù -Ÿ§/¸9È Ï?×ØÓ ³ Ç ½
¯
U 7Qa'S6G97Lã{A<=/C)7L143–GL:Y=?85NCD1–äÈåœÈ Ï/×ØÓ ³ Ç_.“¢XW7Q5N5)W1u^5NC)W=/Cȕ867Lf_7Q8:EÇ_.“¢XWkB65
Ètœ[Ç'=?38“W:Y36A`:NÊœæ_]˪œvç. Õ C795X:Y=?5DgtCD1Ef?:<; 7LTog¤C)W6=/CXC)W:Y5D:7Q38:<:Y8‡5)=/C)795)TÍg
C)W:>F479f?:Y3“:YÎ_B=/C)7L143$.
ä_. U W1Ÿ^C)W=/CXTo1?;:<f?:<; gn; :Y=?Gq3kB6aO
:<;Xè¤C)W:N:YÎ_B=/C)7L143
é ®6ê Ù -Yë ®ì ³ -Yë ® Ó Ù é ® ³ èEœŒí ¥Ø-Ÿ§
W=?5=/CGL:Y=?5DC143:3143 îï;):Y=?G$; 1k1?C=?368tã638tC)W:5 Bas1?Tr=?G9G
C)W:3143îï;):Y=?G$;)1_1?C)51?T
C)W:>:YÎ_B=/C)7L143q.
x$y{z}|i~uoy{€+ U BO65DC)7LC)BC)7Q3F ® œ ¬³ ¥Ø-ŸÝ/Ç4§V793¤C)W:&:YÎkB6=/C)7L143$]_^V:&1?OC)=?793nC)W:&:YÎkB6=Ûî
C)7L143‡7Q3 ¬{ð
é ¬ ê ³ ç ¬ Ó ³ è Ù Çä œí ¥ñÇ4§
¯
¶ 5)793FC)W:CD; =?35DTo1?; aE=/C)7L143 « œ ¬ Ó ]6^V:NF?:<C=EÎkB6=?8; =/C)79A :YÎ_B=/C)7L143“7Q3 « ð
é « Ó ³ ç « ³ è Ù äÇ œŒí ¥ñä4§
¯
¢XW:8795)A`;79aE793=?3kCp1?TZC)W795X:YÎ_B=/C)7L143ª795ä?Ç¥ñÇ Ù è§p^W79A W¡795314363:<F4=/C)7Lf?:79TZ=?368
143G9g¤7LTòènփÇ_.+¦1?;ènÖÁÇ_]^2:N1?OC)=?7Q3‡C)W:>;)1_1?C)5
œ Ù - ³ó Ç¥ñÇ Ù è§ œ Ù - Ù ó Ç¥ñÇ Ù è §
«Ñ ç ­ «Ó ç ¯
¦1?;F?:<CDC)7Q3F';):Y=?G ¬ ^2:>3:<:Y8 « Üåí . µ Okf 7L14B5 GLg « Ó Ú†í=?38‡W:Y3A`:>7LCpF47Lf?:Y5p143GLg
3143îï;):Y=?Gf/=?G9B:Y5+1?T ¬ .ápBC Üåí79T=?38\143GLg'7LT$ènÖ . Õ 3\C)W795+A<=?5):^2:1?OC)=?7Q3
Câ^V1;):Y=?Gf/=?G9B:Y52TÍ1?; ¬ =?368¤W« :YÑ 36A`:&Câ^V1;):Y=?G;)1_1?C)5pTo1?;2C)ìÓ W:1?; 79F4793=?G{:YÎkB=/C)79143–¥Ø-Ÿ§.
¢XW_B5^2:A`1436A<G9B8:>C)W=/CC)W:<;):l=/;):Câ^V1¤;):Y=?Gq;)1_1?C)5&=?38“Câ^V1n3143 îï;):Y=?Gi;)1k1?C)5TÍ1?;
èvÖ ìÓ =?38´To14B;E3143 îï;):Y=?G2;)1_1?C)5'To1?;EèƒÌ ìÓ . µ O_f 7L14B5)GLgšC)W:“5)Baˆ1?T&=?GQGVC)W:
;)1_1?C)51?T2C)W:\:YÎ_B=/C)7L143•795Ç_.“¦1?;è´Ö ìÓ ]òCô^21–;):Y=?G;)1_1?C)51?T¥ñÇ4§N=/;):¤F47Lf?:Y3KO_g
¬ Ñ œ ³õ « Ñ =?38 ¬ Ó œ Ù õ « Ñ . ² :Y3A`:NC)W:5 Ba1?TZ;):Y=?G$;)1_1?C)5X1?TX¥Ø-Ÿ§X795F47Lf?:Y3‡O_g
¬ Ñ ³ ÓÑ ³–¬ Ó ³ ÓÑ ^W79A W\;):Y8BA`:Y52CD1‡-/. Õ CVTo14G9GL1u^5+C)W:&5)B6a©1?TqC)W: 3143 îï;):Y=?G{;)1k1?C)5
1?T¥Ø-Ÿ§pTo1?;ènÖ ìÓ 795=?G95)1“-/.+¢XW_B5
¢XW:N5)Bas1?Tr3143;):Y=?G
; 1k1?C)5Œœsö Ç- TÍTÍ1?1?;;è¤ Ö ìÓ
è¤Ì ìÓ
ç.¦Z7938v=?G9GN÷Ÿîô87LF479Ct3_BaO
:<; 5\To1?; a':Y8ŒO_gƒB65)793F•143GLg†C)W:–867LF47LC)5‡æ•=?38d÷k]&=?368
879f_795 7LO6GL:XO_gnO1?C)Wªæ'=?38ª÷k.
x$y{z}|i~uoy{€+ 0pGL:Y=/;GLg?]C)W:šG9=?5)Ct867LF47LC¤alB5DCtO
:Kæ•=?38Œ^V:šW=Ÿf?:šCD1†8:<CD:<; aE7Q3:
C)W:;):YaE=?7937Q3F\ëE879F47LC)5<.p¦1?;>87Lf 795)7LO67QG97LCôgO_gª÷k]{7LC7955)Bø\A<7L:Y3kCXCD1nA`14365)798:<;XC)W:
3_BaO:<;i1?OC)=?793:Y8Okg>;):<S{G9=?A<793F÷XOkg>í ù4To1?;i:`ú=?a'S6G9:2æ4÷?÷/æ4÷/æ?æ 795q867Lf_7Q5)7LO6GL:òO_g÷7LT
=?38l143G9gæ/í?í4æ/í4æ?æN795Z879f_795 7LO6GL:rO_g÷k.rû+=?AWE5)BAW'3kB6aO
:<;Z795ò1?OC)=?793:Y8O_g=?886793F
5D14aE:‡1?TC)W:“3_BaO:<;5To;)14a…C)W:“5):<Ctüuæ/í æ/í?í æ/í?í?í æ/í4í?í4í æ/í?í4í?íRí æ/í4í?í4í?í?í ý
=?GL143FE^7LC)W“æ_.2™š:GL1_1?þ‡=/CXC)W:N; :YaE=?7938:<;­ 51?TZ­ C)W:Y5D:N­ ^W:Y3“8­ 7Lf 798:Y8n­O_g“÷kù{C)W:<g
=/;):\üR- ä Ç ë ç æký4.>¢XWkB657LC7Q55)Bø¤A<7L:Y3RCCD1¤A W:YA)þ“To1?;C)W145):A`14aO6793=/C)7L14365X1?T
­ ­ ­ ­ ­
;):Ya\=?7938:<; 5N^W79AW@=?88ÂBS@CD1–=¡3_BaO:<;1?TC)W:\To;)14a…Ç ³ ÷/ÿ]5)793A`:\C)W:¤G9=?5)C
879F47LC27Q5=?GL;):Y=?8gtæ_.+¢XW:Y5D:N=/; :üuÇký4]$üuä ëký4]qüYç æký4]ÉüuÇ ä ç ý4]iüR- ä æký4]ÉüR- Ç ëký4]
üuÇ ä æ ëký4]+üR- ç æ ëkýn=?38@üR- Ç ä ç ­ ëký4.'¢XW­ :Y5D:lA`1?;)­ ;):Y­5DS
1438ªCD­ 1t­ C)W:l3_B­aO
­ :<; 5
÷?÷?÷/­ æ4­÷?÷/­ æ_]$÷?÷/æ4÷/­ æ4÷/­ æ_]
­ æ?æ4÷?÷?÷?÷/æ_]q÷/­ æ4÷/­ æ?æ4­ ÷/æ_­ ]
æ4÷?÷?÷/æ?æ?æ_]$÷?÷/æ?æ4÷/æ?æ_]
æ4÷/æ?æ?æ4÷/æ_]$æ?æ?æ4÷?÷/æ?æ_]
÷/æ?æ?æ?æ?æ_.
æ_.hi:<C2H&IEJ[O
:&=CD;79=?3F4GL:^7LC)W\5 798:Y52è6] u]u.V0p1435)798:<;2=CD;79=?3F4GL:&H Ñ I Ñ J Ñ ^7LC)W
5)7Q8:Y5p:YÎkB6=?G$CD1\è ³ Ç ] ³ Ç ]  ³ èÇ . U W1u^[C)W=/C
H
Ñ I Ñ J Ñ

Ü ç  HIEJ 
­
^W:<;): °  ±i8:Y31?CD:Y5XC)W:=/;):Y='1?TòC)W:NCD; 7Q=?3F4GL:  .
x$y{z}|i~uoy{€+ Õ C795 :Y=?5)g„CD1¡1?O65D:<;)f?:EC)W6=/CNC)W:<;):E795=tCD; 79=?3F4GL:'^7LC)Wš5)798:Y5>è ³ Ç ]
 è
³ Ç ]  ³ Ç . ¶ 5)793F ² :<;)143 5TÍ1?; alBG9= ]^2:NF?:<C
-Yë ° H IEJ&± Ó œ ¥}è ³ ³ < §`¥}è ³ Ù Y §`¥ ³  Ù è §`¥ ³ è Ù <§
­
=?38
-Yë  H Ñ I Ñ J Ñ Ó œ -Yä ë ¥}è ³ ³ Y§`¥ Ù è ³ ³ äY§`¥ Ù ³  ³ ä/è§`¥ Ù  ³ è ³ ä<§
¯
U 7Q3A`:è =/;):XC)W:X5)798:Y5Z1?T$=>CD; 7Q=?3F4GL:?]?C)W:<;):=/;):S
145)7LC)7Lf?:;):Y=?G63kBaO
:<; 5{È Ê
5)B6A W‡C)W­ =/C­ èœŒÊ ³ Xœ ³ È XœÂÈ ³ Ê_. ¶ 5 793FC)W:Y5D:N; :YG9=/C)7L1435p^2:N1?OC)=?­ 793 ­
Û­  ­
H° &I'J ± Ó œ -YëÈ{Ê
H I J Ó ä¥ñÇÈ ³ Ê4§`¥ñÇ/Ê ³  §`¥ñÇ ³ È
§ ¯
Ñ Ñ Ñ  
¢XW_B57LCX7Q55)Bø\A<7L:Y3kCCD1ES; 1Ÿf?:NC)W=/C
¥ñÇÈ ³ Ê?§`¥ñÇ/Ê ³ §`¥ñÇ ³ È$§p܆Ç4÷ È{Ê
  u­
To1?;XS
145)7LC)7Lf?: ;):Y=?Gi3kBaO
:<; 5òÈ Ê . ¶ 5)793 F ‡î …7Q3:YÎkB6=?G97LCôg?] ^2:NF?:<C
­ ­ 
ÇÈ ³ ÊÜÁ䥨È{Ó<Ê4§ Ñâ× ì Ç/Ê ³ ÜÁä¥}Ê/Ó § Ñâ× ì Ç ³ ȪÜÁä¥ ÓÞÈ
§ Ñâ× ì
­   ­   ¯
ªBGLC)7LS{GLg_7Q3FC)W:Y5D:?]?^2:p1?OC)=?793lC)W:p8:Y5)7L;):Y8;):Y5)B6GLC<.əš:=?G95)11?O65):<;)f?:pC)W=/CZ:YÎkB=?GLî
7LCâgnW14G9857LTZ=?38‡143G9gn7LT{Ètœ[Ê>œ .+¢XW7957Q5:YÎkB67LfÛ=?G9:Y3RCCD1EC)W:5DC)=/CD:Ya':Y3kCC)W6=/C
H&IEJb795p:YÎ_B79G9=/CD:<; =?Gï. 

ë_. Õ 3= GL1?CDCD:<; g?]?C)79A)þ?:<C)5=/; :VF47Lf?:Y337Q3:`îô87LF47LCÉ3kBaO


:<; 5òB5)793F143GLg>C)W:p87LF479C)5V- Ç ä_.
¢XW:<g–=/; :¤=?G95D1„A`14GL14B;):Y8@;):Y8$]òO6G9B:\1?;F?;):<:Y3•793@5 BA WÂ=¡^p=YgšC)W=/ClCô^21„C)79A)­ þ?:<­ C)5
^W145D:l3_BaO
:<; 5 87 
:<; 7Q3„=?GQGiC)W:E36793:S{G9=?A`:Y5 F?:<CN87 
:<;):Y3kC A`14G914B; 5<. U BSS
145D:
C)W:+C)79A)þ?:<CÉO:Y=/;793FC)W:+3_BaO
:<;+-YÇ?Ç?Ç?Ç?Ç?Ç?Ç?Ç 795$;):Y8=?38>C)W=/CÉO
:Y=/; 793FpC)W:23kBaO
:<;
Ç?Ç?Ç?Ç?Ç?Ç?Ç?Ç?Ç'7Q5F?;):<:Y3$.Ž&:<CD:<; a\793:?] ^7LC)W\S;)1_1?Tâ]kC)W:&A`14G914B;V1?T$C)W:C)79A þ?:<CpO:Y=/;793F
C)W:N3_BaO
:<;N-YÇ?ä -YÇ?ä -YÇ?ä_.
x$y{z}|i~uoy{€+ ¢XW:To14G9GL1u^793F5):YÎkB:Y36A`:1?TiaE1Ÿf?:Y5XGL:Y=?8¤CD1C)W: A`14GL14B;21?TiC)W:&C)79A)þ?:<C
O
:Y=/; 793FlC)W:N3kBaO
:<;N-YÇ?ä -YÇ?ä -YÇ?ä ð
hi7Q3: !BaO
:<; ¢X79A þ?:<"C !B6aO
:<; 0214GL14B; #:Y=?5D143
- -YÇ?Ç?Ç?Ç?Ç?Ç?Ç?Ç ;):Y8 >7Lf?:Y3
Ç Ç?Ç?Ç?Ç?Ç?Ç?Ç?Ç?Ç F?;):<:Y3 >7Lf?:Y3
ä ä -Yä -?-Yä -?-Yä O6G9B: hÉ793:Y5N%- $©Ç
ç Ç?ä -Yä?ä -Yä?ä - F?;):<:Y3 hÉ793:Y5N%- $©ä
æ ä?ä -Yä?ä -Yä?ä - O6G9B: hÉ793:Y5N%- $©Ç
ë -YÇ?ä -YÇ?ä -YÇ?ä ;):Y8 hÉ793:Y5&ç $©æ
Õ T&-YÇ?ä -YÇ?ä -YÇ?ä–7Q5 ;):Y=?A W:Y8ÂO_g–5)14a':E1?C)W:<;;)1k1?C<]ò;):Y8KA`14G914B;alB5DCNO
:'1?OC)=?793:Y8
:<f?:Y3´=?GL143FªC)W=/Cl;)1k1?C<.„¦1?;'7LTTo1?;:`ú=?a'S6GL:„-YÇ?ä -YÇ?ä -YÇ?äšF?:<C)5lO6G9B:ETÍ; 14a5D14aE:
1?C)W:<;;)1k1?C<]6C)W:Y3‡C)W:NTo14G9GL1u^793F'5D:YÎ_B:Y3A`:NG9:Y=?85CD1\=EA`143kCD; =?87QA`C)7L143 ð
hi7Q3: !BaO
:<; ¢X79A þ?:<"C !B6aO
:<; 0214GL14B; #:Y=?5D143
- -YÇ?Ç?Ç?Ç?Ç?Ç?Ç?Ç ;):Y8 >7Lf?:Y3
Ç -YÇ?ä -YÇ?ä -YÇ?ä O6G9B: >7Lf?:Y3
ä Ç?ä -Yä -?-Yä -?- F?;):<:Y3 hÉ793:Y5N%- $©Ç
ç Ç -?-Yä?ä -Yä -?- F?;):<:Y3 hÉ793:Y5N%- $©Ç
æ ä?ä?Ç?Ç -YÇ?Ç -YÇ ;):Y8 hÉ793:Y5& ç $©Ç
ë -?-Yä -Yä?ä -Yä?ä O6G9B: hÉ793:Y5 ä $©æ
÷ ä?ä -Yä?ä -Yä?ä - F?;):<:Y3 hÉ793:Y5N%- $©Ç
é Ç?Ç?Ç?Ç?Ç?Ç?Ç?Ç?Ç ;):Y8 hÉ793: ë $©÷
¢XW_B5pC)W:NA`14G914B;X1?TÇ?Ç?Ç?Ç?Ç?Ç?Ç?Çn795p;):Y8“A`143kCD; =?879A`C)7Q3F'C)W=/C7LCX7Q5F?;);):Y3$.


  

!#"%$!&
')(+*-,.0/21$!&435,687:9
/!*-.)(+;</>=),.?/5;@BADCFEG-(+&IHKJL(<1MJONPG-Q#GLRSG-TU/V.?,#=?JL,FWX(+*-Y $5(+&:=?'0$VZ[=)JL,\')(+*-,]'%@BA8G
A2C2GCFEG
EI@^.),]'_Y ,1=)(+35,`;+a5cb-9LYLY $!'),

d /!efAEhgL(<'),1i=M'KNRj/>=lkXm
d genk\@poqk\AropkCsoqk\EmL/5&
*
d 1efQcNnt>Q0RuopCFEftVCFA8

v .)$>3!,l=?J
/>=#@BADCFEw(<'K/ xyz-{>|

[  %}


D M
C

N L
Q

A K B

Fig. 1.
~ g'_,.)3!,X=)J
/V=2NQ0R^T€(+'2/Y
/5.?/5;,;;+$5‚!.?/5WfGƒkh(+'=)J
,IWX(+*LY$!(&:=D$5ZBR^N„/V&*…JL,`&1,IT†Q‡/V;<'_$†Y/5'?'_,]'
?= JL.?$59L‚!JXkXƒˆ,`=Š‰Pg ,‹=)JL,KY $5(+&:=Š$VZ (&:=),.?'),1=)(+$5&2$VZŒ@BCq/V&
*XAE†m!/V&*D;+,=cg ,‹=)JL,‹Y $5(+&!=Š$5Z (+&:=),.?'),1=)(+$5&
$5ZƒADEw/V&
*nR^T…
"%$!&
')(+*-,.Ž=?JL,8=?.)(</V&L‚!;,fR^T†N†l$V=),I=)J
/V=XŠk‘(+'Y
/V.M/V;+;,;Š=?$T†Nu/5&
*kw(<'\=?JL,OWX(<*-Y $5(+&!=2$VZBR^NP
’ ,`&
1`,DP(+'‹=?JL,ŽWX(<*4“”Y $5(+&:=#$VZŠR^T…%b4(+&
1`,R^TU(<'KY
/5.?/5;;+,`;Œ=?$8@BC2G
(=#Z•$!;;+$>H#'0=)J
/V=#‰q(<'‹=?JL,ŽWX(+*-“”Y $5(+&:=
$5Zc@BC2F#$>Hrk–(<'K=?JL,21`(.M19
WX1`,`&:=).?,Ž$5Zc—f@BADC–/5&
*n=)J
,WX,*-(</V&AŽ‰˜Y
/!')'),'#=)JL.?$59L‚!JkX ’ ,`.?,Ž=)J
,`.?,
/5.),™=šH0$XY$:')')(g
(;+(›=?(,]'`œ

d (<ef@lA2Cr(<'#/2.?(‚!J!=‹=?.)(</V&L‚!;,™HK(=)J0@lA2Csopž5Ÿ: F/V&
*O‰¡opkXmL/5&
*
d (+(<eO‰op
¢ k‡(&nHKJL(<1MJn1`/5'),\AŽ‰s(+'‹Y ,`.?Y ,`&
*-(<19
;+/5.K=)$£@BC2

b-9LYLY $!'),20@lA2C‡osž5Ÿ: \/V&
*f‰qo^kX ~ g
'),`.?35,™=)J/>=Fk(<'K=?JL,1(+.M19LW£1,&:=).?,\$VZƒ=)JL,Ž=).?(+/5&L‚5;+,ŽEICFA¤/5&
*
J
,`&
1`,n0EICFA¥o¦ž!Ÿ! >b-(WX(+;+/5.);+a§0E8@BAo¦ž5Ÿ: >¨©=ŽZ•$!;;+$>H#'F=?J
/>=80@lEIC‘o¦ž5Ÿ: >†/V&*…@lADCFEª(+'D/
.?,1=?/5&L‚5;+,5ƒ«‹JL(<'%(+WXYL;+(,]'0=)J
/V=‹NQ%R^Tw(<'0/D.?JL$5WDgL9
' ’ ,`&
1`,™Q%Nft>Q0RuorB/V&*£=)J
(+'0‚5(+35,]'0CFE¬opCFA8
«‹J49
'l@lADCFEh(<'K/£')7:9
/5.),!
¨­&§=?JL,I'),1`$5&
*§1`/!'_,!G[$!g
'_,.)3!,2=)J
/V=DAE®(<'\Y ,`.?Y,&
*-(<19L;</V.\=?$@BC2G¯NQq(<'ŽY/V.M/V;+;,;ƒ=)$@™C¦/5&
*§Q%R°(+'
Y/V.M/V;+;,;=)$OAE ’ ,`&1,(=lZ•$!;;+$>H#'K=?J
/>=\R^Q±(<'#Y ,`.?Y,&
*-(<19L;</V.B=)$IQ%NBb-(WX(+;+/5.l.?,/5')$5&
(&L‚8')JL$>H#'#=)J
/V=
NQ%R^TU(+'#/2.?,1=?/V&
‚5;+,5
² _' (+&L‚PQ%NftVQ%R o³CFEOtVCFAIG#H%,‚5,=I=)J
/V=nCFAE„(+'O'_(+WX(;</V.£=?$´Q0R^Ns¨­&¡Y
/V.)=)(<19
;+/5.Gl0Q%R^Nµo
‹CFADEho^¶´')/·a!#b4(&1,Q0R¸(<'lY/V.M/V;+;,;y=)$IEIA8G
H0,/5;+')$£‚5,`=F0ADk\N®os¶clb-(&
1`,DNQ%R^T¥(+'B/81a-1;+(<1
7:9
/!*-.?(;</>=?,`.M/V;[H0,X/V;<'_$8‚!,=0QcT†NªoS0Q%R^Nªo‡¶c ² '_(+&L‚O=)JL,DZ/51=B=?J
/>=\AEU(+'™Y
/5.?/5;;+,`;=)$fT†NGŒH0,
‚!,=™0Q0k\Aroq0Q%T†Nwop¶%cb4(&1,ŽAEhgL(<'),1i=M'l‹CFA@G
H0,F/5;+')$XJ
/·35,Ž0NA2ksop¶%Š«‹J:9'

kŽN‘opNAropAQPo¡Q%k

/5&
*nAQ¹(<'KY
/V.M/V;+;+,`; =)$8k\NPƒ«‹J
(+'K‚!(3!,'KkŽR¸(+'KY/V.M/V;+;,;=?$8Q%C‡/V&
*

kŽRºopkŽQ…opAQPo¡Q%C

¨©=lZ•$5;+;$>H#'%=)J/>=™k\Q0C\R»(<'#/XY
/V.M/V;+;+,`;+$5‚5.M/VWn¼%9L=F0Q%C\Rºopž!Ÿ! V ’ ,`&1,2‹R˜k\Qopž5Ÿ: >c«‹JL(<'K(+WXYL;(+,'
=?J
/>=2NQ%R^T½(<'/'?7!9/V.?,5I¾l;+')$†$5g'_,.)3!,D=?J
/>=£0Q%k\N¿o¤ž!Ÿ! £/5&
*ÀJL,`&1,f‹CFAD@¦o¬0Q0k\N„oÁž5Ÿ: V
«‹J
(+'%‚5(+35,]'0‹CFE8@popž5Ÿ: #/V&*£JL,`&1,B@BADCFE¬(+'%/Ž.?,1i=M/V&L‚!;,!ƒb-(&
1`,BAD@po¡A2C2G:(=cZ•$!;;+$>H#'¯=?J
/>=‹@lA2CFE
(<'#/X'?7!9/V.?,5

ÂLKb-9LYLY $!'),KÃf(<'#/2YL.?(WX,F‚5.?,/V=),.%=?J
/V&Ä
ā(+&
*/V;+;yY
/5(.M'‹$VZ(&:=),‚5,.?' dÆÇMÈ e0')/V=)(<'šZ•a4(+&L‚2=)JL,\,]7:9
/>=?($!&

Æ4ÉcÊ Ä Æ4È¯Ê Âià dÆ™Ê´È e Ê Ã É opŸLË

[  %} Ì ,\HK.)(=),F=?JL,\,7:9


/V=)(+$5&f(&O=)J
,FZ•$5.?W

Æ É Ê Â Æ Ã Ê Ã É Ê´È>d Ä Æ™Ê Âià eŠo¡Ÿ

’ ,`&
1`,
dÆFÊ ÃŒe É
È oÎÍ
Ä ÆBÊ ÂiÃ
(<'K/V&f(+&!=?,`‚!,`.]«‹JL(<'K')JL$>H#'%=)J
/V=KÄ B
Æ Ê ÂÃn*L(34(<*-,' d ÆFÊ / &
*IJL,`&1,\/V;<'_$X*-(+34(+*-,]' d Ä ÆBÊ
à e É V ÄÌe É Š¼%9L=G-H0,
J/·35,
d Ä ÆFÊ Ä`à e É o d Ä ÆFÊ ÂiÃ Ê Ã e É o d Ä ™ Æ Ê i Ìe É Ê Âià d Ä Æ™Ê Âà e Ê Ã É Ë

¨©=lZ•$5;+;$>H#'0=?J
/>=lÄ ÆFÊ ÂiÆ*-(+34(+*L,'¯Ã É %b4(+&
1`,#Ã(+'l/XYL.)(+WX,5G-=?JL,\$5&
;af*-(34(<'_$!.?'‹$5Z Ã É /V.?,DÏD Ç Ï%Ãf/5&
*†Ï%à É
b-(&
1`,KÃйÄ
G-H%,\/5;+')$XJ
/·35,0ÃOo±Ä:Ñ Ê F$5.#Ä:Ñ Ê Â-
Ò zL]|…Ó } b49LYLY $!'),‹ÃOo±Ä!Ñ Ê 5 ~ g434($!9
'_;+a£Ä ÆFÊ ÂiÃOo˜F(+'‹&L$5=#Y$:')')(+gL;,!ƒ¨­&-Z/51=GLH0,F‚5,=\lopÄ ÆBÊ ÂiÃIo
Ä X Æ Ê Â d Ä!Ñ Ê ]eXÔÕÄ Æ£Ê¡Ö Ñ´o IHKJL(<1MJ(<'D(+WXY$:')')(+gL;,! ~ &…=)J
,f$5=)JL,.DJ/V&
*´Ä Æ2Ê Âño O‚5(+35,]'
Í Í
Ä Æ o Âià #o Ö Ñ ÄÔ Æ o Â5Ñ \/5&
* Æ™Ê ÃOo Â5Ñ  Ê Ä:Ñ Ê loqь
Í Í Í Í Í Í Í Í
dÆ\Ê ÃŒe É
«‹J49
' È o‘Í oqÑ É Š«‹J49
' dÆ ÇMÈ eŠo d Â5Ñ  Ç Ñ É e0HKJ
,`&2ÃOo¡Ä:Ñ Ê 5Šb4(+W2(+;</V.?;a!GLÄ ÆBÊ ÂiÃIo…ÃOÔ
d Ä!Ñ Ê Âià e Í Í
Ä Æ o ÆHKJL(<1MJ†(<'#&L$5=BY $!'?'_(+gL;+,5B"%$!&
'_(<*-,.)(+&L‚IÄ ÆFÊ ÂiÆo ÃG
H0,Ž‚5,`=™Ä Æ o ÄÃ$5. Æ o Ã†Ô È o^ŸL
Í Í Í Í
’ ,`&
1`, dÆ Ç?È eŠo d Ä!Ñ  Ç Ÿ!e%HKJL,.),‹ÃOopÄ!Ñ Ê 5
Í Í
ˆ[,=™9
'l1`$5&
')(+*L,`.BÄ ÆFÊ ÂÃno¹Ã É ’ ,&
1,2Ä Æ o´Ã É ÂiÃo¹Ã d à Â5eK/5&
*n&L,(›=?JL,`.0Æ&L$!.cà ÂX(<'B*L(34(<'_(+gL;+,
Í Í Í
g4afÄLƒ¨©ZƒÄ Æ™Ê ÂiÃIo Ã É G4=?JL,`&Ä Æ o à d Ã Ê Â5eŠÔ Æ o d Ä!Ñ Ê ]e d Ñ Ê ·ei
Í Í Í
’ ,`&
1`, ÆDÊ Ão d Ä:Ñ Ê ]e d Ñ  Ê ]e\o d Ä!Ñ Ê ·e_ьI«‹J
(+'‚5(+35,' È o¤Ñ É O¾l‚!/5(& dÆ ÇMÈ eŽo³× d Ñ Ê
Í Í Í Í
·e d Ä!Ñ Ê ·e Ç Ñ É`Ø HKJL,`&XÃOo±Ä:Ñ Ê !

Ò zL]|} b49LYLY $!'),‹ÃOo¡Ä:Ñ 5ƒ¨©Z¯Ä ÆFÊ ÂÃ8or5G4=?JL,`&Ä Æ o Ö Ñ Ê Ä2$5. Æ o Â!Ñ Ê 5 Ì ,Ž/V;<'_$X‚!,=


Í Í Í
dÆFÊ ÃŒe É d Â5Ñ Ê  Ê Ä:Ñ ]e É
È o Í o Í Í Í o Ñ É
  Í
/5&
*H%,X‚5,`=F=)JL,8')$5;+9-=)(+$5& dÆ ÇMÈ eBo d Â5Ñ Ê  Ç Ñ É ei ~ &=)JL,£$5=)JL,.\J
/V&*Ä ÆÊ ÂiÅo X*L$:,]'F&L$V=\J
/·3!,
Í Í
/5&4a8'_$!;9-=?($!&I(+&:=),`‚!.?/5;y'_$!;9-=?($!&8Z•$!. Æ Šb4(+WX(;</V.?;a!G4=)JL,.),™(<'%&
$X')$5;+9-=?($!&I(+&I=?JL,\1`/5'),BÄ ÆlÊ ÂiÃOo…Ã[Š«/VÙ4(+&L‚
Ä Æ2Ê Âiùo ÃGƒH%,8‚!,= Æ o ù/V&
*§JL,&
1, È o¦ŸL Ì ,f‚5,==)JL,f'_$!;9-=?($!& dÆ ÇMÈ eDo d Ä:Ñ Ê  Ç Ÿ!e¨©Z
Í Í Í
Ä Æ8Ê ÂiÃ^o¬Ã É G0=)JL,&¡Ä Æ o¦Ã d à Â5efo d Ä:Ñ ]e d Ä!Ñ Ä!e2‚5(+34(&
‚ Æ o d Ä!Ñ ·e d Ñ ]eX/V&*´J
,`&
1`,
Í Í Í Í Í
Æ2Ê Ã…o d Ä!Ñ ]e d  Ê Ñ ]eFoÁÑ d Ä!Ñ ]ei£«‹JL(<'\‚!(3!,' È o Ñ É /5&
*ÀJL,`&
1`, dÆ Ç?È eFo d Ä!Ñ  Ç Ñ É ei
Í Í Í Í Í Í
Å(+&
/V;+;+aOÄ ÆFÊ ÂÃIo Ã É *L$:,]'‹&L$V=#J/·35,F/V&4aI')$5;+9-=)(+$5&
Í

Ä
0¨©Z¯¶…(<'K/.?,/5;y.)$4$5=0$5Z[=)J
,\,7:9
/>=?($!&IÚ Û ÚÜ Ê Ú ÂŽo¡ŸLG-YL.?$>35,B=)J/>=nݛ¶Þß#opÄL\à”ÅL$!.‹/5&4a8.),]/V;Œ&49LWg ,`.
Í Í
Æ GLH0,\*-,`&
$V=),\g4aá Æ!â =)J
,\‚5.?,/>=?,'_=0(+&:=),‚5,`.K&
$V=K,6L1,,*-(+&L‚ Æ ã

[  %} b49


YLY$:'_,Ž¶(+'K/X.?,/5; .?$4$V=K$VZ=)J
,\‚5(+35,`&O,7:9
/V=)(+$5&Š«‹JL,&

¶ Û ¶ Ü Ê ¶ Â\o¡Ÿ
Ë ää`ä d Ó e
Í Í
«‹J
(+'l‚5(+35,'K¶Û ¶Ü Ê ¶ ™orŽ/V&
*nJL,&
1, d ¶ ]e d ¶å Ê ¶Ü Ê ·eco‡5 ~ g
'_,.)3!,™=)J
/V=l¶[å Ê ¶Ü Ê Žæ
Í Í Í
Â5¶ É Ê ¶ Ü o½¶ É d ¶ Ê Â!ei‡¨©Z ±ç¶¥èŸ
GK=)J
,`&q¶ Ê ÂsÐéŸLG#‚!(34(+&L‚P¶ É d ¶ Ê Â5eÀÐ韴/V&
*±J
,`&
1`,
d¶ Í
·e d ¶[å Ê ¶Ü Ê ]l e 衟L#¨©Zc¶¹è !GL=)J
,`&¶å Ê ¶܎o^¶Ü d ¶ Ê ]elÐpŸ8/5&
*JL,&
1,¶å Ê ¶Ü Ê DÐpŸL
Í Í
«‹J
(+'#/5‚!/5(&O‚5(+35,' d¶ ]e d ¶å Ê ¶Ü Ê ]e0èêŸL
Í
«‹J
,F/5g$>3!,#.?,')$5&L(+&L‚D')JL$>H#'c=)J
/V=‹Z•$5.‹¶è´Ÿ
G-H%,BJ
/·35,™¶ Û ¶ Ü Ê ¶ FèêŸD/5&
*8J
,`&
1`,™1/V&L&L$5=0g ,™,7:9
/V;
Í Í
=?$5 Ì ,Ž1$5&1;+9
*-,F=)J/>=#/X.),]/V;Œ.?$:$5=#¶…$VZƒÚÛ ÚÜ Ê Ú ÂŽo±Ÿ2(<'KY $!')(›=?(3!, d $5g434($!9
');a8¶¡o¡
¢ Ÿ:ei
Í Í
l$>Hp9
')(+&L‚8¶Û ¶Ü Ê ¶ ÂFopŸLG-H0,F‚5,`=
Í Í
¶ Þ op¶ å ¶ É Ê ÂV¶
Í
«‹J
,Ž'š=M/>=),WX,`&:=á ¶Þ â o¡Ä2(<'K,7:9L(+3>/V;+,`&:=‹=)$8Ä2繶ÞFè¹ë

"%$!&
')(+*-,.Ž¶[å ¶ É Ê ÂV¶^è‡ëIb4(+&
1,8¶˜ÐŸLG=?JL(<'Ž(<'\,7:9L(+3>/V;+,`&:=F=)$†¶Û ¶Ü Ê ÂV¶ É è‡ë!¶% ² '_(+&L‚f=)JL,
Í Í
.?,`;</>=?($!& d ]eG-H%,\1`/5&OHK.?(=),ŽÂV¶ É ¶ Ê ÂDè´ë:¶§$5.lÂV¶ É ¶ Ê Â2èêŸL¯«.),]/>=)(+&L‚D=)JL(<'#/!'0/X7:9
/!*-.?/V=)(<1VG4H0,
 Í Í§ì
‚!,=0=)JL(<'‹(<'c,]7!9
(3>/V;+,`&:=0=)$ è´¶PèêÂLl$>H¡$5g
'),`.?35,#=)J
/V=‹(Z¶Pæ¹ÂŽ=)JL,&lo d ¶ ]e d ¶ å Ê ¶ Ü Ê ·e0æêÂ
Â Í ì

HKJ
(+1MJ†(<'#(+WXY$:')')(+gL;,!#¨©ZcŸOèp¶¹ç G=)JL,&Žo d ¶ ]e d ¶å Ê ¶Ü Ê ]elèpŸ8HKJ
(+1MJ†/5‚!/V(+&n(+'B(+W2Y $!'?')(gL;+,5
 Í

Ì ,1`$5&L;+9
*-,F=?J
/>= 豶´èêÂL0b4(+WX(;</V.?;+aO¶å ¶ É Ê Â5¶æ±Ä2(+'l,7:9L(+3·/5;,&:=K=)$8¶Û ¶Ü Ê Â5¶ É Ä5¶P江
Â Í Í Í
HKJ
(+1MJ£(<'Š,]7!9
(3>/V;+,`&:=Š=?$DÂV¶ É ë!¶ Ê ÂŽæêŸLƒ¼09-=c=?JL(+'c(+'0 d ¶ ]e É æêŸ\HKJL(<1MJ£(<'Š3>/V;+(+* ’ ,`&
1`,BÄ2繶Þ\è´ë
Í Í
/5&
*fH%,F‚!,=Žá ¶Þ â o¡Ä

니[,=#ír*-,&L$V=?,l=?JL,Ž1(+.M19LWX.M/5*-(+9
'%$VZ/=).?(</V&L‚!;,F@lA2C2m Æ G È GLî#(=?'K')(+*L,'KADC2G
CF@G
@lAImL/V&
*Oï]ð-G-ï]ñ`GLï]ò‹(=?'
,`6-.?/!*-((y$!YLY $!')(›=?,F@G
A8G C2¯¨©Z¯Â5írçPï·ð4GLYL.?$>35,B=)J/>=

d (<e Æ Ð È /V&
* Æ Ðêî>m
d (+(<enÂVírдï ñ /V&
*Â5í˜Ð´ï ò

Æ-È î —
[  %} Ì ,KÙ4&L$>H…=)J/>=%ÂVíso /V&
*Dï ð o G!HKJL,`.?, Æ Ç?È·Ç îc/V.?,0=)JL,#')(<*-,'ƒ$5Z
=)J
,‹=).?(+/5&L‚5;+,0@BADC2G
Â:— ó Æ
ÆFÊPÈ¯Ê î Í
ó o /V&
*—w(<'0=)JL,Ž/5.),]/$5Zƒ@BADC2c«‹J49
'0=?JL,\‚5(+35,&f1`$5&
*L(›=?($!&nÂ5í˜ç´ï ð =?.?/5&
');+/V=),'0=?$
Â

Â!— É
Æ4È îBç
ó Æ
Í
v 9-=_=?(&
‚ ó Æ o…Ã Ç ó È o¡ô Ç ó î#o¡ï4G-H%,™‚5,`= Æ o¡ô Ê ï ÇMÈ o¡ï Ê Ã Ç îKo…Ã Ê ôŽ/V&*£=)J
,™1`$5&
*-(=)(+$5&O&L$>H
Í Í Í
(<'
à dÃ Ê ôVe d ô Ê ïVe d ï Ê Ã e0ç±Â!— É
¼09-= ’ ,.)$!&õ 'KZ•$5.?W9
;+/I‚5(+35,'G— É o ó d ó Æ e dó È e dó î`e#o±Ãô·ï d Ã Ê ô Ê ïVei Ì ,2$5gL=?/V(+& dÃ Ê ôVe d ô Ê
Í Í Í
d
ïVe ï Ê Ã e0ç±ÂVô·ï à d Ê ô Ê ïVei%ö¯6-Y
/V&*-(&
‚X/5&
*O,÷Œ,1i=?(&
‚8'_$!W2,\1/V&
1`,`;+;+/V=)(+$5&
'G4H%,F‚5,`=

Ã É dô Ê ïVe Ê Ã dô É Ê ï É e0çêô·ï d ô Ê ïVeË d<ø e

b 9LYLY $!'), Æ ç È Š«‹JL(<'K(+W2Y


;(+,'‹=?J
/>=lô Ê ïDç¹ï Ê Ã†/5&
*OJL,`&
1`,Žô2çÃc«‹JL(<'‹(WXYL;+(,]'‹=)J/>=Bô É ïDçÀÃ É ï2/5&
*
-
ô·ï É ç±Ã
ï É ‚!( 34(+&L‚fô·ï d ô Ê ïVeDç±Ã É ï Ê Ã
ï É è±Ã É ï Ê Ã
ï É Ê Ã É ô Ê Ã ô É oqÃ É d ô Ê ïVe Ê Ã d ô É Ê ï É eBHKJL(<1MJ
1`$5&:=).M/5*L(+1=?' d<ø ie cb4(+WX(;</V.?;+a5G Æ ç¹î™(+'#/5;+')$2&L$V=#Y $!'?'_(+gL;+,5¯«‹JL(<'KYL.?$>35,]' d (<e
b-9LYLY $!'),ÂVí˜ç¹ï ñ Š¾l'K/5g$>3!,l=?JL(<'‹=?/5Ù5,'0=?JL,FZ•$5.?W

ô É dï Ê Ã e Ê ô dï É Ê Ã É e0çÃï d Ã Ê ïVeiË dùø5ø e

b-(&
1`, Æ Ð È /5&
* Æ Ðêî>GLH0,™J/·35,FôÐÀÃ Ç ïÐÀÃ[Š«‹J49
'Kô É ï2ÐÀÃ É ï2/V&
*nô·ï É ÐÀÃ
ï É ’ ,`&
1`,

ô É dï Ê ÃŒe Ê ô dï É Ê Ã É e0йô É ï Ê ô·ï É ÐÃ É ï Ê Ãï É o…Ãï d Ã Ê ïVe

HKJ
(+1MJ1$!&:=).M/5*-(<1i=M' dùø5ø ei ’ ,&
1,ŽÂ5írÐPï ñ cb4(WX(+;+/5.);+a5G-H0,Ž1`/5&OY
.)$>3!,l=?J
/>=BÂ5í˜ÐPï ò Š«‹JL(+'‹Y
.)$>3!,' d (+(<e

‹ˆ[,=¯*-,`&
$V=),c=)JL,0'_,`=$5Z
/V;+; Ö “=?9LYL;+,' dÆ Ç?È·Ç î Ç)úÇ)û:Ç?ü ey$VZLY $!')(›=?(3!,Š(&:=?,`‚5,.?''_91MJ\=)J
/V= Æ É ÊDÈ É Ê î É Êú É ÊŽû É o
ì
ü É %"%$5&
')(<*-,`.‹=?JL,Ž'_,`=
‰±oSý Æ-È î ú:û·ü œ dÆ Ç?È·Ç î Ç)úÇ)û:Ç?ü e%þ†lÿË

Å(+&
*O=)JL,\‚!.),]/>=?,'_=K1`$5WXWX$5&n*-(+3:(<')$5.‹$VZ¯/V;+;Œ=)JL,\WX,Wg ,`.M'‹$VZƒ‰Ž
[  %} Ì ,F')JL$>H¹=)J
/V=%=?JL,B.?,7:9L(+.?,*X‚!1*£(<'0ÂVë
Š"%$5&
')(+*L,`.‹/V&8,;,W2,&:= dÆ Ç?ú
Ç î Ç)úÇ)û:Ç?ü e0þ‹ Ì ,BJ
/·3!,
Æ É ÊPÈ É Ê î É ÊPú É Êû É o ü É Ë

Ì 
,
.M'_=Š$5g
'),`.?35,0=)J/>=Š&L$5=c/V;+; Æ Ç?È·Ç î Ç)úÇ)û 1`/V&Xg ,K$-*L*y ~ =)JL,.)HK(<'),5GVH0,KJ
/·35, Æ É È É î É ú É û É      

d WX$4* :e‹/V&
*nJ
,`&
1`, ü É  
d WX$-* !eiGHKJL(<1MJf(<'K(+WXY$:')')(g
;,Žg ,1/V9
'),\&L$8'?7:9
/V.?,\1`/V&ng ,1$!&L‚5.?9L,&!=‹=?$
 ì
WX$-*-9L;+$ L¯«‹J49
'#/V=K;,]/5'_=‹$!&L,F$VZ Æ Ç?È·Ç î Ç)úÇ)û (<'‹,`3!,`&
ì

b-(WX(+;+/5.);+aD(Zy&L$5&
,#$VZ Æ Ç?È·Ç î Ç)úÇ)û (+'%*-(+3:(<')(gL;+,#g4aXÄLGV=?JL,`& Æ É È É î É ú É  û É  
 d WX$-*8Ä:e¯/V&
*XJ
,`&
1`,  
ü É   d W2$-*Ä:e‹HKJL(<1MJ†/V‚!/5(&n(<'#(+WXY$:')')(g
;,Žg ,1/V9
'),Ž&L$O')7:9
/5.),\(<'l1`$5&L‚!.)9L,&:=K=)$OÂ2WX$-*-9L;+$IÄLK«‹J49
'BÄ
*L(34(<*-,' Æ-È î ú!û>ü
«‹J
,`.?,F/5.),\'),`3!,`.M/V;ŒY$:')')(+gL(;+(=)(+,'0Z•$5. Æ ÇMÈ·Ç î Ç?ú
Ç?û
Ò zL]|…Ó } b49LY
Y$:'_,Ž$!&L,Ž$5Z=?JL,`Ww(<'#,35,`&†/5&
*f=)J
,$V=?JL,`.lZ•$59L.™/V.?,\$-*L*ymy')/·a Æ (<'#,35,&G È>Ç î Ç?ú
Ç?û /V.?,Ž$-*L*y
«‹J
,`& È É Ê î É Ê¹ú É Ê¹û É 
ë d WX$-* !el¨©Z Æ É 

ë d W2$-* !eiG=)J
,`& ü É 
Ÿ d WX$-* !el/V&
*nJ
,`&
1`,2Â Æ Ç ë ü
   
‚!(34(+&L‚ 
-
Æ ü ™¨©Z Æ É d
Ÿ WX$-* !eG =)JL,&  ü É  
ë WX$-* :e#HKJL(<1MJ/5‚!/V(+&†‚!(3!,'K=?J
/>=Fë Æ /V&
*Â ü '_$I=)J
/V=
d  

Æ-ü ƒ¨©=KZ•$5;+;+$>H#'0=)J
/V= 
Æ4È î ú!û>ü /5&
*OJL,`&
1`,Â>ë Æ-È î ú:û>ü 
Ò zL]|} b49LYLY $!'), Æ ÇMÈ /5.),l,`35,&£/V&
*8î )Ç úÇ)û 5
/ .),l$4*
*y¯«‹JL,`&8î É Ê†ú É Êû É Ä dX
W $-* !e¯b-(&
1`, Æ É Ê†È É Ÿ  

$ .BëOWX$4*L9L;$ LG (›=™Z•$5;+;+$>H#'K=)J/>= ü É
! Ä $5.
I 
 
d W2$-* :eKHKJL(<1MJ†(<'l(+WXY$:')')(+gL;,! ’ ,`&
1`,=?JL(<'B1/5'),D*-$4,'B&L$5=
/5.)(<'),5
Ò zL]| [} ¨©Z=?JL.),,F$VZ ÆÇMÈ·Ç î Ç)úÇ)û /V.?,F,`3!,`&n/V&
*O=šH0$D$-*L*yGL=)JL,&  Æ-È î ú!û>ü 
/5&
*fJL,`&
1`,ŽÂ>ë Æ-È î ú:û>ü
Ò zL]|† } ¨©ZZ•$!9L.#$5Z Æ Ç?È>Ç î Ç?ú
Ç?û /V.?,F,`3!,`&G-=?JL,`&†/5‚!/5(& 
Æ È î ú:û·ü /V&
*ÂVë Æ4È î ú!û>ü
-  ’ ,`.?,Ž/V‚!/5(&OZ•$5.B/V&4aI')(›6
=?9LYL;+, dÆ Ç?È>Ç î Ç?ú
Ç?û!ÇMü e%( &†‹G-H0,\$5g
'),`.?35,l=)J
/V=BÂ>ë Æ-È î ú:û>ü ƒb-(&
1`, 
 É Ê  É Ê  É Ê Â É Ê Ä É o¡ë É Ë

Ì ,Ž'),`,™=)J
/V= d !G!G!G ÂLG Ä
G ë4eiþf´/V&*OJ
,`&
1`,\ÂVëXþf‰ƒ«‹J49
'#ÂVëX(+'0=?JL,\‚!1*I$5Z‰Ž
Ö v .)$>3!,l=?J
/>=‹=?JL,\&49LWg ,`.#$5Z “=?9LYL;+,'‹$VZƒY $!')(›=?(3!,™(+&:=),‚5,`.M' dÆ ÇMÈ·Ç î Ç?ú
Ç?û e%'?/>=?(+'_Z•a4(&
‚D=?JL,\,7:9
/V=)(+$5&
ì
Æ4È î ú!û o à È î ú!û‹ÊÆ î ú:û0ÊPÆ-Èiú!û‹ÊÆ-È î û%ʏÆ-È î ú ã
ì
(<'#/5& 
 (+&!=?,`‚!,`.]
[  %} Ì ,\HK.)(=),F=?JL,\,7:9
/V=)(+$5&f(&O=)J
,FZ•$5.?Wfœ

     
Ê Ê Ê Ê o Ë
Æ È î ú û
ì

« J
,\&:9
Wg ,`.K$VZ
3!,B=?9LYL;+, dÆ Ç?È>Ç î Ç?ú
Ç?û e%HKJL(<1MJ')/V=)(<'šZ•a8=)JL,\‚!(3!,`&f.),;+/V=)(+$5&n/V&
*IZ•$5.KHKJL(<1MJ Æ o ¢ È (<'‹,`3!,`&G
‹
g ,1/V9
'),2Z•$5.Ž(Z dÆ Ç?È·Ç î Ç)úÇ)û eB(<'\/f')$5;+9-=?($!&G=?JL,`&…')$f(<' dÈ>Ç)Æ Ç î Ç?ú
Ç?û e­HKJL(<1MJ(<'Ž*-(<'_=)(+&
1i=\Z•.?$5W dÆ Ç?È·Ç î Ç)úÇ)û ei

b-(WX(+;+/5.);+a=)J
,£&:9
Wg ,`.Ž$5Z
3!,D=?9LYL;+,'\HKJ
(+1MJ…'?/>=)(<'_Z•a=)JL,£,]7:9
/>=?($!&À/5&
*†Z•$!.\HKJL(<1MJÀîo ¢ ú (<'Ž/V;<'_$f,`3!,`&
 
’ ,`&
1`,n(›=8')9 81,'D=?$À1`$59L&:=X$5&L;+a§=)JL$:'_,
3!,I=?9LYL;+,' dÆ ÇMÈ·Ç î Ç?ú
Ç?û e\Z•$!.2HKJ
(+1MJ Æ o È>Ç îo ú P«‹J49
'D=)JL,
,]7:9
/>=?($!&O.?,*-91,'0=?$
   
Ê Ê o Ë
Æ î û
ì
’ ,`.?,#/V‚:/V(+&=?JL,‹=)9LY
;, dÆ Ç)Æ Ç î Ç î Ç)û e[Z•$5.cHKJL(+1MJ Æ o± ¢ î0(+'Š,`35,&Xg,]1`/V9'_,‹H0,#1`/V&£/!')')$-1(</>=),‹*-(÷ ,.),&:=c'_$!;9-=?($!&
 
d î Ç î Ç)Æ Ç)Æ Ç)û ec=)$X=?JL(<'
35,™=?9LYL;+,5Š«‹J49
'K(=l'_9 £1`,'0=?$81$5&'_(<*-,`.0=?JL,\,7:9
/V=)(+$5&

ë  
Ê o Ç
Æ û
ì
/5&
*n'_J
$>H±=)J
/V=K=)JL,\&49LWDg,.#$VZY
/V(+.M' dÆ Ç)û e‹'?/>=)(<'_Z•a:(+&L‚D=)JL(<'K,]7!9/>=)(+$5&f(<'K$-*L*y
«‹J
(+'K.?,*L9
1,]'0=)$
Æ:û opÂ5Ÿ û%Ê Æ
ì
$!.
dÆ Â5Ÿ!e dû eŠorŸ!ŸLË
Í Í…ì
¼09-=#$!g
'),`.?35,B=)J
/V=


Ÿ!ŸŽo˜ ÀŸ!ŸFoq ! ì
ŸŽoêë   ì
o
ì
ÂVŸ
orŸ ]ŸŽopÂ5Ÿ ! " ì
oqÂ
ì
OëDo
ì
" !
Ÿ Žo^]Ÿ5Ÿ !Ë

l$V=?,F=)J
/V=#&L$2Z/51i=?$5.?(+'?/>=?($!&I$5Z%Ÿ!ŸD/!'‹YL.?$4*L9
1i=K$VZ=šH0$2&
,`‚!/V=)(+35,™&49LWg ,`.M'‹a4(,;+*f/XY$:'_(=)(+35,™=?9LYL;+, d Æ Ç)û ei
’ ,`&
1`,H0,†‚5,`=I=?JL,'),ž')$5;+9-=)(+$5&'`r«‹JL(<'OYL.)$>3!,'2=?J
/>=O=)JL,=)$V=M/V;B&:9
Wg ,`.f$VZ
35,†=)9LYL;+,' dÆ ÇMÈ·Ç î ?Ç ú
Ç?û e #
'?/>=?(+'_Z•a4(&
‚D=?JL,\‚5(+35,&O,]7:9
/>=?($!&O(<'‹$-*L*y

  
!"$#% '&()*!+

,.-0/2143658791:3';1=<?>A@CB9DE>GF3+DIH
JK>A@C1=L?MNDIH
O?3'PQ>GOEFRESG1:T+LUM?-/21433';V1W<X1Y@C>AOIFT65Z>GF3'1[P\J'1Y]^33Q;1W>GFV]4>GP\]4SG1:DIH
TLUM_OI36`aOIFV@b+c9`d7e1Y>fFVRgF1hOIPQ1[Pi3QDjTk3';VOEFbi-lmH*T+` n6`b%n6b%5cBPQDpoE1q3Q;VOp33';1XJK>A@C1hJ+DEH
3'PQ>GOEFRESG1
TLUMOIPQ16>fFr3';V1:PQOI3'>GDtsUu,hv?uV,YwU>GFxJ'DE<X1qDEP\@C1YPY-

y2z9{}|~Y}z9€*
/2143:‚g791W3Q;1?>fFV][1[F3'PQ1=DEH
3'PQ>GOEFRESG1=TLUM_OEFV@ƒ„791U>f3QJBPQDI…1h]^3'>GDEF†D.FLUM?-W‡7VJ'1[PQoE1:3Q;VOp3qT+L$nk
ˆ TM‰OEJ
TLŠn‹TMŒ>f<XBSG>f1hJ3';VOI3qƒn‹bnN5-=ŽDrO.J'J'<X1W3Q;VOp3:TMŠ‘’T+Lg-=/“1[3q”d7e1U3';V1?BVP'DE…1Y]^3Q>fD.FDIH%‚jD.F
`•bi-—–0;1[FX3Q;1+B91[PQB91[FV@C>A]4VSGOEP*‚”ŠH˜P'D.<N‚:3QD=`b™>GJ*Oq7>GJ'1Y]43'D.P DIH2`•bšOIFV@tO.J T+`n›b*5cE>f3*>AJ OW7>GJ'1Y]43'D.P
DEH—T65ZOESGJ'DV-
œi1YFV]41WT‚Un‚5-

r N
r
I
L
r

B D M C

Fig. 1.

ž C3+T‚?n Ÿ n ]4DJK1h]   T6£E¤ ¥ OEFV@


JK>GF¡ ¢Tq£I¤.¥ Ÿ

‚5’¦§n ‚ƒ¡¦ ¨©ƒg5’¦in ¦%¨ª}L?5¬« L=ƒj­¦


Ÿ
n ¦ ¨®9¯ «°ª¢±+« ²4­K³ ¦
´
Ÿ ¤

œ+1[FV][1 ¦ ][D.J'1Y] ¦   Tq£I¤ ¥ n ¦ ¨   ª £E¤E­


«›ªµ±«¶²4­ ¦ ¥ ¦ RE>Go>fFR ¦ ][DI3 ¦   Tq£I¤ ¥ n   ª¢²0«™·[­Q£I¤ ¥ ¦ -qŽ>GFV][1?²U‘·pc
Ÿ Ÿ ¯ Ÿ
¸ 1:DE73QOI>GF Ÿ ]4DE3š µT6£E¤E¥+nŒ 'ªµ²%«¹·Y­'£I¤.¥p- ŽD¡±+« ¯ nŒ Qª¢²%« ·[­'£E¤E¥I- –0;>GJ0R.>fo.1YJ ¯ n›¤I·I-
º Ji`”„n”b°OIFV@rT+`n`b™n»b%5c ¸ 1q;VO¼o.1:”b™n›T65›£p½-—¾¹1WOISAJKD?;VO¼o.1qT”¿n›”5-
À+D ¸

¦ n›‚.b ¦ n›‚” ¦ ¨¶”b ¦ n T+‚ ¦ « T+” ¦ ¨¶”b ¦


Ÿ , ,
n T+‚ ¦ « ¦Ã ¨ ¦Ã
Á“Â wE½ Â
¤
n ¦ ]4DJK1h] ¦V  T6£E¤ ¥ « ¦Ã ´
Ÿ Ä Â
œ+1[FV][1 ¦ ]4DE3 ¦  µT6£E¤E¥+n ¦Å ¦Ã- ÆPQDE<Œ3';1WOE7eDpo.1Ec ¸ 16RE1[3
Ÿ Â
Ç ²%« · ¦ ¤ ,
n   ¤E²^¦ ¨©¤I·4¦0« ¦ ¥ ´
¤ZÈ Ä É Á
x ¯

ŽC>f<XBSG>ËÊ9][Op3Q>fD.FgR.>fo.1YJ0sE² ¦ ¨¶,hwE· ¦ «š,hÌ.²^·0n›vV-


–0;>GJ0]YOIF¡791 ¸ PQ>f3K3'1YFjO.Jªµ²—«†·[­4ªµsE²—« ,Yw.·[­
n›vV- º J²:n
ˆ ·pc ¸ 1
R.143s.² «Í,Yw.·0nv-
–D¡]4DEFe]4SGV@C1Ec n¤E·pÎQs.²*nk,Yw.·iϏ>f1YSG@
¯
² · ´
¯ n n
,Yv ,hw s

,
¤-0/2143™OEFV@ 7e1:B9D.J'>Ë3Q>fo.1q>fF3'1YRE1YPQJ0J'V]\;g3Q;VOp3
Á w ,  ´
,hÄ  

Æ >GFV@j3';1:<X>GF>G<=<ŠBeDJ'J'>G7Sf1qopOISG1qDIH  -

y2z9{}|~Y}z9€*
¾¹1:;VO¼o.1
   ,YÄ ´
,    Á w
–0;eOp3i>AJ[c
Á ¨
Ä  Á ¨
¤.s ´
,    Á w
s-Ž>GFV]41ÍOIFV@©OIPQ1WBeDJK>f3'>GoE1U>fF3Q1[RE1YPQJYc ¸ 1=<tO¼Ï ¸ P'>f3'1šn Á †¨2c ¸ ;1YP'1=v
–0;VJ Á

À+D ¸¸ 1:RE1[3
   -

Á ¨
Ä
Á ¨
 Á ¨ ¤Es ´
,    Á w
ŽCD
Ä  ¤Es 3';VOI3i>GJYc w
 Á ,   ´
,    Á w ¤Es   Ä
¾¹1†ÊVFe@¶3Q;1J'<tOISGSf1hJ3topOISG1†DIH»H˜D.P ¸ ;>G]\;‹7e1h]4DE<X1hJtO ¸ 1YSfSm@14ÊVF1h@Í>fF3'1YRE1YPY-›ÆVDEPkna,EÎQ¤Î'w3';1
79DEVFV@JjDIH‹OEP'1†PQ1YJ'Be1h]^3Q>fo.1[SGÏ
Ç ,YÌ
, YÎ ,
Ì
Î w
Ç ,.,
Î'w
 Î Ç s Á QÎ s ¤ ´ À+DEF1DEHq3';1hJK1•BVOI>GPQJg]4DEF3\OI>GF°OEF
¤Es Ä È ¤.s Ä“È Ä w È
>GF3'1YRE1[P79143 ¸ 1[1YFr3';1Y<r-
Á
w  ,h¤ ,   ! s
ÆVDEP xn Á c ¸ 1
;VO¼oE1 n›½ OIFe@ n  - œi1[Fe]41Ech>GFq3';>AJ2]YOEJ' 1 n CchOEFV
" @ n Á —¨#jn¤Ȉ¨ Á nw.¤-
¤Es ¤Es Ä Ä
Á
w  Á w
–0;V>GJ=>GJUOISAJKDx3Q;1¡SG1YOEJK3WB9D.JQJK>G7SG1¡opOISG1Ec 7e1h][OEVJK1.c>f$ H &%_sCc3';1Y F k‘ % ‘‰Ìc—OIFe@šJK D  ‘‰ w C-
¤.s s
œ+1[FV][1q3';1:<X>GF>G<=< BeDJ'J'>f7VSf1:opOISG1qDI H  >GJw¤C-
wV-0/2143'Î)(Î 7e1:B9D.J'>f3'>GoE1qP'1hOIS F<U7e1YPQJYcCFDI3+OESfS 1+*VOISµcJKe]\;g3Q;VOp3+J'DE<X163 ¸ DXDEH3Q;1:1,*VOI3'>GDEFVJ
Ÿ
'-e¦ ¨©¤.( ?¨ Ÿ n»vÎ/( 9¦*¨©¤ Ÿ ?¨'jn›vVÎ Ÿ
9¦%¨©¤0'1U¨(:n›vVÎ
;eO¼oE1:OX]4DE<X<XDEFrPQDDE3YcJQO¼Ï2 -4 3 PQDpoE1i3Q;VOp3
ª¢O.­š>AJPQ1YOIS OEFV@rF1[ROp3'>GoE1 OIFV@

ª˜7e­g3';V1q3';>GPQ@r1+.* eOp3'>GDEFr;VO.J0FDEF5m PQ1YOIS PQDDI3QJY-

y2z9{}|~Y}z9€*
6 D.FVJ'>G@C1YP03';1W@>GJQ]4PQ>f<X>GFVOIF3\J0DIH23Q;1q3';VP'1Y1:1,*VOp3Q>fD.FVJ
1' 9¦*¨( Ÿ ¨ Ÿ n v ª,¼­
(  ¦ ¨ Ÿ ?¨' n v ª¢¤.­
 ¦*¨7'1U
9  ¨( n v ´ ª}w­
Ÿ
/2143iVJ+@C1[FDE3'163';V1[< 7Ïrƒ8pÎ'ƒ ÎQƒ:9qPQ1YJ'B91Y]^3Q>fo.1[SGÏE-
–0;1YF ¸ 1q;VO¼oE1
¦
ƒ 8 n Á ª;( ¦ « '9­4Î'ƒ n Á ª ¦ «'-(I­4Î'ƒ 9 n Á ª<' ¦ «=( ­ ´
Ÿ ¦ Ÿ Ÿ
¾¹1:DE7eJK1YP'o.1+3Q;VOp3

ƒ8
¨¶ƒ ¨™ƒ9§n Á >ª ' ¦ ¨( ¦ ¨ ¦ «?'@(+«A( « '9­
¦ Ÿ
n C¤ Beª<'t«A(I­ ¦ ¨ ;ª (6« Ÿ ­ ¦ ¨Ÿ ª Ÿ «D
Ÿ
'9­ ¦E ‘°v

¤
J'>GFV]41 'Î)(Î OIPQ1UFVDI3=OISGS
1,*VOES¢-tœ+1[FV][1¡Op3WSG1YOEJK3:DEF1XDEH%ƒ 8 ÎQƒ Î'ƒ 9 <=eJ3:791tBeDJK>f3'>GoE1.-t¾•1t<tO¼ÏO.J'J'<X1
Ÿ ¦
ƒ 8 ‘°v-
ŽCBB9D.J'1?ƒ vgOEFV@ƒ 9 v-ql F3Q;>AJ6]YOEJ'1=79DI3Q;3';V1?1+*VOp3Q>fD.FVJUªµ¤E­OIFV@¶ª}w.­+;VO¼o.1=DEFVSfÏxFD.F5µPQ1YOESPQDDI3\J
OEFV@†+1 *.eOp3'>GDEF™ ªK,h­i;VOEJ+DEFSGÏj PQ1YOES2PQDDI3QJY-iœi1YFV]41W3';1X]4D.<?<XD.F†P'DDI3Í<UVJ37e1U7e1[3 ¸ 1Y1[F¶ªµ¤E­iOEFV@šª}w­^- ž 3
¦
3Q;1[F ¹>  GJ 3';1DI3Q;1[P%P'DDE3 DIH“79DI3Q;•ª¢¤.­ OEFV@ª}w­^- œ+1[FV][1+>f3*H˜D.SfSGD ¸ J
3Q;VOp3qª¢¤E­*OIFe@ª}w­
;VO¼o.1+J'OE<X1+JK1[3%DEH“P'DDE3QJY-
–0;V>GJ>G<XBSG>f1hJ03';VOI3
( n_Ÿ n ' ´
Ÿ
' (
–0;V4 J 'n (?n ]4D.F.3QPQO.@C>A]^3'>GFRX3Q;1UR.>fo.1[F][DEFV@C>f3'>GDEF2-+œi1YFV]41?7eDE3';ƒ OIFV@ƒ 9 ][OIFVFDI3+791?F1[ROp3'>GoE1.-¾•1
Ÿ ¦
<tO¼ÏgO.J'J'<X1:ƒ
¦
%
ÍvV-—–0;VJ ¸ 1q;VO¼o.1
(p¦0« Ÿ '‘©vÎ Ÿ ¦«'-(%°v ´
–0;V1YJ'1q3 ¸ D?RE>GoE1
(¦Ÿ¦ ‘ ' ¦(Ÿ
J'>GFV]41 '2Î (Î OEP'16OISGS9B9D.J'>f3'>GoE1E-
œ+1[FV][1 ¸ 1D.7C3QOE>f F ( ‘ ' ¦ DEP0 ƒ 9
Ÿ
<UVJK3i7e1:79143 ¸ 1[1YFr+1 *VOp3Q>fD.FVJqª,h­0OEFV@ ª¢¤.­^-
Ÿ  vV-—¾•1q]4D.FV]4SGV@C13';VOI3*3Q;1:]4D.<?<XD.FgPQDDE3
–0;VJ
'- ¦*¨ ( x¨ Ÿ n v
( ¦*¨ Ÿ x¨' n v
 Sf>G<X>fFeOp3'>GFR ¦ c ¸ 1qDE7C3\OI>GF
­ n ' ¦ «A( ´
¤ª ( ¦ «?'
Ÿ Ÿ
Ž >fFV][1( ¦ «' ‘°vXOIFV@:'
C ¦ «=( vVc ¸ 1:][DEFV][SfV@13Q;VOp3 vV-
Ÿ Ÿ  
–0;V1W]4DEFe@C>Ë3Q>fD.F' ¦ «A( vU>G<XBSG>f1hJ03';VOI33';1:1+*VOp3Q>fD.F¹ª¢w.­%;VO.J0DEFSGÏ¡FD.F5µPQ1YOES P'DDI3\J[-
Ÿ 
º ËS 3Q1[PQFVOp3Q1[SGϕD.F1¡][OEFšOIPQRE1tOEJ:H˜DESGSfD ¸ JY-jŽBVBeDJK12>GJUO†]4DE<X<XDEF PQDDI3:DIH03 ¸ Dr1+*VOp3Q>fD.FVJ[c J'O¼Ï.c*ªK,h­:OEFV@
ªµ¤E­4-rlmHC>GJ=FDEF5mPQ1YOISµc 3Q;1[F >  GJ=OISAJKDOxPQDDI3WDIH79DI3Q;›ª,h­WOIFV@°ª¢¤.­^-†œi1YFV]41¡–0;V1g]4D1¡][>f1YF.3\J:DIHWªK,h­:OEFV@
ªµ¤E­OIPQ1=BPQDEB9DEP'3'>GDEFeOISµ-6–0;>AJH˜DEP\]41h
J '¹n (Xn c2Og][DEF3'P\OE@>G]43'>GDEF“-qœi1YFV]41U3';1X]4D.<X<?D.FPQDDI379143 ¸ 1[1YF•OEFÏ
3 ¸ DX,1 *VOI3'>GDEFVJi]YOIFFVDI37e1=FDE5 F mPQ1YOISµ-
/“DD >GFRtŸ Op33Q;1W]4D1 ¡][>f1YF.3\J[c ¸ 1:]4D.FV]4SGV@C1q3Q;VOp33Q;1=][DE<X<XDEFrP'DDIC3
<UVJK3+7e1UF1[ROp3Q>fo.1E-%lmH+ª,h­iOIFV@šªµ¤E­0;VO¼o.1W]4D.<?<XD.F†P'DDI 3 *ce3';1Y F ( ¦ % '•OEFV@ ¦ % '-(-œi1YP'1=OI3+SG1YOEJK3iDEFV1
Ÿ Ÿ
>GF+1 *.eOISG>Ë3Ïj>GJJ3QP'>A]^3H˜D.
P ( ¦ n ' OIFe@ ¦  n '-(UH˜DEP\]41h J 'xn (Wn -0œi1YFV]4 1 ( ¦ ¦ A
‘ ' ¦ ( -%–0;V>GJ+RE>GoE1Y
J ' ¦
Ÿ
OEFV@r;1[Fe]41tª}w­%;VOEJ0FD.FPQ1YOIS PQDDI3\J[-
Ÿ Ÿ Ÿ Ÿ  (Ÿ
Á - º SfS B9D.JQJK>G7SG1½.m@C>GRE>f3 F<U7e1YPQJYc>fF¡1hOE]\;¡DEH ¸ ;>A]\;t3';16@C>GRE>f3QJ%DC][]4VP >GF € ze€Q¢€
 h}€ D.PQ@1[Pª˜H˜P'D.<‰Sf1[H 3*3QD
PQ>GRE;3Yc1.- Re-fc9Ì IsEsEv.­ OIPQ1 ¸ P'>f3K3Q1[FOEJOtJ'1,*1YFV]41:>GF }€   h}€ DEP\@C1YPY- Æ >GFV@r3';V1U¤Iv.v.s µ3';rF<=791[P+>fFx3Q;>GJ
J',1 *1YFV]41.-
y2z9{}|~Y}z9€p
6 D.FVJ'>G@C1YPOU.½ m@C>GRE>f30FV<=791[P ¸ ;D.J'1q@C>fR.>Ë3\J*H˜P'D.<_Sf1[H 3%3QDXP'>GRE;3%OEP'16>GFgFD.Fj>GFV]4PQ1YO.JK>GFRUDEP\@C1[Ph-lmH%,>AJ*3';16ÊVP\JK3
@>fR.>Ë3%DIH“JKe]\;¡O:F<=791[PhcE3Q;1[F¡3';16JK7eJK1+*.V1[F3*@C>GRE>f3QJ*][OIFVFDI3 1]41[1h@x,E-
–0;1+J'143%DIH“OISGSeJ'V]\;tF<U7e1YPQJ ¸ >Ë3Q;
>GF>f3'>AOIS2@>fR.>Ë3i+1 *.eOIS 3'Dx,:>AJ

 ,hvEv.vEvEvVÎ[,.,YvEv.vEvVÎ[,I,.,[vEvIvVÎ[,I,.,I,YvEvÎ[,E,E,I,.,[vÎY,I,E,I,.,I,E, ´
–0;V1[PQ1qOEP'1q1YSf1Y<?1YF3QJ0>GFj3Q;>AJiJK1[3Y-
/21436VJ+][DEFVJ'>G@1[P+.½ m@C>GRE>f3+F<U7e1YPQJ ¸ >Ë3Q;>fFV>Ë3Q>GOES@C>GRE>f3q¤-+Ž3QOEPK3Q>fFRXH˜D.P'<„¤Iv.vEv.vEvc ¸ 1=]YOIF†REDgBx3'Dx¤E¤E¤.¤E¤.¤C-
¾¹1W]4D.F33';1hJK1qF<U7e1YPQJO.J%H˜DESGSfD ¸ YJ u
¤Iv.vEvEv.v  ¤C,.,E,.,E, u ½
¤E¤EvEvEv.v  ¤E¤,E,.,E, u s
¤E¤.¤IvEv.v  ¤E¤.¤C,.,E, u Á
¤E¤.¤E¤Iv.v  ¤E¤.¤E¤,E, u w
¤E¤.¤E¤E¤Ev  ¤E¤.¤E¤.¤C, u ¤
¤E¤.¤E¤E¤.¤  ¤E¤.¤E¤.¤E¤ u§,

w
–0;V1¡F<=791[P:DEHJ'V]\;¹F<=791[P\J:>AJ=¤C,.-jŽ>G<?>GSAOIPQSfÏ ¸ 1¡]4D.F3WF<=791[P\J ¸ >f3'; >fFV>Ë3Q>GOES*@C>GRE>f3=w 3';V1¡JK1+*.V1[FV][1
JK3QOEPK3\J%H˜P'D.< wEv.vEv.vEv=OEFV@j1[FV@J ¸ >Ë3Q;xw.wEw.wEwEwV-2¾¹1:;VO¼oE1 
wEvEv.vEv.v  w.¤.¤E¤E¤.¤ u¤,
wEwEv.vEv.v  wEw¤E¤E¤.¤ u ,¼s
wEwEw.vEv.v  wEw.w.¤E¤.¤ u ,hv
wEwEw.wEv.v  wEw.wEw.¤.¤ u ½
wEwEw.wEw.v  wEw.wEwEw¤ u w
wEwEw.wEw.w  wEw.wEwEw.w u ,

¾¹1:DE73QOI>GFr3';1q3QDI3\OIS F<=791[PDEH F<U7e1YPQJJK3QOEPK3Q>fFVRUH˜PQDE< wU1+*VOIS 3QDgsI½- Ž>G<X>fSAOIPQSGÏEc

Á Ev v.vEv.v  Á .w wEwEw.w u sI½


ÁEÁ v.vEv.v  ÁEÁ wEwEw.w uw.s
ÁEÁEÁ vEv.v  ÁEÁ.Á wEw.w u ¤Iv
ÁEÁEÁ.Á v.v  ÁEÁ.ÁEÁ w.w u ,Yv
ÁEÁEÁ.ÁEÁ v  ÁEÁ.ÁEÁEÁ w u Á
ÁEÁEÁ.ÁEÁ.Á  ÁEÁ.ÁEÁEÁ.Á u ,
,h¤E½

sIvEv.vEv.v  s .Á ÁEÁEÁ.Á u ,h¤E½


sEsIv.vEv.v  sEs ÁEÁEÁ.Á u pv
sEsEsEvEv.v  sEs.s ÁEÁ.Á u w.s
sEsEs.sIv.v  sEs.sEs Á.Á u ,hs
sEsEs.sEsEv  sEs.sEsEs Á u s
sEsEs.sEs.s  sEs.sEsEs.s u ,
¤Es.¤

½EvEv.vEv.v  ½.s.sEsEs.s u ¤Es.¤


½E½Ev.vEv.v  ½E½sEsEs.s u ,h¤E½
½E½E½.vEv.v  ½E½.½.sEs.s u sI½
½E½E½.½Ev.v  ½E½.½E½.s.s u ¤C,
½E½E½.½E½.v  ½E½.½E½E½s u ½
½E½E½.½E½.½  ½E½.½E½E½.½ u ,
Á ½¤

pvEv.vEv.v  p½.½E½E½.½ u Á ½ ¤

p v.vEv.v  
I ½E½E½.½ u ¤C,hv


I vEv.v  
 p ½E½.½ u Ì Á


 p v.v  
 
p ½.½ u ¤IÌ


 
I v  
 

I ½ u 


 
   
 

  u ,
pĤ
–0;VJ 3';1+FV<=791[P DEH ½.m@>fR.>Ë3 F<U7e1YPQJ ¸ ;V1[PQ1i@C>fR.>Ë3\J OEP'1FD.F5m>fFV][P'1hOEJ'>fFVR6JK3QOEPK3Q>fFRqH˜PQDE< ,Yv.vEvEv.vOEFV@X1[Fe@C>fFVR
¸ >f3'7
;  

 
:>GJ
IÄ.¤¨ Á ½.¤¨©¤EsE¤¨›,h¤E½¨©sI½+¨©¤C, ¨¶½=n , ,¼s ´
ŽC>fFV][1W¤IvEvs \, C,hs.nq¤EÄEvVc ¸ 16;VO¼oE1+3'Dt]4D.FVJK>A@C1YP*D.FSfÏr¤IÄEv=F<=791[P\J%>fFj3';1:J'1,*1YFV]41 ¸ >Ë3Q;j>GF>f3'>AOIS“@C>GRE>f3Ì-—¾•1
;eO¼oE1

ÌEvEv.vEv.v  Ì.s.sEsEs.s u ¤Es.¤


ÌE½Ev.vEv.v  ÌE½.wEwEw.w u w.s
ÌE½ Á vEv.v  ÌE½ Á ,E,hv u w

Á
–0;VJ3';1:PQ1,*>GP'1h@¡F<U7e1YPi>GJÌ.½ Á ,E,Yv -
y2z9{}|~Y}z9€*  
lm3U>GJ FD ¸ F•3';eOp3:3';V1¡F<=791[PWDEH ¸ O¼ÏCJ6DIH]\;DD.J'>fFR DE7C…1h]^3\J6H˜P'D.<  @C> 1[PQ1[F3:3ϏB91YJ:DEH%D.7C…1Y]43QJ¡ª ¸ >Ë3Q;
8 Ÿ
PQ1[B9143Q>Ë3Q>fD.FVJXOISGSfD ¸ 1Y@e­:>AJ   
¥ -™l F¶BVOEPK3Q>G][SAOIPhc
>ËH ¸ 1 ¸ OEF.3?3'D ¸ PQ>Ë3Q1  @C>fR.>Ë3XF<=791[P\J?VJ'>fFR  @C>GRE>f3QJ
OESfSGD ¸ >GFR=H˜D.PP'1YBe1[3'>f3'>GDEFVJ ¸ >f3';r3Q;1:OE@@C>f3'>GDEFeOIS“]4D.FV@C>f3'>GDEFj3';VOI303';1W@C>GRE>f3QJOEŸ BB91YOIP0>GFxFDEF5m>GFV]4PQ1YO.JK>GFR=D.PQ@1[Phc
8
¸ 1WJ'1[163';VOI33';>AJi]YOIFr7e1W@CD.F1q>fF»   
¥ ¸ O¼ÏCJY-

À+D ¸’¸ 1:R.P'D.Br3';1WR.>fo.1[FxF<=791[P\Ji>GF3'D¡@C >  1[PQ1[F3]4SAOEJQJ'1YJiOEFV@ ¸ P'>f3'1q3Q;1=F<U7e1YPiDIH ¸ O¼ÏCJ>fF ¸ ;V>G]\;†1hOE]\;
][SGO.J'J6][OEF791UD.7C3QOE>fF1h@ -:–2D E1Y1[B†3QPQO.]  ¸ 1XOISAJKD ¸ PQ>f3'1U3';1X]4<USAOp3'>GoE1?JKV<XJ6DEH
3';1?F<=791[PqDIH F<U7e1YPQJ
J'DxDE7C3\OI>GF1Y@“-j‡7VJK1YP'o.1=3Q;VOp3:3Q;1¡F<U7e1YPQJ63Q;1[<tJ'1[SGoE1YJ:OEP'1 ¸ PQ>Ë3'3'1YF¹>GFšOEJQ]41YFV@C>GFRjD.PQ@1[Ph-tŽCD ¸ 1X1 C;VOEVJK3
F<U7e1YPQJ0791[R.>fFVF>fFVR ¸ >Ë3Q;¹,.c3';1YFx7e1YRE>GFF>GFR ¸ >Ë3Q;¤UOEFV@xJKDXD.F“-

Ài<U7e1YPQJ >GRE>f3QJVJ'1Y@jDI3Q;1[P   


¥
8 6 V<=SAOp3Q>fo.1

Ÿ
3Q;VOIFr3Q;1qÊC1Y@jBVOIP'3 
J'<
7e1YRE>GFF>GFR ¸ >Ë3Q; , ,.c v ¤ s  \¥n ½ ½
¤ ¤cf,.c v w s   ¥ n ¤, ¤ 
w wVc ¤cf,.c v Á s  Å  ¥ n sE½ ÌEw
Á Á c wVc ¤cf,.c v s s    ¥+n ,¼¤I½ ¤IvEÄ
s sc Á c wVc ¤cf,.c v ½ s
8
   ¥ nZ¤EsE¤ Á ½,
½ ½Vc sc Á c wVc ¤cf,.c v  s
88
   ¥ n Á ½.¤ Ä.¤Iw
 Cc ½Vc sc Á c wVc ¤cf,.c v Ì s
8
   ¦ ¥+n pÄ.¤ , ,¼s
H˜P'D.< ÌEv.vEv.vEv:3QDtÌ.sEs.sEs.s sc Á c wVc ¤cf,.c v ½ s
8
   ¥ nZ¤EsE¤ ,hÄE½
H˜P'D.< ÌE½.vEv.vEv:3QDtÌE½Ew.wEw.w wVc ¤cf,.c v Á Á  ¥ n
 w.s ¤EvEv¤


–0;V1:F133';VP'1Y1q½.m@>fR.>Ë3F<=791[P\JOIPQ1:ÌE½ Á vEv.vcÌE½ Á ,Yv.vcÌE½ Á ,E,hv-


œ+1[FV][1q3';1=¤EvEvs¼3';jF<=791[Pi>GFj3';1WJ'1,*1YFV]41q>AJiÌE½ Á ,E,hv-
s-0/2143C 8 e
7 1WO?RE>GoE1YFjB9D.J'>f3'>GoE1q>fF3Q1[RE1YPY- º J'1,*1[Fe]41  8 n  8 Î) Î  9 Î DIHB9D.J'>f3'>GoE1:>GF.3Q1[R.1[P\J*>AJJ'V]\;
3Q;VOp3C Cc H˜D.P  %°¤CcC>AJDE7C3\OI>GF1Y@gH˜PQDE<

8 7ÏgO.@@C>GFRt  JKD.<X16FVDEFY1[¦ PQD?@C>GREÉ[>fÉ[3iÉ  DIH  8 -3 P'Dpo.1+3Q;VOp3



¢ª O.­g3';V1WJK1+*.V1[FV][16;eOEJOIF "! € F<U7e1YP



ª˜7e­g3';V1WJK1+*.V1[FV][16;eOEJ0>GFCÊVF>f3'1YSfÏj<XOEFÏ¡1[o.1[FrFV<=791[P\J[-

y2z9{}|~Y}z9€*

ª¢O.­r/“1[3 eJ O.J'J'<X1*3';VOI3 3Q;1[PQ10OIPQ1 FD61[o.1[FWF<U7e1YPQJ >GF=3Q;1%J'1,*1YFV]41.- –0;V>GJ <X1YOEFVJ“3Q;VOp3  8 >GJ D.7C3QOE>fF1h@
H˜PQDE< c7ÏgOE@V@C>fFVRXO?FD.F[1YP'D?1[o.1[F†@C>GRE>f3DIH  3'D cH˜DEP1YO.]\;  %,.-


3 #‹791q3';V1:Sf1[H 3+<XD.JK301YoE1YFr@C>GRE>f3i>GF 8 ¸ ;>A]\;r<tO¼Ï¡791q3QOE1YFj>GFg3Q;1qH˜DEPQ<


  
/“1[$
 8in &% 8
'%
¦ É[ÉYÉ
%)(*# Wƒ28^ƒ
¦ ÉYÉ[É
ƒ,+
; 1YP'1-% 8 Î.% Î ´[´Y´ Î/% ( OIPQ1?DC@@ @C>GRE>f3QJtª0 ´Î [´Y´ Î'ƒ
¸ 
1 %,.-
¦  8 ÎQƒ ¦ +

8 OIPQ1U1YoE1YF•D.P:D@V@  OEFV@ ƒ + DC@@ c


%Nv.­ ƒ
Ž>GFV]411YO.]\;U3Q>f<X1 ¸ 1+OEP'1OE@@C>GFR=OI3 SG1YOEJK3%¤63QDUOq3Q1[PQ<NDEH 3';16JK1+*1[FV][13'D=RE143 3';1F13 3Q1[PQ<xcOI3*JKD.<X1
JK3QOIR.1Ec ¸ 1 ¸ >fSGS“;VO¼o.16OU3'1YP'< DIH3';1qH˜D.P'<
 
n&%
¦ ÉYÉ[É
8
% ( #WÄ.ÄEÄ %
ÉYÉ[É
Ä32

¸ ;1YP'12 n‰wVÎQsCÎ0tD.P:Ä-XÀiD ¸‹¸ 1tOIPQ1=H˜D.PQ][1Y@3'D†OE@V@4# 3'DD  3QDxRE143 5 8¼c2O.Jq>Ë3W>AJ63';1XD.FSfÏ1[oE1YF


@C>GRE>f3+O¼opOI>GSGOE7Sf1.- º H 3'1YP+Op3i<XD.JK30H˜DEPiJK3'1YBVJDEH OE@V@C>Ë3Q>fD.F“c ¸ 1:JK1Y13Q;VOp3+J'DE<X1qF133Q1[PQ< >AJ0DIH 3';1qH˜D.P'<

76 0n8%
¦ [É ÉYÉ
% (9 8
vEv.v
É[ÉYÉ
5
%

¸ ;1YP'1 9 PQ1[BVSGO.]41YJ:#„DIHC  c 9 n;#k¨ ,.c


5 n,EÎQwÎ\sCc DEPC- ž C376X;VOEJ=FDFD.F[1YP'D1[oE1YF™@C>GRE>f3
]4D.F3'P\OE@C>A]^3Q>fFR?DEVPiOEJQJK<XBC3Q>fD.F“-
œi1YFV]41q3Q;1WJK1+*1[FV][1q;VOEJJ'DE<X1q1[o.1[FrFV<=791[P+O.J0>Ë3\J%3'1[PQ<x-

s
ª˜7e­jlmH93';1YP'1OIPQ1%DEFSGÏ:ÊVF>f3'1YSfÏU<tOIFÏ:1[o.1[FU3'1[PQ<tJ
OIFV@>GJ 3Q;1SAOEJK3 3Q1[PQ<xcp3';1YF?3';1iJ'1,*1YFV]41   8 n
  8 Î   ¦ Î ´[´[´  >GJ:D.7C3QOE>fFV1Y@•>fFšOxJK>G<X>fSAOIP:<tOEFF1[PWOEFV@¹;1[Fe]41X<=VJK3W;VO¼o.1XOEF•1YoE1YF3Q1[PQ<7  ϶ ª}O­^c O
]4D.F3'P\OE@C>A]^3Q>fD.F“-
–0;eJ   8 c;VO.J0>fFCÊeF>Ë3Q1[SGÏg<tOIFÏ¡1YoE1YFg3Q1[PQ<XJY-
  
 

½V-0Æ >GFV@xOISGS H˜FV]43'>GDEFVJ†u  J'V]\;r3';VOI3

j  e¦%¨
—ª ­K¥in—ª  ­¨ —ª
­ ª,¼­

 Î
Î X>GF-iª}œi1YP'1
H˜D.PiOISGS 2 @C1YFDI3Q1YJ%3Q;1WJK1[3iDIH—OISGS P'1hOIS F<=791[P\JY- ­

y2z9{}|~Y}z9€* – O>fFVR n


rnkvt>GF©ª,¼­^c ¸ 1URE1[3 —ª}v.­0n—ª¢v.­H˜DEPOESfS -œ+1[FV][1 ¸ 1=DE73QOI>GF—ª¢v.­0n v-
– O >GFR
Xn›vX>fF ª,¼­^c ¸ 1:RE1[3

—ª; ¦ ­ n—ª 9­ ª¢¤.­

ŽC>f<X>GSGOEP'SGÏ2rnv?>fFšª,¼­*RE>GoE1YJ

—ª
—ª ­K­
n —ª
C­ ª}w­

3 C3K3Q>fFVR!
Xn’,q>GFšª}w.­4c ¸ 16R.143
—ª"—ª ­K­ n# —ªK,h­%$&  ªÁ ­

À+D ¸ VJ'>GFRª¢¤.­*OEFV@ ª Á ^­ c ¸ 1qDE7C3\OI>GF

—ª;—ª;9­K­*n#—ª'—ª; ¦ ­K­ n& ¦ —ª,¼­ ª¢s.­

3 C3(
Xn# Un †>GFšª}w­0OISAJKD?RE>GoE1YF
—ª —ª 9­'­ n&—ª  ­ ª}½­

6 D.<XBVOIPQ>fFVRxªµsE­OEFV@ }ª ½­^cV>f3iH˜D.SfSGD ¸ J03';eOp3 ¦ —ª,¼­*n"—ª;9­4-*lmH ™n ˆ vVcC3';V1[F)—ª  ­*n·0cVH˜D.P+JKD.<X1W]4DEFeJ3\OIF3


·I- Ž>fFe]41*—ª}v­ n»vVc ¸ 1qV ; O¼o.1+—ª;9­ n·0xH˜D.P$jn›vXOEJ ¸ 1[SGSµ- Ž7VJK3'>f3'C3Q>fFVRX3';>AJ>GF ªK,h­4c ¸ 1:J'1[163';eOp3

·Eª e¦%¨™·,
- ­ n»·09¦*¨¶·,
-

D.P
· ¦
- =n›·.
 /$0
Î 1 ´

–0;V>GJ+>f<XBSG>f1hJ03';eOp3· ¦ n·I-%œ+1[FV][1W·+nvXD.PW,E-*¾•1:D.7C3QOE>fF2—ª;9­*nv?H˜D.P+OISGS   D.P—ª  ­*n"†H˜DEPiOESfS 2 -%lm3+>GJ


1hOEJ'Ït3'DXo.1[PQ>ËH˜Ït3Q;VOp33Q;1YJ'1q3 ¸ DXOEP'1qJ'DESGC3'>GDEFVJ0DEH3Q;1:RE>GoE1YFg1+*.eOp3'>GDEF2-

34353534353437686469353534353435353435:

½

 
"!$# &%(') *

+-,/.103240650879;:=<?>A@B2-CD9FEG2H@?CDEG>B2H06I@J9LKNMPORQ=CTS69VUT>AW69;U*XYGZ[YT\L]^5HEG_`2H02-EG>JCTS?_P9FCT>Aacb?EG5HIHET9;UTUT>J50 ,d 9FCef2H0W


g W69;065HCD9hCTS69h>B0?ai9;0jCDEG9h2H0?WkaF>JEGaF<_Pai9;0jCDEG9h5H]lCTS?9hCDEG>A2H06I@A9NEG9;UDbm9;aiCT>JnH9;@JoH,

p >^qr"ET5snH9hCTS2-C/e g >AUtbm9FETbm9;0?W?>AaF<@A2-E"CD5MRe,
p >A>^qvu8<?b?bm5UD9wM4e9ix=CD9;0W69;Wy_f9F9FCTULKVOz>A0v{|Q2H0?Wy}yQ~`2-ET9CTS69€_P>AW?b5>B0jCTU5H]‚MPORQ?MRKƒEG9;UDbm9;a„7
CT>JnH9;@AoH,(r(ET5[nH9€CTS?2-C/e3>AUtCTS69haF>AEGaF<?_Pai9;0…CDET9h5H]†CDEG>A2H06I@A9h}3{‡~P,

ˆ
‰‹ŠŒ Ž[‰‹‘"’

p >^q“(x=CD9;0?W‡MRevCD5v_f9F9FCwCTS69PaF>AEGaF<?_PaF>JE”aF@J9>A0–•f,4—cS69;0–˜*9f™8065[˜šCTS?2-Cw•hKz›š•hev›š•4OR, p u89F9
œ >JI<6EG9[q

F
E O K
L

C
D
B

d 9FCcM4ejej•ž› ŸyH¡‚,(¢/b?b‹@Jo=>B06Ifu8CD9F˜2-ETCF£¤ULCTS69F5HET9;_¥CD5CDEG>A2H0?I@J9wM4KN•fQ6˜9hIH9FC

ŸKL•€¦"§|¡ KLMR¦c›©¨ªŸ§|¡ «‹¨¬KNej¦"§­MReh®;e…•€«H¯

u8>B_P>A@A2-EG@AoHQ8u=CD9F˜°2-ETCF£¤UcCTS?9F5HET9;_¥CD5PCDE”>A2H06I@J9hMPOw•±I>AnH9;U

ŸOw• ¦ §­¡ MfO ¦ › ¨ Ÿ4§|¡ «?¨ Ohe ¦ §­MReh®;e…• « ¯

u8>B0?ai9€Ow•²›KN•fQUG<6³?CDEG2HaiCT>J50I>AnH9;U

¡ ¨ KLMR¦°´µMPOw¦ « › ¨ ŸR§¡ «¨ KNej¦°´Ohej¦ « ¯

¶ TU >A06ICTS694UTCT2H0?W?2-EGW–065HCT2-CT>J50?UVKNM·›ž\Q
MPOš›žXQ OwK±›±Zw2H0?W‡¸› p X€§¹Z(§º\;qD»-¼=Q
˜9IH9FC
KNe ¦ ›¾½ ¦ § p ¸L´µX6q ¦ 2H0?W¿Owe ¦ ›¾½ ¦ § p ¸/´µ\;q ¦ ˜/S?9FET9w½f>AUtCTS69h>A067EG2HW?>A<?U5H]KLMPOf,(—cS…<U

¡/¨¬\i¦t´µX…¦;«N›©¨&ŸR§¡ «¨ p ¸/´ÀX8qª¦t´ p ¸L´µ\;qª¦;«L›¥¨&ŸR§¡ «¨\°´µX8«sZ[¯

.ªC(]5@A@J5s˜LU‚CTS?2-C(9;>JCTS?9FE*\c›X45HE"¡ p \§kX8q*› p Ÿ°§¡lq1Z[,(Á°<6C"\/›X4>A_fb@B>J9;UCTS?2-C"Xf›ÂZc›\tUT>B0?ai9


X‹YGZ[YT\2-ET9f>A0–2-EG>JCTS_f9FCT>Aab?ET5HIHET9;UTUG>J50
,wÃ/5[˜*9FnH9FEsQ ˜*9fS?2snH9fCT2-™H9;0Ä20650879;:=<?>A@A2-CD9FEG2H@lCDEG>A2H0?I@J9
KNMPOR,v—cS=<?Uh\µ›¥ Å X–2H0WÀ˜93S?2snH93¡ p \c§¾X6qh› p ŸÆ§¹¡lq1Z[,kÁ°<6C\c§¾Xµ›¥¼HZf2H0WÀ˜95H³?CT2H>B0

+
¼ Z”¡› p ŸP§º¡lq1Z[,NÇÄ9fai50?aF@B<?W694CTS?2-C€Ÿy›ž¡‚,w—cS?>BU/>A0¿CT<6EG0¿CD9;@A@AUNCTS?2-CVek>AULCTS69R_P>AW87bm5>A0…Cc5H]
H
MR•f,*u8>B0?ai9 g •²› g MÆQ6˜*9€ai50aF@A<?W69wCTS?2-C g e>BUtbm9FETbm9;0?W?>AaF<?@B2-E"CD5MR•f,
ÈŠ^ŽsÉ=Ê‘lË6Ž[^̋É8Š^Í
¢Lb?b@Jo8>A06IRr(CD5@A9;_€o£¤UtCTS69F5HET9;_ÎCD5PCTS69€aio8aF@A>Aaw:=<?2HW6E”>A@A2-CD9FEG2H@mKNMfOh•fQ6˜9hIH9FC
KLMž®Oh•Ï§­KL•ƒ®;MPO ›MR•Â®OhK€¯
u8>B0?ai9"Ow•²›KN•fQ˜*9"IH9FCOw• p \s§wX6q‚›MR•f®ÐZt›MR• p \s§wX6qD»-¼=,‚—cS?>AUI>JnH9;UlMR•²›Â¼Ow•ž›Â¼-ej•f,
ÃL9;0?ai9€e>AUtCTS?9€_P>AW87bm5>A0…C°5H]‚M4•z2H0?Wy2HU/9;2-EG@A>J9FEc˜9ai50aF@A<?W69wCTS?2-C g eÆ>AUtbm9FETbm9;0?W?>BaF<?@A2-ECD5
MR•f,
ÈŠ^ŽsÉ=Ê‘lË6Ž[^̋É8Š^Í
Ñ 5>A0M g ,ÒÇÄ9wS?2snH9VCD54bET5snH9VCTS?2-CcÓ°MRe g ›ÂÔ-ÕÖQ8˜/S?>Aa”S>AU*9;:…<>Jn2H@A9;0jCCD54MRe ¦ §‡e g ¦ ›ÂM g ¦ ,
× GE 2s˜ÏejØÄbm9FETbm9;0?W>AaF<?@A2-ElCD5hKLMÆ,d 9FC"ÙW69;065HCD9tCTS?9caF>JEGaF<?_fE”2HW?>A<?U 5H]KNMPO2H0?Wf@J9FC(ښW69;0?5HCD9
>JCTUt2-EG9;28,°ÛN³UT9FETnH9wCTS?2-C/M g ›ÂÙRQ?e g ¦ ›ÂÙ ¦ ´¼-ÙV½=Q
MRØ \„X
MRe4› › p ¸/´ZiqiÜ ¯
ai5U p Mf»-¼q p
¸ ¸L´ÀZFq
—cS=<?U°˜9h5H³?CT2H>A0
X=\
MRe ¦ ›X=\ p ¸L´ZiqD»-¸V› Y
Ý
UT>B0?ai9wXYGZ[YT\h2-ET9>A0v2-EG>JCTS?_P9FCT>AaNb?ET5HIHET9;UTUG>J50
,(—cS=<?U°˜9€069F9;WvCD5PbET5snH9CTS?2-C
X=\
§­Ù ¦ ´À¼-ÙV½€›Ù ¦ ¯
Ý
—cS>AUET9;W?<?ai9;UtCD5b?ET5sn8>A06I¼-ÙV½€›X=\;»-Ý=,*Á°<6C
X=Z„\ Ú X8Z„\ X8Z„\ X…\
¼-ÙV½€›Â¼w® ® › › › Y
Þ Ú ¸ ¼H¸ Xh§ßZ‚§­\ Ý
<?UG>A06IRXw§ß\t› ¼HZs,*—cS?>AU°b?EG5snH9;UcCTS?9€aF@A2H>A_v,
p >A>^q Ñ 5 >A0ye…}y,wàL5HCD94CTS2-CwÓ°MRe g ›šÓ°MR} g ›±Ô-Õ Ö ,NÃL9;0?ai9fMÆYT}yYTeY g 2-ET9Rai50?aio=aF@B>Aa2H0W¿S69;0ai9
Ó°MRe…}á›`ÓtM g }á›$K€,3u8>B0?ai9Ó°}3MRek›·Ó°{‡MRKz›zMP»-¼=Q†>JCw]^5@B@J5s˜/UVCTS2-ChCDEG>A2H06I@A9;UwM4KN{
2H0?WvMRe…}â2-ET9UT>A_P>A@B2-E;,†—/S?>AU°I>JnH9;U
M4K MR{ KL{
› › ¯
MRe MR} ej}
à 5s˜9FnH9FE;Q˜*93S?2;nH9UT9F9;0µ9;2-EG@A>A9FEwCTS?2-C€MRe¿›$X…\;»-Ý=,vãk5HEG9F5snH9FERKL{›`Z”\;» p f
L X §¾\;q”,—cS=<?Uw˜9
5H³?CT2H>B0
MRK¹®sMR} + KL{®;MRe + X…\
M4{ä› › Ý-X=\-Y e…}·› › Ü ¯
MRe ¼ å MRK ¼ Ý
+ X=\
ÁoUDo=__f9FCDEToHQ?˜9€_4<?UDCcS?2snH9ej~±› Ü , œ >A0?2H@A@Ao
¼ Ý
Z + + X…\
ej{ä› ®sMR{æ› M4{ä› Ü ¯
Xh§­Z(§­\ Ý ¼ Ý
—cS=<?Ute…}·›e…~±›Âej{ç2H0?Wke3>AU°CTS69€aF>JE”aF<?_Pai9;0…CDET9w5H]‚}3{‡~P,
ÈŠ^ŽsÉ=Ê‘lË6Ž[^̋É8Š^Í
ÛN³‹UD9FETnH9NCTS?2-CcKL{ä›ÂZ„\;» p XL§À\;q(›\;»-¼€›KN~P,u6>A0?ai9VKLef³‹>AUD9;aiCTU°2H06I@J9wKQ6˜*9wUD9F9VCTS?2-CcKNej~
>AUtai50?IHEG<69;0…C/CD5KNej{|,"—cS>AU°I>JnH9;U/e…~è›ej{|,u8>B_P>A@A2-EG@Ao3Owej}>AUcai50?IHEG<69;0…CcCD5Ohej{éI>Jn=>B06I
e…}·›e…{|,‚ÇÄ9ai50?aF@A<W69VCTS?2-C/ej}¥›e…{ä›ej~,

¼
¼=,Lr"ET5snH9PCTS2-Cw]^5HEh9FnH9FEGo–bm5UT>ACT>JnH94>B0jCD9FIH9FEê|CTS?9FET9f9ix8>BUDCTUh2 Œ†‘lë ŒlÉ 5HEGW?9FET9;WÄb2H>JE p X‹YGZFqN5H]*bm5UT>ACT>JnH9
>A0…CD9FIH9FEGUcUG<?aGSvCTS2-C
+
꿛 p Xw§ßZ°´º+sq p Xh§ßZ´À¼q §­X‹¯
¼

ˆ
‰‹ŠŒ Ž[‰‹‘"’ ÇÄ9€S?2;nH9CD5Pb?ET5[nH9hCTS?2-CL얝Hí²îví¾ïðíƒW?9Fñ069;W³=o
+
ì p X‹YGZFq‚› p Xw§ßZ°´ß+sq p X€§ßZ´À¼q §ßXYóòmXYGZNôvítY
¼
>AUt2P³> õD9;aiCT>J50
, p à/5HCD9CTS?2-CcCTS69€EG>JIS…CcUT>AW69w>AUc2f02-CT<6EG2H@ 0=<?_³9FEs,Ðq4— 5fCTS?>BUt9;0?WvW69Fñ0?9

ê p ê3§Â+sq
ö p êlq‚› Y êµôví÷¿øsÕ=ù¯
¼
¢N0>AW69;245H] CTS69wbET5…5H]laF2H0³9V5H³?CT2H>B069;WƳ…oÆ@J5…5H™8>A06IR2-CtCTS69V]5@A@J5s˜L>A06I4CT2-³@J9V5H] n-2H@A<69;U°5H]ì p XYGZFq"]^5HE
UD5_P9hUT_P2H@A@mn-2H@A<69;Ut5H]X‹YGZs,

b
a 1 2 3 4 5 6

1 1 2 4 7 11 16

2 3 5 8 12 17

3 6 9 13 18

4 10 14 19

5 15 20

6 21

ÇÄ9R5H³UD9FEGnH9CTS?2-CNCTS694êm7CTS‡W?>A2-IH50?2H@lEG<?0?U/]^EG5_ p +-YDêlq&7CTSÄbm5UT>ACT>J50vCD5 p ê‚Y;+sq&7CTS‡bm5UT>JCT>J50¿2H0?WyCTS?9


9;0…CDEG>J9;Uw2-ET9fêßai50?UT9;aF<6CT>JnH9>A0…CD9FIH9FEGUFúlCTS69fñ?EGUTCL9;0…CDETo‡>A0‡CTS69Rêm7CTSµW?>A2-IH502H@†>BUV5069P_f5HET9fCTS?2H0‡CTS?9
@A2HUTC*9;0…CDETo5H]lCTS69 p ê3´|+sq&7CTSkW?>A2-IH502H@¬, œ 5HEt9ix62H_fb@J9VCTS69Vñ?EGUTC*9;0…CDETo3>B0Æû[7CTSkW>A2-IH50?2H@m>AUV+H+V˜/S?>Aa”S
>AU45069¿_f5HET9vCTS?2H0­CTS69k@B2HUDCf9;0jCDEGo5H] Þ 7CTSºW?>A2-IH502H@°˜/S?>Aa”Sß>AUv+FÕ8,¹ÛN³UD9FEGnH9ÆCTS2-C3û[7CTSºW?>A2-IH502H@
UDCT2-EGCTU]ET5_ü+H+Æ2H0?W|9;0?W?U˜/>JCTS¾+;ûy˜LS?>AaGS2HaFai5<?0…CTUf]^5HEPû¿ai50?UD9;aF<6CT>AnH902-CT<6EG2H@0…<?_³m9FEGUF,–—cS…<U
˜9UD9F9RCTS?2-Cwì p êk´¾+-Y;+sql§ +w›±ì p +-YDêlq”,LÇÄ9f2H@AUD55H³‹UD9FETnH94CTS?2-CLCTS?9ñ?E”UDCcêW>A2-IH50?2H@AUL9ix8S2H<?UDCL2H@B@
CTS69f0?2-CT<?EG2H@‚0…<_€³m9FEGUN]ET5_ý+RCD5vö p êlq”, p —cS=<?Uw2™=>B0?W–5H]"n8>AUT<?2H@†³> õD9;aiCT>J50Ä>AUV2H@JEG9;2HW6oyCTS69FEG9H,4ÇÄ9
]5HEG_P2H@A@AoPb?ET5[nH9€CTS69wb?EG5Hb9FEGC&oH,Ðq
ÇÄ9hñEGUDCt5H³UD9FEGnH9hCTS?2-C
ì p XYGZFq´Äö p XV§ºZ*´¼q(›XÆþºÕ8Y
2H0?W
p XV§ßZ°´Ï+sq p Xw§ºZiq p Xw§ºZ*´Ï+sq p Xw§ºZ*´¼q
ö p XV§ßZ´º+sq‚´Àì p XYGZFq(› ´ ´µXP›Z°´º+wÿºÕ8¯
¼ ¼

Ý
—cS=<?U°˜9€S?2snH9
p Xh§ßZ´º+sq p Xh§ßZ´À¼q
ö p Xw§ßZ°´µ¼qºì p X‹YGZFq(› §ßX­ö p Xw§ºZ*´Ï+sq”¯
¼

u8<?b?bm5UD9Vì ¨ X;YGZ « ›Âì ¨ X YGZ « ,"—/S69;0CTS69hb?EG9Fn=>J5<U°5H³UD9FETn-2-CT>J50kUTS?5s˜/UtCTS2-C


¦ ¦
öƨX†§­Z"´À¼q ìt¨¬XFYGZ”« ßöƨ¬X†§ßZ‚´Ï+sq”Y
ö ¨X §­Z ´À¼q ì ¨X  ö ¨ X ¦ §ßZ ¦ ´Ï+sq”¯
YGZ « ß
¦ ¦ ¦ ¦
u8>B0?ai9cCTS69NUD9;:…<?9;0?ai9
^ö p êlq 
 >BU"UDCDEG>BaiCT@Jof>A0?aiET9;2HUT>B06I6Qj>JC*]^5@A@A5s˜/U"CTS?2-CX§¿Z*›X ¦ §–Z ¦ ,‚Á°<6C*CTS?9;0
CTS69wEG9;@A2-CT>J50yìt¨¬X  YGZ  «L›Âìt¨¬X YGZ «h>A_fb‹@A>J9;UCTS?2-C/X  ›¾X 2H0WyZ  ›ÂZ ,*Ã/9;0?ai94ì‡>AU°5069i75069H,
¦ ¦ ¦ ¦
d 9FC(êk³m9c2H0joP02-CT<6EG2H@?0=<?_³9FEs,‚u8>B0?ai9tCTS69/UD9;:=<69;0?ai 9
^ö p êl q  >AU(UDCDE”>AaiCT@JoR>A0?aiET9;2HUT>B06I6Q˜9/aF2H0ñ‹0?W
2f0?2-CT<?EG2H@ 0=<?_€³m9F E UT<aGSvCTS?2-C

ö p R´Ï+s q ßê­ö p ?q”¯
“*:…<?>An2H@J9;0…CT@JoHQ
 ´º+sq
pf
ßê
 p€
 §+sq ¯ p +sq
¼ ¼

àL5s˜žUD9FC/Xf›¾êk´
 pf
 ´º+sq 2H0WyZt›f´µXw§+-,ÛN³‹UD9FETnH9wCTS?2-C/XÆþºÕ8,à/5[˜ p +sq°UTS?5s˜/UtCTS2-C
¼

XR›êk´
 pR
 ´º+sq   p 
 §+sq ´
 pf
 ´Ï+sq 
› ¯
¼ ¼ ¼
ÃL9;0?ai9Zt›R´µXh§¾+hÿƒ+-,"—cS=<?UcXÆ2H0?WyZN2-ET9h³m5HCTSbm5UT>JCT>JnH9h>B0jCD9FIH9FEGU/2H0?W

ì p XYGZFq(›
+
p Xh§­Z°´º+sq p Xh§ßZ´À¼q §­XR›
 pf
 ´º+sq §­Xf›¾ê‚¯
¼ ¼
— S>AU"UTS?5s˜/UCTS?2-C°9FnH9FEToÆ0?2-CT<6EG2H@0…<_€³m9FE°>AU*>B0CTS69NEG2H06IH9V5H]lì†,‚—cS=<?Utìk>BU*2H@AUT550…CD56,"ÇÄ9wai50?aF@A<W69
c
CTS?2-CNì¿>BUc2R³> õD9;aiCT>J50
,

Ý=,Ld 9FCðW69;0?5HCD9hCTS69€UD9FC/5H]†2H@A@mCDE”>Jb@J9;U p X‹YGZ[YT\Fqt5H]‚>A0jCD9FIH9FE”UF, × 9Fñ069h2f]<?0?aiCT>A50k얝 ïð³=o

ì p X‹YGZ[YT\;q(› p Xh§­Z(§­\Y/X8Z§ºZ”\(§­\iX‹Y/X8Z”\;q”¯

œ >A0?Wv2H@B@mCDEG>Jb@J9;U p X‹YGZ[YT\Fq/>A0ðUT<?aGSkCTS?2-CLì ¨ ì p X‹YGZ[YT\Fq « › p X‹YGZ[YT\Fq”,

ˆ ‰‹ŠŒ Ž[‰‹‘"’ ÇÄ94UTS?5s˜ CTS?2-CLCTS69€UD5@B<6CT>J50?UD9FCcai50UT>AUDCTUt5H]ø p  YTÕ8YTÕjq‚ú  ô!Nù÷¿ø p ´h+-Y;´€+-Y;+sqGù,/d 9FC/<U


b<?CcXw§ÏZ(§­\L›#"?QX=Z(§ßZ„\*§ß\„X›%$42H0?WkX8Z„\N› ì†,°—cS69€I>JnH9;0yai50?W>JCT>J50yìt¨ì p X‹YGZ[YT\;q1«h› p X‹YGZsYT\;q


>A_Pb@A>J9;U*CTS?2-C
"V&§ $§ßìk›X‹Y "'$°& § $[ìP§ß ì "›ÂZ[(Y "'$[ìy›\¯
—cS=<?UtX8Z„\)"*$sìy›ìm\V2H0?WyS69;0?ai9h9;>JCTS?9FE/\Fì›ÂÕf5HE/X8Z+"*$N›š+-,

,Ë -;/É .-’ u8<6b?bm5UD9f\F캛·Õ8,v—cS69;0À9;>JCTS69FE€\›·Õy5HE4ìߛ·Õ8,vÃL5s˜9FnH9FEP\f›©Õy>A_fb‹@A>J9;Uhìߛ·Õy2H0W


n8>Aai9i7nH9FEGUT28,V—cS=<?Uc˜945H³?CT2H>A0–Xw§¹ZL›0"Q1"h§2$€›žXQmX8ZL›3$R2H0?W!"'$›žZ[,L—/S69€ñ?E”UDCcC&˜5ET9;@A2-CT>J50U
I>JnH9€Zc›š ´ $H,(—cS=<?
U $N›X8Zc›±´c4 X $h2H0?5
W "*$N› Z°›š
´ $H,‚ÇÄ9IH9FC/9;>JCTS69F
E $V›Õf5HELXR6
› "f›±´h+-,
.ª]7$h› Õ8QCTS69;0‡Z/›²Õ2H0?W¿X3›8"›  Q‹UT2soH,/ÇÄ94IH9FCLCTS69CDEG>Jb@A9 p XYGZ[YT\;q°› p  YTÕ8YTÕjq”Qm˜/S69FET9  ô!V,". ]
$›
Å Õ8Q?CTS69;0kXf6
› "f›±´h+-,(Á°<6CcCTS?9;09"N§&$§­ìk›X3>B_fb@A>J9;U*CTS?2-C€´€+"§/$*§­Õ€›±´€+w]^5HE”aF>A06I:$N›ÂÕ8,
—cS=<?U°˜9hIH9FCcCTS69UD5@A<6CT>J50]2H_P>A@Ao p X‹YGZsYT\;q*› p  YTÕ8YTÕjq”Q‹˜/S69FET9  5
ô V,

Þ
,Ë-;É.;.-’ Å Õ8,‚.10CTS?>BU"aF2HUD9wX8Z+"*$N›ž+-,*Ã/9;0?ai9N9;>JCTS69FE°2H@A@‹2-ET9L9;:=<?2H@‹CD5Æ+-ú85HE°C&˜5R9;:…<2H@
u <?b?bm5UD9/\F쭛
8
CD5y+w2H0WvCTS69h5HCTS69FE/C&˜59;:=<?2H@
CD5y´h+-ú5HE/2H@A@m9;:=<?2H@
CD5¿´€+-,
u <?b?bm5UD9/XP›ÂZt›6"f›6$N›±+-,*—cS69;0ÆX/§ÀZ §À\c›6"PUTS65[˜/U°CTS?2-Ct\/›±´h+-,*u6>A_P>A@A2-E”@JoRìv›±´€+-,(Ã/9;0?ai9
8
$N›X=Z‚§ßZ„\"§­\iXf›±+c´º+/´º+/›±´h+hai50…CDEG2HW?>BaiCT>A06I<$V›ž+-,
u <?b?bm5UD94X–›·Z›â+P2H0?W="¿›>$›â´h+-,P—/S69;0µX§¾Z°§Ï\f›?"kI>JnH9;U\R›â´/Ý2H0W"€§6$c§ì›$X
8
I>JnH9;Ucìv›ÂÝ=,*Á<?CCTS?9;0kì›ÂX=Z„\/›±+"®+*® p ´/Ýq"›±´/Ý=Q?2Rai50jCDE”2HW?>AaiCT>J50
,*u6>A_P>A@A2-E”@Jo4XP›ÂZt›±´€+N2H0W
"R › $N›š+w>AUc065HCtbm5UTUT>A³@J9H,
.ª](Xƛ$+-Q ZN›`´€+-Q1"›z+-Q@$€›`´€+-Q‹CTS69;0‡Xh§¹Z(§º\w›0"I>JnH9;UV\V›`+-,wu8>A_P>B@A2-EG@Jo얛`+-,VÁ°<6CLCTS?9;0
ìv›X8Z„\/›±+†®i+†® p ´h+sq›±´h+L2ai50…CDEG2HW?>AaiCT>A50
,‚. ]
XP›±+-Q6Zc›±´h+-4 Q "4›±´€+-A Q $N›±+-Q8CTS69;0Æ\t›š´h+L2H0W
$R›±X8Z"§ÏZ„\°§º\iXk›·´h+t§ +N´¾+›·´h+R2H0?W‡2ai50jCDE”2HW?>AaiCT>J50–CD5 5 $4›¥+-,h—cS69RUDo8_P_f9FCDETo¿³9FC ˜*9F9;0
p X‹YGZ[YT\Fq42H0W p "BY $HYGì q4UTS65[˜/U4CTS?2-CfX›é´h+-Q*Zƛð+-7 Q "›é+-C
Q $y›é´h+Æ>AUR065HCbm5UTUG>J³@J9H, œ >B0?2H@A@JoÄ>J]
Xf›±´h+-Q8Zc›±+-' Q "f›±´h+c2H0 W $V›±+-QjCTS69;03\/›±´h+c2H0W3ìk›±´h+-,‚Á°<6C*CTS69;03ìv›X8Z„\c>AU(065HCUT2-CT>AUDñ9;W
,
—cS?9N50@JoaF2HUD9€@A9F]^Ct>AU°CTS?2-Ct5H]†XYGZ[Y("YB$€³m9;>A0?I42H@A@9;:=<?2H@mCD5k´h+R,(—cS?9;0v\t›š+w2H0?Wkìk›±+-,‚.ªCc>AU°9;2HUDo
CD5a”S69;aT™vCTS2-C p ´h+-Y;´h+-Y;+sqc>AUc>A0?W?9F9;Wv2fUD5@A<6CT>J50 ,
ÈŠ^ŽsÉ=Ê‘lË6Ž[^̋É8Š^Í
Å ÕP>A_Pb@A>J9;UCTS?2-C D \ED8ÿƒ+h2H0?WFD ì7D=ÿ +-,ÛV³UD9FETnH9hCTS2-C
\Fì|›

" ¦ ´À¼$V›X ¦ §ºZ ¦ §|\ ¦ Y X ¦ ´À¼HZc›6" ¦ §&$ ¦ §ºì ¦ ¯

¢NW?W?>A0?ICTS69;UT9hC&˜56Q˜9hIH9FC€´/¼ p Z(§G$[q(›ÂZ ¦ §­\ ¦ §/$ ¦ §ßì ¦ ,"—cS?>AU°_P2so³9w˜/EG>JCDCD9;0v>A0CTS69€]^5HEG_

p Z(§Â+sqª¦"§ $ §Â+sqª¦"§­\i¦"§ßìm¦°´À¼€›Õ8¯
p

ÇÄ9ai50?aF@A<?W?9PCTS?2-C\ ¦ §ì ¦ $¼=, ¶ UT>A0?IHD \ED‚ÿ¥+Æ2H0?WID ì7D†ÿâ+-Q˜*935H³?CT2H>A0D \D › +2H0WID ì7D ›ä+-Q
Z‚§+/›Õ2H0?J W $*§Â+L›Õ8,"—cS=<?UcZt›$V›±´h+-,"àL5s˜žXV§&"4›6"w§&$*§ºìR§ßXV§ßZ‚§­\w2H0WvCTS?>AU°I>JnH9;U
Z‚§­\(&
§ $°§ßìv›Õ8,(.ªCc]5@A@J5s˜LU°CTS?2-C/\/›Âìv›±+h2H0Wvñ0?2H@A@AoXR
› "R›±´€+-,

Þ ,Vu=5_P9 ÞLK UG:…<?2-EG9;UL2-ET9EG2H0?W65_@JovaGS65UD9;0¿]ET5_â2ÆÔPî–ÔPaGS?9;UTUN³52-E”Wy2H0?W¿2-ET9ai5@J5<?ET9;WyEG9;W


,Lu8S65[˜
CTS?2-C/CTS69FET9w9ix6>AUDCTUc2¼Pîk¼R³@J58aT™Æ5H] Þ UT:…<2-ET9;Ut5H]˜/S?>Aa”Sv2-C/@J9;2HUDC/CTS6ET9F9h2-ET9€ai5@A5<6ET9;WvET9;W ,

ˆ
‰‹ŠŒ Ž[‰‹‘"N
’ M 50UT>AW69FEw2b2-ETCT>ACT>J50‡5H]°Ô3îÄÔaGS69;UGUw³m52-EGWÄ<?UT>A0?IÆUT>Jx=CD9F9;0À¼3îļƳ@J58aT™–5H] Þ UG:…<?2-EG9;U
9;2Ha”Sy2H0?WvEG9;_P2H>A0?>A0?I4UD9FnH9;0…CD9F9;0–UT>A06I@J9hUG:…<?2-EG9;Ut2HU/UTS65[˜/0k>A0CTS?9wñ?I<6ET9w³m9;@J5[˜h,

1 2 3 4

7 6 5

16

8 9

15 14

10

11 12 13

.ª]‚2H0jov506945H]‚CTS69;UD9Æ+ K ³‹>JIUT:=<?2-ET9;UNai50jCT2H>B0–ÝET9;WyUG:…<?2-EG9;U/CTS69;0y˜*9R2-ET94W65069H,LÛV0¿CTS69ai50…CDEG2-EToHQ
9;2Ha”S3_P2soai50…CT2H>A032-C_f5UDC°¼wET9;W3UT:…<2-ET9;U"2H0?WCTS69;UD9V2HaFai5<?0…C*]5HE*2-C_f5UDCL+ K ®D¼›ÂÝH¼hET9;WUT:=<?2-ET9;U;,
—cS?9;0CTS69FET9€2-ET9P+ OPUT>A06I@A9VUG:…<?2-EG9;Ucai50?069;aiCD9;W¿>A
0 P;>JI-Q7 P;2-I]2HUTS?>A50
,. C/@A5…5H™8Uc2HUt]5@A@J5[˜/UF

û
ÇÄ9ÆUDb@A>JCVCTS?>BUh2-I2H>A0À>B0ÄCD5¿UD9FnH9FE”2H@*_P>AETET5HEh>A_P2-IH9;U€5H]td
7&UTS?2-bm9;WÄñI<6ET9;U€2HUUTS65[˜/0À2-³m5snH9H,v—cS?9FET9
2-ET93]^5<?EUT<?a”Sµ]5HET™8UF,. ]t2H@A@"CTS69ñ?nH9Æ<?0?>JChUG:…<?2-EG9;Uh5H]tCTS693ñ?EGUDCh]5HET™Ä2-ET9ÆET9;W
Q†CTS69;0µ˜93aF2H0IH9FCR2
¼fPUT:=<?2-ET9S?2;n8>A06IPCTS?ET9F9€ET9;W¿UT:=<?2-ET9;UF,tÃL9;0?ai9€CTS69FEG9aF2H0¿³9h2-CV_f5UDCL]^5<?E/<?0?>JCLUT:…<2-ET9;U/S?2sn=>B06I
ET9;Wßai5@A5<6E;,ºu8>A_P>A@B2-EG@JoHQCTS69FET9kaF2H0ß³m9v2-C_f5UTCfCTS6ET9F9vET9;WßUT:=<?2-ET9;UR]ET5_ç9;2Ha”Sº5H]NCTS69vET9;_P2H>A0>A06I
CTS6EG9F9w]^5HET™8UF,"—l5HIH9FCTS69FEN˜*9wIH9FC Þ §ßÝN®[Ý›±+;ÝRET9;WkUG:…<?2-EG9;UF,"—cS69;UT9wCD5HIH9FCTS69FEL˜L>JCTSkÝH¼R]ET5_©CTS69h³‹>JI
UT:=<?2-ET9;U/2HaFai5<?0jCN]5HE/50?@Jo Þ ûPET9;WkUT:=<?2-ET9;UF,°Á°<6C/˜9€™8065[˜ƒCTS?2-C L Þ K UT:=<?2-ET9;ULS?2;nH9ET9;Wyai5@J5<?E;,—/S69
ai50?aF@B<?UT>J50Æ]5@A@J5s˜LUF,

û=,/.10¿2Æaio8aF@A>Aa4:…<?2HW?EG>A@A2-CD9FEG2H@lKLMPOh}yQKLM©›±X‹QMPO蛱Z[Q Ow}⛚\[Q Ó°KNMPOš›`+;¼-ÕÖ-Q


2H0?WÄÓ°KLMR}æ›
Ý-Õ Ö ,"r"ET5snH9€CTS?2-C
p >^q\VÿºXw§ßZ[ú
p >A>^qHR \(§­X4´ \"§­Z*R › \°´µX´Zs,
Rå å R å
ˆ
‰‹ŠŒ Ž[‰‹‘"’
¢Lb?b@Jo8>A06IRai5UT>A0?9wEG<?@J9VCD5PCDEG>B2H06I@J9wKLMPOfQ6˜*9hIH9FC

KwO ¦ ›X ¦ §ßZ ¦ ´À¼-X8Z‹ai5U†+;¼-Õ Ö ›X ¦ §ßZ ¦ §­X8Zs¯

ÛN³‹UD9FETnH9wCTS?2-CNÓt}3KwO › Ó°}ÆMPO ›±+;¼-ÕÖt´Ý-ÕÖc›ÂÔ-ÕÖ-,(—cS=<?Ut˜9wIH9FC
K O ¦
w Þ
\ ¦ › › ¨ X ¦ §ßZ ¦ §|X8Z « ¯
ai5U ¦ Ý-Õ Ö Ý
u=5
Þ p X4´ÀZFq ¦
\i¦t´ p Xh§­ZFqª¦/› ¨X…¦"§ßZ„¦"§­X8Zi«c´ ¨&X…¦"§ßZ„¦(§ß¼-X8Zi«N› ÿºÕ8¯
Ý Ý
—cS>AUb?ET5snH9;UL\wÿºXw§ßZN2H0?WvCTS=<?U p >^q>AUcCDEG<69H,

B
b
C

30 30
a

A
c

K
œ 5HELb?ET5[n=>A0?I p >B>^q”Q?ai50?UT>AW?9FEtCTS69hb?ET58W?<?aiC

S ›©¨UTƧHVv§XWm«¨YT–´=Vy´ZW
«¨UTƧHV–´[Wm«‹¨YT–´=VƧXW
«HY

˜/S?9FET9 T‡› \(§­XQV–› \"§ßZN2H0?W9Wv› \°´µXR´µZ[,"“(x=b2H0W?>A06I4CTS69hb?EG5=W?<aiCFQ=˜9hIH9FC


å å å
S › p \"§­X6qª¦"§ p \"§ßZFqª¦"§ p \t´µX4´ÀZFqª¦t´À¼ p \"§­X6q p \"§ßZFq´À¼ p \"§­X6q p \t´µX4´ÀZFq´À¼ p \"§ßZFq p \t´µX4´ÀZFq
› ´/Ý-\i¦"§ Þ X=¦"§ Þ Z„¦"§ Þ X=Z
› Õ8¯

—cS=<?Ut2-C/@A9;2HUDCc5069h5H]†CTS69h]2HaiCD5HEGUL_<?UTCc³m9w9;:…<?2H@ CD5Õ8,u8>A0ai9\T§/VƧXWÄþºÕP2H0?WJT§HV–´ZW–þ¹Õ8Q
>JCt]5@A@J5[˜/U°CTS?2-CcCTS?9hb?ET58W?<?aiC°5H]†CTS69hET9;_P2H>B0?>A06I4C&˜5]2HaiCD5HEGU/>BUcÕ8,"—cS?>BU*I>AnH9;U

\‚§ßX´ \"§ßZt› \°´µX4´ÀZL5HE \(§­X4´ \(§ßZc›±´ \t´µX´Zs¯


å å å å å å
ÇÄ9€ai50aF@A<?W69wCTS?2-C
RR \"§|XR´ \"§ºZ RR › \t´µX4´ÀZ[¯
Rå å R å
K , p 2jqvr"ET5snH94CTS?2-Cw>J]ê|>BU/23b5UG>JCT>JnH9>B0jCD9FIH9FEwUT<?a”S–CTS?2-CLêßÿ Þ Õ6+H+ ¦ QCTS69;0–CTS69FET99ix8>BUDCTUV2H0¿>A0…CD9FIH9FE^]
+
UT<aGSvCTS?2-C/Z
ê 2] ¦ `_ +*§ ê‚,
¼-ÕHÕb
û a
p ³q œ >A0?W€CTS69*UG_P2H@A@J9;UDClbm5UT>JCT>JnH9>A0jCD9FIH9FEdcü]5HEl˜LS?>AaGS˜/S69;0?9FnH9FE†2H04>A0…CD9FIH9FE†êÆ>AU†UT<?aGSCTS?2-C†êµÿ2czQ
+
CTS69FEG9w9ix8>AUTCTUc2H0k>A0…CD9FIH9Fe
E ]ªQUT<aGSvCTS?2-C/ ê &]^ ¦ `_ +*§ ê‚,
¼-ÕHÕb
û a

ˆ
‰‹ŠŒ Ž[‰‹‘"’

p 2jqvd 9FCtêµÿ Þ Õ6+H+ ¦ 2H0?W5fäôví³m9hUT<?aGSkCTS?2-Cgf ¦ ß


 ê f §Â+sq ¦ ,"—cS69;0
p`

+ +
_ +*§
¼-ÕHÕû a êv´ f §Â+sq ¦
p` ÿ _ +"§
¼-ÕHÕû a f ¦ ´ f §¾+sq ¦

›
f ¦
´ ¼hfâ´º+
µ
¼-ÕHÕû
+
› ¨Qf ¦ ´ Þ Õ6+FÕifâ´À¼-ÕHÕûH«
¼-ÕHÕû
+
›
¼-ÕHÕû
_ p fδÀ¼-ÕHÕûq ¦ ´À¼-ÕHÕû ¦ ´À¼-ÕHÕû a
+
ÿ
¼-ÕHÕû
_ p Þ Õ6+H+L´À¼-ÕHÕûq ¦ ´À¼-ÕHÕû ¦ ´À¼-ÕHÕû a
+
›
¼-ÕHÕû
_ ¼-HÕ Õ K ¦ ´À¼-ÕHÕû ¦ ´¼-ÕHÕû a
+ ¼-ÕHÕ K
› p Þ Õ6+H+N´À¼-ÕHÕûq"› þºÕ8¯
¼-ÕHÕû ¼-ÕHÕû
—cS=<?U°˜9hIH9FC
+
ê f §+sqª¦
p$ _ +"§
¼-ÕHÕû a ê‚Y

2H0?W9] ¦ › f §Â+sq ¦ >AU°CTS69€W?9;UT>JET9;WvUT:=<?2-ET9H,


p$

O
p ³qÆÇÄ9UTS65[˜`CTS2-Cc › Þ Õ6+FÕ ¦ §±+Æ>AUhCTS?93ET9;:=<?>JEG9;WÀ@J9;2HUDC40…<_€³m9FE;,–u6<6b?bm5UD9Pêžÿ>cz,yÇÄEG>ACD9
ê|› Þ Õ6+FÕ ¦ j § Q ˜/S69FET9N‡>AUV2bm5UT>JCT>JnH9R>A0…CD9FIH9FE;,4àL5HCD9PCTS?2-CV˜9P_P2;o‡2HUTUT<_f9Rê2 Þ Õ6+H+ ¦ ³=o
b2-EGC p 2jq”,*àL5s˜

+ +
_ +"§
¼-ÕHÕûba
êk´ Þ Õ6+H+ ¦ › _ +"§
¼-ÕHÕûba
_ Þ Õ6+FÕ ¦ §&
a ´ Þ Õ6+H+ ¦

› Þ Õ6+FÕ ¦ §ß¼V® Þ Õ6+FÕV§&€§


 ´ Þ Õ6+H+ ¦
¼-ÕHÕû
› p Þ Õ6+FÕN§+sq ¦ §  ´º+sq §
pf
 ´ Þ Õ6+H+ ¦
¼-ÕHÕû
›  ´º+sq §
pR
 þºÕ8¯
¼-ÕHÕû
—cS=<?U°˜9h5H³?CT2H>A0
+
Þ Õ6+FÕ ¦ ºê Þ Õ6+H+ ¦ _ +*§
¼-ÕHÕû@a
ꂯ

ÇÄ9€a”S69;aT™vCTS2-Cec › Þ Õ6+FÕ ¦ ˜/>A@B@0?5HCt˜*5HET™, œ 5HENUT<6b?bm5UD9V꿛 Þ Õ6+FÕ ¦ ,"—cS69;0

+
_ +"§ ¼-ÕHÕû a Þ Õ6+FÕ ¦ › Þ Õ6+FÕ ¦ §ß¼N® Þ Õ6+FÕR› Þ Õ6+H+ ¦ ´º+\ Þ Õ6+H+ ¦ ¯

+
—cS=<?U°CTS69FEG9h>AUc065PUT:=<?2-ET9h>B0jCD9FIH9FEL³9FC ˜*9F9;0yêµ2H0?W _ +"§
¼-ÕHÕû@a
ê‚,

—cS>AUb?ET5snH9;U p ³q”,

k2kGkGkGkGk2k îßîºî kGk2kGkGkGkGk2kGkGl

m

 
  "!$#&% ')(+*-, ,/.
0213547698;:=<>624?A@CBEDFHGI:=<>?2J+8
KL624M?A@>BONP6Q8GSRT8;JBVUWBON<X624Y8;J:ZA[?AJ\F^]<>_Ba`O8=_8;JBH624?A@CB
]TBO85bBaBO47FVDc624MNd8=JBE]<>_Ba`O8=ZA:eZ2f-gF91h-:Zji2Bk8=J6Q8
l DF
m\n FHGonpMq
rtsTuwvyxjz{sT|~}
€ <>4`OBV<>_e8;JMB‚US<>NƒK…„†Z2<>4$8‡Z2fˆD‰GSRbBVJ6Ši2BHDŒ‹
‚G‹Ž‘Amƒ1 € <C4`aB"FH’“]<C_=BO`O8;_WgeDFHG”R-b•BY62@C_=Z
– 4Z—b˜8;J6Q8kGk’™‹›š+Ž‘œwš‚Ÿža ¡1 € <>4M`aBSFH¢]<C_=Ba`‘8;_
geDFV’PRMbBV62@C_=Z£J6Ši2B
A

FH’9¤ ‹ ’¥k¤ b/2

FVD ¤ ‚D ¤ q
¦ Z—b•B‘iABa:OR c E

FH’ ¤ ‹ FHG ¤ §Gk’ ¤ ‹¨š ¤  œwšš ©¤ Ž ža¤   ¤ª D

’¥ ¤ ‹ « m¥Ž ¬ šš+§Ž ž®­ ¤ q


B/4
B/4 ab/(a+c)

B/2
C

¯ _=<C4?£8;JBO_=Bk<>4¥8;JMB‚6Q]†Z—iABkB‘°ƒ„:Ba__=<>Z247624N"_<>US„@><±f³²ƒ<C4?R+b•BE?AB‘8
B
a

š ¤2´ œwš‰§žO  ¤ §Ž ¤2µ ‹¨ž ¤ œwž ¬ š®  ¤ q


¯ _=<C4?Sž ¤ ‹¨š ¤ §Ž ¤ 624MNdBO@><CUW<C46Q8;<C4?HŽ—RbB‚Z2]M8¶62<C4
ž‘· ¬ m2š+ž ¤ ¬ š ¤ ž ¬ mQš+·•‹¨¸ q
354$8=:=Z®N[`a<C4?£¹‹ºža2šR8=J<>_e:BaN[`OBa_8;ZS6£`a[]<C`EBa»+[6Q8=<>ZA4½¼
¹ · ¬ m¾¹ ¤ ¬ ¹ ¬ m‚‹º¸ q
¿ Z24_=<CNBa:e8;JMBkf{[4M`O8;<CZA4YÀˆœ{¹  ‹º¹ · ¬ m¾¹L¤ ¬ ¹ ¬ mHf{ZA:/¹eÁŸ¸Yœw62_žO2š9<>_e„TZA_<C8;<±iABŠ ‘1ÂÃZ2:
¸ n ¹¥ÄÅmƒRbBd_BaBÆ8;J6Q8SÀˆœ³¹ W‹Ç¹ ¤ œ³¹ ¬ mA  ¬ ¹ ¬ m n ¸M1ÈÉB"6Q@>_=Z\Z2]_=BO:iAB"8;J6Q8
Àʜ{¹ -‹Ëœ³¹ ¬ mA aœ³¹L¤ ¬ 0j  ¬ÍÌ <>_/_
8;:=<C`O8=@C²W<>4`O:=BŠ6Q_=<>4M?£ZA4ΜÏm ª¡Ð  ¡1-3…8‡<>_/Ba62_²98;Z”`aZAUS„[M8=B
À/ÑLlAAm2Ò‹ ¬ 0AÓ¢0 n ¸ ª 624MN Àˆœ p  -‹ Ì Á¸ q
¦ Ba4`OBS8=JBa:BW<C_k6d[4M<>»+[B”i26Q@>[B”Z2f¹<>4Ô8;JMBS<C4$8;Ba:
iQ62@œÏlA2m ª p  ‰_[`¶JÉ8;J6Q8VÀˆœ{¹ E‹^¸ƒ1
ÈÎBE`aZA4M`a@>[MNB8;J6Q8 l ž
m n š nŸpMq
rtsTuwvyxjz{sT|ÎÕ½ Ö BO8[_ˆ8;6 – Bg‡F£ Ì ‹¨×®1ˆØeJBO4¥g‡VFHGً¨g’9FHc‹¨×624NWg‡G‚FV’^‹
mQ׃1 ¯ _=<C4?”_=<C4BE:=[@CB<>4¥8;:<X624M?A@>BO_‡FH‚DÙ624MNPFH‚GSRbBE?ABO8
FV ‹ D
_=<>4eD _<>4‡× ª
FH ‹ G‚
_<>4‡ÚA¸2Û _<>4 p × q
0
€ <>4`OBHDI‹ŒGkWRtbBHZA]ƒ8¶62<C47_=<C4 p ×Ê_=<C4eDˋ˜_<>4‡×®1 ¦ Z—b•B‘iABa:ED˜‹ŒÚA¸ Û ¬©Ì ׃1ØeJ+[_
bBE?ABO8_<>4 p ׈`OZA_ Ì ×k‹º_<>4׃1ÜZ28;B8;J6Q8
ž ‹ 0 ‹ _<>4 p × ‹
š `OZA_ Ì × _<>4× p ¬)Ì _=<C4 ¤ × q
ØeJM<>_Æ_=JMZjb‡_Æ8=J6Q8džO2š np 1 ¯ _=<C4?ݞaQšÞ‹ p ¬&Ì _<>4 ¤ ×®R<±8Æ<>_¥Ba62_²8;ZÝ`aZAUS„[M8=B
`OZA_Tm2×E‹¢Ñ¡œßža2š®  ¬ 0ŠÒQ2mƒ1 ¦ BO4`aB
š ‹¨`OZA_ ×k‹ 0 ž 0—á ¤ 0
ž Ì mPà š ¬ ¬ q
€ [„„TZA_=BkžO2š9ĺlAAm®1-ØeJBa4ѶœwžO2š®  ¬ 0—Ò ¤ ĺÚ$ Ì 624NYšƒQžâºmAAlƒ1-ØeJ+[_
m Ä š ‹ 0 ž 0 á ¤ 0‚Ä Ú 0‰‹ 0
l ž mPà šY¬ ¬ ÓƬ Óª
b‡JM<>`¶Jd<>_‡62]_[:=Nt1ÈÎBE`aZA4M`a@>[MNB8;J6Q8žO2š9ÁŸlAAmƒ1
®m 1 Ö B‘8ã"]TB‰646¾8;[:=62@4+[UH]TBa: _=[`¶JS8;J6Q8 ãY‹~š$¤MPŽ¶¤Pž¡¤ŠR$f{ZA:_=ZAUSB‡46Q8=[:;62@4+[UV]†BO:=_
š ª Ž ª žj1h-:=Z—iABE8;J6Q8
ÚQãd‹¢ÑCät嚝Ýæ¾åŽ-ÝçQå
ž‘Ò ¤ ˜Ñ>ä ¤ šÝæ ¤ Ž-Ýç ¤ ž‘Ò ¤ ˜ÑCä · šÝæ · ŽˆÝç · ž‘Ò ¤ ª
b‡JMBa:=Bä+è—éê_OR†æ=è¾éë_aRTç¶èjéê_62:BV62@C@ | sT|íì$î+ï¾s <C4+8=Ba?ABO:=_O1‡ÂÃ[M:8;JMBa:aRT<Cf p NMZƒBO_ |ísÃx N<Ci®<CNBk6Q8
@CBŠ62_
8£ZA4B9Z2fš ª Ž ª ž¾R„:Zji2B¥8;J6¾8”Ú2ã`a624Ý]†BÆB¡°M„M:=Ba__=BON<>4Î8;JB¥f{ZA:Uðä/¨ñ®¤/ºò2¤jR
b‡JMBa:=Bð ª ñ ª òS62:=B46Q8;[:=62@y4+[UH]TBa:_ |ísT| î Z2fÊb‡J<C`¶JY<C_eN<Ci®<C_=<>]M@>B‰]$² p 1
rtsTuwvyxjz{sT|  3…8`a624"]†BEBa62_=<C@C²¥_BaBO4Y8;J6Q8
Ú2ãd‹ŒœÏm2ŽÊm2ž ¬ š®  ¤ ¨œÏm2žÊm2š ¬ ŽO  ¤ ¨œÏm2š‰m2Ž ¬ ža  ¤ q
ØeJ+[_b•B£`a624P8¶6 – B‰ätå/‹Ÿä ¤ ‹Ÿä · ‹ŒmƒR†æ¾åe‹cæ ¤ ‹Ùæ · ‹Œm96Q4N7çQåe‹cç ¤ ‹Ùç · ‹ ¬ 0A1
€ [„„TZA_=B NZ®Ba_•4MZ28/N<±i®<>NB?A`aNyœwš Ž žO ‘1ˆØeJBO4 NZ®Ba_N<Ci®<>NMB‰6Q8@>Ba62_8ZA4BZ2f š Ž žj¼
_=6Š² p NMZƒBOp _ 4MZ28N<Ci®<CNB‡š1-ÜZ28=Be8;J6Qª 8ª Ba62`¶J9ZQfym2ŽMPp m2ž ¬ šRm2žPm2š ¬ Ž/624N9m2šÊPm2ª Ž ¬ ª ž
<C_BO<C8=JBa:N<Ci®<>_<>]@CB‡]+² p ZA:4ZA4BZ2fy8;JMBaUI<>_ N<Ci®<>_<>]@CB‡]+² p RM62_8=JBN<±ótBO:=Ba4M`aB‰ZQf 624$²
85bZH_[UW_ <>_62@±b/6Š²®_•N<±i®<>_=<C]@>Be]$² p 1ˆ3…f p NZ®Ba_4Z28N<Ci®<>NMBm2ŽTÔm2ž ¬ šR®8;JBO49b•BJ6ŠiAB
8=JB¥:Ba»+[<>:BaNÝ:=BO„:=BO_=BO4+8;6Q8;<CZA4½1Ô3…f p N<Ci®<CNBa_”mQŽeÞm2ž ¬ šR-8;JBO4 p NZ®Ba_H4ZQ8£N<Ci®<CNB
mQŽˆmQž-§š1ô4"8=JBEZ28;JMBa:‡J624NtRb•B‚62@>_Z£4Z28=BE8=J6Q8
Ú2ãY‹ŒœÏmQŽˆmQž-§š®  ¤ ~œwm2ž ¬ m2š ¬ Ž‘  ¤ ~œ ¬ m2šm2Ž ¬ ža  ¤ ‹Ÿð ¤ §ñ ¤ §ò ¤ ª
b‡JMBa:=Bð"‹Þm2Ž29m2žAWšRAñ£‹~m2ž ¬ m2š ¬ Ž 624N”ò‚‹ ¬ mQš½9mQŽ ¬ žj1 € <C4`aB ð ¬ ñS‹ p œwŽ2Wš® 
6Q4N p NZ®Ba_‰4Z28N<±i®<>NBEðíR½<C8f{ZA@>@CZ—b‡_8=J6Q8 p NZƒBO_4Z28‰N<Ci®<CNBVñY62_bBa@C@Ï1 € <>US<>@X6Q:=@C²2R
bBE`aZA4M`a@>[MNB8;J6Q8 p NZ®Ba_‡4Z28‡NM<Ci®<>NBEòƒ1
p 1 :Ö BŠB‘628@yõö 624NPãÉ]TBV„TZA_=<±8;<Ci2B‚<C4$8;Ba?2Ba:=__=[`¶J78=J6Q88;JB‚Ba»+[6Q8=<>ZA4Pðä ¬ õ¥ð”ãԋ˜¸SJ62_
:ZƒZQ8;_÷)624NYø 1 h-:=Z—i2B‚8;J6Q8÷)624NPøÉ62:=BE<C4+8=Ba?ABO:=_<CfÊ624NdZA4@±²Æ<Cf/ùúõ"÷íû®ÞùüõÆøyû
<C_e8;JB‚_=»+[62:BHZ2f-6Q4P<>4$8=Ba?ABO:a1‚œ ¦ Ba:=B¥ùüðûýNBO4Z28;BO_8=JBV@>62:=?ABO_8<>4$8=Ba?ABO:‰4Z28B‘°ƒ`OBaBaNM<>4?
ðí1þ 
rtsTuwvyxjz{sT|  3…fˆ÷Ý624N"øÉ62:=BE]TZ28;J"<>4$8=Ba?ABO:=_aR8=JBa4
ùüõ"÷ û®¨ùúõÆøyû†‹ºõ"÷"ÝõÆøԋ¨õ Ñ ÷ƝÝø Ò ‹¨õ ¤ q
m
ØeJM<>_e„:Zji2Ba_ZA4BE<>US„@><C`Š6Q8=<>ZA4t1
ô]_Ba:
iABE8;J6Q8e÷ƝÝø7‹¨õÅ624Nd÷ýø7‹¨ãÊ1ˆÈÎBE[_=B8=JBE„:=ZA„TBa:
85²ÆZ2f <>4$8=Ba?ABO:/f{[4`O8=<>ZA4tÿ
ð ¬ 0 n ù ðûíÄðPf{ZA:‡624$²":BŠ62@½4+[UV]†BO:‡ðí1ØeJ+[_
õ ¤ ¬ mE‹¨õ©Ñ5÷9Íø Ò ¬ m‚‹ õ"÷ ¬ 0ÍõÆø ¬ 0 n ùüõ"÷ ûAŸùúõÆøyûíğõ©Ñ5÷¥Éø Ò‹ºõ ¤ q
€ <>4`OBkõo624NPã62:B‚„TZA_<C8;<±iABE<>4$8=Ba?ABO:=_aRÃ]TZ28;JY÷Ý624NYøÍUH[_
8]TBk„†ZA_<C8=<CiABQ13…fˆõoâÞm®R
bB‡ZA]_Ba:i2B8=J6Q8-8=JBa:B<C_-4ZE_=»+[62:B]TBO85bBaBO4¥õƤ ¬ m624NSõƤ—1 ¦ Ba4`OB2R®Ba<C8=JBa:-õ¢‹˜0
Z2:kùúõ"÷íûƒ~ùüõÆøíû†‹ºõƤ—1 3…fÊõ ‹˜02R8;JBO4P÷"Ýøԋ˜0E<>US„@><CBa_•8=J6Q8‡]TZ28;JY÷)624Ndø)62:B
„TZA_<C8;<±iAB‚:=BŠ6Q@>_‰_U96Q@>@>BO:e8;J624)021 ¦ BO4`aB£ã΋Œ÷ ø)`Š6244MZ28]TB£69„TZA_=<±8;<Ci2BV<C4+8=Ba?ABO:a1‰ÈÎB
`OZA4`O@>[NB8=J6Q8‚ùüõ"÷ û®¨ùúõÆøyû†‹¨õ ¤ 1h-[M88;<>4M?Sõ¢‹~÷ƝÝøÉ<>4Æ8=J<>_e:Ba@X6¾8;<>Z24½RƒbB‚?2BO8
 ÷ ¤ Ýã    ø ¤ Ýã ‹ Ñ5÷"ÝøýÒ ¤
q
¯ _=<C4?Yùü𣠝 ®ûT‹Œùüðûƒ  ¥f{Z2:‡624$²":=BŠ6Q@½4+[UH]TBa:eð\624MNY<C4+8=Ba?ABO: †R8;JM<>_e:=BON[`OBa_8=Z
 ÷ ¤    ø ¤  ‹~÷ ¤ ©ø ¤
q
ØeJM<>_e_=JMZjb‡_8;J6¾8÷ ¤ 624N"ø ¤ 62:B]†ZQ8;Jd<>4$8;BO?ABa:_a1ô4"8=JBEZ28;JMBa:‡J624NtR
÷ ¤ ¬ ø ¤ ‹ Ñ ÷"Ýø ÒÃÑ ÷ ¬ ø Ò ‹ºõ Ñ ÷ ¬ ø Ò q
ØeJ+[_
Ñ5÷ ¬ øý҉‹ ÷ ¤ õ ¬ øí¤ ª
<C_6:;6Q8=<>ZA46Q@4+[UH]TBa:a1 € <>4`OB÷Edø\‹ºõ <>_6E:;6¾8;<>Z2462@M4+[UV]†BO:aR+<C8-f{ZA@C@>Z—b‡_Ê8=J6Q8 ]†ZQ8;J
÷Ÿ6Q4NÉø62:=B”:;6¾8;<>Z2462@ˆ4+[UH]TBa:_a1 ¦ Z—b•B‘iABO:aRˆ]TZ28;JÎ÷ý¤‚624Nøí¤V6Q:=B”<>4$8;BO?ABa:_a1 ¦ BO4`aB
Ba62`¶JdZ2fˆ÷)6Q4NYøÎ<>_‡624d<>4$8=Ba?ABO:a1
Ì 1 <CÖ _-B‘_; 8 62É<CNH‹ 8;ZEœw`ašMZ2å :=ª :=š BO¤ _=ª „Tš ZA· 4ª NSqŠqŠ8;q Zkª 62 š 4”
A <>4$]TiABHBO:=6¥_<>„TZABa4”:UHZ2f[Mˆ8¶6¾R+8;<Cf<>Z2 4PZ2fœL]M0 [Mª 8 m š ª p ýª ÁqŠqŠš¾q èjª 1eãyœ ¡Ê 1 ° Ë6QUW„„6Q@C<>B2:Vÿýœw35 š 4” ª 8=šjJèOB  
„TBa:UH[M8;6Q8;<CZA4SœÏm ª Ì ª l ª p ª 0j ‘RŠ8=JBa:B 62:=B ‡<>4$iABO:=_=<CZA4 n _í`a Z2:=:=BO_=„TZA4N<C4?‡8;Z‡8;JB„62<C:=_œÏm 0j ‘R
œ Ì ª p  ¡Rkœ Ì ª 0j ‘Rkœwl ª p  ¡RkœÏl ª 0— ‘R‚œ p ª 0j ‘1þ  ¦ Z—bÇU96Q4+²§„TBa:=UV[M8¶6Q8=<>ZA4M_9Z2fWœL0 ª m ª p ª qŠqŠq ª ãy ¡R
œ{ãÔâ p  ‘RJ6—i2B‚B¡°M62`O8=@C² x Hs <>4$iABO:=_<>ZA4_ 
rtsTuwvyxjz{sT|  354Æ6‚„TBa:UH[M8;6Q8;<CZA49Z2fœ
0 m ã½ ‘Rƒ85bZH<C4+i2Ba:_=<>Z24_/`Š6Q4¥Zƒ`O`a[:<>4WZA4@C²
Z24BEZ2fý8=JBf{ZA@>@CZ—b‡<>4?k85b•ZSb/6Š²®_aÿ ª ª p ª qŠqaq ª
œ  ÆØ/b•ZWN<>_ ZA<>4$8e`OZA4_=BO`a[M8=<Ci2BV„62<C:=_‡62:B<>4$8;BO:=`¶J6Q4?ABaNtÿ
œ
0 ª m ª p ª  ¬ 0 ª  ª   0 ª  m qŠqŠq  ¬ 0 ª  ª E 0 ª m ª qŠqŠq ª ã½ 
¬  œ
0 ª m ª qŠqaq  ¬ 0 ª  º0 ª  ª  m ª qŠqŠq ª  ¬ 0 ª Eº0 ª  ª Em ª qŠqŠq ª ã½  q

œ e ! 6Q`¶J§]@CZƒ` – Z2f8;J:BaBÆ`OZA4_=BO`a[M8=<Ci2BY<C4+8=Ba?ABO:=_£`a624Ý]†BƄTBa:UH[M8;BONÝ<>4Ý624$²ÍZQf8=JB
f{ZA@>@CZjb‡<C4?”mVb•6—²®_O¼
œL0 ª m ª p ª qŠqŠq  ª E 0 ª Em ª qŠqŠq ª ã½  "¬  œ
0 ª m ª qŠqŠq ª Em ª  ª Eº0 ª qŠqŠq ª ã½ ‘¼
œL0 ª m ª p ª qŠqŠq  ª E 0 ª Em ª qŠqŠq ª ã½  "¬  œ
0 ª m ª qŠqŠq ª Eº0 ª k§m ª  ª qŠqŠq ª ã½  q
¿ Z24_=<CNBa:E`Š6Q_=BPœ  ¡1íÂZA:  ‹ 0ARy8;JBO:=BW62:=B£ã ¬ p „†ZA__=<C]@>BHiQ62@C[Ba_EZ2#f †¼ f{Z2:  ‹ m®R
8=JBa:Bk62:=Bã ¬ÎÌ „TZA__=<>]M<>@><±8;<CBa_f{Z2$: d6Q4N"_=Z”ZA4½1ØeJ+[_‡8;JB4+[UH]TBa:‡Z2fý„TBa:=UV[M8¶6Q8=<>ZA4M_
b‡<±8;JÆ85bZW<C4+i2Ba:_=<>Z24_‡Z2fý8=J<>_e85²®„TBk<C_
0 §m‡&  %'%'%j¨œßã ¬ p  -‹ œ{ã ¬ p  am œ{ã ¬ mA  q
p
53 4"`a62_=BWœ(/ ‘RMbBE_=BaBE8;J6Q8e8;JBO:=BE62:BEã ¬ mH„†BO:=UH[ƒ8¶6Q8=<>ZA4_•Z2f BŠ62`¶Jd85²ƒ„TB2R_<>4`OB)¥`Š624
8;6 – BiQ62@C[Ba_ef{:=Z2U08;Z”ã ¬ mƒ1 ¦ BO4`aBbB‚?2BO8‰mœßã ¬ mA /„†BO:=UH[M8;6Q8;<CZA4_/ZQfý8=J<>_e85²®„†BQ1
Âý<>462@C@C²2R®8=JBE4®[MUH]†BO:‡Z2fʄTBa:UH[M8;6Q8;<CZA4_•b‡<±8;J *x Vs <C4$iABa:_=<CZA4_<>_
œßã ¬ p  Oœßã ¬ mA  §mœßã mA ‹ œßãW 0j aœ{ã ¬ mA 
m ¬ m q
l®1 Ö B‘8•D‰FHGc]TB6k8;:<X624M?A@>Be<C4Wb‡J<C`¶J¥DF˜‹ºD‰GS1 Ö BO8•’^]TB‡8;JBUW<CNƒK5„TZA<>4$8Z2f½FHGc624N
+ ]TB6V„TZA<>4$8•ZA4¥D’Y1 € [„„TZA_=BŒ<>_ 8;JB‰f{Z®Z28Z2fí„TBa:=„TBa4MN<>`O[@X62: f{:=ZAU + ZA4¥D‰GS1-3…f
D + ‹ F + ‹-,íR FH’ ‹ õo624MNPò‚‹¨õƤjœL0•,† ¡R„:=Z—i2Bk8=J6Q8
+ ’ +  D’
ò ¤ ¬ Ñ , · ¬ , ¤ ¬ m Ò ò‰ 0‹º¸ q
¦ Ba4`OB‚_JZ—b~8=J6Q 8 ,Yâºm”624.N ,Ƌ~m£<Cfý624NdZA4@±²¥<±fýD‰FHGË<>_eBO»+[<>@>6Q8;BO:;62@w1
rtsTuwvyxjz{sT| 
Ö B‘8D’Ç0 ‹ /yR + ’ ‹Ùñd624N\FH’Ç‹c’¥G›‹˜š1ÈÉB
Z2]_=BO:iABE8;J6Q8F + ¤/‹~š$¤ý©ñ®¤Š1 1YZ2:=BaZ—i2Ba:aR
’¥G ‹ šTœ / ¬ ñ 
A

+ Ù‹ + D\_=<C4g’WDG&‹Ë(œ / ñM 


¬ D‰G Ž ª
b‡JMBa:=B"Ž9‹ D‰G™‹ DF91 ¯ _=<>4M?7D +  + ’ ‹ (œ / ¬
A/2

b
h-y
E
ñM =¾ñ†R+b•BkZA]M8;62<>4Æñ£- ‹ /Ðœ
0  ,T ‘1-ØeJ+[_
F + ¤ ‹ œwš ¤ Ýñ ¤  Ž ¤
P

, ¤ ‹
y
+  ¤ œ(/ ¬ ñM  ¤ š ¤ q
B
[M8‚œ/ ¬ ñ  -‹-,ñ£‹2,/ÐœL0 ,T ‡6Q4NPŽ¶¤•‹~š$¤ý/¤—1-ØeJ+[_bBEZA]M8¶6Q<>4
a D a C

,43/‹
œßš$¤¾œ
0,T 
¤ý/¤‘ Oœwš$¤ý/¤‘ 
š¤/ ¤ q
¯ _=<C4?”õ ‹¨š®  /Y624NYòV‹¨õƤ—œ
0  ,T ‘R8=J<>_e_<>US„@>6< 5Ba_•8=Z
ò ¤ ¬ òœ ,· ¬ , ¤ ¬ mA yº0‹~¸ q
7 <±i®<>N<C4?H]$²"òMR8=J<>_‡?A<±iABa_
ò ò0 ‹2, · ¬ , ¤ ¬ m q
¦ Z—b•B‘iABa:EòÞœ
0jQò$ âÙmSf{ZA:‰624$²P„TZA_=<±8;<±iABk:=BŠ6Q@ý4+[UH]TBa:òM1ØeJ+[8_ , · ¬ ,ä ¬§Ì âÙ¸ƒ1
ØeJM<>_eUW6—²9]†Bb‡:<C88;Ba4"<C4"8=JBf{ZA:U“9Ñ , ¬ mQÒÃ:Ñ ,¤ý  ,‚§mQ҂⠸M;1 •[ƒ 8 ,¤ ,‚§mHÁŸ¸ƒ1
œßÂZA:‰B¡°6QUW„@CB2R†ZA4B£UW6—²Y`¶JBO` – 8;J6¾8<C8=_‰N<C_=`a:<>US<>46Q4+8<>_‰4Ba?$6¾8;<Ci2B21þ  ¦ Ba4`OB<,ÍâŒm®1
3…=f ,Œ‹ mƒR8=JBa4ºòW œL0j2ò$ \‹ mÍ624N JBa4M`aBÎò~‹ 0A1ŒØeJ<C_Æ?A<Ci2Ba_Æõ ¤ ‹ 0j p ZA:
8;624íœ{DAmA -‹ õ ‹˜0—? > p 1 ØeJ+[_D¨@ ‹ A¸ Û 624MNdJMBa4`OB‚D‰FHGË<>_eBO»®[M<>@X6¾8;Ba:=62@Ï1
¿ Z24+i2Ba:_=Ba@±²AR/<±f‡8;:=<>624?A@CB¥DFVGo<>_£Ba»+[<C@X6Q8=Ba:=62@ÏRˆ8=JBa4©õ ‹8¶624íœ{DAm2 W‹ 0j > p 624N
JMBa4`OBEòV‹ËœL0ý,†  p 1 € []_8=<C8=[M8;<C4?V8;JM<>_•<C498;JMBBa»+[6Q8=<>ZA49_;6Q8=<>_A5BaNÆ]$²ÆòMR®b•BZ2]M8¶62<C4
Ñ 0 , Ò ¤ ¬ p Ñ 0  , ÒTÑ , · ¬ , ¤ ¬ m Ò §Ú‚‹¨¸ q
ØeJM<>_EU96Š²P]†B£b‡:=<±8=8=Ba4<>4\8;JB£f{ZA:U Ñ , ¬ m ÒTÑ p , · &  B, ¤ ŸBÓ ,9ŸÓ Ò ‹^¸ƒ1 ¦ Ba:B”8=JB
_Ba`OZA4NYfß6Q`O8;Z2:e<>_e„TZA_=<±8;<Ci2B]†BO`Š62[M_=BC,YÁŸ¸M1ˆÈÎBE`aZA4`O@>[NMBE8=J6Q8D,Ƌ¨mƒ1
Ì
ƒ13…f ð
ª ñ ª ò”62:=BE„TZA_=<±8;<±iAB:=Ba62@y4+[UH]TBa:_aR„:Zji2Bk8=J6Q8
Ñ ð”©ñ§ò$  ¤ Ñ ñMò§ò¾ð”ÝðÃñ Ò ¤ Ä p Ñ ñ ¤ ©ñƒò§ò ¤ ÒTÑ ò ¤ §òQð£Ýð ¤ ÒÃÑ ð ¤ ÝðÃñEÝñ ¤ Ò q
rtsTuwvyxjz{sT|~} ÈÎBk]TBa?2<>4Æb‡<C8=J"8=JBEZA]_Ba:iQ6Q8=<>ZA4Æ8=J6Q8
ð ¤ )ðÃñE©ñ ¤ ‹ p œ³ð”©ñM  ¤  0 œ{ð ¬ ñM  ¤ â p œ{𣝩ñM  ¤ ª
Ì Ì Ì
6Q4N"_=<>US<>@>62:•]TZA[4MN_ef{ZA:eñƒ¤ýÝñMò§òQ¤—Rò2¤-©òQð”)ðä—1-ØeJ+[_
Ñp wð ¤ )ðÃñE©ñ ¤ ÒTхñ ¤ ©ñƒò©ò ¤ ÒÃÑ5ò ¤ ©òQðS)ð ¤ ÒVâ ÓM 0 х𣝩ñƒÒ ¤ хñ©òAÒ ¤ хò‰)ðTÒ ¤ q
Ì
ØeJ+[_<C8/<>_e_=F[ EW`O<>BO4+88;Z”„:Z—iABE8;J6Q8
Ñw𔝩ñ§òA҆ÑwðÃñEÝñMò§ò¾ðTÒHÄ ÓÚ Ñwð”ÝñMÒÃÑÏñE©òAÒTÑ5òÝðTÒ q
-»+[<±i26Q@>Ba4$8=@C²ARƒb•BE4BOBaNY8=Z”„:=Z—iABE8;J6Q8
Ó Ñ ð£ÝñE§ò ÒTÑ ðÃñÝñMò©òQð Ò Ä Ú Ñ ð£Ýñ ÒÃÑ ñ§ò ÒTÑ ò‰)ð Ò q
¦ Z—b•B‘iABa:ORTbBE4Z28=BE8=J6Q8
Ñ ð”©ñ ÒÃÑ ñE©ò ÒTÑ òÝð Ò ‹ Ñ ð£ÝñE§òA  Ñ ñMò©òQð”)ðÃñ Ò ¬ ðÃñMò q
ØeJ+[_‡8;JMBk:Ba»+[<C:=BaNd<>4MBa»+[62@C<C85²S8¶6 – Ba_‡8;JBf{ZA:U
Ñwð”ÝñMÒÃÑÏñ§òAÒTÑ5ò‰)ðT҂âºÓQðÃñƒò q
ØeJM<>_/f{ZA@C@>Z—b‡_/f{:=ZAG U H1"9K IJ1o<C4Ba»+[62@C<C8=<>Ba_O¼
ð”Ýñ9â¨m > ðÃñ ª ñE©òSâºm > ñMò ª òÝðPâºm > ò¾ð q
rtsTuwvyxjz{sT|ŸÕ½Ö B‘8_<C4+8=:=Z®N[`OBE𣝧ñS‹~žjRÃñE§ò£‹Þš9624NYòÝðY‹¨Žj1ØeJBa47š Ž ž
Q6 :=B98=JBÆ_=<CNBa_”Z2f6P8;:<X624M?A@>BQ13…f K7‹ œßš”~Ž‡~žO =Am®R8=JBa4Ý<±8£<>_HBa62_²Î8;ZÔ`Š6Q@>`a[Mª @X6Q8=ª B
ðƋ2K ¬ šÃRñ£‹2K ¬ Ž—RÃò‚‹-K ¬ ž‰624N"ð£ÝñE§ò‚‹-KA1ˆÈÉBk62@C_=Z”ZA]_Ba:
iABE8;J6Q8
ð ¤ WðÃñ 9ñ ¤ ‹˜œ{ð 9ñM  ¤ ¬ ðÃñS‹¨ž ¤ ¬ 0 œwž®¥š ¬ Ž‘ aœßžƒ¥Ž ¬ š® -‹ p ž ¤  0 œwš ¬ Ž‘  ¤ âIp ž ¤ q
Ì Ì Ì Ì
1dZA:=BOZji2Ba:ORðÃñÍñƒòe)òQðd‹Ë(œ K š® a(œ K Ž‘ ½ œ K Ž‘ a(œ K žO t œ K žO a(œ K š® ‘1-ØeJ+[_
<±8e_=<y_L[ ES`a<CBa4$88=Z”„:=Z—iABE8;J6Q8 ¬ ¬ ¬ ¬ ¬ ¬
KNM œ K š® Oœ K Ž‘ /Ä š+Ž¡ž
Ú
¬ ¬ Ó q
[M8ŠLR O›œ K ¬ š® a(œ K ¬ Ž‘ ‹ çœ ÌBP Ôç2 ‘R$b‡JBO:=Bç ª P 62:B:Ba_„†BO`O8;<±iABO@C²S8;JB<>4ƒK…:;62N<C[_aRA8=JB
`O<>:`a[U£K5:=62N<>[M_eZ2fÊ8=JB8;:=<>624?A@CBb‡JZA_BV_=<CNBa_62:B‚š ª Ž ª žjRT624NPš®Ž¡ž‹ BÌ P Qç K$1 ØeJ+[_8=JB
<C4Ba»+[6Q@><C85²W:=BON[`OBa_8=Z
çœ BÌ P ©ç2 eÄ Úm P ç q
ØeJM<>_H<C_V_=<CUW„M@C²ÎmQçÎÄ P 1ÔØeJ<>_kf{ZA@>@CZjb‡_kf{:=ZAS U RUT ¤ ‹ P œ Pc¬ mQçA ¡RÊb‡JBO:=!B R<>_‚8=JB
<C4`aBO4$8;:=Bk624MVN T˜8=JBE`a<C:=`a[MUW`OBa4$8;:B21
l
rtsTuwvyxjz{sT|XW½ 3…fÊbBE_=B‘8ðƋ2,ÚÃRñ£‹2,ÎjRÃòV‹2,Þ¾R8=JBa4"8;JMBk<C4Ba»+[6Q@><C85²9`¶J624?2Ba_‡8;Z
œwš©Ž-§žO  ¤ œwš+Ž-§Ž¡ž©žOš®  ¤ Ä p œßš ¤ ©š®Žˆ§Ž ¤  aœßŽ ¤ §Ž¡ž©ž ¤  Oœwž ¤ ©žOš‰§š ¤   q
ØeJM<>_‡_=JZ—b‡_8;J6Q8‡bB‚UW6Š²"62__=[USBk𣝩ñE§òH‹Œ0A1 Ö BO8‰÷΋¨ðÃñ©ñƒò‰©òQðí1•ÈÎB‚_BaB
8=J6Q8
ð ¤ )ðÃñ©ñ ¤ ‹ œ{ðH©ñM  ¤ ¬ ðÃñ
‹ œ{ðH©ñM aœL0 ¬ ò$  ¬ ðÃñ
‹ ð£Ýñ ¬ ÷‹c0 ¬ ò ¬ ÷ q
ØeJ+[_
Y œ{ð ¤ ÝðÃñ©ñ ¤  ö‹ œ
0 ÷
¬ ¬ òA aœ
0 ¬ ÷ ¬ ð† Oœ
0 ¬ ÷ ¬ ñM 
‹ œ
0 ¬ ÷Ê  · ¬ œ
0 ¬ ÷Ê  ¤ ~œL0 ¬ ÷ˆ ÷ ¬ ðÃñƒò
‹ ÷ ¤ ¬ ÷ · ¬ ðÃñƒò q
ØeJ+[_‡bBk4MBaBaNd8;Z”„:Zji2Bk8=J6Q8÷ ¤Ä p œß÷ý¤ ¬ ÷ · ¬ ðÃñƒò$ ‘1ØeJ<C_e:=BON[`aBO_8=Z
p ðÃñMò”Ä ÷ ¤ œwm ¬ p ÷ˆ  q
¦ Z—b•B‘iABa:
p ÷ԋ p œ{ðÃñEÝñMò©òQð† ‡Äcœ³ð£ÝñE§òA  ¤ ‹˜0 ª
_Z£8;J6Q8‰m ¬ p ÷ÍâÙ0A1-ØeJ+[_<C8‡_L[ ES`aBO_8=ZS„M:=Z—iABE8;J6¾8 p ðÃñMò£Äº÷ý¤Š;1 [M8
÷ ¤ ¬ p ðÃñƒò ‹ œ{ðÃñEÝñMò§ò¾ð†  ¤ ¬ p ðÃñMòœ{ð”Ýñ©ò$ 
‹ M ð ¤ ñ ¤ ¬ ðÃñMòœ³ð”Ýñ§òA 
ZZ\^]^Z
‹ m0 M œ³ðÃñ ¬ ñƒò$  ¤ ⺸ q
Z[AZ\^]^Z



  !"$#&%('*)+-,/..0
1&243 57698 :<;>=?5A@6BDCE@GFHGIE5GJLKMCN6DOPCEFRQTS75UF&6DB5&VW8YX-J:YX-J;X9=Z5[6\5BD57]5^S_6CE`GF?OP`bacKMCEFd:<;eJ;Y8cJ 8 :
BD5UOfZ5US_6DCNg&5hIEiG2jlkff?`mO
56\5YS7CEBnS7koeQTS7CEBDShIN5`bap6DBCq@bFH&IN5W8WXh:YXh;XLf?@GODO
5^O/6\B`&kHG\r8s2/tBD`-gG56D\@-6:uX-J;X-J
K?JKUXW@GB5uSh`GFShilShINCqSbJRv\5hBD5YK^XCEO46D\5uCEFRQwSh5hFm6BD5Y`bax6BDCE@GFHGIE5u8WX_:uXU;4X-2
y zZ{}|x~U}zZ€

A
C1

I1 K B1
B
I

A1

C
‚ b` 6D5s6\?@-6Km8WXYƒ„Km:YXWƒ(Kl;4XYƒ†…b‡lJ?v\5UB5c‡eCqO6D\5cCEFRQwBD@&ˆRCEkOL`Ga‰6D\5s6DBCq@bFHGIE5s8 :<;e2ŠL5hF?S75<K$CEOL6\5
ShCNBnS7ko<QTS75UFm6BD5W`Ga 6D\5Y6BDCE@GFHGIE5u8WX7:YXU;X‹2
3 576sŒ=?5e6\5Žf?`&CNFm6Y`baCEFm65UBDO5US76CE`GF`GaKm:YX<@GFˆ8W;e2e4\5UFAKmŒƒ‡lJ Km8‘ƒ…b‡$@bFˆ’K&Œ“8‘ƒ”G•m–b2
— 6ea˜`GIEIE`-vLOs6D\@-6$’K&8Œ™ƒ›šG•m–$@GFˆ!\5UFS75[’K&8:YX9ƒœG•&–b2!4\mk?Oe8K&:uXPCEOž@bFŸ5U mkCEIE@b65hBn@bI6BDCq@bFH&IN5&2
jRCNožCEIE@GBIEiW6BDCE@GFHGIE58 Kl;X/CqOx@GIEO`L5^ &kCNIq@-6D5hBn@bI¡2p¢“5\5hF?S75`&=R6D@GCNFs8:YX ƒ£8W;4X ƒ¤8 Kžƒ¤Km:YXƒdKl;4X ƒ£…-‡l2
¢A5<@GIEO`Ž`G=?O
5UBg&56D\@-6u’: X Km; X ƒ‘1^…G• – @bF?ˆ¥K&: X 8W; X CEO@ŽBD\`Go<=kOU24\lkOY’: X 8; X ƒ1‹…b• – @bFˆ9=li
Sh`GFShiRS7IECEShC¦6i¥’8WXWƒdœ&•&–G24jRCNFSh5c8 :uXWƒ(8;X‹J§8†CqO46\5<ožCEˆRfZ`GCEFm6 `ba6D\5c@bBnSu:uX_8;Xb2 — 6La˜`GIEIN`-vLO6\@b6
8YX78„=CEO5US76DO ’8WXW@GFˆ9KUXIECN5^O`GF¨6\5uIECEF5s8WX78s24\CqOCEoefINCE5UO46D\@-6
’: X K X ; X ƒ£”&• –© ’8 XUª …uƒ£”&• –© šG• – ƒ„1‹…b• –b«
jRCNFSh5<’:YX7Km;XLƒ¬1^…G•&–‹Jv5uS7`&FS7IEkˆR5Y6D\@-6L:uX-JK?JKUXbJ?;XW@bBD5YS7`GF?S7iRS7IECESG2L­}®kB
6D\5hBL8„CqO46D\5sS75UF&6DB5&2 ¯
…24®‰CEFˆ9@bIEI°6BDCEfIN5^Ou­²±x³
´ ³µ¯4O
kSn\¨6\?@-6±?¶cƒ£µl· ©!¸ JRv\5hBD54±¥CEO@efBDCEoe5s@GFˆ¨´ ³
µr@bBD5YF@-6DkBD@GI§Fmkoc=Z5hBnOh2
y zZ{}|x~U}zZ€ ¢A5u=?5UHGCEFrvCN6\r6D\5sO6n@bFˆ@bBnˆPa}@&S_6`&BCqOD@-6CE`GF
µ · ©!¸ ƒ¬­˜µR¹º!…-µ © …G¯h­˜µR¹ © …-µ © …&¯ «
4\lkOWv5<\?@‹gG5cµ ¹ ºŸ…-µ © …$ƒ£±?»@bF?ˆ[µ ¹ © …-µ © …Pƒd±?¼9a˜`&BYO`Gož5<f?`mO
CN6CEgG5eCNFm6D5hHG5UBDO½>@bFˆ¾ŸOkSn\
6D\@-6Y½ © ¾!ƒ†´ 2sjlCEFS75<µ ¹ ºŸ…-µ © …¨¿Àµ ¹ © …-µ © …RJ§v5<\@‹gG5c½Á¿d¾ŸO`Ž6D\@-64±?»‘ˆRCEgmCqˆR5^O±Z¼§2W4\lkO
µ ¹ º…-µ © …cˆRCEglCEˆ5UOµ ¹ © …-µ © …R2¢ABCN6CEFHžµ ¹ © …-µ © …sƒ£µ ¹ º“…bµ © … ©A¸ µ§Jmv5WCNFRa˜5UB46\@b6µ ¹ º“…bµ © …
ˆCNglCqˆR5UO ¸ µ$@bF?ˆ$\5UFS75uµ ¹ º!…-µ © …eˆRCEglCEˆR5^O ¸ µ ¹ 2ÂkR6
¸ µR¹Lƒ ¸ ­˜µR¹4º!…-µ © …&¯ ©Äà ­˜µcºÅ1^¯ «
4\lkOµ ¹ ºÄ…-µ © …PˆRCEglCEˆ5UO à ­˜µžºÆ1^¯72jlCEFS75<µ ¹ ºŸ…-µ © …ŽˆRCEgmCqˆR5^O =Z`b6D\ ¸ µ9@bFˆ à ­˜µeº¤1^¯7J?v5cSh`GFShINkˆ5
6D\@-6LCN6 @bIqO`žˆCNglCqˆR5UO à 24\CEO4H&CNg&5UO4µ ¹ º!…-µ © …uƒ†1&J…RJ ¸ `&B à 2
— aµ ¹ ºŸ…bµ © …žƒ1GJ6D\5hFµPƒ1c@bFˆ9µ · ©Ÿ¸ ƒ„ÇRJ?HGCEgmCEFHs±¥ƒ„ÇŽ@bF?ˆ9´[ƒ1&2 — aµ ¹ ºŸ…-µ © …žƒ„…RJ6\5UF
µ ¹ ºÆ…-µÄƒÈ•¥H&CNglCEFH¥µÄƒÉ…29ÂkR6<6\5UF!µl· ©¤¸ ƒÊ…b•¥CqOsF`b6s6D\5PfZ`-v5UBs`baL@¥fBDCEoe5&2¥4\5$5^ &k?@-6CE`GF?O
µ ¹ º¨…-µ © …ƒ ¸ @GFˆžµ ¹ º¨…bµ © …ƒ à \@‹g&5F`uCNFm6D5hHG5UBO`GIEkR6CE`GFOU2 ¢A5Sh`GFShINk?ˆR546\@b6­²±x³
´x³
µ¯ƒ¬­¡ÇR³U1G³h1‹¯
CqO46D\5u`GFIEi$O`GIEkR6CE`GF 2
Ë I¦6D5hBDF@-6DCNg&5hIEiGJRkOCEFHŽµ ¹ º…bµ © …YƒÄ± » @GFˆrµ ¹ © …bµ © …YƒÄ± ¼ Jv5Yož@‹i$HG576
¸ µeƒÄ± »rÌ ± ¼lÍZ» ºM1‹Î «
— a/½ÐϤ•JR6D\5hFŽ±[ˆRCEglCEˆ5UO ¸ `&Bµ§2 — a°±[ˆRCEglCEˆR5^O ¸ J6\5UFŽ±rƒ(…R2 — a§±[ˆCNglCqˆR5UO µZJ6\5UF¥µ ¹ º!…bµ © …cƒŸ±?»
O\`-vLO6D\@-6x±$ˆRCEgmCqˆR5^O…u@bF?ˆ<\5hFSh5±¨ƒd…2‰ÂkR66\5UF$…b¶cƒÆµl· ©r¸ J&v\CqSn\PO
\`-vLO‰6D\@-6µ<CqO5UgG5hF 2‰‰@bÑlCEFH
µŽƒd…GÒJ&v5HG5h64…b¶GÍ ¹ ƒ ¸ Ò&· © 1&24\CEOCNožfIECE5UO6\@b6Ò<ƒ£•c@bF?ˆŽ\5hF?S75 µŽƒ¤•J&v\CqSn\PCqO@<S7`&F&6DBD@&ˆRCqS_6CE`GF 2
4\lkOL½Óƒ¤•ž@bF?ˆ¨µ ¹ º!…-µ © …uƒ¬1G24\CqO4HGCEgG5^O4µeƒ†1u@GFˆr\5hFSh54±¨ƒdÇJR´¨ƒ†1&2
š243 5768Æ=Z5L@uO
5h6`baZB5^@bIRFlkoc=Z5hBnO/OkSn\e6\@b68Æ\@GO/@b6IN5^@GO
6‰a˜`&kB5UIN5Uoe5UFm6DOU2jlkff?`mO
58¤\?@GO‰6\5fB`&f?5UB
6i
6D\@-6sÔ ¹ ©ÆÕ_Ö CqOu@PBn@-6DCN`&F@bI‰Flkoc=Z5hBWa˜`GBu@GINI/ˆRCqO6DCNF?S_6uFlkoc=Z5hBnOYÔ?³ Õ ³ Ö CNFA8s2etBD`-gG5u6D\@-6W6\5UB5e57×RCqO6nOY@
fZ`&OCN6CEgG5YCNFm6D5hHG5UB ؙOkSn\r6\@b6 Ô٠ؙCEO@žBn@-6DCN`&F@bI§Flko<=?5UB4a˜`GB5hg&5hBDi$ԎCEF¥8s2
y zZ{}|x~U}zZ€ jRkffZ`&O5c•ŽÚ[8s244\5UF9Ô ¹ ƒdÔ ¹ © •žÛ Õ CEOLBD@b6CE`GF@GI @bFˆ9Ô Õ ƒÀ• ¹ © Ô Õ CEO@bIqO
`žBn@-6CE`GF?@bI§a˜`GB
@GINIxÔ?³ Õ CNF¥8sJZÔ[ƒd Ü •J Õ ƒd
Ü •JÔƒ Ü Õ 2ŠL5hF?S75cԎƒ£ÔX Ù ØÝa˜`GBO
`&oe5uBn@-6DCN`&F@bI°ÔXY@GFˆrF@-6DkBD@GI°Flkoc=Z5hBؑ2
®`GBL@GFmi Õ ƒ£ Ü •Jv5Y\@‹gG5
ÕhÙ Øރ ÔÔ ÕX «
v\CESn\9CqO@<Bn@-6DCN`&F@bI°Flko<=?5UBU2
Š 5hFSh5sv5uož@‹ir@GODO
koe5s•žCqOF`G6 CEF98c2 — a6D\5hBD5sCqOL@žFlkoc=Z5hB Ô¨CEF[8¬O
k?Sn\¨6D\@-6<ºLÔPCqOL@bIqO
`žCEF[8sJ6\5UF
@GH&@GCNFŽv5 Sh@GFŽHG5h66\5S7`&FS7IEkOCN`&FP@GOa˜`&INIE`-vLOU2/ß`&FO
CqˆR5UB6v`s`b6D\5hB5hIE5hož5hFm6DO Ö ³DàsCEF$8s2/4\5UF Ö ¹ © à&Ô<CEO
Bn@-6DCN`&F@bI§@bF?ˆ Ö ¹ ºàmԞCEO4@bIqO`cBn@-6CE`GF?@bI¡2 — 64a˜`GIEIE`-vLO6D\@-6 Ö ¹ CEOBn@-6DCN`&F@bI§@bFˆ¨à&ÔPCEOBn@-6CE`GF?@bI¡2jlCEožCNIq@bBDIEiGJRà ¹
@GFˆ Ö ÔP@bBD5Y@bIqO
`eBD@b6CE`GF@GIEOU2/4\lkO4à ª Ö ƒá­}àmÔR¯ ª ­ Ö ÔR¯CEO4Bn@-6DCN`&F@bI¡2 ‚ `G65W6\@b6v5uSh@GF¨g-@GBiPàŽ`-g&5hB48(vC¦6D\
àŸƒ Ü Ö @GFˆàŸƒ Ü Ô?2 Ë H&@GCNF Ö ¹ CEOWBn@-6DCN`&F@bI‰CNožfIECE5UOW6\?@-6 Ö ƒ Ö X Ù Øâa˜`&BYO`Gož5eBD@b6CE`GF@GI Ö X @GFˆF?@-6kBD@GI
Flko<=?5UBؑ2¢“5u`&=O5hBDgG56\@b6 Ö Ù Øãƒ Ö X ؙCEO4Bn@-6DCN`&F@bI¡J@bF?ˆ
Ô Ù Øރ ÖÖ Ô X ³
O`e6\@b6 Ô٠ؙCEO @<Bn@-6DCN`&F@bI§Flko<=?5UBU2jlCEožCNIq@bBDIEiPCEO6\5sSU@GO5vCN6\ºLÔ٠ؑ2®`GBL@GFliŽ`b6\5hB5hIE5hož5UF&6 à?J
Õ Ù ØރÀØ Ö X àÖ
CqOL@eBn@-6CE`GF?@bI§Flkoc=Z5hB^2
4\lkOLv5YoŽ@‹iPF`-v(@GODO
kož56\@b6 •žCEO4F`b6LCEFr8†@GFˆ9Ô ©MÕ ƒ£ Ü •ea˜`GBL@GFmiPÔZ³ Õ CEF¥8s23p5h6 ÔZ³ Õ ³ Ö ³àž=Z5Ya˜`GkB
ˆCEO
6CEFS_6W5hIE5hož5hFm6nO`ba/8s24¢A5soŽ@‹i$@GODO
koe5$ä Ô°äϬä 244\5hF¥à ¹ © Ô Õ @GFˆ9à ¹ ©ŸÕ7Ö @bBD5WBn@-6DCN`&F@bI°Flko<=?5UBDO
Õ
@GFˆŽO`sCEO6\5UCNB4ˆRCNåZ5UB5UFS75Ô Õ º Õ7Ö 2x¢ABDC¦6DCNFH<Ô ¹ © Ô Õ ƒdÔ ¹ ©Õ_Ö§© ­}Ô Õ º Õ_Ö ¯7JG@bF?ˆžkO
CEFHu6D\5La}@&S_6nOÔ ¹ ©[Õ_Ö J
Ô Õ º Õ7Ö @bBD5 Bn@-6DCN`&F@bIqOUJGv5WSh`GFShINkˆ5 6D\@-6Ô ¹ © Ô Õ CqO4@bIqO
`e@cBn@-6DCN`&F@bI?Fmkoc=Z5hB^2/jRCNožCEIE@GBIEiGJ Õ ¹ © Ô Õ CEO4@GIEO`
@eBn@-6DCN`&F@bI°Flko<=?5UBU2
ß`&FOCEˆR5UB © Õ
æ ƒ ÔÕ ƒ ÔÕ ¹¹ © ÔÔ Õ «
‚ `b6D5u6\@b6 Ô ¹ © Ô Õ Ï¤•24\mk?O æ CqOL@žBn@-6DCN`&F@bI°Flko<=?5UB @bFˆ9Ԏƒ Õ æ 24\CqOHGCEgG5^OÔ ¹ © Ô Õ ƒ Õ ¹ ­ æ ¹ © æ ¯_2
3 576LkO6D@bÑ&5 Õ ¹ ­ æ ¹ © æ ¯ƒ£ç24\5UF
ä Õ äGƒ¬è æ ¹ ç© æ ƒ¬é ´µ ³
v\5hBD5¨´¤@bF?ˆÄµÄ@bBD5$F?@-6kBD@GIFlko<=?5UBDOU2Ÿ‰@bÑG59Ø ƒê´µ§2Ÿ4\5UF(ä Õ ä Ù Ø ƒ›´ÆCEOž@“BD@b6CE`GF@GIFlko<=?5UBU2
®‰CEF@bIEIEiGJRa˜`&BL@bFli Ö CNF¥8sJv5u\@‹g&5
Ö Ù Øãƒ Õ Ù Ø ÖÕ ³
CqOL@GIEO`e@eBn@-6DCN`&F@bI°Flko<=?5UBU2
¸ 2 Ë NI I§6D\5Wf?`&CNFm6DOvCN6\¨CNFm65UHG5UBSh`m`&BDˆCNF@b65^OCEF$6D\5´?µmQ¡fIq@bF5@bBD5Sh`GIE`GkBD5UˆPkO
CEFHc6\BD5h5YS7`&IN`&kBDOUJmB5^ˆ°Jl=INk5
@GFˆ9HGBD5h5UFpJ5^@GSn\9S7`GIE`GkBL=?5UCNFHPkO5Uˆ9@-6 IE5U@&O6L`&FS75&2 — 6 CqOLÑlF`-vFr6D\@-66D\5sfZ`GCEF&6e­}•³D•&¯CqO Sh`GIE`GkBD5Uˆ¨B5^ˆ
@GFˆ$6\5YfZ`GCEF&6s­ë•³h1‹¯CEO4S7`&IN`&kBD5UˆP=INk5&2tBD`-gG56D\@-646D\5hBD5Y57×RCEO
646\BD5h5WfZ`GCEF&6nO4vC¦6D\¨CEFm65UHG5hBSh`m`&BDˆCNF@b65^O
`GaˆCEO
6CEFS_6 Sh`GIE`GkBnOv\CqSn\¨a˜`&BoÈ6\5ug&5hB6CqS75UO`Ga@eì }íZî§~7ïnð€xíZ{}ñlò 6DBCq@bFH&IN5&2
y zZ{}|x~U}zZ€ ß`GF?O
CqˆR5hBs6\5¨IE@b6
6CqS75¨f?`&CNFm6nOU­}f?`&CNFm6DO<vCN6\ÄCEFm65hH&5hBcSh`l`GBnˆRCNF?@-65^OD¯u`GF6\5$IECNF5^OcµÄƒÓ•[@GFˆ
µŽƒ†1&Jb`b6D\5hB6\@GF9­ë•³D•&¯@bF?ˆr­ë•³U1^¯_J — aZ`&F5`baZ6\5ho9JmOD@‹ic8dƒá­²±x³h1^¯7JGCqOS7`&IN`&kBD5Uˆ<HGBD5h5hF J-6\5UFžv5\@‹g&5@
BDCEHG\m6
QT@bFH&IN5^ˆ<6BDCE@GFHGIE5vC¦6D\­}•³•m¯_J§­}•³U1^¯/@GFˆŽ8d@&Og&5hB6CqS75UOUJ&@GINI?\@‹glCNFHsˆRC¦å§5hBD5hFm64S7`GIE`GkBDOU2­¡jl5h5W®xCEHGkB5^O
1u@GFˆ¥…R2ó¯

…
g
(0,q) D (p,q)

A (p,1)
b g b b

r g
r A(0,q) r
B (c,0) (p,0)
b r/ b

Fig-1 Fig-2 Fig-3

— axF`b6^Jl6\5WIE@b6
6DCESh5 fZ`GCEF&6nO`GFPµŽƒ£•<@bFˆ$µeƒ†1W@bBD5@GINIZB5^ˆP`&B=IEk5G2/¢A5YS7`GF?O
CqˆR5hB6\B5U5ˆRCNåZ5UB5UFm64SU@GO5UOU2
ô<ðõ^ñ(ö?÷ jlkff?`mO
5¨@¥f?`&CNFm6ž:ƒÐ­ Ö ³D•&¯YCqOs=INk5&2Aß`GF?O
CqˆR5hBe@¥HGBD5h5UFf?`&CNFm6eøùƒÐ­¦± ³ æ ¯YCNFÄ6\5¨fIE@GF5G2
jRkffZ`&O5u±dƒÈ Ü •2 — a4C¦6nOufBD`bú5US76CE`GFÅ­²± ³D•&¯W`&F6D\5P´QT@-×RCqOYCEOYBD5UˆpJp6\5hF¤­²±x³ æ ¯_J4­²±x³•m¯@GFˆÆ­ Ö ³D•&¯Y@GB5<6\5
g&5hB6CqS75^Oc`Ga@BD5U mkCEBD5Uˆ6ilfZ5r`Ga BDCNH&\&6Qw@GFHGIE5UˆA6DBCq@bFHGIE5G2 — a<­¦± ³•m¯sCqOe=IEk5GJ6\5UFMv5rSU@bFŸSh`GFOCqˆR5hB<6\5
6DBCq@bFHGIE5<v\`mO
5egG5UB
6DCESh5UOW@bBD5r­ë•³D•&¯_J­¦± ³D•&¯@bFˆÅ­²±x³ æ ¯_2 — ax±Aƒ‘•J§6\5UF6D\5žf?`&CNFm6nOuø¥J­}•³•&¯@bF?ˆM­ Ö ³•m¯
vCEIEIpv`&Bѧ2q­}®‰CEHGkBD5Wš2 ¯
ô<ðõ^ñ“ûp÷ Ë fZ`GCEFm6 øȃ¬­ Ö ³h1‹¯_JR`GF¨6\5uINCEF5uµžƒ¬1GJRCqO4BD5Uˆ°2 Ë OCNožCEIE@GBL@bBDHGkož5UF&64v`GBDÑlOCEFr6\CqOLSh@&O
5&2

E (p,q)
g

b L b

r
K r M

Fig-4

ô<ðõ^ñMü ÷ jlkffZ`&O5Ž@bIEI/6\5ŽIq@-66CqS75žfZ`GCEF&6nOu`GF“6\5ŽINCEF5Žµ“ƒ•9@bBD5žB5^ˆ“@GFˆA@bIEI`GF“6\5ŽIECEF5Žµ“ƒÓ1Ž@bBD5
=INk5<f?`&CNFm6nOh2Wß`&FOCEˆR5UB@ŽH&B5U5hF9fZ`GCEF&6Yýƒþ­²± ³ æ ¯_JZv\5hBD5 æ ƒ„ Ü •P@bF?ˆ æ ƒ‘ Ü 1&2E­¡jl5h5c®‰CEHGkBD5 ¸ 2 ¯[ß`&FO
CqˆR5UB
@GF¥CEO`&ODS75UIN5^OBDCNH&\&6Qw@GFHGIE5Uˆ$6BDCq@bFH&IN5sýsŒAØÿvC¦6D\A’ýƒ(”G•m–sOkSn\96\?@-6L6D\5c\lilf?`G65UFmk?O
5sŒAØ CqO @žf?@bB6
`Ga6\5c´QT@-×RCqOh2Y3p5h6WýcØ CNFm65UBDO5US76 µ9ƒCEF  2W4\5UF“ŒùCEOW@PBD5Uˆ¥fZ`GCEFm6W@GFˆ  CqOY@P=IEk5<f?`&CNFm6U2uŠ 5hFSh5
ýcŒ  CEO@Žˆ5UOCNBD5Uˆ$6BDCE@GFHGIE5G2
Ç243 576ž8 :<;ê=?59@96BDCq@bFH&IN5&V J  J †=Z5P6D\BD5h5$5^ mk@bI¡JˆRCqO˜ú`&CNFm6eShCNBnS7IE5UOcCNF?O
CqˆR5¨8:<;êOkSn\6D\@- 6 
6D`GkSn\5UOW8 :É@GFˆ[8W;e
V ¤6D`GkSn\5UOW8 :PV°@GFˆ[:<;eJp@GF ˆ ¤6`Gk?Sn\5UOW:<;@GFˆ;Y8s2Y3p5h 6 Ÿ=?5ž@PS7CEBnS7IE5
6D`GkSn\CNFH¨S7CEBDShIN5^ O J  J ¤57×l6D5hBDF@bIEINi&2LtBD`-gG5Y6D\@-66\5<INCEF5Lú`&CNFCNFH$6\5eS7CEBDShkoeQwSh5hFm6BD5 þ@GFˆr6\5
CEFRQTS75UF&6DB5uKŽ`Ga 6DBCq@bFH&IN5u8:<;áf@GODO
5^O6\BD`GkHG\¨6\5sSh5hFm6BD5Y`ba 2
y zZ{}|x~U}zZ€ 3p5h6 YXbJ  J  W=Z56D\5uS75UF&6DB5^O`Ga 6D\5WShCNBnS7IE5UO   J   J   BD5UOf?5^S_6CEgG5UINi&Jm@GFˆ¨IN5h6 á=?56\5
ShCNBnS7ko<QTS75UFm6BD5u`ba6\5c6¹ BDCE@GFHGIE5YX ¹ G2L3 576 ´AˆR5hF`b65s6\5cSh`Gožož`GF¥Bn@GˆRCEkO`Ga/6\BD5h5<S7CEBDShIN5^O  J  J
  2 ‚ `G65u6D\@- 6 CqO @GIEO`Ž6D\5cS75UFm6BD5s`ba6\5cS7CEBDShIN5 J?@&O YX žJ  žJ !5^@GSn\957×Sh5h5Uˆr6D\5cBn@GˆCNkOL`ba
=liP´x23p5h6Lø¥J"ÄJŒŸJ  J?ؙ=?5uBD5UOf?5^S_6DCNg&5hIEiŽ6\5ufB`Gú5US_6DCN`&FO`ba¹ KJžJeJ#WXbJ ¹ `GF9:<;É2

š
A

O1

I P
O
O2 O3

B L D X K M C

® B`&o :<: ø  ƒ  øP K ¹ J v5ŽHG5h6c:  ƒ´‰­%$Yº Õ ¯ ª ‡lJ‰@&OuK&ø>ƒ‡9@bFˆ:<ø ƒ­%$Yº Õ ¯_2¨jRCNožCEIE@GBIEiGJ;uØ ƒ


´‰­%$Lº Ö ¯ ª ‡l24\5hBD57a˜`&B5&J  ØރdÔYº ´‡ ­%$4º Õ© $4º Ö ¯ƒ ԇ ­}‡ º[´§¯_2jlCEFS75& ¹  Ø' CqO@cBD5US76D@bFHGIE5W@GFˆ
("ÐCqO46\5ufZ5hBDfZ5hFˆRCqS7kIE@GB=CEO5US76`&B`ba) ¹   JC¦6LCqO4fZ5hBDf?5UFˆRCqS7kIq@bBL=CEO5US76`GB4`Ga  ؙ@GO4v5hIEI¡24\lkO
 " ƒ 1  Øãƒ Ô ­˜‡YºA´Z¯_V
… …-‡
: " ƒ :  ©  "ƒ ´‡ %­ $ º Õ ¯ © …bÔ ‡ ­˜‡Yº“´§¯ƒ ԅ º ´‰­ Õ …-º ‡ Ö ¯ V

ø$Œ ƒ :cŒ›º:cøȃ ԅ ºÆ%­ $ º Õ ¯ƒ Õ º … Ö V
"Œ ƒ :cŒ›º& : " ƒ ԅ º ԅ © ´x­ Õ …bº ‡ Ö ¯ ƒ ´x­ Õ …bº ‡ Ö ¯ «
Š 5hFSh5uv5YH&576
"“Œ ƒ ´ «
øPŒ ‡
¢A5u`G=?O
5UBg&5 6D\@-66D\5sO
CqˆR5^O4`ba‰6BDCE@GFHGIE(5 Y*X  ¹  u@GB5
 ¹   ƒ  Øރ Ô ­}‡ºA´Z¯7+
‡ ³    X ƒ ‡Õ ­}‡ºA´Z¯7+ ³  X  ¹ ƒ ‡Ö ­}‡WºA´Z¯ «
4\lkOL6\5eO
CqˆR5^O`b,a YX  ¹ c@GFˆ¨6\`mO
5c`ba8 :e;@bBD5uCNF¥6D\5sBn@-6DCN`[­}‡uºA´§¯ ª ‡m2®kB6\5UBUJZ@GOL6\5cOCqˆR5UO `ba
YX*  ¨@bBD5Žf@bBn@bIEIN5UI6`¥6D\`&O5Ž`baL8:<;eJv5PO5h5P6\@b6<K[CqOY6\5PCNFQwSh5hFm6BD5Ž`ba YX ¹ $@&Ouv5UINI¡294\CEO
H&CNg&5UO¹ -K  ª .K (ƒá­˜‡WºA´Z¯ ª ‡lJ?@bF?ˆ$\5UFS7&5 & ª -K †ƒ¤´ ª ‡l24\lkO4v5u`G=6D@bCEF
"Œ ƒ & «
øPŒ K-
— 6a˜`&INIE`-vLO6\?@-6 K?/³ 40³ @bBD5uS7`&INIECNF5U@bB^2
Ë I¦6D5hBDF@-6D5hIEiGJ/v5$@bIqO`[CNFa˜5hB<6\@b6eK“CEOs6\5rS75hFm6DB5¨`ba\`Gož`G6\5h6iAv\CESn\6n@bÑG5^Ou6D\25 1H&kBD35 W*X  ¹ Ž6D`
8:<;e2Š 5hFSh5uC¦66n@bÑ&5UO¬6D`2e2 — 6a˜`GIEIN`-vLO6D\@-6 K?³/4³4@GB5uSh`GIEINCEF5U@GB
œ243 57 6 Ž­}´Z¯4=Z5s@eHGCEgG5hFrfZ`GIEimF`GožCE@GI°vC¦6D\9CNFm65UHG5UBLS7`l5 5PS7CE5hFm6nOh2/tBD`-gG56\?@-66\5hBD5W5h×RCEO
66v`žfZ`GIEilF`GožCq@bIqO
6 ­˜´Z¯@GF8 ˆ 7ž­˜´§¯_J@GH&@GCNFPvCN6\¨CNFm6D5hHG5UBSh`l5 5PS7CE5hFm6DOUJOkSn\Ž6\?@-6s­˜C 9¯ Ž­˜´Z¯ 6 ­˜´§¯CEO4@cf?`&INilF`&oeCq@bI§CEF$´ ¹ V@GFˆ
­}CNC ¯ Ž­}´Z:¯ 7ž­}´Z¯4CqO@žf?`&INilF`&ožCE@GI°CNFr´  2
y zZ{}|x~U}zZ€ 3p5h 6 Ž­˜´§¯ƒ£-Ô ; © ÔX_´ © Ô ¹ ´ ¹ = © <*<<^© Ô ¼ ´ ¼ =?5s@žfZ`GIEilF`GožCq@bI°vCN6\9CEF&6D5hH&5hBS7`l5 5PShCN5UF&6nOh2
> ðì@ ~ ?B A ¢“5uoŽ@‹iPvBDC¦6D5
Ž­˜´§¯ƒ¤Ô ; © Ô ¹ ´Z¹ © Ô · ´ · C © <<*<‹© ´ Ì Ô X © Ô  ´Z¹ © -Ô Dh´ D C © <<*< Î «
E 5 1?F5
6 ­˜´§¯ƒdÔF; © Ô ¹ ´Z¹ © Ô · ´ · C © <<*< ºA´ Ì ÔX © Ô.7´Z¹ © Ô D ´ D = © <*<< Î «
¸
4\5hF 6 ˜­ ´§¯CEO b@ IqO
`ž@žf?`&INilF`&ožCE@GI°vC¦6D\rCEFm65UHG5hBSh`m55PS7CE5hFm6DO@GFˆ
Ž­}´Z¯ 6 ­˜´§¯ƒ Ì Ô-; © Ô ¹ ´ ¹ © Ô · ´ · ©=<*<< Î ¹ º´ ¹ Ì ÔX © Ô.7´ ¹ © Ô D ´ D ©C<<*< Î ¹
CqOL@efZ`GIEimF`GožCE@GI°CNF9´ ¹ 2
> ðì@ ~ ?ëG A ¢A5uvBCN65u@bHm@bCEF
Ž­˜´§¯ƒd8c­˜´§¯ © ´?:¨­˜´Z¯ © ´Z¹U;ž­}´Z¯_³
v\5hBD5
8c­}´Z¯ ƒ -Ô ; © .Ô h´  © .Ô H7´ H = © <*<< ³
:$­}´Z¯ ƒ ÔX © Ô · ´  © .Ô I7´ H = © <*<< ³

;ž­}´Z¯ ƒ Ô ¹ © .Ô Dh´ © .Ô J7´ = H © <*<<p«
‚ `b6D5<6D\@-6s8<­˜´Z¯7J‰:¨­˜´§¯Y@bFˆA;ž­˜´Z¯s@bBD5cfZ`GIEimF`GožCE@GIEOWvCN6\CEF&6D5hH&5hBuS7`l5 5PShCN5UF&6nOY@bFˆ“5U@&Sn\`ba6\5^O
5eCqOu@
fZ`GIEilF`GožCq@bI°CEF9´  2/¢“5uoŽ@‹iPCEFm6BD`lˆkS75
K ­˜´Z¯ÿƒ 8c­}´Z¯ M © L ´Z:$­˜´§¯ N © L ¹ ´ ¹ ;ž­˜´§¯_³
O ­˜´§¯™ƒ 8c­}´Z¯ M © L ¹ ´?:$­}´Z¯ M© L ´ ¹ ;ž­˜´§¯_³
v\5hBD5 L CqO@bFrCNoŽ@bH&CNF?@bBDiPS7k=Z57QwB`l`G64`Ga‰kFC¦6i&24\5hF
K ­}´Z¯ O ­˜´Z¯ƒ Ì 8c­}´Z¯
Î ¹ © ´?¹ Ì :$­˜´§¯
Î ¹ © ´ ·ZÌ ;ž­}´Z¯
Î ¹
ºA´?8<­˜´§¯:$­}´Z¯ºA´  :$­˜´§¯
;ž­}´Z¯/ºA´?¹‹;ž­˜´§¯8<­˜´Z¯
OCEFS75 L  ƒ†1Y@GFˆ L[N © L ¹ ƒ¬ºu1G2/x@GÑlCNFPH 7ž­˜´§¯ƒ K ­˜´Z¯ O ­˜´§¯_Jv5u`G=6D@bCEF
  
Ž­˜´§:¯ 7ž­˜´§¯ƒ Ì 8<­˜´Z¯ Î © ´  Ì :$­}´Z¯ Î © ´ H Ì ;ž­˜´Z¯ Î ºAšG´  8c­}´Z¯:¨­˜´§¯
;ž­˜´§¯_³
v\CESn\9CqO@<fZ`GIEilF`GožCq@bI°CEF9´  24\CEOa˜`GIEIN`-vLOa˜BD`Goþ6D\5uCEˆ5hFm6CN6i
­ëÔ ©MÕ/©ÄÖ ¯h­}Ô ¹ ©MÕ ¹ ©ŸÖ ¹ ºÔ Õ º Õ_Ö º Ö Ôl¯ƒdÔ  ©ŸÕ  ©ÄÖ  ºAš&Ô Õ7Ö-«
Ë I¦6D5hBDF@-6D5hIEiGJ 7ž­}´Z¯oŽ@‹iP=Z5sˆRCNBD5US76IEi$ˆR5 1F5Uˆr=li
7ž­˜´§¯ƒ Ì 8c­}´Z¯ Î ¹ © ´ ¹ Ì :$­}´Z¯ Î ¹ © ´ ·§Ì ;ž­}´Z¯ Î ¹
º“´Z8c­}´Z¯:¨­˜´§¯ºA´  :$­}´Z¯
;ž­}´Z¯º“´Z¹^;ž­}´Z¯
8c­˜´§¯ «

Ç

    !" #$&%'()"*+  -,/.0102
354 16+!87  :9;<=  7
>@?BACEDGFBHJILKMCONDQPSRTN=UWVXTCYN=U!Z[X\CED&]KMCON=U[RTU^DQCEPSR\_=P&`M_RTU^Dbadc!eSfgDdfhN@DjikHl]lILm:n@oprq;ikHlFb]m
i/HJIG]O?)A1CEDtsvudwxKMCDdfWCGyJRTZWz`_R{U|Dda+_=};FIOq$HJIPSC~aQ`MC~eDdR\€=C~X\=? ‚$cW`!`_adCbHY]ƒm…„@]js†?‡Pd_ˆ€=C
DdfhN@DBF‰ud]kudwŠN@PdCGe_X{XTRTUWC~N@P~?
‹ŒŽ
‘’Œ“”
•– DQC~UhZ—I&]˜DQ_š™›adcheœf[DdfhN@DJI&]m]G™š?žA1CED
i/HJIG]Ÿm¡ ¢m£i/HYFB]O?¢¤bK!aQCEPS€=CgDdfhN@DšHY]
RTaYDdfhCž`CEPS`C~U!ZhRTeEchXTN@PjK!RTaQC~eDQ_=P_=}&IG™'?¦¥BC~UheC
HJI§m¨HY™¡N=U!Z…HOIG™¡RTaN=U8RTaQ_adeC~XTC~aDQPSRTN=U$z
VX\C=?ž©-fªchaYi/HY™5]«m¬ ?®­/cWDjDdfWC~U¯i/HY™°]«m D A
 ±m²i/HYFB]O?±©-fhRTašRTyO`!X{R\C~ažDdfhN@D³H°ud]kudF‰ud™
N=XTX)X{R\CO_U´N®eERTPSeEX\C=?µ¶UgDdcWPœUq1·+C5e_UheEXTc!ZWCODdfhN@D α
α
i/™°FbH¸mik™5]GH¸mn@o p ?B‚$R{UheC‰]LRTa-DdfWClyJRTZ$z α
`M_RTU^DO_=}‰I&]¹’Kªºe_UhaQDQPScheDdRT_U»YN=UhZ"s R{aODdfWC P
M
yJR{Z$z`M_RTU|D¼_=}MIGFl¹’VR\€=C~U»œq|R\D½}¾_XTXT_ˆ·ta;DdfhN@D)]‰s¿RTa α
`!N@PœN=XTX\C~XDQ_™°FƒN=UhZ'™5F…m…„@]‰s²m"HY]O?©-fªcha
™°HY]GFRTalN=UºRTaQ_adeC~X\C~ajDQPSN@`MCEÀ~RTchy±N=UhZ—™°HÁRTa α
`!N@PœN=XTX\C~XDQ_]GFj?
 C‰fhC~UheCGV=CED B N C

i/™°]GF…m…i/HYFB]mÃi/HJI&]vmÃikwš]jIOÄ
DdfWCJX{N=aQDGC~ŪchN=XTR\DƁš}’_XTX\_
·ta&}’Pd_y¢DdfWCJ}N=eDGDdfhN@DYi/HY]jImÇn@oprq¼N=UhZ´wÈR{aDdfWCyJRTZWz`_R{U|D_=}-I&H
aQ_žDdf!N@DjwÉ]˜mLw³ImLwÉH}¾_=PGDdfhCJPSR\Vf^D¶zN=UWVX\C~ZgDQPœRTN=UWVX\CJHY]jIO?JµÆD&}’_XTX\_
·ta&DdfhN@DjF‰ud]kudwÊN@PSC
e_XTX{RTUWC~N@P~?
ˍ¾ˆÌªÍr“¼ÎWˆÌ[‹MŒ’Ž
‘’Œ“”
 CchaQCbe_^_=PSZ!RTUhN@DQCtV=CE_yJCEDQPd=? ACED cha DdN@Ï=C&]ƒmйo$udo|»œqhN=UhZ5DdfWCBe_ª_=PSZhR{UhN@DQCBN – C~a/N=X\_UWVj]lIvN=U!Z
]GH°Ä  CDdN@Ï=C°IÇm˜¹Ñrudo|»N=UhZgHm˜¹o$uSÒE»œ?JA1CEDGFLm˜¹’Ó¼uQÔ$»œ?  CJaQCECJDdf!N@DGwÕm˜¹Ñ~Ö@„ªuSÒEÖ@„»GN=U!Z
s×mØS¹’ÓOٗÑ~»QÖ@„ªuQÔ$Ö@„=Úª?©tfWCGe_UhZhR\DdRT_Už]GHmÄ@]‰s DQPSN=UhadXTN@DQC~a-DQ_
¹’ÓOÙºÑE»ÜÛٗÔ^Ûtm…ÒÝÛrÞ
 Cl_=K!aQCEPd€=CYDdfhN@DbDdfWCYadX\_=`MCG_=}+IG]¸mƒo$ÄDdfhN@DB_=}/I&HRTalßtÒEÖrÑrÄDdfhN@DB_=} ]GFÐRTaBÔ$Ö
ÓÄ1N=U!Z'DdfhN@Db_=}
HYFÐRTaY¹’ÔJ߯ÒE»QÖ
Ó¼?©½N@Ï$RTUWV°`hPd_=`MCEPbaSR\VUhaEq· CYeEN=U³e_U|€=CEPSDji/]jI&HÇm†i/]GFbH°q€$RTN5àÜá^â´}chUheDdRT_Uq
DQ_JDdfhC&}¾_XTXT_ˆ·tRTUhVlPSC~XTN@DdR\_Uã
Ó Û Ù¯Ô Û ßgÔªÒkmßtќӼÞ
©-fªcha/·+CG_=KhDdN=RTU
Ó;¹’ÓOÙºÑE»m"Ô¹äÒ/ß³ÔW»œu Ñ=¹Ñ Ù¯Ó»)m…Ò@¹äÒ/ßgÔ$»œÞ
µÜD/}¾_XTXT_ˆ·ta+DdfhN@D/Ô$Ö
Ó®m…ÒEÖrÑr?­/cWD/DdfWC~U·+CbV=CEDkDdf!N@D+DdfhCaSX\_=`MCB_=}½FB]ÐN=UhZ]GwåN@PdCDdfWC&adN=yOC=?  C
e_UheEX{chZWCDdfhN@DtFjud]kudwŠN@PdC‰e_XTX{RTUWC~N@P~?
„ª?BæbCEç!UWCGNaQC~Å^chC~UheC5èá^é$ê¶é=ë ì¼í N=ak}¾_X{X\_ˆ·ta~ã
á|éOmÇî o$>@uu R\R\}½}½DdDdfWfWCGCGUªUªchchy‰y‰KMKMCECEPkPk_=_=};};`M`M__adadR\R\DdDdR\R\€=€=C&C&ZhZhRTRT€ª€ªRTRTadad_=_=PSPSa+a+_=_=}½}½â[ R{aGï~ðð@q
â[R{a‰ñò@ñEóhÞ
¹©-fhCb`M_adRTDdR\€=C&ZhR\€$RTaQ_=PSa _=}¼âgRTU!eEXTchZWCl>&N=ak· C~X{XN=a-â)?ô»lA1CEDkõ®m8o$Þöá í á á^÷;ÞEÞEÞtKCDdfWCPdC~N=XU^chyjKMCEP
·tfh_aQC®ZWC~eERTyJN=X-C – `!N=U!adR\_U¦e_U^DdN=RTUha°á|é—RTU—DdfWCšâMzDdfº`!XTN=eC=q/âÇøŠ>@Û ?"æbCEDQCEPSyJR{UWC=q/·tR\Ddfº`hPS_^_=}Üq
·tfhCEDdfWCEPkõÉR{a/PSN@DdR\_UhN=X_=PBR\PdPSN@DdR\_U!N=Xä?
‹ŒŽ
‘’Œ“”
 C®adfh_ˆ·ÇDdfhN@DOõ¦RTaYR\PdPœN@DdR\_UhN=Xä?´‚$cW`!`_adCDdfhN@DYõùRTalPSN@DdRT_UhN=Xä?[©-fWC~U¯DdfWCžaQC~ŪcWC~UheC[èá^é$ê é=ë ì¼í R{a
`MCEPSR\_$ZhRTe+N@}’DQCEP+aQ_yJCBaQDdN@V=C=Ä$DdfWCEPdCtC – RTadDUhN@DdcWPSN=XU^c!y‰KMCEPSaúudû1aScheSfDdfhN@D á^éYm8á é=üMý }’_=P N=XTXWâ[øþú?
ÿ fh_^_aQC ÁaScheSfžDdf!N@D ®û;øùúN=Uh Z ®û;RTa-NY`MCEPd}’C~eDtadŪchN@PdC=?ACED
«m í   Û
ÞEÞEÞ   u û1 m í  Û
ÞEÞEÞ   u
KMC-DdfWCb`hPSR{yOC-ZWC~e_yO`M_adR\DdRT_Uha_=} gudû¼aQ_‰DdfhN@Dk   Ù BRTaCE€=C~U°}’_=Pb> $? B_ˆ·8DdN@Ï=C&N‰`hPœRTyOC
[ZhR MCEPdC~U^DB}’Pd_ ÿ
y 1í~u  uEÞEÞEÞ¼u   ? _UhadRTZhCE!P ®û N=Uh" Z ®ûÜ?l‚$RTUhe#C ®û1 ß ®û R{abZ!R\€ªR{adR\K!X\CKªšûÜq·+C
fhNˆ€=CJ%á $'& ý m†(á & ý ?‰¥BC~Û Uhe*C )+¹ ®û »BN=Uh, Z )-¹ ®û»BfhNˆ€=CJadN=yOCY`!N@PSR\DƁ=?‰­/cWD )+¹ ®û»-m¸.„ )-¹ ®û»œqadR{UheC
VeEZ+¹ ¼/u ®ûä» mÐ>GN=U!0Z šRTaBNO`hPSR{yOC=?+‚$RTU!eC ®û)RTa-NJadŪchN@PdC=q )-¹ ®û»/RTa-_ªZhZ1?µÜD-}¾_X{X\_ˆ·ta-DdfhN@!D )+¹ ®û»
RTa/CE€=C~UšN=UhZ®fWC~U!eCG%á $1& 3ý m8 2 á & ý ?©-fhRTa/e_U^DQPSN=ZhRTeDdRT_U'RTyO`XTR\C~a DdfhN@D-õšR{a-R\PdPSN@DdRT_UhN=Xä?
ˍ¾ˆÌªÍr“¼ÎW
5‘ 4̯‹ŒŽ
‘’Œ“” 6 abC~N@PSXTRTCEP~qN=adaSchyOClDdfhN@DBõ[R{a-PSN@DdR\_U!N=X¼N=UhZÉeSfh_^_aQCOUhN@DdchPSN=X¼UªchyjKMCEPSa
úudûaScheSfšDdfhN@Dá é mƒá é@üMý }¾_=PN=XTXâ¯ø…ú? ÿ _U!adRTZWCEP-DdfhClUªchyjKCEPœa-á & ü¼í udá & ü uEÞEÞEÞudá & üMý q·tfhCEPdC
ÕøúÉRTaN=U|³U^c!y‰KMCEP~?G©-f!RTayjchadDe_U^DdN=RTUgN@DGX\C~N=adDb_UhCJo$?Y¤bDdfWCEPS·tRTaQCYá|é®Û m>Y}’_=PGN=XTX½âùøú?
­/cWD/á  m8oYR\}N=U!Z_UhXTR\} jR{a+NYadŪchN@PdC=?)¥BC~UheCGR\D+}¾_X{X\_ˆ·ta DdfhN@D/DdfWCEPdC&N@PdC&UW_YadŪchN@PdC~a+}’_=P+8 â 7þúq
·tf!RTeSfJR{aN@K!adchPSZ?;©-f^c!aCE€=CEPdûe_UhaQC~eEcWDdR\€=CbDQCEPSyJa _=}DdfWCbaQC~Å^chC~UheCYèá^é$êylchaQDe_U^DdN=RTUN‰olN@}¾DQCEP
eCEPdDdN=R{UÉadDdN@V=C=?lACEDbàtmyN :– 9 úud<û ;qN=UhZge_U!adRTZWCEPbà Û N=UhZº¹’à½Ùv>ˆ» Û ?l‚$RTU!eCjDdfWCEPSCYN@PSCYUh_aSÅ^chN@PSC~a
KMCED·+CEC~UYà Û N=UhZž¹’àˆÙž>ˆ» Û q=·+Cke_UheEXTchZhC DdfhN@D)á =
ü> m>+}’_=P/?> @þ„ràœ?)­/cWD½DdfWC~Uq=·+C/f!N~€=C-„ràA¹ 7þû»
e_UhadC~eEcWDdR\€=CGDQCEPSyak_=};DdfWCGaQC~ŪcWC~UheC5èá^é$ê+·BfhRTeSfžyRTadako$q!e_U^DQPSN=ZhRTeDdR{UWVJ_cWPkC~N@PSX{R\CEP-_=K!aQCEPS€rN@DdR\_U1?
Bª?!C;RTUhZžN=X{XMPdC~N=X1Uªchy‰KMCEPSa/õÉadcheœfžDdfhN@D
D õ Û Ùº„r:õ Ekm D õ E Û Ùº„ D õ E
Þ
C F õ G¼ZWC~UW_=DQC~akDdfhC‰XTN@PdV=C~aQDBRTU|DQCEV=CEPBUW_=D-C – eCEC~ZhRTUWVJõ¼?ô»
¹¥BCEPd"
‹ŒŽ
‘’Œ“”
6 ZhZhRTUhV°>&KM_=Ddf®aSRTZWC~aEq$DdfWCGC~ŪchN@DdR\_U®PdC~ZhcheC~akDQ_
D ¹’õOÙ8>ˆ»ÜÛ E mHØ F õYÙ8I> G¾Ú Û Ä
·+COfhN~€=Cc!aQC~ Z F Jõ G!K
Ù ŠmLF õ5K Ù MG}¾_=P&CE€=CEPdÉRTU^DQCEV=CENP g?Y‚$ch`h`M_aQC‰õ5Ùv> ƒo$?l©-fhC~O U F õ5ÙvI> GP
õOÙ8> ¦o$?©-fªcha
Ø F õOÙ8I> G Ú Û øv¹’õJÙ8>ˆ»ÜÛ&ø D ¹’õYم>ˆ»Ü1Û Ekm Ø F õOمI> G Ú Û Þ
©-fªchaC~ŪchN=XTR\DƁÉfh_XTZha&CE€=CEPdª·tfWCEPSC=?©-fhRTaVRT€=C~" a F õ°ÙI> G m:õ°Ùƒ>JN=UhZ[Ddf^chaGõ°Ùƒ>JRTaGN=URTU|DQCEV=CEPˆ?
Q adRTUWVYõOÙ8N > ¦o$qh·+C‰e_UheEX{chZWCDdfhN@D
õ R 9 ßG>@u~ßt„ªu~!ß BªuEÞEÞETÞ ;Þ
S
‚$ch`h`M_aQCBõOÙ8N> 7¦o$?  CjfhNˆ€=C
¹’õOÙ8>ˆ» Û ø D ¹’õOÙ8>ˆ» Û E-m Ø F õYÙ8I> G Ú Û Þ
U _=PdCE_ˆ€=CEPˆq!·+C‰N=XTaQ_f!N~€=C
¹’õOم>ˆ»ÜÛ Ã> Ù D ¹’õOÙ8>ˆ»ÜÛ E m> نVØ F õOمI> G¾Ú Û Þ
©-fªcha/·+CG_=KhDdN=RTU
F Jõ G$م>tmWF õOم>IGXù’õOم>ˆ»ZY\[ >Ù Ø F õYÙ8>IG Ú Û mW[ > Ù Ø F õJGªÙ…> Ú Û Þ
©-f!RTa/adfW_ˆ·BakDdfhN@D
õ R D â)Tu ] >م¹’â5Ù8>ˆ» Û ß¦> Ú u

·tfhCEPdCâ[øÃßG>GRTakN=U'RTU|DQCEV=CEPˆ? ©-fªcha/DdfWCGaQ_X{cWDdR\_UžaQCEDtR{a
9 ߉>@u~ßt„ªu~#ß BªuEÞEÞE^Þ ;`_ab_ é=ë ìXcí F â)u ] >م¹’â5Ù8>ˆ» Û ß¦>~Úed1Þ
µÜD-RTa/C~N=aQ€=CEPSR\}’°DdfhN@DtN=XTXDdfhC&PdC~N=X1Uªchy‰KMCEPSakR{UDdfhRTakaQCEDtR{UhZWCEC~ZžadN@DdRTad}¾°DdfWCGVR\€=C~U®C~Å^chN@DdRT_U?
f ? 6 XTX-DdfhCÉ`M_RTU^DdaR{UùDdfWC³`!X{N=UWCšN@PdC³e_X\_cWPSC~Z"c!adRTUWV[DdfhPdCECge_X\_cWPSaE?‡PS_ˆ€=C³DdfhN@DDdfWCEPdCÉC – R{aQDdaN
DQPSR{N=UWVX\Ct·BR\DdfJ€=CEPdDdR{eC~akfhN~€$RTUWVjDdfWCbaSN=yOC&e_X\_cWP/adcheSf°DdfhN@DGhñ g i-j$1ñ kR\D/RTa R{aQ_adeC~X\C~a.ï k RTDda N=UWVXTC~a+N@PSC
RTUV=CE_yJCEDQPSRTe&`hPS_=V=PdC~adadR\_U1?
‹ŒŽ
‘’Œ“”
ÿ _U!adRTZWCEP½NbeERTPSeEX\C+_=}!`M_adR\DdR\€=C PSN=ZhRTc!a¼RTUjDdfWC/`!XTN=UhC N=UhZYRTUhaSePSR\KMCNbPdCEVchXTN@P;fWCE`hDdN@V=_UOFbHJI&™ml?npo
RTUYR\DE?‚$R{UheC/DdfWC-aQCE€=C~UJ€=CEPSDdRTeC~a)_=}DdfhRTa)fWCE`hDdN@V=_U5N@PdC-e_X\_cWPSC~ZOK^lDdfWPdCEC-e_XT_cWPSaEq|ad_yOCkDdfWPdCEC-€=CEPdz
DdRTeC~atfhN~€=CGDdfhCGadN=yOCGe_X\_cWPˆqWK^5`!R\V=CE_U$zfW_XTC&`hPSRTUheERT`!X\C=? ÿ _UhadRTZWCEP/DdfWC&DQPœRTN=UWVX\C}’_=PSyOC~ZKª5DdfWC~adC
DdfWPSCECj€=CEPdDdR{eC~aE?jACEDc!abeEN=X{X¼DdfhCl`N@PdDB_=} DdfWCOeER\PSeEchyJ}¾CEPdC~U!eClaQCE`N@PSN@DQC~Z³K^šN=U|šD·+_®e_UhaQC~eEcWDdR\€=C
€=CEPdDdR{eC~a _=}DdfWCbfhCE`hDdN@V=_UN=, U q kAr~?©-fWCbDdfWPdCECb€=CEPdDdRTeC~a+_=}1DdfWCadN=yOCe_X\_cWP/N@PdCaQCE`!N@PSN@DQC~ZKªs
 q.k/rht
_=}/X\C~UWV=DdfgûÜ/u guQâ¦N=a&·+CJyO_ˆ€=C=q½adN~ge_chU^DQCEPQzeEX\_$edϪ·tRTadC=q¼N=X\_UWVžDdfhCYeERTPSeEX\C=qaQDdN@PSDdRTUWV}¾PS_yÁNç – C~Z
€=CEPdDQC – N=yO_UhVJDdfWC~aQCGDdfhPdCEC=qh·tfWCEPSCGû!u Ù :Ù¯âšw m v^?k‚$RTUheC=q$DdfWCG_=PœZWCEP-_=})ûÜ/u guQâ[ZW_^C~aBUW_=D-yJN@DQDQCEP
}’_=P&N5DQPœRTN=UWVX\C=qMDdfWCEPdCYN@PdCY}’_cWP`_aSadR\K!RTX{R\DdR\C~aEãb>ˆÙY>ˆÙ xNm v^Ä+>ˆÙ&„Ù f Nm v^Ä>ˆNÙ BÙ BNm v^ļ„Ù&„NÙ BNm v^?Gµ¶U
DdfWCçhPSaQDEq½DdfhR\PSZ´N=UhZ[}’_cWPdDdfeEN=aQC~aEq;· CfhNˆ€=C°RTad_adeC~X\C~aGDQPœRTN=UWVX\C~aE?µ¶U[DdfWC°aQC~e_UhZeEN=aQC=q)· C°fhN~€=C
NšDQPSRTN=UWVXTC·tfW_adC°N=UWVXTC~alN@PSC°RTUV=CE_yOCEDQPSR{e5`!Pd_=V=PdC~adadRT_U?É©tfWC}’_cWPOe_=PdPdC~aQ`M_UhZ!RTUWV®çhVchPdC~ajN@PSC
adfh_ˆ·tUKMC~X\_
·G?

D E E

C
C

B B B
A A A A
(i) (ii) (iii) (iv)

µ¶U[¹R¾»œq!FbH†m8HJIYÄWRTU[¹RTRTR’»œq$FylÐm8HblJÄhRTU[¹R\€h»œq!F&IÃmÃINlJÄ!N=UhZžR{U[¹RTR¾» ·+C‰aQCECGDdfhN@DBik™Çm{zÖ v^q


i/F…m….„ z Ö vYN=U!Zši/H†m f z Ö vl·tf!RTeSfžN@PdCjRTUV=CE_yOCEDQPSRTe‰`hPd_=V=PdC~aSadR\_U?
xª?BACEDGFBHJI:KMCON=U[N=eEcWDQCzN=UWVX\C~Z[DQPSRTN=UWVX\CON=UhZgXTCEN D |¨KMCOR\Dda_=PdDdfW_@zeC~U^DQPdC=?JA1CE?D }J~€‚5ZWC~UW_=DQCODdfhC
XTN@PSV=C~aQD-N=X\DdR\DdchZhC&_=}¼DdfWC‰DQPSRTN=UWVX\C&FbHJIY?‡PS_ˆ€=C‰Ddf!N@D
yF |ÇٗbH |Ù¦I |ƒù „ }>~€‚ªÞ
‹ŒŽ
‘’Œ“”

A1CED"i-I×KC8DdfWCƒadyJN=XTX\C~adD—N=UhVX\C=q'aQ_†Ddf!N@D
I&Fø¦FBH:N=UhZžI&HLø¦FBH°?)µ¶UDdf!RTa eEN=adCbDdfhC
A

N=X\DdR\DdchZWC&DdfhPd_cWVfšI¸RTakDdfWCjX\_UWV=C~aQD-_UhC=?+ACED
DdfWC&N=X\DdR\DdchZWCBDdfWPd_cWVf®IyOCECEDkFbH:R{U5™¬N=U!Z
K D X\CE„D |˜KMCDdfWC/_=PdDdfW_@zeC~U^DQPdCk_=}!FbHJIY?½A1CEDI&™
C – DQC~UhZhC~Z"yJCECED°DdfhCÉeER\PœeEchyYzeER\PSeEXTC®_=}‰FbHOI
H
RTK U …¯?  C³fhNˆ€=CgIG™¿ƒ m }J~€‚³ad_[DdfhN@DDdfhC
B C
RTUWC~ŪchN=XTR\DƁDQ_JKMC&`hPd_
€=C~Z'RTa
yF |ÇÙº† H |ÙºI |‡ù„IG™'Þ
Q adRTUWVOI&™ÇmÃI|ÐÙ,|ə'qªDdfhRTa PdC~ZhcheC~a DQ_OF!|ÐÙ³Hb|ˆ"I&™…, Ù |š™š?)¥t_
· CE€=CEPˆqh· Cb_=K!aQCEPS€=CbDdf!N@D
Fy| m:F!…¯qHb| m:Hb…ÈN=UhZ8|əåm:™"…¯?{¹ CW_=PjC – N=yO`X\CJbH | m:†H …ÈN=UhZgm ™ | m:" ™ …È}’_XTX\_
·
}’Pd_yÇDdfWC‰e_UhV=PScWC~Uhe_=}½DdfWCGPSR\Vf^D¶zN=UWVX\C~Z®DQPSRTN=UWVX\C~ak™°bH …åN=U!Z®™5bH |?ô»
©-fªchaB· CJUhCEC~ZšDQ_ž`!Pd_ˆ€=CODdfhN@D&Fy…«ÙþHb…‰I …¯? 6 `h`!X\$RTUWV‡DQ_X\C~yj: Šöa&DdfhCE_=PdC~y DQ_žDdfWCOeªeEX{RTe
ŪchN=ZWPSR{XTN@DQCEPSN=XMHJI&Fy…q$·+CGV=CED
FBŒH ‹
NI …¢m…F I ‹
†H …ٗHJ I ‹ˆyF …Šø¦FbŽ H ‹~bH …¬Ù—FbŒ H ‹~!F …šÞ
©-f!RTa/RTyO`!XTRTC~a Ddf!N@DbI …±ø¦yF …Ùº†H …¯qW·tf!RTeSfžRTa/`!PdC~eERTaQC~X\5·tfhN@D-·+C‰N@PdCGXT_^_=Ï$RTUWVY}’_=P~?
©-fhCEPdCk· CEPSC-_=DdfWCEPKMC~N=cWDdRT}’chX$aQ_X{cWDdR\_Uha;VR\€=C~UJK^OaQDdchZhC~U|Dda)·BfW_&`!N@PdDdRTeERT`!N@DQC~ZJRTUYAµ  U ¤BzƄ@o=onª?  C
PdC~e_=PœZžDdfWC~y¬fWCEPSC=?
 ACEDtFb™'q!b H lqI nÐKMC&DdfWCGN=X\DdRTDdchZWC~a-N=Uh Z |±KMCDdfWCG_=PdDdfh_@zeC~U|DQPdC=?k¤bK!aQCEPS€=CGDdfhN@D
!F | m F yF |šp H G m F yF |gyI G Þ
Fb™ F FB™°p H G F Fb™‘I G
©-f!RTa+VR\€=C~a
yF | m F !F |Ép H Gª’ Ù F !F |gyI G mÐ>-ß F bH |³‘I G Þ
FB™ F FB™°pH Gª’ Ù F FB™žyI G F FbHJyI G
‚$R{yJRTXTN@P+C – `!PdC~adadR\_U!a/}¾_=P-DdfhCGPSN@DdR\_a-bH |žÖrbH l¸N=UhZšNI |Ö=NI nÐyJN~KMC&_=KhDdN=RTUhC~Z? 6 Z!ZhRTUWVWq$·+CGV=CED
!F | Ù b H | I |
Fb™ bH l Ù I n m…„ªÞ
‚$ch`h`M_aQCbFb™RTakDdfWCGXTN@PSV=C~aQDtN=X\DdR\Ddc!ZWC=?  CGV=CED
yF | Ù † H | Ù NI |  yF | Ù bH | Ù NI | m…„ªÞ
FB™ FB™ Fb™ Fb™ Hbl INn
©-f!RTa+VR\€=C~a-DdfWC&PSC~adchX\DE?
“ A1CEyD ”ÐKC&DdfhCGeER\PSeEchyYzeC~U^DQPdC‰N=U!Z®X\CED •-qs†q!wåKMC&DdfWCGyJRTZWz`_R{U|Dda/_=};HJIYqI&FjqhFbH:PSC~aQ`MC~eÝz
DdR\€=C~XT=?B©-fWC~Uš·+CjÏ$UW_ˆ·DdfhN@DyF |±m „ ”•tq†H |±m „ ”Gs N=UhZgI |Šm „ ”&w? 6 abC~N@PSXTR\CEP~qN=adadchyJC
Fb™«R{a/DdfWC‰XTN@PdV=C~adDtN=X\DdR\DdchZhC=?;©tfWC~U®HJIRTa/DdfWCjX\C~N=aQDtadR{ZWC=?  CjfhNˆ€=C
f F FBHJ‘I Gm f F * H ”‰yI GªÙ f Fö?I ”&#F G$Ù f F F ”&pH Gxm HOW I –' „ ”•gÙ¦I&Ž F –' „ ”GsÊٗFb— H –' „ ”&w
m HOW I –®!F |¬ÙºIGŽ F –®bH |ٗFb˜ H –ÉI |
ø FBHž¹yF |Ù¯bH |Ù¦I |g»œÞ
©-fªcha
yF |ÇÙºH†|Ù¦IN|ˆ f F FbFbHJH yI G m8„@Fb™'Þ
™š  C&yJN@Ï=C&chadCB_=}1DdfhCb}N=eD/DdfhN@DkF!|¢m…„.›ge_aMi/F‰qWHb|¢m…„.›ge_ai/H°qhI| m…„.›[e_aitIvN=U!Z
Fb™ÇmÄ.›[adRTUti/H¯adRTUtitIYqW·tfWCEPSNC ›ÐRTa/DdfWC‰eER\PœeEchyYzPSN=ZhR{cha+_=};FbHJIY?  ClN@PdC‰N=adadc!yJRTUWVYDdfhN@DtFb™
RTa/DdfhC‰XTN@PdV=C~aQD-N=XTDdR\DdchZWCGaQ_ODdf!N@Dbi/FRTakDdfWCGX\C~N=adDtN=UWVX\C=?©tf^cha/·+C‰fhNˆ€=CGDQ_`!Pd_ˆ€=CGDdf!N@D
e_ai/FºÙ—e_aikH8ٗe_aMi-œ I ù„;aSRTUti/HJi-Ibu
chU!ZWCEPDdfWCN=adadc!yO`hDdR\_U´i/ F ÐikHN=U!Z´i/ F †i-IO?J¤&U[yjc!X\DdR\`!X\$RTUWV5DdfhRTaKª³„;aSRTUti/FjqDdfhRTa&R{a
C~ŪchR\€@N=X\C~U^D+DQ_
„W¹adRTUti/FÉe_aikF—Ù—aSRTUti/FÉe_ai/H…Ù—adRTUtikFše_aMi-I‰»
 f aSRTUBi/FÉadRTUti/HJi-I…m…adRTU-„@F¦Ù—adRTUk„@HÃٗadR{U-„IbÞ
©-f!RTa/RTakC~ŪchR\€@N=X\C~U|D/DQ_
e_ai/Hž¹adRTU-i/F"ß[adR{Uti/HJ»ٗe_ai-I¹adRTUti/F"ß[adRTU-i-Ij`» ¦o$Þ
‚$R{UheC&FBHJIRTatN=eEcWDQCzN=UWVX\C~ZšN=UhZ®FRTakDdfWCGX\C~N=adDtN=UWVX\C=qhDdfhC&PdC~adchX\D/}’_XTX\_
·taE?
žª?BACED-á!uSÒ
udÑ&KMC&`M_adR\DdR\€=CPdC~N=XU^chyjKMCEPSakadcheœfžDdfhN@Dtá ÷ ÙºÒ ÷ m8Ñ ÷ ?‡PS_ˆ€=CGDdfhN@D
á Û Ù—Ò Û ß[Ñ Û 7ŸžW¹Ñkß[áW»¹Ñ-ß[ÒE»œÞ
‹ ŒŽ
‘’Œ“”
©-fhCbVRT€=C~U'RTUWC~ŪchN=XTRTD5yN~KMCb·tPSR\DQDQC~U®RTUDdfhC&}¾_=PSy
vrÑÛ-@ß žW¹áGÙºÒE»ÜÑ/ßù¹áªÛÙºÒÝÛkß@ž@áªÒE»ZYùo$Þ
‡ cWDQDdRTUWVjõw m vrÑ Û eq  OmÐ#ß žW¹á-ÙgÒE»ÜÑ
Jq ¡lm߉¹á Û Ù[Ò Û s ß ž@á$Ò»œqW·+CfhN~€=CDQ_Y`hPd_
€=CbDdf!N@D+õ&ÙS tÙ,¡Yùo$?
¤bKaQCEPd€=C&DdfhN@D-õ¼/u  Vu ¡5N@PdC‰UW_=D-N=X{XC~Å^c!N=X¹’ õ 7¦o$q  mY¦o|»œ? Q adR{UWVODdfWCGRTZWC~U^DdR\DƁ
õ ÷ Ù  ÷ ÙO¡ ÷ ßBrõ e¡jm „> ¹’õOÙu &Ù¡^» D ¹’õ5ߢ h» Û Ù…¹- Yß8¡^» Û Ù…¹ ¡jßgõM» ÛhE u
·+CRTUW}’CEPDdfhN@DjRTDGRTa&ad>c £eERTC~U|D&DQ_®`!Pd_ˆ€=C5õ ÷ K Ù  ÷ Ÿ Ù ¡÷ K ß Br:õ   ¡¢Yo$?‚WcWK!aQDdR\DdchDdRTUWV°DdfWCO€rN=X{cWC~a&_=}
õ¼/u  Vu ¡Wq!· C‰adCECGDdfhN@D-DdfhR{akRTa/C~Å^c!R\€rN=XTC~U|DkDQ_
B f B@hÑ ¤-ß„$'> žW¹á‰ÙºÒE» ÷ Ñ ÷ ßþ¹á Û ÙºÒ Û @ ß ž@á$ÒE» ÷ ߦ>~ „ ž@Ñ ÷ ¹á&ÙºÒE»¹á Û Ù¦Ò Û @ ß ž@á$ÒEZ» Y¦o$Þ
Q adRTUWVOÑ ÷ m8á ÷ ÙºÒ ÷ qhDdfhRTa/PdC~Z!cheC~akDQ_
B f BØÆá ÷ ÙžÒ ÷ Ú Û ß„$'> žW¹á)ٞÒE» ÷ ¹á ÷ Ù®Ò ÷ »ªß'¹áªÛhٞÒÝÛM ß ž@áªÒE» ÷ ß®>~ „ žW¹Q¹á ÷ ÙžÒ ÷ »¹á;Ù®Ò»¹áªÛ!ٞÒÝÛß ž@áªÒEZ» Yùo$Þ
©-f!RTa/yJN~KMCGadRTyO`XTR\çhC~ZɹN@}¾DQCEPaQ_yOCGDQC~Z!R\_chakeEN=XTeEchX{N@DdR\_Uhaœ» DQ_Wq
ß-á Û Ò Û ¹¶>~„=n@á Û ß´ „ x f á$ÒÙ8>~„=n=Ò Û Z» Yùo$Þ
­/cWDG>~„=n@á Û ß´ „ x f á$ÒÙ8>~„=n=Ò Û m>~„=nW¹áOßÒ» Û Ù f á$‘Ò 7¦o$?¥BC~UheCjDdfWC&PdC~adchXTD+}’_XTX\_
·taE?
¥ Ì ¦Îh^Í §;” ©tfWCKC~adD-e_UhaQDdN=U^#D ¨ORTU°DdfWCGRTUWC~ŪchN=XTRTDá Û ÙÒ Û ß³Ñ Û K ø ¨!¹Ñ+ßgá$»¹Ñ/ßgÒE»œqh·tfWCEPdC&áuSÒ
udÑ
N@PdC&`M_adRTDdR\€=C&PdC~N=XTa-aScheSfžDdf!N@Dtá ÷ ÙºÒ ÷ m8Ñ ÷ qhRTa#¨‰m…„ Ø > Ù¦„ <í © ÷ Ùº„ c<í © ÷ Ú ?
¥BCEPdCGN@VN=RTUq!DdfWCEPdCG·+CEPdC‰aQ_yJC&KC~N=chDdR\}’c!Xad_XTcWDdR\_Uha/VRT€=C~UžK^žaQDdc!ZWC~U|Dda~?
  ClfhN~€=C
á ÷ m8Ñ ÷ ß[Ò ÷ mйÑkß´ÒE»¹Ñ Û Ù—ÑEÒ)ÙºÒ Û »œu
·tf!RTeSfžRTakaSN=yOC‰N=a
á Û m Ñ Û Ù—ÑEÒ)ÙºÒ Û Þ
Ñkß´Ò á
‚$R{yJRTXTN@PSXT=q|·+CGV=CED
Ò Û m Ñ Û Ù—ÑáGٗá Û Þ
Ñ/ß[á Ò
 CG_=KaQCEPd€=CGDdfhN@D
á Û Ù Ò Û m Ñ@¹á Û ÙºÒ Û »;ß[á ÷ ß´Ò ÷ m Ñ=¹á Û ÙºÒ Û ß[Ñ Û » Þ
Ñ+ßÒ Ñkß[á ¹ÑkßgáW»¹Ñ-ß´ÒE» ¹Ñkß[á$»¹ÑkßÒ»
©-f!RTa/adfW_ˆ·BakDdfhN@D
á Û ÙºÒ Û ß[Ñ Û m Ñ Û Ù—ÑEÒÙºÒ Û Ù Ñ Û Ù—ÑáGÙ¯á Û Þ
¹Ñkß´á$»¹Ñkß´ÒE» Ñá ÑEÒ
©-fªchakR\D-R{akad>c £eER\C~U^DkDQ_J`hPS_ˆ€=CGDdfhN@D
Ñ Û Ù—ÑEÒ;Ù¦Ò Û Ù Ñ Û Ù—ÑáGٗá Û øŸžªÞ
Ñá ÑEÒ
¥B_ˆ·+CE€=CEP~q!·+C‰fhNˆ€=CjÑ Û ÙºÒ Û øù„@ÑEÒN=UhZ®Ñ Û Ù—á Û øù„@Ñá!? ¥tC~UheC
Ñ Û Ù—ÑEÒ)ÙºÒ Û Ù Ñ Û Ù—Ñá&Ùºá Û øŸB0ª Ò Ù á øŸB0–'„‰mwžªÞ
Ñá ÑÒ á Ò «
 C‰f!N~€=Cjc!aQC~Z 6 U ‚z ¬ U RTUhC~Å^chN=X{R\D=?
“ A1CEDGc!a&aQCED&õ´m†áWÖrÑYN=Uh, Z  šm¸ÒEÖrÑr?O©-fWC~U³õ ÷ Ÿ Ù   ÷ m¬>YN=UhZ³DdfhCJRTUWC~ŪchN=XTR\DƁ®DQ_žKMCl`!Pd_ˆ€=C~Z[R{a
õÛO Ù   Û ßºN> 7­žW¹¶>tßgõ»¹¶>-¢ ß  h»œ?©-fhRTa+PdC~Z!cheC~akDQ_
¹’õYO Ù  W» Û  Ù žW¹’õJu Ù  h»½@ ß ®rõ  O ß v†7ùo$Þ ¹¶>ˆ»
­/cWD
>Bm"õ ÷ O Ù   ÷ mй’õOu Ù  h»¹’õ Û ßg:õ  &u Ù   Û »œu
·tf!RTeSfOVRT€=C~a:õ  OmØS¹’õB¯ Ù  h» ÷ ß³>~ڈ.Ö BW¹’õBs Ù  h»œ? ‚$cWK!adDdR\DdcWDdRTUWVDdfhR{aRTU'¹¶>ˆ»N=UhZR{U|DQPd_$ZhcheER{UWVõB¯ Ù  Om"àÝq
DdfWCGR{UWC~Å^c!N=XTR\D5DdN@Ï=C~atDdfhC&}¾_=PSy
÷
àÜÛÙžrà)ß ®B ¹’à àߦ>ˆ» ßuv†7¦o$Þ ¹ä„»
©-f!RTa/yJN~KMCGadRTyO`XTR\çhC~ZDQ_š#ß xrà ÷ Ù8'> ®rà Û ß„rà1K Ù ®07ºo$? • Å^chRT€rN=X\C~U^DdX\
߉°¹ xrà)@ ß ®»¹’à)ߦ>ˆ»ÜÛ 7¦o$Þ
©-fªcha · CGUhCEC~ZDQ_l`!Pd_ˆ€=C&DdfhN@#D xrZà YŸ®ª?+¤K!aQCEPd€=CbDdfhN@D‰¹’õj8 Ù  W» ÷ 7ºõ ÷ 8 Ù   ÷ m>@qhad_lDdf!N@D+Zà 7Ã>@?  C
N=XTad_JfhN~€=C
ª õYO Ù    õ÷ u Ù  ÷ m >Þ
„ « „ „
©-f!RTa/adfW_ˆ·BakDdfhN@D-à ÷  f ?©-fªcha
xª rà ÷  ~> „ x– f m x@o=o YÃ>@Þ
®N« $x >~„ $x >~„
¥BC~UheCpxràZYŸ®ªqh·tf!RTeSf`hPS_ˆ€=C~a-DdfhCGVR\€=C~U®RTUhC~Å^chN=X{R\D=?
™š  Cj·-PSR\DQCGÒ ÷ m8Ñ ÷ ß[á ÷ N=UhZ®á ÷ m8Ñ ÷ ß´Ò ÷ aQ_JDdfhN@D
÷ ÷
Ñ/ß[áOm Ñ Û ß[ÑÒ áGٗá Û u Ñ/ß´Òkm Ñ Û ß[á ÑEÒÙºÒ Û Þ
©-fªcha/DdfWCGR{UWC~Å^c!N=XTR\D5PdC~ZhcheC~a-DQ_
÷ ÷
áªÛÙºÒÝÛ+ß´ÑÝÛ7­ž ¹Ñ Û ß[Ñáٗá á Û »Ò ¹Ñ Û ßgÑEÒÙºÒ Û » Þ
©-f!RTa/adRTyO`!X{R\çhC~aˆ¹N@}’DQCEPkaQ_yOC‰X\C~UhV=Ddf|eEN=XTeEchXTN@DdRT_Uhaœ» DQ_
ß[Ñ ¤ ßù¹á&ÙºÒE»ÜIÑ ±kß´áªÒÝIÑ ² م¹á ÷ ÙºÒ ÷ »ÜÑ ÷ Ùù(á ² ٗá ÷ Òٗá Û Ò Û Ù—á$Ò ÷ ÙºÒH²
»ÜÑ Û
¹á Û Ò)ٗá$Ò Û Ù—á ÷ ÙºÒ ÷ »ÜáªÒÝÑم¹á ² Ò Û @
ß ž@á ÷ Ò ÷ ٗá Û Ò ² »Z7¦o$Þ
‚$chK!aQDdR\DdcWDdR{UWV
Ñ ÷ m…á ÷ ÙºÒ ÷ u IÑ ²Bm8Ñ=¹á ÷ ÙºÒ ÷ »œu hÑ ±tm8Ñ Û ¹á ÷ Ù¦Ò ÷ »œu Ñ ¤ m†¹á ÷ ÙºÒ ÷ » Û u
DdfWCGR{UWC~Å^c!N=XTR\D5}’chPdDdfWCEPkPdC~Zhc!eC~akDQ_
á^Û~ÒÝÛ=¹á^Û٦ҜÛÙºÑÝÛÙºá^ÑÙºÒÝÑk@ ß ž@á$ÒEZ» 7¦o$Þ
©-fªcha)·+CBUhCEC~ZJDQ_‰`!Pd_ˆ€=CBDdfhN@D á Û ÙšÒ Û Ù®Ñ Û Ù®áªÑٚҜÑm ß ž@á$Ò 7¦o$? ‚$R{UheC-á Û Ù'Ò Û øù„@á$Ò
qªR\D RTa)C~UW_cWVf
DQ_Y`hPd_
€=C&DdfhN@DkÑ Û Ù[Ñ=¹áBÙ´ÒE»ß f áª‘Ò 7¦o$? U chX\DdRT`!X\$RTUWV&DdfhRTa+Kª5ÑbN=UhZchadRTUhVjá ÷ Ù´Ò ÷ m8Ñ ÷ q$· C&UWCEC~Z
DQ_J`!Pd_ˆ€=CGDdf!N@D
á ÷ Ù—Ò ÷ Ù—Ñ Û áÙ—Ñ Û yÒ 7 f á$ÒÝÑ
Þ
Q adRTUWV 6 U ‚z ¬ U RTUWC~ŪchN=XTR\DƁDQ_JDdfWC~adC f DQCEPSyJaBN=UhZžchadRTUhVYÑ 7ùá!qhÑ 7ùÒB·+CGV=CED
á ÷ Ù—Ò ÷ Ù—Ñ Û áÙºÑ Û yÒ 7 f ¹á ÷ Ò ÷ Ñ Û á^Ñ Û ÒE» <í © ² m f á$ҜÑru
·tf!RTeSf`hPS_ˆ€=C~a-DdfhCGRTUWC~ŪchN=XTR\DƁ=?
INMO-2010 Problems and Solutions

1. Let ABC be a triangle with circum-circle Γ. Let M be a point in the interior of triangle
ABC which is also on the bisector of ∠A. Let AM , BM , CM meet Γ in A1 , B1 , C1
respectively. Suppose P is the point of intersection of A1 C1 with AB; and Q is the point
of intersection of A1 B1 with AC. Prove that P Q is parallel to BC.
Solution: Let A = 2α. Then ∠A1 AC = ∠BAA1 = α. Thus

∠A1 B1 C = α = ∠BB1 A1 = ∠A1 C1 C = ∠BC1 A1 .

We also have ∠B1 CQ = ∠AA1 B1 = β, say. It follows that triangles M A1 B1 and QCB1
are similar and hence
QC B1 C
= .
M A1 B1 A1

C1

A
Γ α α
B1
α α

α
α
P M Q

γ γ β
β

B γ β C

A1

Similarly, triangles ACM and C1 A1 M are similar and we get


AC C1 A1
= .
AM C1 M
Using the point P , we get similar ratios:
PB C1 B AB A1 B1
= , = .
M A1 A1 C1 AM M B1
Thus,
QC A1 C1 · B1 C
= ,
PB C1 B · B1 A1
and
AC M B1 · C1 A1
=
AB A1 B1 · C1 M
M B1 C1 A1 M B1 C1 B · QC
= = .
C1 M A1 B1 C1 M P B · B1 C
However, triangles C1 BM and B1 CM are similar, which gives
B1 C M B1
= .
C1 B M C1
Putting this in the last expression, we get
AC QC
= .
AB PB
We conclude that P Q is parallel to BC.

2. Find all natural numbers n > 1 such that n2 does not divide (n − 2)!.
Solution: Suppose n = pqr, where p < q are primes and r > 1. Then p ≥ 2, q ≥ 3 and
r ≥ 2, not necessarily a prime. Thus we have

n − 2 ≥ n − p = pqr − p ≥ 5p > p,
n − 2 ≥ n − q = q(pr − 1) ≥ 3q > q,
n − 2 ≥ n − pr = pr(q − 1) ≥ 2pr > pr,
n − 2 ≥ n − qr = qr(p − 1) ≥ qr.

Observe that p, q, pr, qr are all distinct. Hence their product divides (n − 2)!. Thus
n2 = p2 q 2 r 2 divides (n − 2)! in this case. We conclude that either n = pq where p, q are
distinct primes or n = pk for some prime p.
Case 1. Suppose n = pq for some primes p, q, where 2 < p < q. Then p ≥ 3 and q ≥ 5.
In this case

n − 2 > n − p = p(q − 1) ≥ 4p,


n − 2 > n − q = q(p − 1) ≥ 2q.

Thus p, q, 2p, 2q are all distinct numbers in the set {1, 2, 3, . . . , n − 2}. We see that
n2 = p2 q 2 divides (n − 2)!. We conclude that n = 2q for some prime q ≥ 3. Note that
n − 2 = 2q − 2 < 2q in this case so that n2 does not divide (n − 2)!.
Case 2. Suppose n = pk for some prime p. We observe that p, 2p, 3p, . . . (pk−1 − 1)p all
lie in the set {1, 2, 3, . . . , n − 2}. If pk−1 − 1 ≥ 2k, then there are at least 2k multiples of
p in the set {1, 2, 3, . . . , n − 2}. Hence n2 = p2k divides (n − 2)!. Thus pk−1 − 1 < 2k.
If k ≥ 5, then pk−1 − 1 ≥ 2k−1 − 1 ≥ 2k, which may be proved by an easy induction.
Hence k ≤ 4. If k = 1, we get n = p, a prime. If k = 2, then p − 1 < 4 so that p = 2
or 3; we get n = 22 = 4 or n = 32 = 9. For k = 3, we have p2 − 1 < 6 giving p = 2;
n = 23 = 8 in this case. Finally, k = 4 gives p3 − 1 < 8. Again p = 2 and n = 24 = 16.
However n2 = 28 divides 14! and hence is not a solution.
Thus n = p, 2p for some prime p or n = 8, 9. It is easy to verify that these satisfy the
conditions of the problem.

3. Find all non-zero real numbers x, y, z which satisfy the system of equations:

(x2 + xy + y 2 )(y 2 + yz + z 2 )(z 2 + zx + x2 ) = xyz,


(x4 + x2 y 2 + y 4 )(y 4 + y 2 z 2 + z 4 )(z 4 + z 2 x2 + x4 ) = x3 y 3 z 3 .

Solution: Since xyz 6= 0, We can divide the second relation by the first. Observe that

x4 + x2 y 2 + y 4 = (x2 + xy + y 2 )(x2 − xy + y 2 ),

holds for any x, y. Thus we get

(x2 − xy + y 2 )(y 2 − yz + z 2 )(z 2 − zx + x2 ) = x2 y 2 z 2 .


However, for any real numbers x, y, we have

x2 − xy + y 2 ≥ |xy|.

Since x2 y 2 z 2 = |xy| |yz| |zx|, we get

|xy| |yz| |zx| = (x2 − xy + y 2 )(y 2 − yz + z 2 )(z 2 − zx + x2 ) ≥ |xy| |yz| |zx|.

This is possible only if

x2 − xy + y 2 = |xy|, y 2 − yz + z 2 = |yz|, z 2 − zx + x2 = |zx|,

hold simultaneously. However |xy| = ±xy. If x2 − xy + y 2 = −xy, then x2 + y 2 = 0 giving


x = y = 0. Since we are looking for nonzero x, y, z, we conclude that x2 − xy + y 2 = xy
which is same as x = y. Using the other two relations, we also get y = z and z = x. The
first equation now gives 27x6 = x3 . This gives x3 = 1/27(since x 6= 0), or x = 1/3. We
thus have x = y = z = 1/3. These also satisfy the second relation, as may be verified.

4. How many 6-tuples (a1 , a2 , a3 , a4 , a5 , a6 ) are there such that each of a1 , a2 , a3 , a4 , a5 , a6


is from the set {1, 2, 3, 4} and the six expressions

a2j − aj aj+1 + a2j+1

for j = 1, 2, 3, 4, 5, 6(where a7 is to be taken as a1 ) are all equal to one another?

Solution: Without loss of generality, we may assume that a1 is the largest among
a1 , a2 , a3 , a4 , a5 , a6 . Consider the relation

a21 − a1 a2 + a22 = a22 − a2 a3 + a23 .

This leads to
(a1 − a3 )(a1 + a3 − a2 ) = 0.
Observe that a1 ≥ a2 and a3 > 0 together imply that the second factor on the left side
is positive. Thus a1 = a3 = max{a1 , a2 , a3 , a4 , a5 , a6 }. Using this and the relation

a23 − a3 a4 + a24 = a24 − a4 a5 + a25 ,

we conclude that a3 = a5 as above. Thus we have

a1 = a3 = a5 = max{a1 , a2 , a3 , a4 , a5 , a6 }.

Let us consider the other relations. Using

a22 − a2 a3 + a23 = a23 − a3 a4 + a24 ,

we get a2 = a4 or a2 + a4 = a3 = a1 . Similarly, two more relations give either a4 = a6


or a4 + a6 = a5 = a1 ; and either a6 = a2 or a6 + a2 = a1 . Let us give values to a1 and
count the number of six-tuples in each case.

(A) Suppose a1 = 1. In this case all aj ’s are equal and we get only one six-tuple
(1, 1, 1, 1, 1, 1).
(B) If a1 = 2, we have a3 = a5 = 2. We observe that a2 = a4 = a6 = 1 or a2 = a4 =
a6 = 2. We get two more six-tuples: (2, 1, 2, 1, 2, 1), (2, 2, 2, 2, 2, 2).
(C) Taking a1 = 3, we see that a3 = a5 = 3. In this case we get nine possibilities for
(a2 , a4 , a6 );

(1, 1, 1), (2, 2, 2), (3, 3, 3), (1, 1, 2), (1, 2, 1), (2, 1, 1), (1, 2, 2), (2, 1, 2), (2, 2, 1).
(D) In the case a1 = 4, we have a3 = a5 = 4 and

(a2 , a4 , a6 ) = (2, 2, 2), (4, 4, 4), (1, 1, 1), (3, 3, 3),


(1, 1, 3), (1, 3, 1), (3, 1, 1), (1, 3, 3), (3, 1, 3), (3, 3, 1).

Thus we get 1 + 2 + 9 + 10 = 22 solutions. Since (a1 , a3 , a5 ) and (a2 , a4 , a6 ) may be


interchanged, we get 22 more six-tuples. However there are 4 common among these,
namely, (1, 1, 1, 1, 1, 1), (2, 2, 2, 2, 2, 2), (3, 3, 3, 3, 3, 3) and (4, 4, 4, 4, 4, 4). Hence the total
number of six-tuples is 22 + 22 − 4 = 40.

5. Let ABC be an acute-angled triangle with altitude AK. Let H be its ortho-centre and O
be its circum-centre. Suppose KOH is an acute-angled triangle and P its circum-centre.
Let Q be the reflection of P in the line HO. Show that Q lies on the line joining the
mid-points of AB and AC.
Solution: Let D be the mid-point of BC; M that of HK; and T that of OH. Then
P M is perpendicular to HK and P T is perpendicular to OH. Since Q is the reflection
of P in HO, we observe that P, T, Q are collinear, and P T = T Q. Let QL, T N and OS
be the perpendiculars drawn respectively from Q, T and O on to the altitude AK.(See
the figure.)

L
Q
H
N T
S O
M P

B K D C

We have LN = N M , since T is the mid-point of QP ; HN = N S, since T is the mid-point


of OH; and HM = M K, as P is the circum-centre of KHO. We obtain

LH + HN = LN = N M = N S + SM,

which gives LH = SM . We know that AH = 2OD. Thus

AL = AH − LH = 2OD − LH = 2SK − SM = SK + (SK − SM ) = SK + M K


= SK + HM = SK + HS + SM = SK + HS + LH = SK + LS = LK.

This shows that L is the mid-point of AK and hence lies on the line joining the midpoints
of AB and AC. We observe that the line joining the mid-points of AB and AC is also
perpendicular to AK. Since QL is perpendicular to AK, we conclude that Q also lies on
the line joining the mid-points of AB and AC.
A
Remark: It may happen that H is above L as in the
adjoining figure, but the result remains true here as
well. We have HN = N S, LN = N M , and HM =
H
M K as earlier. Thus HN = HL + LN and N S = Q
L
SM + N M give HL = SM . Now AL = AH + HL =
2OD + SM = 2SK + SM = SK + (SK + SM ) = N T
SK + M K = SK + HM = SK + HL + LM = SK + M P O
S
SM + LM = LK. The conclusion that Q lies on the
line joining the mid-points of AB and AC follows as
earlier.
K D
6. Define a sequence han in≥0 by a0 = 0, a1 = 1 and

an = 2an−1 + an−2 ,

for n ≥ 2.

(a) For every m > 0 and 0 ≤ j ≤ m, prove that 2am divides am+j + (−1)j am−j .
(b) Suppose 2k divides n for some natural numbers n and k. Prove that 2k divides an .

Solution:

(a) Consider f (j) = am+j + (−1)j am−j , 0 ≤ j ≤ m, where m is a natural number. We


observe that f (0) = 2am is divisible by 2am . Similarly,

f (1) = am+1 − am−1 = 2am

is also divisible by 2am . Assume that 2am divides f (j) for all 0 ≤ j < l, where
l ≤ m. We prove that 2am divides f (l). Observe

f (l − 1) = am+l−1 + (−1)l−1 am−l+1 ,


f (l − 2) = am+l−2 + (−1)l−2 am−l+2 .

Thus we have

am+l = 2am+l−1 + am+l−2


= 2f (l − 1) − 2(−1)l−1 am−l+1 + f (l − 2) − (−1)l−2 am−l+2
= 2f (l − 1) + f (l − 2) + (−1)l−1 am−l+2 − 2am−l+1


= 2f (l − 1) + f (l − 2) + (−1)l−1 am−l .

This gives
f (l) = 2f (l − 1) + f (l − 2).
By induction hypothesis 2am divides f (l − 1) and f (l − 2). Hence 2am divides f (l).
We conclude that 2am divides f (j) for 0 ≤ j ≤ m.
(b) We see that f (m) = a2m . Hence 2am divides a2m for all natural numbers m. Let
n = 2k l for some l ≥ 1. Taking m = 2k−1 l, we see that 2am divides an . Using an
easy induction , we conclude that 2k al divides an . In particular 2k divides an .
Problems and Solutions, INMO-2011
1. Let D, E, F be points on the sides BC, CA, AB respectively of a triangle ABC such that
BD = CE = AF and ∠BDF = ∠CED = ∠AF E. Prove that ABC is equilateral.

Solution 1:
Let BD = CE = AF = x; ∠BDF =
A ∠CED = ∠AF E = θ. Note that ∠AF D =
B + θ, and hence ∠DF E = B. Similarly,
b−x ∠EDF = C and ∠F ED = A. Thus the tri-
x angle EF D is similar to ABC. We may take
F D = ka, DE = kb and EF = kc, for some
kc E positive real constant k. Applying sine rule to
θ
F triangle BF D, we obtain
A θ
B
c−x ka 2Rka
= = ,
c−x ka kb x sin θ sin B b
C where R is the circum-radius of ABC. Thus
θ
we get 2Rk sin θ = b(c − x)/a. Similarly, we
x a−x obtain 2Rk sin θ = c(a − x)/b and 2Rk sin θ =
B D C
a(b − x)/c. We therefore get
b(c − x) c(a − x) a(b − x)
= = . (1)
a b c
If some two sides are equal, say, a = b, then a(c − x) = c(a − x) giving a = c; we get a = b = c
and ABC is equilateral. Suppose no two sides of ABC are equal. We may assume a is the
least. Since (1) is cyclic in a, b, c, we have to consider two cases: a < b < c and a < c < b.
Case 1. a < b < c.
In this case a < c and hence b(c − x) < a(b − x), from (1). Since b > a and c − x > b − x, we
get b(c − x) > a(b − x), which is a contradiction.
Case 2. a < c < b.
We may write (1) in the form
(c − x) (a − x) (b − x)
= = . (2)
a/b b/c c/a
b a
Now a < c gives a − x < c − x so that < . This gives b2 < ac. But b > a and b > c, so
c b
that b2 > ac, which again leads to a contradiction
Thus Case 1 and Case 2 cannot occur. We conclude that a = b = c.
Solution 2. We write (1) in the form (2), and start from there. The case of two equal sides
is dealt as in Solution 1. We assume no two sides are equal. Using ratio properties in (2), we
obtain
a−b b−c
= .
(ab − c2 )/ca (bc − a2 )/ab
This may be written as c(a − b)(bc − a2 ) = b(b − c)(ab − c2 ). Further simplification gives
ab3 + bc3 + ca3 = abc(a + b + c). This may be further written in the form
ab2 (b − c) + bc2 (c − a) + ca2 (a − b) = 0. (3)
If a < b < c, we write (3) in the form
0 = ab2 (b − c) + bc2 (c − b + b − a) + ca2 (a − b) = b(c − b)(c2 − ab) + c(b − a)(bc − a2 ).
Since c > b, c2 > ab, b > a and bc > a2 , this is impossible. If a < c < b, we write (3), as in
previous case, in the form
0 = a(b − c)(b2 − ca) + c(c − a)(bc − a2 ),
which again is impossible.
One can also use inequalities: we can show that ab3 + bc3 + ca3 ≥ abc(a + b + c), and equality
holds if and only if a = b = c. Here are some ways of deriving it:
(i) We can write the inequality in the form

b2 c2 a2
+ + ≥ a + b + c.
c a b
Adding a + b + c both sides, this takes the form

b2 c2 a2
+c+ +a+ + b ≥ 2(a + b + c).
c a b
But AM-GM inequality gives

b2 c2 a2
+ c ≥ 2b, + a ≥ 2a, + b ≥ 2a.
c a b
Hence the inequality follows and equality holds if and only if a = b = c.
(ii) Again we write the inequality in the form

b2 c2 a2
+ + ≥ a + b + c.
c a b
We use b/c with weight b, c/a with weight c and a/b with weight a, and apply weighted
AM-HM inequality:

b c a (a + b + c)2
b· +c· +a· ≥ ,
c a b c
b · b + c · ac + a · b
a

which reduces to a + b + c. Again equality holds if and only if a = b = c.

Solution 3. Here is a pure geometric solution given by a student. Consider the triangle
BDF , CED and AF E with BD, CE and AF as bases. The sides DF , ED and F E make
equal angles θ with the bases of respective triangles. If B ≥ C ≥ A, then it is easy to
see that F D ≥ DE ≥ EF . Now using the triangle F DE, we see that B ≥ C ≥ A gives
DE ≥ EF ≥ F D. Combining, you get F D = DE = EF and hence A = B = C = 60◦ .
2. Call a natural number n faithful, if there exist natural numbers a < b < c such that a divides
b, b divides c and n = a + b + c.
(i) Show that all but a finite number of natural numbers are faithful.
(ii) Find the sum of all natural numbers which are not faithful.

Solution 1: Suppose n ∈ N is faithful. Let k ∈ N and consider kn. Since n = a + b + c, with


a > b > c, c b and b a, we see that kn = ka + kb + kc which shows that kn is faithful.
Let p > 5 be a prime. Then p is odd and p = (p − 3) + 2 + 1 shows that p is faithful. If
n ∈ N contains a prime factor p > 5, then the above observation shows that n is faithful. This
shows that a number which is not faithful must be of the form 2α 3β 5γ . We also observe that
24 = 16 = 12 + 3 + 1, 32 = 9 = 6 + 2 + 1 and 52 = 25 = 22 + 2 + 1, so that 24 , 32 and 52 are
faithful. Hence n ∈ N is also faithful if it contains a factor of the form 2α where α ≥ 4; a factor
of the form 3β where β ≥ 2; or a factor of the form 5γ where γ ≥ 2. Thus the numbers which
are not faithful are of the form 2α 3β 5γ , where α ≤ 3, β ≤ 1 and γ ≤ 1. We may enumerate all
such numbers:
1, 2, 3, 4, 5, 6, 8, 10, 12, 15, 20, 24, 30, 40, 60, 120.
Among these 120 = 112 + 7 + 1, 60 = 48 + 8 + 4, 40 = 36 + 3 + 1, 30 = 18 + 9 + 3, 20 = 12 + 6 + 2,
15 = 12 + 2 + 1, and 10 = 6 + 3 + 1. It is easy to check that the other numbers cannot be
written in the required form. Hence the only numbers which are not faithful are

1, 2, 3, 4, 5, 6, 8, 12, 24.

Their sum is 65.

Solution 2: If n = a + b + c with a < b < c is faithful, we see that a ≥ 1, b ≥ 2 and c ≥ 4.


Hence n ≥ 7. Thus 1, 2, 3, 4, 5, 6 are not faithful. As observed earlier, kn is faithful whenever

2
n is. We also notice that for odd n ≥ 7, we can write n = 1 + 2 + (n − 3) so that all odd n ≥ 7
are faithful. Consider 2n, 4n, 8n, where n ≥ 7 is odd. By observation, they are all faithful.
Let us list a few of them:

2n : 14, 18, 22, 26, 30, 34, 38, 42, 46, 50, 54, 58, 62, . . .
4n : 28, 36, 44, 52, 60, 68, . . .
8n : 56, 72, . . . ,

We observe that 16 = 12 + 3 + 1 and hence it is faithful. Thus all multiples of 16 are also
faithful. Thus we see that 16, 32, 48, 64, . . . are faithful. Any even number which is not a
multiple of 16 must be either an odd multiple of 2, or that of 4, or that of 8. Hence, the only
numbers not covered by this process are 8, 10, 12, 20, 24, 40. Of these, we see that

10 = 1 + 3 + 6, 20 = 2 × 10, 40 = 4 × 10,

so that 10,20,40 are faithful. Thus the only numbers which are not faithful are

1, 2, 3, 4, 5, 6, 8, 12, 24.

Their sum is 65.


3. Consider two polynomials P (x) = an xn + an−1 xn−1 + · · · + a1 x + a0 and Q(x) = bn xn +
bn−1 xn−1 + · · · + b1 x + b0 with integer coefficients such that an − bn is a prime, an−1 = bn−1
and an b0 − a0 bn 6= 0. Suppose there exists a rational number r such that P (r) = Q(r) = 0.
Prove that r is an integer.

Solution: Let r = u/v where gcd(u, v) = 1. Then we get

an un + an−1 un−1 v + · · · + a1 uv n−1 + a0 v n = 0,


n n−1 n−1 n
bn u + bn−1 u v + · · · + b1 uv + b0 v = 0.

Subtraction gives

(an − bn )un + (an−2 − bn−2 )un−2 v 2 + · · · + (a1 − b1 )uv n−1 + (a0 − b0 )v n = 0,

since an−1 = bn−1 . This shows that v divides (an − bn )un and hence it divides an − bn . Since
an − bn is a prime, either v = 1 or v = an − bn . Suppose the latter holds. The relation takes
the form

un + (an−2 − bn−2 )un−2 v + · · · + (a1 − b1 )uv n−2 + (a0 − b0 )v n−1 = 0.



(Here we have divided through-out by v.) If n > 1, this forces v u, which is impossible since
gcd(v, u) = 1 (v > 1 since it is equal to the prime an − bn ). If n = 1, then we get two equations:

a1 u + a0 v = 0,
b1 u + b0 v = 0.

This forces a1 b0 −a0 b1 = 0 contradicting an b0 −a0 bn 6= 0. (Note: The condition an b0 −a0 bn 6= 0


is extraneous. The condition an−1 = bn−1 forces that for n = 1, we have a0 = b0 . Thus we
obtain, after subtraction
(a1 − b1 )u = 0.
This implies that u = 0 and hence r = 0 is an integer.)
4. Suppose five of the nine vertices of a regular nine-sided polygon are arbitrarily chosen. Show
that one can select four among these five such that they are the vertices of a trapezium.

Solution 1: Suppose four distinct points P , Q, R, S(in that order on the circle) among these
d
five are such that P c Then P QRS is an isosceles trapezium, with P S k QR. We use
Q = RS.
this in our argument.
• If four of the five points chosen are adjacent, then we are through as observed earlier. (In
this case four points A, B, C, D are such that AB d = BC d = CD.)
d See Fig 1.

3
B B B
A A A
C C

D D

H H
E E
G G
F F
Fig 1. Fig 2. Fig 3.

• Suppose only three of the vertices are adjacent, say A, B, C(see Fig 2.) Then the remaining
two must be among E, F, G, H. If these two are adjacent vertices, we can pair them with
A, B or B, C to get equal arcs. If they are not adjacent, then they must be either E, G
or F, H or E, H. In the first two cases, we can pair them with A, C to get equal arcs. In
d = CE
the last case, we observe that HA d and AHEC is an isosceles trapezium.
• Suppose only two among the five are adjacent, say A, B. Then the remaining three are
among D, E, F, G, H. (See Fig 3.) If any two of these are adjacent, we can combine them
with A, B to get equal arcs. If no two among these three vertices are adjacent, then they
must be D, F, H. In this case HAd = BD d and AHDB is an isosceles trapezium.
Finally, if we choose 5 among the 9 vertices of a regular nine-sided polygon, then some
two must be adjacent. Thus any choice of 5 among 9 must fall in to one of the above
three possibilities.

Solution 2: Here is another solution used by many students. Suppose you join the vertices
of the nine-sided regular polygon. You get 92 = 36 line segments. All these fall in to 9 sets

of parallel lines. Now using any 5 points, you get 52 = 10 line segments. By pigeon-hole
principle, two of these must be parallel. But, these parallel lines determine a trapezium.
5. Let ABCD be a quadrilateral inscribed in a circle Γ. Let E, F , G, H be the midpoints of the
arcs AB, BC, CD, DA of the circle Γ. Suppose AC · BD = EG · F H. Prove that AC, BD,
EG, F H are concurrent.

Solution:
D
G
Let R be the radius of the circle Γ. Observe that
1 D
∠EDF = ∠D. Hence EF = 2R sin . Sim-
C 2 2
H B
ilarly, HG = 2R sin . But ∠B = 180◦ − ∠D.
2
D
Thus HG = 2R cos . We hence get
2
D D
F EF ·GH = 4R2 sin cos = 2R2 sin D = R·AC.
2 2
A
Similarly, we obtain EH · F G = R · BD.
E
B
Therefore
R(AC + BD) = EF · GH + EH · F G = EG · F H,
by Ptolemy’s theorem. By the given hypothesis, this gives R(AC + BD) = AC · BD. Thus

AC · BD = R(AC + BD) ≥ 2R AC · BD,
using AM-GM inequality. This implies that AC · BD ≥ 4R2 . But AC and BD are the
chords of Γ, so that AC ≤ 2R and BD ≤ 2R. We obtain AC · BD ≤ 4R2 . It follows that
AC · BD = 4R2 , implying that AC = BD = 2R. Thus AC and BD are two diameters of Γ.
Using EG · F H = AC · BD, we conclude that EG and F H are also two diameters of Γ. Hence
AC, BD, EG and F H all pass through the centre of Γ.

4
6. Find all functions f : R → R such that
2
f (x + y)f (x − y) = f (x) + f (y) − 4x2 f (y), (1)

for all x, y ∈ R, where R denotes the set of all real numbers.

Solution 1.: Put x = y = 0; we get f (0)2 = 4f (0)2 and hence f (0) = 0.


Put x = y: we get 4f (x)2 − 4x2 f (x) = 0 for all x. Hence for each x, either f (x) = 0 or
f (x) = x2 .
Suppose f (x) 6≡ 0. Then we can find x0 6= 0 such that f (x0 ) 6= 0. Then f (x0 ) = x20 6= 0.
Assume that there exists some y0 6= 0 such that f (y0 ) = 0. Then

f (x0 + y0 )f (x0 − y0 ) = f (x0 )2 .

Now f (x0 + y0 )f (x0 − y0 ) = 0 or f (x0 + y0 )f (x0 − y0 ) = (x0 + y0 )2 (x0 − y0 )2 . If f (x0 +


y0 )f (x0 − y0 ) = 0, then f (x0 ) = 0, a contradiction. Hence it must be the latter so that

(x20 − y02 )2 = x40 .



This reduces to y02 y02 − 2x20 ) = 0. Since y0 6= 0, we get y0 = ± 2x0 .
√ √
Suppose y0 = 2x0 . Put x = 2x0 and y = x0 in (1); we get
√  √  √ 2 
f ( 2 + 1)x0 f ( 2 − 1)x0 = f ( 2x0 ) + f (x0 ) − 4 2x20 f (x0 ).

But f ( 2x0 ) = f (y0 ) = 0. Thus we get
√  √ 
f ( 2 + 1)x0 f ( 2 − 1)x0 = f (x0 )2 − 8x20 f (x0 )
= x40 − 8x40 = −7x40 .

Now√if LHS is equal to 0, we get x0 = 0, a contradiction. Otherwise LHS is equal to ( 2 +
1)2 ( 2 − 1)2 x40 which reduces to x40 . We obtain x40 = −7x40 and this forces again x0 = 0. Hence
there is no y 6= 0 such that f (y) = 0. We conclude that f (x) = x2 for all x.
Thus there are two solutions: f (x) = 0 for all x or f (x) = x2 , for all x. It is easy to verify
that both these satisfy the functional equation.
Solution 2: As earlier, we get f (0) = 0. Putting x = 0, we will also get

f (y) f (y) − f (−y) = 0.

As earlier, we may conclude that either f (y) = 0 or f (y)= f (−y) for each y ∈ R. Replacing
y by −y, we may also conclude that f (−y) f (−y) − f (y) = 0. If f (y) = 0 and f (−y) 6= 0 for
some y, then we must have f (−y) = f (y) = 0, a contradiction. Hence either f (y) = f (−y) = 0
or f (y) = f (−y) for each y. This forces f is an even function.
Taking y = 1 in (1), we get

f (x + 1)f (x − 1) = (f (x) + f (1))2 − 4x2 f (1).

Replacing y by x and x by 1, you also get

f (1 + x)f (1 − x) = (f (1) + f (x))2 − 4f (x).

Comparing these two using the even nature of f , we get f (x) = cx2 , where c = f (1). Putting
x = y = 1 in (1), you get 4c2 − 4c = 0. Hence c = 0 or 1. We get f (x) = 0 for all x or
f (x) = x2 for all x.

5
Problems and Solutions: INMO-2012

Let ABCD be a quadrilateral inscribed in a circle. Suppose AB =


1. p

2 + 2 and AB subtends 135◦ at the centre of the circle. Find the
maximum possible area of ABCD.

Solution: Let O be the centre of the circle in which ABCD is inscribed


and let R be its radius. Using cosine rule in triangle AOB, we have
√ √
2 + 2 = 2R2 (1 − cos 135◦) = R2 (2 + 2).

Hence R = 1.
Consider quadrilateral ABCD as in the second figure above. Join AC.
For [ADC] to be maximum, it is clear that D should be the mid-point
of the arc AC so that its distance from the segment AC is maximum.
Hence AD = DC for [ABCD] to be maximum. Similarly, we conclude
that BC = CD. Thus BC = CD = DA which fixes the quadrilateral
ABCD. Therefore each of the sides BC, CD, DA subtends equal angles
at the centre O.
Let ∠BOC = α, ∠COD = β and ∠DOA = γ. Observe that
1 1
[ABCD] = [AOB]+[BOC]+[COD]+[DOA] = sin 135◦ + (sin α+sin β+sin γ).
2 2

Now [ABCD] has maximum area if and only if α = β = γ = (360◦ −


135◦)/3 = 75◦ . Thus
√ ! √
1 3 1 1 3+1 5+3 3
[ABCD] = sin 135 + sin 75 =
◦ ◦
√ +3 √ = √ .
2 2 2 2 2 2 4 2

Alternatively, we can use Jensen’s inequality. Observe that α, β, γ are


all less than 180◦ . Since sin x is concave on (0, π), Jensen’s inequality
gives  
sin α + sin β + sin γ α+β+γ
≤ sin = sin 75◦ .
3 3
Hence √
1 3 5 + 3 3
[ABCD] ≤ √ + sin 75◦ = √ ,
2 2 2 4 2
with equality if and only if α = β = γ = 75 .

2. Let p1 < p2 < p3 < p4 and q1 < q2 < q3 < q4 be two sets of prime numbers
such that p4 − p1 = 8 and q4 − q1 = 8. Suppose p1 > 5 and q1 > 5. Prove
that 30 divides p1 − q1 .

Solution: Since p4 − p1 = 8, and no prime is even, we observe that


{p1 , p2 , p3 , p4 } is a subset of {p1 , p1 + 2, p1 + 4, p1 + 6, p1 + 8}. Moreover p1 is
larger than 3. If p1 ≡ 1 (mod 3), then p1 + 2 and p1 + 8 are divisible by 3.
Hence we do not get 4 primes in the set {p1 , p1 + 2, p1 + 4, p1 + 6, p1 + 8}.
Thus p1 ≡ 2 (mod 3) and p1 + 4 is not a prime. We get p2 = p1 + 2, p3 =
p1 + 6, p4 = p1 + 8.
Consider the remainders of p1 , p1 + 2, p1 + 6, p1 + 8 when divided by 5. If
p1 ≡ 2 (mod 5), then p1 + 8 is divisible by 5 and hence is not a prime. If
p1 ≡ 3 (mod 5), then p1 + 2 is divisible by 5. If p1 ≡ 4 (mod 5), then p1 + 6
is divisible by 5. Hence the only possibility is p1 ≡ 1 (mod 5).
Thus we see that p1 ≡ 1 (mod 2), p1 ≡ 2 (mod 3) and p1 ≡ 1 (mod 5). We
conclude that p1 ≡ 11 (mod 30).
Similarly q1 ≡ 11 (mod 30). It follows that 30 divides p1 − q1 .

3. Define a sequence hf0 (x), f1 (x), f2 (x), . . .i of functions by


2
f0 (x) = 1, f1 (x) = x, fn (x) − 1 = fn+1 (x)fn−1 (x), for n ≥ 1.

Prove that each fn (x) is a polynomial with integer coefficients.

Solution: Observe that

fn2 (x) − fn−1 (x)fn+1 (x) = 1 = fn−1


2
(x) − fn−2 (x)fn (x).

This gives
   
fn (x) fn (x) + fn−2 (x) = fn−1 fn−1 (x) + fn+1 (x) .

We write this as
fn−1 (x) + fn+1 (x) fn−2 (x) + fn (x)
= .
fn (x) fn−1 (x)
Using induction, we get
fn−1 (x) + fn+1 (x) f0 (x) + f2 (x)
= .
fn (x) f1 (x)
Observe that
f12 (x) − 1
f2 (x) = = x2 − 1.
f0 (x)
Hence
fn−1 (x) + fn+1 (x) 1 + (x2 − 1)
= = x.
fn (x) x
Thus we obtain
fn+1 (x) = xfn (x) − fn−1 (x).
Since f0 (x), f1 (x) and f2 (x) are polynomials with integer coefficients,
induction again shows that fn (x) is a polynomial with integer coeffi-
cients.
Note: We can get fn (x) explicitly:
     
n n − 1 n−2 n − 2 n−4 n − 3 n−6
fn (x) = x − x + x − x +···
1 2 3

4. Let ABC be a triangle. An interior point P of ABC is said to be good if


we can find exactly 27 rays emanating from P intersecting the sides of
the triangle ABC such that the triangle is divided by these rays into 27
smaller triangles of equal area. Determine the number of good points
for a given triangle ABC.

Solution: Let P be a good point. Let l, m, n be respetively the number


of parts the sides BC, CA, AB are divided by the rays starting from
P . Note that a ray must pass through each of the vertices the triangle
ABC; otherwise we get some quadrilaterals.
Let h1 be the distance of P from BC. Then h1 is the height for all the
triangles with their bases on BC. Equality of areas implies that all
these bases have equal length. If we denote this by x, we get lx = a.
Similarly, taking y and z as the lengths of the bases of triangles on CA
and AB respectively, we get my = b and nz = c. Let h2 and h3 be the
distances of P from CA and AB respectively. Then
2∆
h1 x = h2 y = h3 z = ,
27
where ∆ denotes the area of the triangle ABC. These lead to
2∆ l 2∆ m 2∆ n
h1 = , h1 = , h1 = .
27 a 27 b 27 c
But
2∆ 2∆ 2∆
= ha , = hb , = hc .
a b c
Thus we get
h1 l h2 m h3 n
= , = , = .
ha 27 hb 27 hc 27
However, we also have
h1 [P BC] h2 [P CA] h3 [P AB]
= , = , = .
ha ∆ hb ∆ hc ∆
Adding these three relations,
h1 h2 h3
+ + = 1.
ha hb hc
Thus
l m n h1 h2 h3
+ + = + + = 1.
27 27 27 ha hb hc
We conclude that l + m + n = 27. Thus every good point P determines
a partition (l, m, n) of 27 such that there are l, m, n equal segments
respectively on BC, CA, AB.
Conversely, take any partition (l, m, n) of 27. Divide BC, CA, AB re-
spectively in to l, m, n equal parts. Define

2l∆ 2m∆
h1 = , h2 = .
27a 27b
Draw a line parallel to BC at a distance h1 from BC; draw another line
parallel to CA at a distance h2 from CA. Both lines are drawn such
that they intersect at a point P inside the triangle ABC. Then

1 l∆ m∆
[P BC] = ah1 = , [P CA] = .
2 27 27
Hence
n∆
[P AB] = .
27
This shows that the distance of P from AB is
2n∆
h3 = .
27c

Therefore each traingle with base on CA has area . We conclude that
27
all the triangles which partitions ABC have equal areas. Hence P is a
good point.
Thus the number of good points is equal to the number of positive
integral solutions of the equation l + m + n = 27. This is equal to
 
26
= 325.
2

5. Let ABC be an acute-angled triangle, and let D, E, F be points on BC,


CA, AB respectively such that AD is the median, BE is the internal
angle bisector and CF is the altitude. Suppose ∠F DE = ∠C, ∠DEF =
∠A and ∠EF D = ∠B. Prove that ABC is equilateral.
Solution: Since ∆BF C is
right-angled at F , we have
F D = BD = CD = a/2. Hence
∠BF D = ∠B. Since ∠EF D =
∠B, we have ∠AF E = π − 2∠B.
Since ∠DEF = ∠A, we also get
∠CED = π−2∠B. Applying sine
rule in ∆DEF , we have
DF FE DE
= = .
sin A sin C sin B
Thus we get F E = c/2 and DE = b/2. Sine rule in ∆CED gives
DE CD
= .
sin C sin(π − 2B)
Thus (b/ sin C) = (a/2 sin B cos B). Solving for cos B, we have
a sin c ac
cos B = = 2.
2b sin B 2b
Similarly, sine rule in ∆AEF gives
EF AE
= .
sin A sin(π − 2B)
This gives (since AE = bc/(a + c)), as earlier,
a
cos B = .
a+c
Comparing the two values of cos B, we get 2b2 = c(a + c). We also have

2a2 c
c2 + a2 − b2 = 2ca cos B = .
a+c
Thus

4a2 c = (a + c)(2c2 + 2a2 − 2b2 ) = (a + c)(2c2 + 2a2 − c(a + c)).

This reduces to 2a3 − 3a2 c + c3 = 0. Thus (a − c)2 (2a + c) = 0. We conclude


that a = c. Finally
2b2 = c(a + c) = 2c2 .
We thus get b = c and hence a = c = b. This shows that ∆ABC is
equilateral.

6. Let f : Z → Z be a function satisfying f (0) 6= 0, f (1) = 0 and

(i) f (xy) + f (x)f (y) = f (x) + f (y);



(ii) f (x − y) − f (0) f (x)f (y) = 0,

for all x, y ∈ Z, simultaneously.

(a) Find the set of all possible values of the function f .


(b) If f (10) 6= 0 and f (2) = 0, find the set of all integers n such that
f (n) 6= 0.

Solution: Setting y = 0 in the condition (ii), we get



f (x) − f (0) f (x) = 0,

for all x (since f (0) 6= 0). Thus either f (x) = 0 or f (x) = f (0), for all x ∈ Z.
Now taking x = y = 0 in (i), we see that f (0) + f (0)2 = 2f (0). This shows
that f (0) = 0 or f (0) = 1. Since f (0) 6= 0, we must have f (0) = 1. We
conclude that
either f (x) = 0 or f (x) = 1 for each x ∈ Z.
This shows that the set of all possible value of f (x) is {0, 1}. This
completes (a).
 
Let S = n ∈ Z f (n) 6= 0 . Hence we must have S = n ∈ Z f (n) = 1 by
(a). Since f (1) = 0, 1 is not in S. And f (0) = 1 implies that 0 ∈ S. Take
any x ∈ Z and y ∈ S. Using (ii), we get

f (xy) + f (x) = f (x) + 1.

This shows that xy ∈ S. If x ∈ Z and y ∈ Z are such that xy ∈ S, then


(ii) gives
1 + f (x)f (y) = f (x) + f (y).
 
Thus f (x) − 1 f (y) − 1 = 0. It follows that f (x) = 1 or f (y) = 1; i.e.,
either x ∈ S or y ∈ S. We also observe from (ii) that x ∈ S and y ∈ S
implies that f (x − y) = 1 so that x − y ∈ S. Thus S has the properties:

(A) x ∈ Z and y ∈ S implies xy ∈ S;


(B) x, y ∈ Z and xy ∈ S implies x ∈ S or y ∈ S;
(C) x, y ∈ S implies x − y ∈ S.

Now we know that f (10) 6= 0 and f (2) = 0. Hence f (10) = 1 and 10 ∈ S;


and 2 6∈ S. Writing 10 = 2 × 5 and using (B), we conclude that 5 ∈ S and
f (5) = 1. Hence f (5k) = 1 for all k ∈ Z by (A).
Suppose f (5k + l) = 1 for some l, 1 ≤ l ≤ 4. Then 5k + l ∈ S. Choose
u ∈ Z such that lu ≡ 1 (mod 5). We have (5k + l)u ∈ S by (A). Moreover,
lu = 1 + 5m for some m ∈ Z and

(5k + l)u = 5ku + lu = 5ku + 5m + 1 = 5(ku + m) + 1.

This shows that 5(ku + m) + 1 ∈ S. However, we know that 5(ku + m) ∈ S.


By (C), 1 ∈ S which is a contradiction. We conclude that 5k + l 6∈ S for
any l, 1 ≤ l ≤ 4. Thus 
S = 5k k ∈ Z .

———-00000———-
Problems and solutions: INMO 2013

Problem 1. Let Γ1 and Γ2 be two circles touching each other externally at R. Let l1 be a line
which is tangent to Γ2 at P and passing through the center O1 of Γ1 . Similarly, let l2 be a line
which is tangent to Γ2 at Q and passing through the center O2 of Γ2 . Suppose l1 and l2 are not
parallel and interesct at K. If KP = KQ, prove that the triangle P QR is equilateral.

Solution. Suppose that P and Q lie on the opposite sides of line joining O1 and O2 . By sym-
metry we may assume that the configuration is as shown in the figure below. Then we have
KP > KO1 > KQ since KO1 is the hypotenuse of triangle KQO1 . This is a contradiction to
the given assumption, and therefore P and Q lie on the same side of the line joining O1 and O2 .

b b R b

O1 O2

Q
b

Since KP = KQ it follows that K lies on the radical axis of the given circles, which is the
common tangent at R. Therefore KP = KQ = KR and hence K is the cirumcenter of △P QR.

1
Q P
b
b

K
b

b b b

O1 R O2

On the other hand, △KQO1 and △KRO1 are both right-angled triangles with KQ = KR and
\1 = RKO
QO1 = RO1 , and hence the two triangles are congruent. Therefore QKO \1 , so KO1 , and
hence P K is perpendicular to QR. Similarly, QK is perpendicular to P R, so it follows that K is
the orthocenter of △P QR. Hence we have that △P QR is equilateral.

Alternate solution. We again rule out the possibility that P and Q are on the opposite side of
the line joining O1 O2 , and assume that they are on the same side.
Observe that △KP O2 is congruent to △KQO1 (since KP = KQ). Therefore O1 P = O2 Q = r
(say). In △O1 O2 Q, we have O\ 1 QO2 = π/2 and R is the midpoint of the hypotenuse, so RQ =
RO1 = r. Therefore △O1 RQ is equilateral, so QRO\1 = π/3. Similarly, P R = r and P\RO2 = π/3,
hence \P RQ = π/3. Since P R = QR it follows that △P QR is equilateral.

Problem 2. Find all positive integers m, n, and primes p ≥ 5 such that

m(4m2 + m + 12) = 3(pn − 1) .

Solution. Rewriting the given equation we have

4m3 + m2 + 12m + 3 = 3pn .

The left hand side equals (4m + 1)(m2 + 3).


Suppose that (4m + 1, m2 + 3) = 1. Then (4m + 1, m2 + 3) = (3pn , 1), (3, pn ), (pn , 3) or (1, 3pn ),
a contradiction since 4m + 1, m2 + 3 ≥ 4. Therefore (4m + 1, m2 + 3) > 1.
Since 4m + 1 is odd we have (4m + 1, m2 + 3) = (4m + 1, 16m2 + 48) = (4m + 1, 49) = 7 or 49.
This proves that p = 7, and 4m+1 = 3·7k or 7k for some natural number k. If (4m+1, 49) = 7 then
we have k = 1 and 4m + 1 = 21 which does not lead to a solution. Therefore (4m + 1, m2 + 3) = 49.
If 73 divides 4m + 1 then it does not divide m2 + 3, so we get m2 + 3 ≤ 3 · 72 < 73 ≤ 4m + 1. This
implies (m − 2)2 < 2, so m ≤ 3, which does not lead to a solution. Therefore we have 4m + 1 = 49
which implies m = 12 and n = 4. Thus (m, n, p) = (12, 4, 7) is the only solution.

2
Problem 3. Let a, b, c, d be positive integers such that a ≥ b ≥ c ≥ d. Prove that the equation
x4 − ax3 − bx2 − cx − d = 0 has no integer solution.

Solution. Suppose that m is an integer root of x4 − ax3 − bx2 − cx − d = 0. As d 6= 0, we have


m 6= 0. Suppose now that m > 0. Then m4 − am3 = bm2 + cm + d > 0 and hence m > a ≥ d. On
the other hand d = m(m3 − am2 − bm − c) and hence m divides d, so m ≤ d, a contradiction. If
m < 0, then writing n = −m > 0 we have n4 + an3 − bn2 + cn − d = n4 + n2 (an − b) + (cn − d) > 0,
a contradiction. This proves that the given polynomial has no integer roots.

Problem 4. Let n be a positive integer. Call a nonempty subset S of {1, 2, . . . , n} good if the
arithmetic mean of the elements of S is also an integer. Further let tn denote the number of good
subsets of {1, 2, . . . , n}. Prove that tn and n are both odd or both even.

Solution. We show that Tn − n is even. Note that the subsets {1}, {2}, · · · , {n} are good. Among
the other good subsets, let A be the collection of subsets with an integer average which belongs to
the subset, and let B be the collection of subsets with an integer average which is not a member
of the subset. Then there is a bijection between A and B, because removing the average takes a
member of A to a member of B; and including the average in a member of B takes it to its inverse.
So Tn − n = |A| + |B| is even.

Alternate solution. Let S = {1, 2, . . . , n}. For a subset A of S, let A = {n + 1 − a|a ∈ A}.
We call a subset A symmetric if A = A. Note that the arithmetic mean of a symmetric subset is
(n + 1)/2. Therefore, if n is even, then there are no symmetric good subsets, while if n is odd then
every symmetric subset is good.
If A is a proper good subset of S, then so is A. Therefore, all the good subsets that are not
symmetric can be paired. If n is even then this proves that tn is even. If n is odd, we have to
show that there are odd number of symmetric subsets. For this, we note that a symmetric subset
contains the element (n + 1)/2 if and only if it has odd number of elements. Therefore, for any
natural number k, the number of symmetric subsets of size 2k equals the number of symmetric
subsets of size 2k + 1. The result now follows since there is exactly one symmetric subset with only
one element.

Problem 5. In an acute triangle ABC, O is the circumcenter, H is the orthocenter and G is the
centroid. Let OD be perpendicular to BC and HE be perpendicular to CA, with D on BC and E
on CA. Let F be the midpoint of AB. Suppose the areas of triangles ODC, HEA and GF B are
b
equal. Find all the possible values of C.

Solution. Let R be the circumradius of △ABC and ∆ its area. We have OD = R cos A and
DC = a2 , so
1 1 1
[ODC] = · OD · DC = · R cos A · R sin A = R2 sin A cos A . (1)
2 2 2
Again HE = 2R cos C cos A and EA = c cos A. Hence
1 1
[HEA] = · HE · EA = · 2R cos C cos A · c cos A = 2R2 sin C cos C cos2 A . (2)
2 2
Further
∆ 1 1
[GF B] = = · 2R2 sin A sin B sin C = R2 sin A sin B sin C . (3)
6 6 3

3
Equating (1) and (2) we get tan A = 4 sin C cos C. And equating (1) and (3), and using this relation
we get

3 cos A = 2 sin B sin C = 2 sin(C + A) sin C


= 2(sin C + cos C tan A) sin C cos A
= 2 sin2 C(1 + 4 cos2 C) cos A .
2
Since cos A 6= 0 we get 3 = 2t(−4t + 5) where√ t = sin C. This√implies (4t − 3)(2t − 1) = 0 and
therefore, since sin C > 0, we get sin C = 3/2 or sin C = 1/ 2. Because △ABC is acute, it
follows that Cb = π/3 or π/4.
b = π/3 is
We observe that the given conditions are satisfied in an equilateral triangle, so C
b b
a possibility. Also, the conditions are satisfied in a triangle where C = π/4, A = tan−1 2 and
Bb = tan−1 3. Therefore Cb = π/4 is also a possibility.
b are π/3 and π/4.
Thus the two possible values of C

Problem 6. Let a, b, c, x, y, z be positive real numbers such that a+ b + c = x + y + z and abc = xyz.
Further, suppose that a ≤ x < y < z ≤ c and a < b < c. Prove that a = x, b = y and c = z.

Solution. Let
f (t) = (t − x)(t − y)(t − z) − (t − a)(t − b)(t − c) .
Then f (t) = kt for some constant k. Note that ka = f (a) = (a − x)(a − y)(a − z) ≤ 0 and hence
k ≤ 0. Similarly, kc = f (c) = (c − x)(c − y)(c − z) ≥ 0 and hence k ≥ 0. Combining the two, it
follows that k = 0 and that f (a) = f (c) = 0. These equalities imply that a = x and c = z, and
then it also follows that b = y.

4
29th Indian National Mathematical Olympiad-2014

February 02, 2014

1. In a triangle ABC, let D be a point on the segment BC such that AB + BD = AC + CD. Suppose that the
points B, C and the centroids of triangles ABD and ACD lie on a circle. Prove that AB = AC.
Solution. Let G1 , G2 denote the centroids of triangles ABD and ACD. Then G1 , G2 lie on the line parallel to
BC that passes through the centriod of triangle ABC. Therefore BG1 G2 C is an isosceles trapezoid. Therefore it
follows that BG1 = CG2 . This proves that AB 2 +BD2 = AC 2 +CD2 . Hence it follows that AB ·BD = AC ·CD.
Therefore the sets {AB, BD} and {AC, CD} are the same (since they are both equal to the set of roots of the
same polynomial). Note that if AB = CD then AC = BD and then AB + AC = BC, a contradiction. Therefore
it follows that AB = AC.
2. Let n be a natural number. Prove that
hni hni hni hni √ 
+ + + ··· + n
1 2 3 n
is even. (Here [x] denotes the largest integer smaller than or equal to x.)
Solution. Let f (n) denote the given equation. Then f (1) = 2 which is even. Now suppose that f (n) is even
for some n ≥ 1. Then
       
n+1 n+1 n+1 n+1 √ 
f (n + 1) = + + + ··· + n+1
1 2 3 n+1
hni hni hni h n i √ 
= + + + ··· + n + 1 + σ(n + 1) ,
1 2 3 n
where σ(n + 1) denotes the number of positive divisors of n + 1. This follows from n+1 = nk + 1 if k
   
 n+1   n  √  √ k
divides n + 1, and k = k otherwise. Note that n + 1 = [ n] unless n + 1 is a square, in which case
√  √
n + 1 = [ n] + 1. On the other hand σ(n + 1) is odd if and only if n + 1 is a square. Therefore it follows
that f (n + 1) = f (n) + 2l for some integer l. This proves that f (n + 1) is even.
Thus it follows by induction that f (n) is even for all natural number n.

3. Let a, b be natural numbers with ab > 2. Suppose that the sum of their greatest common divisor and least
common multiple is divisible by a + b. Prove that the quotient is at most (a + b)/4. When is this quotient exactly
equal to (a + b)/4?
Solution. Let g and l denote the greatest common divisor and the least common multiple, respectively, of a
and b. Then gl = ab. Therefore g + l ≤ ab + 1. Suppose that (g + l)/(a + b) > (a + b)/4. Then we have
ab + 1 > (a + b)2 /4, so we get (a − b)2 < 4. Assuming, a ≥ b we either have a = b or a = b + 1. In the former case,
g = l = a and the quotient is (g + l)/(a + b) = 1 ≤ (a + b)/4. In the latter case, g = 1 and l = b(b + 1) so we get
that 2b + 1 divides b2 + b + 1. Therefore 2b + 1 divides 4(b2 + b + 1) − (2b + 1)2 = 3 which implies that b = 1 and
a = 2, a contradiction to the given assumption that ab > 2. This shows that (g + l)/(a + b) ≤ (a + b)/4. Note
that for the equality to hold, we need that either a = b = 2 or, (a − b)2 = 4 and g = 1, l = ab. The latter case
happens if and only if a and b are two consecutive odd numbers. (If a = 2k + 1 and b = 2k − 1 then a + b = 4k
divides ab + 1 = 4k 2 and the quotient is precisely (a + b)/4.)
4. Written on a blackboard is the polynomial x2 + x + 2014. Calvin and Hobbes take turns alternatively (starting
with Calvin) in the following game. During his turn, Calvin should either increase or decrease the coefficient of
x by 1. And during his turn, Hobbes should either increase or decrease the constant coefficient by 1. Calvin wins
if at any point of time the polynomial on the blackboard at that instant has integer roots. Prove that Calvin
has a winning strategy.
Solution. For i ≥ 0, let fi (x) denote the polynomial on the blackboard after Hobbes’ i-th turn. We let Calvin
decrease the coefficient of x by 1. Therefore fi+1 (2) = fi (2) − 1 or fi+1 (2) = fi (2) − 3 (depending on whether
Hobbes increases or decreases the constant term). So for some i, we have 0 ≤ fi (2) ≤ 2. If fi (2) = 0 then Calvin
has won the game. If fi (2) = 2 then Calvin wins the game by reducing the coefficient of x by 1. If fi (2) = 1
then fi+1 (2) = 0 or fi+1 (2) = −2. In the former case, Calvin has won the game and in the latter case Calvin
wins the game by increasing the coefficient of x by 1.
5. In an acute-angled triangle ABC, a point D lies on the segment BC. Let O1 , O2 denote the circumcentres of
triangles ABD and ACD, respectively. Prove that the line joining the circumcentre of triangle ABC and the
orthocentre of triangle O1 O2 D is parallel to BC.
Solution. Without loss of generality assume that ∠ADC ≥ 90◦ . Let O denote the circumcenter of triangle
ABC and K the orthocentre of triangle O1 O2 D. We shall first show that the points O and K lie on the
circumcircle of triangle AO1 O2 . Note that circumcircles of triangles ABD and ACD pass through the points A
and D, so AD is perpendicular to O1 O2 and, triangle AO1 O2 is congruent to triangle DO1 O2 . In particular,
∠AO1 O2 = ∠O2 O1 D = ∠B since O2 O1 is the perpendicular bisector of AD. On the other hand since OO2 is
the perpendicular bisector of AC it follows that ∠AOO2 = ∠B. This shows that O lies on the circumcircle of
triangle AO1 O2 . Note also that, since AD is perpendicular to O1 O2 , we have ∠O2 KA = 90◦ − ∠O1 O2 K =
∠O2 O1 D = ∠B. This proves that K also lies on the circumcircle of triangle AO1 O2 .
Therefore ∠AKO = 180◦ − ∠AO2 O = ∠ADC and hence OK is parallel to BC.
Remark. The result is true even for an obtuse-angled triangle.
6. Let n be a natural number and X = {1, 2, . . . , n}. For subsets A and B of X we define A∆B to be the set of all
those elements of X which belong to exactly one of A and B. Let F be a collection of subsets of X such that
for any two distinct elements A and B in F the set A∆B has at least two elements. Show that F has at most
2n−1 elements. Find all such collections F with 2n−1 elements.
Solution. For each subset A of {1, 2, . . . , n − 1}, we pair it with A ∪ {n}. Note that for any such pair (A, B) not
both A and B can be in F. Since there are 2n−1 such pairs it follows that F can have at most 2n−1 elements.
We shall show by induction on n that if |F| = 2n−1 then F contains either all the subsets with odd number of
elements or all the subsets with even number of elements. The result is easy to see for n = 1. Suppose that
the result is true for n = m − 1. We now consider the case n = m. Let F1 be the set of those elements in F
which contain m and F2 be the set of those elements which do not contain m. By induction, F2 can have at
most 2m−2 elements. Further, for each element A of F1 we consider A \ {m}. This new collection also satisfies
the required property, so it follows that F1 has at most 2m−2 elements. Thus, if |F| = 2m−1 then it follows
that |F1 | = |F2 | = 2m−2 . Further, by induction hypothesis, F2 contains all those subsets of {1, 2, . . . , m − 1}
with (say) even number of elements. It then follows that F1 contains all those subsets of {1, 2, . . . , m} which
contain the element m and which contains an even number of elements. This proves that F contains either all
the subsets with odd number of elements or all the subsets by even number of elements.

——— ? ? ? ? ? ———
Problems and Solutions: INMO-2015

1. Let ABC be a right-angled triangle with ∠B = 90◦ . Let BD be the


altitude from B on to AC. Let P , Q and I be the incentres of triangles
ABD, CBD and ABC respectively. Show that the circumcentre of of
the triangle P IQ lies on the hypotenuse AC.

Solution: We begin with the following lemma:


Lemma: Let XY Z be a triangle with ∠XY Z = 90 + α. Construct an
isosceles triangle XEZ, externally on the side XZ, with base angle
α. Then E is the circumcentre of 4XY Z.
Proof of the Lemma: Draw ED ⊥
XZ. Then DE is the perpendicu-
lar bisector of XZ. We also observe
that ∠XED = ∠ZED = 90 − α. Ob-
serve that E is on the perpendicu-
lar bisector of XZ. Construct the
circumcircle of XY Z. Draw per-
pendicular bisector of XY and let
it meet DE in F . Then F is the
circumcentre of 4XY Z. Join XF .
Then ∠XF D = 90 − α. But we know
that ∠XED = 90 − α. Hence E = F .
Let r1 , r2 and r be the inradii of the triangles ABD, CBD and ABC
respectively. Join P D and DQ. Observe that ∠P DQ = 90◦ . Hence

P Q2 = P D2 + DQ2 = 2r12 + 2r22 .

Let s1 = (AB + q BD + DA)/2. Observe that BD = ca/b and AD =


√ 2
AB 2 − BD2 = c2 − cab = c2 /b. This gives s1 = cs/b. But r1 =
s1 − c = (c/b)(s − b) = cr/b. Similarly, r2 = ar/b. Hence
 2
c + a2

2 2
P Q = 2r = 2r2 .
b2

Consider 4P IQ. Observe that


∠P IQ = 90+(B/2) = 135. Hence P Q
subtends 90◦ on the circumference
of the circumcircle of 4P IQ. But
we have seen that ∠P DQ = 90◦ .
Now construct a circle with P Q as
diameter. Let it cut AC again in K.
It follows that ∠P KQ = 90◦ and the
points P, D, K, Q are concyclic. We
also notice ∠KP Q = ∠KDQ = 45◦
and ∠P QK = ∠P DA = 45◦ .
Thus P KQ is an isosceles right-angled triangle with KP = KQ. Ther-
fore KP 2 + KQ2 = P Q2 = 2r2 and hence KP = KQ = r.

Now ∠P IQ = 90 + 45 and ∠P KQ = 2 × 45◦ = 90◦ with KP = KQ = r.


Hence K is the circumcentre of 4P IQ.

(Incidentally, This also shows that KI = r and hence K is the point


of contact of the incircle of 4ABC with AC.)
Solution 2: Here we use compu-
tation to prove that the point of
contact K of the incircle with AC
is the circumcentre of 4P IQ. We
show that KP = KQ = r. Let r1
and r2 be the inradii of triangles
ABD and CBD respectively. Draw
P L ⊥ AC and QM ⊥ AC. If s1 is
the semiperimeter of 4ABD, then
AL = s1 − BD.
But
AB + BD + DA ca c2
s1 = , BD = , AD =
2 b b
Hence s1 = cs/b. This gives r1 = s1 − c = cr/b, AL = s1 − BD = c(s − a)/b.
Hence KL = AK − AL = (s − a) − c(s−a)b
= (b−c)(s−a)
b
. We observe that

(c + a − b)2 c2 + a2 + b2 − 2bc − 2ab + 2ca


2r2 = = = (b2 −ba−bc+ac) = (b−c)(b−a).
2 2
This gives

(s − a)(b − c) = (s − b + b − a)(b − c) = r(b − c) + (b − a)(b − c)


= r(b − c) + 2r2 = r(b − c + c + a − b) = ra.

Thus KL = ra/b. Finally,

r 2 a2 r 2 + c 2
KP 2 = KL2 + LP 2 = + = r2 .
b2 b2
Thus KP = r. Similarly, KQ = r. This gives KP = KI = KQ = r and
therefore K is the circumcentre of 4KIQ.

(Incidentally, this also shows that KL = ca/b = r2 and KM = r1 .)

2. For any natural number n > 1, write the infinite decimal expansion
of 1/n (for example, we write 1/2 = 0.49̄ as its infinite decimal expan-
sion, not 0.5). Determine the length of the non-periodic part of the
(infinite) decimal expansion of 1/n.

Solution: For any prime p, let νp (n) be the maximum power of p


dividing n; ie pνp (n) divides n but not higher power. Let r be the
length of the non-periodic part of the infinite decimal expansion of
1/n.
Write
1
= 0.a1 a2 · · · ar b1 b2 · · · bs .
n
We show that r = max(ν2 (n), ν5 (n)).
Let a and b be the numbers a1 a2 · · · ar and b = b1 b2 · · · bs respectively.
(Here a1 and b1 can be both 0.) Then
!  
1 1 X b 1 b
= r a+ = r a+ s .
n 10 k≥1
(10s )k 10 10 − 1

Thus we get 10r (10s − 1) = n (10s − 1)a + b . It shows that r ≥
max(ν2 (n), ν5 (n)). Suppose r > max(ν2 (n), ν5 (n)). Then 10 divides b − a.
Hence the last digits of a and b are equal: ar = bs . This means
1
= 0.a1 a2 · · · ar−1 bs b1 b2 · · · bs−1 .
n
This contradicts the definition of r. Therefore r = max(ν2 (n), ν5 (n)).

3. Find all real functions f from R → R satisfying the relation

f (x2 + yf (x)) = xf (x + y).


Solution: Put x = 0 and we get f yf (0) = 0. If f (0) 6= 0, then yf (0)
takes all real values when y varies over real line. We
2
 get f (x) ≡ 0.
Suppose f (0) = 0. Taking y = −x, we get f x − xf (x) = 0 for all real
x.
Suppose there exists x0 6= 0 in R such that f (x0 ) = 0. Putting x = x0
in the given relation we get

f x20 = x0 f (x0 + y),




for all y ∈ R. Now the left side is a constant and hence it follows
that f is a constant function. But the only constant function which
satisfies the equation is identically zero function, which is already
obtained. Hence we may consider the case where f (x) 6= 0 for all
x 6= 0.

Since f x2 − xf (x) = 0, we conclude that x2 − xf (x) = 0 for all x 6= 0.
This implies that f (x) = x for all x 6= 0. Since f (0) = 0, we conclude
that f (x) = x for all x ∈ R.
Thus we have two functions: f (x) ≡ 0 and f (x) = x for all x ∈ R.

4. There are four basket-ball players A, B, C, D. Initially, the ball is


with A. The ball is always passed from one person to a different
person. In how many ways can the ball come back to A after seven
passes? (For example A → C → B → D → A → B → C → A and
A → D → A → D → C → A → B → A are two ways in which the ball
can come back to A after seven passes.)

Solution: Let xn be the number of ways in which A can get back the
ball after n passes. Let yn be the number of ways in which the ball
goes back to a fixed person other than A after n passes. Then

xn = 3yn−1 ,

and
yn = xn−1 + 2yn−1 .
We also have x1 = 0, x2 = 3, y1 = 1 and y2 = 2.
Eliminating yn and yn−1 , we get xn+1 = 3xn−1 + 2xn . Thus

x3 = 3x1 + 2x2 = 2 × 3 = 6;
x4 = 3x2 + 2x3 = (3 × 3) + (2 × 6) = 9 + 12 = 21;
x5 = 3x3 + 2x4 = (3 × 6) + (2 × 21) = 18 + 42 = 60;
x6 = 3x4 + 2x5 = (3 × 21) + (2 × 60) = 63 + 120 = 183;
x7 = 3x5 + 2x6 = (3 × 60) + (2 × 183) = 180 + 366 = 546.

Alternate solution: Since the ball goes back to one of the other 3
persons, we have
xn + 3yn = 3n ,
since there are 3n ways of passing the ball in n passes. Using xn =
3yn−1 , we obtain
xn−1 + xn = 3n−1 ,
with x1 = 0. Thus

x7 = 36 − x6 = 36 − 35 + x5 = 36 − 35 + 34 − x4 = 36 − 35 + 34 − 33 + x3
= 36 − 35 + 34 − 33 + 32 − x2 = 36 − 35 + 34 − 33 + 32 − 3
= (2 × 35 ) + (2 × 33 ) + (2 × 3) = 486 + 54 + 6 = 546.

5. Let ABCD be a convex quadrilateral. Let the diagonals AC and BD


intersect in P . Let P E, P F , P G and P H be the altitudes from P on
to the sides AB, BC, CD and DA respectively. Show that ABCD has
an incircle if and only if
1 1 1 1
+ = + .
PE PG PF PH

Solution: Let AP = p, BP = q, CP = r, DP = s; AB = a, BC = b,
CD = c and DA = d. Let ∠AP B = ∠CP D = θ. Then ∠BP C = ∠DP A =
π − θ. Let us also write P E = h1 , P F = h2 , P G = h3 and P H = h4 .
Observe that
h1 a = pq sin θ, h2 b = qr sin θ, h3 c = rs sin θ, h4 d = sp sin θ.
Hence
1 1 1 1
+ = + .
h1 h3 h2 h4
is equivalent to
a c b d
+ = + .
pq rs qr sp
This is the same as
ars + cpq = bsp + dqr.
Thus we have to prove that a+c = b+d if and only if ars+cpq = bsp+dqr.
Now we can write a + c = b + d as
a2 + c2 + 2ac = b2 + d2 + 2bd.
But we know that

a2 = p2 + q 2 − 2pq cos θ, c2 = r2 + s2 − 2rs cos θ


b2 = q 2 + r2 + 2qr cos θ, d2 = p2 + s2 + 2ps cos θ,
Hence a + c = b + d is equivalent to
−pq cos θ + −rs cos θ + ac = ps cos θ + qr cos θ + bd.
Similarly, by squaring ars + cpq = bsp + dqr we can show that it is
equivalent to
−pq cos θ + −rs cos θ + ac = ps cos θ + qr cos θ + bd.
We conclude that a + c = b + d is equivalent to cpq + ars = bps + dqr.
Hence ABCD has an in circle if and only if
1 1 1 1
+ = + .
h1 h3 h2 h4
6. From a set of 11 square integers, show that one can choose 6 num-
bers a2 , b2 , c2 , d2 , e2 , f 2 such that

a2 + b2 + c2 ≡ d2 + e2 + f 2 (mod 12).

Solution: The first observation is that we can find 5 pairs of squares


such that the two numbers in a pair have the same parity. We can
see this as follows:
Odd numbers Even numbers Odd pairs Even pairs Total pairs
0 11 0 5 5
1 10 0 5 5
2 9 1 4 5
3 8 1 4 5
4 7 2 3 5
5 6 2 3 5
6 5 3 2 5
7 4 3 2 5
8 3 4 1 5
9 2 4 1 5
10 1 5 0 5
11 0 5 0 5

Let us take such 5 pairs: say (x21 , y12 ), (x22 , y22 ), . . . , (x25 , y52 ). Then x2j − yj2
is divisible by 4 for 1 ≤ j ≤ 5. Let rj be the remainder when x2j − yj2
is divisible by 3, 1 ≤ j ≤ 3. We have 5 remainders r1 , r2 , r3 , r4 , r5 .
But these can be 0, 1 or 2. Hence either one of the remainders
occur 3 times or each of the remainders occur once. If, for example
r1 = r2 = r3 , then 3 divides r1 + r2 + r3 ; if r1 = 0, r2 = 1 and r3 = 2, then
again 3 divides r1 +r2 +r3 . Thus we can always find three remainders
whose sum is divisible by 3. This means we can find 3 pairs, say,
(x21 , y12 ), (x22 , y22 ), (x23 , y32 ) such that 3 divides (x21 − y12 ) + (x22 − y22 ) + (x23 − y32 ).
Since each difference is divisible by 4, we conclude that we can find
6 numbers a2 , b2 , c2 , d2 , e2 , f 2 such that

a2 + b 2 + c 2 ≡ d 2 + e 2 + f 2 (mod 12).
31st Indian National Mathematical Olympiad-2016
Time: 4 hours January 17, 2016
Instructions:
• Calculators (in any form) and protractors are not allowed.
• Rulers and compasses are allowed.
• Answer all the questions. Maximum marks: 100.
• Answer to each question should start on a new page. Clearly indicate
the question number.

1. Let ABC be triangle in which AB = AC. Suppose the orthocentre of the


triangle lies on the incircle. Find the ratio AB/BC.
2. For positive real numbers a, b, c, which of the following statements
necessarily implies a = b = c: (I) a(b3 + c3 ) = b(c3 + a3 ) = c(a3 + b3 ),
(II) a(a3 + b3 ) = b(b3 + c3 ) = c(c3 + a3 ) ? Justify your answer.
3. Let N denote the set of all natural numbers. Define a function T : N → N by
T (2k) = k and T (2k + 1) = 2k + 2. We write T 2 (n) = T (T (n)) and in general
T k (n) = T k−1 (T (n)) for any k > 1.
(i) Show that for each n ∈ N, there exists k such that T k (n) = 1.
(ii) For k ∈ N, let ck denote the number of elements in the set {n : T k (n) = 1}.
Prove that ck+2 = ck+1 + ck , for k ≥ 1.
4. Suppose 2016 points of the circumference of a circle are coloured red and the
remaining points are coloured blue. Given any natural number n ≥ 3, prove
that there is a regular n-sided polygon all of whose vertices are blue.
5. Let ABC be a right-angled triangle with ∠B = 90◦ . Let D be a point on AC
such that the inradii of the triangles ABD and CBD are equal. If this common
value is r0 and if r is the inradius of triangle ABC, prove that
1 1 1
= + .
r0 r BD

6. Consider a nonconstant arithmetic progression a1 , a2 , . . . , an , . . .. Suppose there


exist relatively prime positive integers p > 1 and q > 1 such that a21 , a2p+1 and
a2q+1 are also the terms of the same arithmetic progression. Prove that the terms
of the arithmetic progression are all integers.

———-000000———-
INMO-2016 problems and solutions

1. Let ABC be triangle in which AB = AC. Suppose the orthocentre of the triangle lies on the
in-circle. Find the ratio AB/BC.
Solution: Since the triangle is isosceles, the or-
thocentre lies on the perpendicular AD from A on
to BC. Let it cut the in-circle at H. Now we are
given that H is the orthocentre of the triangle.
Let AB = AC = b and BC = 2a. Then BD = a.
Observe that b > a since b is the hypotenuse and
a is a leg of a right-angled triangle. Let BH meet
AC in E and CH meet AB in F . By Pythagoras
theorem applied to 4BDH, we get

BH 2 = HD2 + BD2 = 4r2 + a2 ,


where r is the in-radius of ABC. We want to compute BH in another way. Since A, F, H, E are
con-cyclic, we have
BH · BE = BF · BA.
But BF · BA = BD · BC = 2a , since A, F, D, C are con-cyclic. Hence BH 2 = 4a4 /BE 2 . But
2

2
2a2 4a2 (b2 − a2 )

2 2 2 2 2 2
BE = 4a − CE = 4a − BF = 4a − = .
b b2

This leads to
a2 b2
BH 2 = .
b2− a2
Thus we get
a2 b2
= a2 + 4r2 .
b2 − a2
This simplifies to (a4 /(b2 − a2 )) = 4r2 . Now we relate a, b, r in another way using area. We know
that [ABC] = rs, where s is the semi-perimeter of ABC. We have s = (b + b + 2a)/2 = b + a. On
the other hand area can be calculated using Heron’s formula::

[ABC]2 = s(s − 2a)(s − b)(s − b) = (b + a)(b − a)a2 = a2 (b2 − a2 ).

Hence
[ABC]2 a2 (b2 − a2 )
r2 = = .
s2 (b + a)2
Using this we get
a4 a2 (b2 − a2 )
 
=4 .
b2 − a2 (b + a)2
Therefore a2 = 4(b − a)2 , which gives a = 2(b − a) or 2b = 3a. Finally,

AB b 3
= = .
BC 2a 4
Alternate Solution 1:
We use the known facts BH = 2R cos B and r = 4R sin(A/2) sin(B/2) sin(C/2), where R is the
circumradius of 4ABC and r its in-radius. Therefore
π 
HD = BH sin ∠HBD = 2R cos B sin − C = 2R cos2 B,
2
since ∠C = ∠B. But ∠B = (π − ∠A)/2, since ABC is isosceles. Thus we obtain
 
2 π A
HD = 2R cos − .
2 2

However HD is also the diameter of the in circle. Therefore HD = 2r. Thus we get
 
π A
2R cos2 − = 2r = 8R sin(A/2) sin2 ((π − A)/4).
2 2

This reduces to
sin(A/2) = 2 (1 − sin(A/2)) .
Therefore sin(A/2) = 2/3. We also observe that sin(A/2) = BD/AB. Finally
AB AB 1 3
= = = .
BC 2BD 2 sin(A/2) 4

Alternate Solution 2:
Let D be the mid-point of BC. Extend AD to meet the circumcircle in L. Then we know that
HD = DL. But HD = 2r. Thus DL = 2r. Therefore IL = ID + DL = r + 2r = 3r. We also know
that LB = LI. Therefore LB = 3r. This gives
BL 3r 3
= = .
LD 2r 2
But 4BLD is similar to 4ABD. So
AB BL 3
= = .
BD LD 2
Finally,
AB AB 3
= = .
BC 2BD 4
Alternate Solution 3:
Let D be the mid-point of BC and E be the mid-point of DC. Since DI = IH(= r) and DE = EC,
the mid-point theorem implies that IE k CH. But CH ⊥ AB. Therefore EI ⊥ AB. Let EI meet
AB in F . Then F is the point of tangency of the incircle of 4ABC with AB. Since the incircle is
also tangent to BC at D, we have BF = BD. Observe that 4BF E is similar to 4BDA. Hence
AB BE BE BD + DE DE 3
= = = =1+ = .
BD BF BD BD BD 2
This gives
AB 3
= .
BC 4
2. For positive real numbers a, b, c, which of the following statements
necessarily implies a = b = c: (I) a(b3 + c3 ) = b(c3 + a3 ) = c(a3 + b3 ),
(II) a(a3 + b3 ) = b(b3 + c3 ) = c(c3 + a3 ) ? Justify your answer.
Solution: We show that (I) need not imply that a = b = c where as (II) always implies a = b = c.

Observe that a(b3 + c3 ) = b(c3 + a3 ) gives c3 (a − b) = ab(a2 − b2 ). This gives either a = b or


ab(a + b) = c3 . Similarly, b = c or bc(b + c) = a3 . If a 6= b and b 6= c, we obtain

ab(a + b) = c3 , bc(b + c) = a3 .

Therefore
b(a2 − c2 ) + b2 (a − c) = c3 − a3 .
This gives (a − c)(a2 + b2 + c2 + ab + bc + ca) = 0. Since a, b, c are positive, the only possibility is
a = c. We have therefore 4 possibilities: a = b = c; a 6= b, b 6= c and c = a; b 6= c, c 6= a and a = b;
c 6= a, a 6= b and b = c.
Suppose a = b and b, a 6= c. Then b(c3 + a3 ) = c(a3 + b3 ) gives ac3 + a4 = 2ca3 . This implies that
a(a − c)(a2 − ac − c2 ) = 0. Therefore a2√− ac − c2 = 0. Putting a/c = x, we get the quadratic
equation x2 − x − 1 = 0. Hence x = (1 + 5)/2. Thus we get
√ !
1+ 5
a=b= c, c arbitrary positive real number.
2

Similarly, we get other two cases:


√ !
1+ 5
b=c= a, a arbitrary positive real number;
2

√ !
1+ 5
c=a= b, b arbitrary positive real number.
2
And a = b = c is the fourth possibility.
Consider (II): a(a3 + b3 ) = b(b3 + c3 ) = c(c3 + a3 ). Suppose a, b, c are mutually distinct. We may
assume a = max{a, b, c}. Hence a > b and a > c. Using a > b, we get from the first relation that
a3 + b3 < b3 + c3 . Therefore a3 < c3 forcing a < c. This contradicts a > c. We conclude that a, b, c
cannot be mutually distinct. This means some two must be equal. If a = b, the equality of the first
two expressions give a3 + b3 = b3 + c3 so that a = c. Similarly, we can show that b = c implies b = a
and c = a gives c = b.
Alternate for (II) by a contestant: We can write

a3 b3 c3
+ = + a2 ,
c c a
b3 c3 a3
+ = + b2 ,
a a b
c3 a3 b3
+ = + c2 .
b b c
Adding, we get
a3 b3 c3
+ + = a2 + b2 + c2 .
c a b
Using C-S inequality, we have
√ √ √ !2
2 2 2 2 a3 √ b3 √ c3 √
(a + b + c ) = √ · ac + √ · ba + √ · cb
c a b
 3 3 3

a b c 
≤ + + ac + ba + cb
c a b
= (a2 + b2 + c2 )(ab + bc + ca).

Thus we obtain
a2 + b2 + c2 ≤ ab + bc + ca.
However this implies (a − b)2 + (b − c)2 + (c − a)2 ≤ 0 and hence a = b = c.
3. Let N denote the set of all natural numbers. Define a function T : N → N by T (2k) = k and
T (2k + 1) = 2k + 2. We write T 2 (n) = T (T (n)) and in general T k (n) = T k−1 (T (n)) for any k > 1.
(i) Show that for each n ∈ N, there exists k such that T k (n) = 1.
(ii) For k ∈ N, let ck denote the number of elements in the set {n : T k (n) = 1}. Prove that
ck+2 = ck+1 + ck , for k ≥ 1.
Solution:
(i) For n = 1, we have T (1) = 2 and T 2 (1) = T (2) = 1. Hence we may assume that n > 1.
Suppose n > 1 is even. Then T (n) = n/2. We observe that (n/2) ≤ n − 1 for n > 1.
Suppose n > 1 is odd so that n ≥ 3. Then T (n) = n + 1 and T 2 (n) = (n + 1)/2. Again we see that
(n + 1)/2 ≤ (n − 1) for n ≥ 3.
Thus we see that in at most 2(n − 1) steps T sends n to 1. Hence k ≤ 2(n − 1). (Here 2(n − 1) is
only a bound. In reality, less number of steps will do.)

(ii) We show that cn = fn+1 , where fn is the n-th Fibonacci number.


Let n ∈ N and let k ∈ N be such that T k (n) = 1. Here n can be odd or even. If n is even, it can
be either of the form 4d + 2 or of the form 4d.
If n is odd, then 1 = T k (n) = T k−1 (n + 1). (Observe that k > 1; otherwise we get n + 1 = 1 which
is impossible since n ∈ N.) Here n + 1 is even.
If n = 4d + 2, then again 1 = T k (4d + 2) = T k−1 (2d + 1). Here 2d + 1 = n/2 is odd.
Thus each solution of T k−1 (m) = 1 produces exactly one solution of T k (n) = 1 and n is either odd
or of the form 4d + 2.
If n = 4d, we see that 1 = T k (4d) = T k−1 (2d) = T k−2 (d). This shows that each solution of
T k−2 (m) = 1 produces exactly one solution of T k (n) = 1 of the form 4d.
Thus the number of solutions of T k (n) = 1 is equal to the number of solutions of T k−1 (m) = 1 and
the number of solutions of T k−2 (l) = 1 for k > 2. This shows that ck = ck−1 + ck−2 for k > 2. We
also observe that 2 is the only number which goes to 1 in one step and 4 is the only number which
goes to 1 in two steps. Hence c1 = 1 and c2 = 2. This proves that cn = fn+1 for all n ∈ N.
4. Suppose 2016 points of the circumference of a circle are coloured red and the remaining points are
coloured blue. Given any natural number n ≥ 3, prove that there is a regular n-sided polygon all
of whose vertices are blue.
Solution: Let A1 , A2 , . . . , A2016 be 2016 points on the circle which are coloured red and the remain-
ing blue. Let n ≥ 3 and let B1 , B2 , . . . , Bn be a regular n-sided polygon inscribed in this circle with
the vertices chosen in anti-clock-wise direction. We place B1 at A1 . (It is possible, in this position,
some other B’s also coincide with some other A’s.) Rotate the polygon in anti-clock-wise direction
gradually till some B’s coincide with (an equal number of) A’s second time. We again rotate the
polygon in the same direction till some B’s coincide with an equal number of A’s third time, and
so on until we return to the original position, i.e., B1 at A1 . We see that the number of rotations
will not be more than 2016 × n, that is, at most these many times some B’s would have coincided
with an equal number of A’s. Since the interval (0, 360◦ ) has infinitely many points, we can find a

value α ∈ (0, 360◦ ) through which the polygon can be rotated from its initial position such that
no B coincides with any A. This gives a n-sided regular polygon having only blue vertices.
Alternate Solution: Consider a regular 2017 × n-gon on the circle; say, A1 A2 A3 · · · A2017n . For
each j, 1 ≤ j ≤ 2017, consider the points {Ak : k ≡ j (mod 2017)}. These are the vertices of
a regular n-gon, say Sj . We get 2017 regular n-gons; S1 , S2 , . . . , S2017 . Since there are only 2016
red points, by pigeon-hole principle there must be some n-gon among these 2017 which does not
contain any red point. But then it is a blue n-gon.
5. Let ABC be a right-angled triangle with ∠B = 90◦ . Let D be a point on AC such that the in-radii
of the triangles ABD and CBD are equal. If this common value is r0 and if r is the in-radius of
triangle ABC, prove that
1 1 1
0
= + .
r r BD
Solution: Let E and F be the incentres of tri-
angles ABD and CBD respectively. Let the in-
circles of triangles ABD and CBD touch AC in
P and Q respectively. If ∠BDA = θ, we see that

r0 = P D tan(θ/2) = QD cot(θ/2).

Hence
2r0
 
θ θ
P Q = P D + QD = r0 cot + tan = .
2 2 sin θ
But we observe that

BD + DA − AB BD + DC − BC
DP = , DQ = .
2 2
Thus P Q = (b − c − a + 2BD)/2. We also have

ac (AB + BD + DA) (CB + BD + DC)


= [ABC] = [ABD] + [CBD] = r0 + r0
2 2 2
(c + a + b + 2BD)
= r0 = r0 (s + BD).
2
But
P Q sin θ PQ · h
r0 = = ,
2 2BD
where h is the altitude from B on to AC. But we know that h = ac/b. Thus we get
PQ · h (b − c − a + 2BD)ca(s + BD)
ac = 2 × r0 (s + BD) = 2 × (s + BD) = .
2 × BD 2 × BD × b
Thus we get
2 × BD × b = 2 × BD − (s − b))(s + BD).
This gives BD2 = s(s − b). Since ABC is a right-angled triangle r = s − b. Thus we get BD2 = rs.
On the other hand, we also have [ABC] = r0 (s + BD). Thus we get

rs = [ABC] = r0 (s + BD).

Hence
1 1 BD 1 1
= + = + .
r0 r rs r BD

Alternate Solution 1: Observe that


r0 AP CQ AP + CQ
= = = ,
r AX CX AC
where X is the point at which the incircle of ABC touches the side AC. If s1 and s2 are respectively
the semi-perimeters of triangles ABD and CBD, we know AP = s1 − BD and CQ = s2 − BD.
Therefore
r0 (s1 − BD) + (s2 − BD) s1 + s2 − 2BD
= = .
r AC b
But
AD + BD + c CD + BD + a (a + b + c) + 2BD s + BD
s1 + s2 = + = = .
2 2 2 2
This gives
r0 s + BD − 2BD s − BD
= = .
r b b
We also have
[ABD] [CBD] [ABD] + [CBD] [ABC] rs
r0 = = = = = .
s1 s2 s1 + s2 s + BD s + BD
This implies that
r0 s
= .
r s + BD
Comparing the two expressions for r0 /r, we see that
s − BD s
= .
b s + BD
p
Therefore s2 − BD2 = bs, or BD2 = s(s − b). Thus we get BD = s(s − b).
We know now that
p √
r0 s s − BD BD s(s − b) s
= = = = p =√ √ .
r s + BD b (s − b) + BD (s − b) + s(s − b) s−b+ s

Therefore r
r s−b
=1+ .
r0 s
This gives r !
1 1 s−b 1
= + .
r0 r s r
But r ! !  
s−b 1 s−b 1 s−b 1
= p = .
s r s(s − b) r BD r

If ∠B = 90 , we know that r = s − b. Therfore we get
 
1 1 s−b 1 1 1
= + = + .
r0 r BD r r BD

Alternate Solution 2 by a contestant: Ob-


serve that ∠EDF = 90◦ . Hence 4EDP is similar
to 4DF Q. Therefore DP · DQ = EP · F Q. Tak-
ing DP = y1 and DQ = x1 , we get x1 y1 = (r0 )2 .
We also observe that BD = x1 + x2 = y1 + y2 .
Since ∠EBF = 45◦ , we get
tan β1 + tan β2
1 = tan 45◦ = tan(β1 + β2 ) = .
1 − tan β1 tan β2

But tan β1 = r0 /y2 and tan β2 = r0 /x2 . Hence we obtain

(r0 /y2 ) + (r0 /x2 )


1= .
1 − (r0 )2 /x2 y2
Solving for r0 , we get
x2 y2 − x1 y1
r0 = .
x2 + y2
We also know
AB + BC − AC x2 + y2 − (x1 + y1 ) (x2 − x1 ) + (y2 − y1 )
r= = = .
2 2 2
Finally,
1 1 2 1
+ = +
r BD (x2 − x1 ) + (y2 − y1 ) x1 + x2
2x1 + 2x2 + (x2 − x1 ) + (y2 − y1 )
= .
(x1 + x2 )((x2 − x1 ) + (y2 − y1 ))
But we can write
2x1 + 2x2 + (x2 − x1 ) + (y2 − y1 ) = (x1 + x2 + x2 − x1 ) + (y1 + y2 + y2 − y1 ) = 2(x2 + y2 ),
and

(x1 + x2 )((x2 − x1 ) + (y2 − y1 )) = 2(x1 + x2 )(x2 − y1 )


= 2(x2 (x2 + x1 − y1 ) − x1 y1 ) = 2(x2 y2 − x1 y1 ).
Therefore
1 1 2(x2 + y2 ) 1
+ = = 0.
r BD 2(x2 y2 − x1 y1 ) r
Remark: One can also choose B = (0, 0), A = (0, a) and C = (1, 0) and the coordinate geometry
proof gets reduced considerbly.
6. Consider a non-constant arithmetic progression a1 , a2 , . . . , an , . . .. Suppose there exist relatively
prime positive integers p > 1 and q > 1 such that a21 , a2p+1 and a2q+1 are also the terms of the same
arithmetic progression. Prove that the terms of the arithmetic progression are all integers.
Solution: Let us take a1 = a. We have

a2 = a + kd, (a + pd)2 = a + ld, (a + qd)2 = a + md.

Thus we have

a + ld = (a + pd)2 = a2 + 2pad + p2 d2 = a + kd + 2pad + p2 d2 .

Since we have non-constant AP, we see that d 6= 0. Hence we obtain 2pa + p2 d = l − k. Similarly,
we get 2qa + q 2 d = m − k. Observe that p2 q − pq 2 6= 0. Otherwise p = q and gcd(p, q) = p > 1
which is a contradiction to the given hypothesis that gcd(p, q) = 1. Hence we can solve the two
equations for a, d:
p2 (m − k) − q 2 (l − k) q(l − k) − p(m − k)
a= , d= .
2(p2 q − pq 2 ) p2 q − pq 2
It follows that a, d are rational numbers. We also have

p2 a2 = p2 a + kp2 d.

But p2 d = l − k − 2pa. Thus we get

p2 a2 = p2 a + k(l − k − 2pa) = (p − 2k)pa + k(l − k).

This shows that pa satisfies the equation

x2 − (p − 2k)x − k(l − k) = 0.

Since a is rational, we see that pa is rational. Write pa = w/z, where w is an integer and z is a
natural numbers such that gcd(w, z) = 1. Substituting in the equation, we obtain

w2 − (p − 2k)wz − k(l − k)z 2 = 0.

This shows z divides w. Since gcd(w, z) = 1, it follows that z = 1 and pa = w an integer. (In
fact any rational solution of a monic polynomial with integer coefficients is necessarily an integer.)
Similarly, we can prove that qa is an integer. Since gcd(p, q) = 1, there are integers u and v such
that pu + qv = 1. Therefore a = (pa)u + (qa)v. It follows that a is an integer.
But p2 d = l−k−2pa. Hence p2 d is an integer. Similarly, q 2 d is also an integer. Since gcd(p2 , q 2 ) = 1,
it follows that d is an integer. Combining these two, we see that all the terms of the AP are integers.

Alternatively, we can prove that a and d are integers in another way. We have seen that a and d
are rationals; and we have three relations:

a2 = a + kd, p2 d + 2pa = n1 , q 2 d + 2qa = n2 ,

where n1 = l − k and n2 = m − k. Let a = u/v and d = x/y where u, x are integers and v, y are
natural numbers, and gcd(u, v) = 1, gcd(x, y) = 1. Putting this in these relations, we obtain

u2 y = uvy + kxv 2 , (1)


2
2puy + p vx = vyn1 , (2)
2
2quy + q vx = vyn2 . (3)
Now (1) shows that v|u2 y. Since gcd(u, v) = 1, it follows that v|y. Similarly (2) shows that
y|p2 vx. Using gcd(y, x) = 1, we get that y|p2 v. Similarly, (3) shows that y|q 2 v. Therefore y divides
gcd(p2 v, q 2 v) = v. The two results v|y and y|v imply v = y, since both v, y are positive.
Substitute this in (1) to get
u2 = uv + kxv.
This shows that v|u2 . Since gcd(u, v) = 1, it follows that v = 1. This gives v = y = 1. Finally
a = u and d = x which are integers.

———-000000———-
32nd Indian National Mathematical Olympiad-2017
Time: 4 hours January 15, 2017
Instructions:
• Calculators (in any form) and protractors are not allowed.
• Rulers and compasses are allowed.
• All questions carry equal marks. Maximum marks: 102.
• Answer all the questions.
• Answer to each question should start on a new page. Clearly indicate
the question number.

1. In the given figure, ABCD is a square sheet


of paper. It is folded along EF such that A
goes to a point A′ different from B and C, on
the side BC and D goes to D′ . The line A′ D′
cuts CD in G. Show that the inradius of the
triangle GCA′ is the sum of the inradii of the
triangles GD′ F and A′ BE.
2. Suppose n ≥ 0 is an integer and all the roots of x3 + αx + 4 − (2 × 2016n ) = 0
are integers. Find all possible values of α.
3. Find the number of triples (x, a, b) where x is a real number and a, b belong to
the set {1, 2, 3, 4, 5, 6, 7, 8, 9} such that
x2 − a{x} + b = 0,
where {x} denotes the fractional part of the real number x. (For example
{1.1} = 0.1 = {−0.9}.)
4. Let ABCDE be a convex pentagon in which ∠A = ∠B = ∠C = ∠D = 120◦
and side lengths are five consecutive integers in some order. Find all possible
values of AB + BC + CD.
5. Let ABC be a triangle with ∠A = 90◦ and AB < AC. Let AD be the altitude
from A on to BC. Let P, Q and I denote respectively the incentres of triangles
ABD, ACD and ABC. Prove that AI is perpendicular to P Q and AI = P Q.
6. Let n ≥ 1 be an integer and consider the sum
Xn  
n n
 
n n−2
 
n n−4 2
x= 2n−2k 3k = 2 + 2 ·3+ 2 · 3 + ··· .
2k 0 2 4
k≥0

Show that 2x − 1, 2x, 2x + 1 form the sides of a triangle whose area and inradius
are also integers.
———-000000———-
32व ँ भ रतीय र ्रीय गणित ओल पं िय ड – 2017
समय: 4 घंट जनवरी 15, 2017
ननदे श :

 किसी भी तरह ि गणि (calculators) तथा चादा (protractors) ि रयोग िी अनुमतत नह ह.

 पमाना ( u e ) तथा परिार (c a e ) ि रयोग िी अनुमतत ह.


 सभी र्नं ि अि एिसमान हं. अधिितम अि : 102.
 सभी र्नं ि उ्तर द जिय.
 र्यि र्न िा उ्तर नए पि स रारभ िीजिय. र्न रमाि ्प्ट प स इधगत िीजिय.

1. ददए हुए धचर मं एि वगाािार िागि ह. इस ि पररत: इस


रिार मोड़ा िाता ह कि , ब दु तथा स भभlन भुिा पर ब द ु

पर आ िाता ह तथा , ब द ु ′ पर िाता ह. रखा ′ ′, िो
पर िाटती ह. ददखाईय कि बरभुि ′ िी अत:बर्या बरभुिं ′
तथा ′ िी अत: बर्याओ ि योग ि रा र ह.
2. मान ल जिय कि � ≥ एि पण
ू ांि ह तथा � + �� + − × �
= ि सभी मल
ू पण
ू ांि हं.
� ि सभी सभाववत मान ञात िीजिय.
3. उन सभी बरयुममं �, , िी स्याए ञात िीजिय जिसमं कि � एि वा्तववि स्या ह तथा ,
समु्चय { , , , , , , , , } स इस रिार स धित ह कि
� − {�} + =
िहा {�} वा्तववि स्या � िा भभlना्मि भाग ह. (उदाहरण ि भलए { . } = . = {− . }.
4. मान ल जिय कि एि उ्तल पचभुि (convex pentagon) ह जिसमं ∠ =∠ =∠ =∠ =
° ह तथा भुिाओ िी ल् ाई किसी रम मं पाच रमागत पूणांि (consecutive integer) हं.
+ + ि सभी सभव मान ञात िीजिय.
5. मान ल जिय कि एि बरभुि ह जिसमं ∠ = ° तथा < . मान ल जिय कि , स
पर शी्ाल् ह. मान ल जिय कि , तथा � रमश: बरभुि , तथा ि अत:िंरं िो
तन वपत िरत हं. भसध िीजिय कि �, ि ल् वत ह तथा �= .
6. मान ल जिय कि � ≥ एि पूणांि ह. तन्न योग पर ववचार िीजिय
� �− � �
� �
� �−
� �−
�=∑ = + ⋅ + ⋅ +⋯ .

�≥0
ददखाइए कि �− , �, �+ उस बरभुि िी भुिाओ िो नात हं जिसिा षरफल तथा अत:बर्या
भी एि पूणांि हं.
------------000000-----------
32nd Indian National Mathematical Olympiad-2017
Problems and Solutions

1. In the given figure, ABCD is a square paper.


It is folded along EF such that A goes to a
point A0 6= C, B on the side BC and D goes
to D0 . The line A0 D0 cuts CD in G. Show
that the inradius of the triangle GCA0 is the
sum of the inradii of the triangles GD0 F and
A0 BE.

Solution: Observe that the triangles GCA0 and A0 BE are similar to the triangle GD0 F . If
GF = u, GD0 = v and D0 F = w, then we have

A0 G = pu, CG = pv, A0 C = pw, A0 E = qu, BE = qw, A0 B = qv.

If r is the inradius of 4GD0 F , then pr and qr are respectively the inradii of triangles GCA0
and A0 BE. We have to show that pr = r + qr. We also observe that

AE = EA0 , DF = F D0 .

Therefore
pw + qv = qw + qu = w + u + pv = v + pu.
The last two equalities give (p−1)(u−v) = w. The first two equalities give (p−q)w = q(u−v).
Hence
p−q u−v 1
= = .
q w p−1
This simplifies to p(p−q −1) = 0. Since p 6= 0, we get p = q +1. This implies that pr = qr+r.
2. Suppose n ≥ 0 is an integer and all the roots of x3 + αx + 4 − (2 × 2016n ) = 0 are integers.
Find all possible values of α.

Solution 1: Let u, v, w be the roots of x3 + αx + 4 − (2 × 2016n ) = 0. Then u + v + w = 0


and uvw = −4 + (2 × 2016n ). Therefore we obtain

uv(u + v) = 4 − (2 × 2016n ).

Suppose n ≥ 1. Then we see that uv(u + v) ≡ 4 (mod 2016n ). Therefore uv(u + v) ≡ 1


(mod 3) and uv(u + v) ≡ 1 (mod 9). This implies that u ≡ 2 (mod 3) and v ≡ 2 (mod 3).
This shows that modulo 9 the pair (u, v) could be any one of the following:

(2, 2), (2, 5), (2, 8), (5, 2), (5, 5), (5, 8), (8, 2), (8, 5), (8, 8).

In each case it is easy to check that uv(u + v) 6≡ 4 (mod 9). Hence n = 0 and uv(u + v) = 2.
It follows that (u, v) = (1, 1), (1, −2) or (−2, 1). Thus

α = uv + vw + wu = uv − (u + v)2 = −3

for every pair (u, v).


Solution 2: Let a, b, c ∈ Z be the roots of the given equation for some n ∈ N0 . By Vieta
Theorem, we know that
a+b+c=0
ab + bc + ca = α
abc = 2 × 2016n − 4
If possible, let us have n ≥ 1. Since 7|2016, we have that

7|abc + 4 =⇒ 7|3(abc + 4) =⇒ 7|3abc + 12 =⇒ 7|3abc + 5

Since we have a + b + c = 0, we get that 3abc = a3 + b3 + c3 . Substituting this in the earlier


expression, we get that
a3 + b3 + c3 + 5 ≡ 0 (mod 7)
Consider below, a table calculating the residues of cubes modulo 7.

x 0 1 2 3 4 5 6
x3 0 1 1 −1 1 −1 −1

Hence, we know that if x ∈ N, then we have x3 ≡ 0, 1, −1 (mod 7). Since a3 + b3 + c3 ≡ 2


(mod 7), we see that we must have one of the numbers divisible by 7 and the other two
numbers, when cubed, must leave 1 as remainder modulo 7. Without of generality, let us
assume that
a ≡ 0 (mod 7), b3 , c3 ≡ 1 (mod 7)
Hence, we have b, c ≡ 1, 2, 4 (mod 7). We will consider all possible values of b + c modulo 7.
Since the expression is symmetric in b, c, modulo 7, we will consider b ≤ c.

b 1 1 1 2 2 4
c 1 2 4 2 4 4
b+c 2 3 5 4 6 1

We see that, in all the above cases, we get 76 | b+c. But this is a contradiction, since 7|a+b+c
and 7|a together imply that 7|b + c. Hence, we cannot have n ≥ 1. Hence, the only possible
value is n = 0. Substituting this value in the original equation, the equation becomes

x3 + αx + 2 = 0

Solving the equations a + b + c = 0 and abc = −2 in integers, we see that the only possible
solutions (a, b, c) are permutations of (1, 1, −2). In case of any permutation, α = −3. Substi-
tuting this value of α back in the equation, we see that we indeed, get integer roots. Hence,
the only possible value for α is −3.

3. Find the number of triples (x, a, b) where x is a real number and a, b belong to the set
{1, 2, 3, 4, 5, 6, 7, 8, 9} such that
x2 − a{x} + b = 0,
where {x} denotes the fractional part of the real number x. (For example {1.1} = 0.1 =
{−0.9}.)
Solution: Let us write x = n + f where n = [x] and f = {x}. Then

f 2 + (2n − a)f + n2 + b = 0. (1)

Observe that the product of the roots of (1) is n2 + b ≥ 1. If this equation has to have a
solution 0 ≤ f < 1, the larger root of (1) is greater 1. We conclude that the equation (1) has
a real root less than 1 only if P (1) < 0 where P (y) = y 2 + (2n − a)y + n2 + 2b. This gives

1 + 2n − a + n2 + 2b < 0.

Therefore we have (n + 1)2 + b < a. If n ≥ 2, then (n + 1)2 + b ≥ 10 > a. Hence n ≤ 1. If


n ≤ −4, then again (n + 1)2 + b ≥ 10 > a. Thus we have the range for n: −3, −2, −1, 0, 1.
If n = −3 or n = 1, we have (n + 1)2 = 4. Thus we must have 4 + b < a. If a = 9, we
must have b = 4, 3, 2, 1 giving 4 values. For a = 8, we must have b = 3, 2, 1 giving 3 values.
Similarly, for a = 7 we get 2 values of b and a = 6 leads to 1 value of b. In each case we get a
real value of f < 1 and this leads to a solution for x. Thus we get totally 2(4 + 3 + 2 + 1) = 20
values of the triple (x, a, b).
For n = −2 and n = 0, we have (n + 1)2 = 1. Hence we require 1 + b < a. We again count
pairs (a, b) such that a − b > 1. For a = 9, we get 7 values of b; for a = 8 we get 6 values of
b and so on. Thus we get 2(7 + 6 + 5 + 4 + 3 + 2 + 1) = 56 values for the triple (x, a, b).
Suppose n = −1 so that (n + 1)2 = 0. In this case we require b < a. We get 8 + 7 + 6 + 5 +
4 + 3 + 2 + 1 = 36 values for the triple (x, a, b).
Thus the total number of triples (x, a, b) is 20 + 56 + 36 = 112.

4. Let ABCDE be a convex pentagon in which ∠A = ∠B = ∠C = ∠D = 120◦ and whose side


lengths are 5 consecutive integers in some order. Find all possible values of AB + BC + CD.

Solution 1: Let AB = a, BC = b, and CD = c. By symmetry, we may assume that c < a.


We show that DE = a + b and EA = b + c.

Draw a line parallel to BC through D. Extend EA to meet this line at F . Draw a line
parallel to CD through B and let it intersect DF in G. Let AB intersect DF in H. We have
∠F DE = 60◦ and ∠E = 60◦ . Hence EF D is an equilateral triangle. Similarly AF H and
BGH are also equilateral triangles. Hence HG = GB = c. Moreover, DG = b. Therefore
HD = b + c. But HD = AE since F H = F A and F D = F E. Also AH = a − BH =
a − BG = a − c. Hence ED = EF = EA + AF = b + c + AH = (b + c) + (a − c) = b + a.

We have five possibilities:


(1) b < c < a < b + c < a + b;
(2) c < b < a < b + c < a + b;
(3) c < a < b < b + c < a + b;
(4) b < c < b + c < a < a + b;
(5) c < b < b + c < a < a + b.

In (1), we see that c < a < b + c are three consecutive integers provided b = 2. Hence we
get c = 3 and a = 4. In this case b + c = 5 and a + b = 6 so that we have five consecutive
integrs 2, 3, 4, 5, 6 as side lengths. In (2), b < a < b + c form three consecutive integrs only
when c = 2. Hence b = 3, a = 4. But then b + c = 5 and a + b = 7. Thus the side lengths
are 2, 3, 4, 6, 7 which are not consecutive integers. In case (3), b < b + c are two consecutive
integrs so that c = 1. Hence a = 2 and b = 3. We get b + c = 4 and a + b = 5 so that the
consecutive integers 1, 2, 3, 4, 5 form the side lengths. In case (4), we have c < b + c as two
consecutive integers and hence b = 1. Therefore c = 2, b + c = 3, a = 4 and a + b = 5 which
is admissible. Finally, in case (5) we have b < b + c as two consecutive integers, so that c = 1.
Thus b = 2, b + c = 3, a = 4 and a + b = 6. We do not get consecutive integers.

Therefore the only possibilities are (a, b, c) = (4, 2, 3), (2, 3, 1) and (4, 1, 2). This shows that
a + b + c = 9, 6 or 7. Thus there are three possible sums AB + BC + CA, namely, 6, 7 or 9.

Solution 2: As in the earlier solution, ED = d = a + b and EA = e = b + c. Let the sides


be x − 2, x − 1, x, x + 1, x + 2. Then x ≥ 3. We also have x + 2 ≥ x − 1 + x − 2 so that x ≤ 5.
Thus x = 3, 4 or 5. If x = 5, the sides are {3, 4, 5, 6, 7} and here we do not have two pairs
which add to a number in the set. Hence x = 3 or 4 and we get the sets as {1, 2, 3, 4, 5} or
{2, 3, 4, 5, 6}. With the set {1, 2, 3, 4, 5} we get
(a, b, c, d, e) = (2, 3, 1, 5, 4), (4, 1, 2, 5, 3).
From the set {2, 3, 4, 5, 6}, we get (a, b, c, d, e) = (4, 2, 3, 6, 5). Thus we see that a+b+c = 6, 7
or 9.

Solution 3: We use the same notations and we get d = a + b and e = b + c. If a ≥ 5, we see


that d − b ≥ 5. But the maximum difference in a set of 5 consecutive integers is 4. Hence
a ≤ 4. Similarly, we see b ≤ 4 and c ≤ 4. Thus we see that a + b + c ≤ 2 + 3 + 4 = 9.
But a + b + c ≥ 1 + 2 + 3 = 6. It follows that a + b + c = 6, 7, 8 or 9. If we take
(a, b, c, d, e) = (1, 3, 2, 4, 5), we get a + b + c = 6. Similarly, (a, b, c, d, e) = (2, 1, 4, 3, 5)
gives a + b + c = 7, For a + b + c = 8, the only we we can get 1 + 3 + 4 = 8. Here we cannot
accommodate 2 and consecutiveness is lost. For 9, we can have (a, b, c, d, e) = (3, 2, 4, 5, 6)
and a + b + c = 9.
5. Let ABC be a triangle with ∠A = 90◦ and AB < AC. Let AD be the altitude from A on
to BC. Let P, Q and I denote respectively the incentres of triangles ABD, ACD and ABC.
Prove that AI is perpendicular to P Q and AI = P Q.

Solution: Draw P S k BC and QS k AD.


Then P SQ is a right-angled triangle with
∠P SQ = 90◦ . Observe that P S = r1 + r2
and SQ = r2 − r1 , where r1 and r2 are the
inradii of triangles ABD and ACD, respec-
tively. We observe that triangles DAB and
DCA are similar to triangle ACB.
Hence
c b
r1 = r, r2 = r,
a a
where r is the inradius of triangle ABC. Thus we get
PS r2 + r1 b+c
= = .
SQ r2 − r1 b−c

On the otherhand AD = h = bc/a. We also have BE = ca/(b + c) and

b2 c 2 c4
BD2 = c2 − h2 = c2 − = .
a2 a2
Hence BD = c2 /a. Therefore

ca c2 cb(b − c)
DE = BE − BD = − = .
b+c a a(b + c)

Thus we get
AD b+c PS
= = .
DE b−c SQ
Since ∠ADE = 90◦ = ∠P SQ, we conclude that 4ADE ∼ 4P SQ. Since AD ⊥ P S, it
follows that AE ⊥ P Q.
We also observe that

P Q2 = P S 2 + SQ2 = (r2 + r1 )2 + (r2 − r1 )2 = 2(r12 + r22 ).

However
c2 + b2 2
r12 + r22 = r = r2 .
a2
√ √
Hence P Q = 2r. We also observe that AI = rcosec(A/2) = rcosec(45◦ ) = 2r. Thus
P Q = AI.

Solution 2: In the figure, we have made the construction as mentioned in the hint. Since P, Q
are the incentres of 4ABD, 4ACD, DP, DQ are the internal angle bisectors of ∠ADB, ∠ADC
respectively. Since AD is the altitude on the hypotenuse BC in 4ABC, we have that
∠P DQ = 45◦ + 45◦ = 90◦ . It also implies that

4ABC ∼ 4DBA ∼ 4DAC

This implies that all corresponding length in the above mentioned triangles have the same
ratio.
In particular,
AI DP DQ
= =
BC AB AC
AI 2 DP 2 DQ2 DP 2 + DQ2
=⇒ = = =
BC 2 AB 2 AC 2 AB 2 + AC 2
2 2
AI PQ
=⇒ = , by Pythagoras Theorem in 4ABC, 4P DQ
BC 2 BC 2
=⇒ AI = P Q

as required.

For the second, part, we note that from the above relations, we have 4ABC ∼ 4DP Q. Let
us take ∠ACB = θ. Then, we get

∠P SD = 180◦ − (∠SP D + ∠SDP )


= 180◦ − (90◦ − θ + 45◦ )
= 45◦ + θ

This gives us that

∠ARS = 180◦ − (∠ASR + ∠SAR)


= 180◦ − (∠P SD + ∠SAC − ∠IAC)
= 180◦ − (45◦ + θ + 90◦ − θ − 45◦ )
= 90◦

as required. Hence, we get that AI = P Q and AI ⊥ P Q.

Solution 3: We know that the angle bisector of ∠B passes through P, I which implies that
B, P, I are collinear. Similarly, C, Q, I are also collinear. Since I is the incentre of 4ABC,
we know that
∠A
∠P IQ = ∠BIC = 90◦ + = 135◦
2
Join AP, AQ. We know that ∠BAP = 21 ∠BAD = 12 ∠C. Also, ∠ABP = 21 ∠B. Hence by
Exterior Angle Theorem in 4ABP , we get that
1
∠AP I = ∠ABP + ∠BAP = (∠B + ∠C) = 45◦
2
Similarly in 4ADC, we get that ∠AQI = 45◦ . Also, we have
∠C ∠B
∠P AI = ∠BAI − ∠BAP = 45◦ − =
2 2
∠C
Similarly, we get ∠QAI = 2 .

Now applying Sine Rule in 4AP I, we get


IP AI √ B
= =⇒ IP = 2AI sin
sin ∠P AI sin ∠AP I 2
Similarly, applying Sine Rule in 4AQI, we get
IQ AI √ C
= =⇒ IQ = 2AI sin
sin ∠P AI sin ∠AQI 2
Applying Cosine Rule in 4P IQ gives us that

P Q2 = IP 2 + IQ2 − 2 · IP · IQ cos ∠P IQ

 
2 2 B 2 C B C
= 2AI sin + sin + 2 sin sin
2 2 2 2

We will prove that sin2 B2 + sin2 C2 + 2 sin B2 sin C2 = 21 . In any 4XY Z, we have that


X X Y X
sin2 =1−2 sin
cyc
2 2

Using this in 4ABC, and using the fact that ∠A = 90◦ , we get
A B C A B C
sin2 + sin2 + sin2 = 1 − 2 sin sin sin
2 2 2 2 2 2
1 2 B 2 C
√ B C
=⇒ + sin + sin = 1 − 2 sin sin
2 2 2 2 2


B C B C 1
=⇒ sin2 + sin2 + 2 sin sin =
2 2 2 2 2

which was to be proved. Hence we get P Q = AI.

The second part of the problem can be obtained by angle-chasing as outlined in Solution 2.

Solution 4: Observe that ∠AP B = ∠AQC = 135◦ . Thus ∠AP I = ∠AQI = 45◦ (since
B − P − I and C − Q − I). Note ∠P AQ = 1/2∠A = 45◦ . Let X = BI ∩ AQ and
Y = CI ∩ AP . Therefore ∠AXP = 180 − ∠AP I − ∠P AQ = 90◦ . Similarly ∠AY Q = 90◦ .
Hence I is the orthocentre of triangle P AQ. Therefore AI is perpendicular to P Q. Also
AI = 2RP AQ cos 45◦ = 2RP AQ sin 45◦ = P Q.
6. Let n ≥ 1 be an integer and consider the sum
Xn  
n n
 
n n−2
 
n n−4 2
n−2k k
x= 2 3 = 2 + 2 ·3+ 2 · 3 + ··· .
2k 0 2 4
k≥0

Show that 2x − 1, 2x, 2x + 1 form the sides of a triangle whose area and inradius are also
integers.

Solution: Consider the binomial expansion of (2 + 3)n . It is easy to check that
√ √
(2 + 3)n = x + y 3,

where y is also an integer. We also have


√ √
(2 − 3)n = x − y 3.

Multiplying these two relations, we obtain x2 − 3y 2 = 1.



Since all the terms of the expansion of (2 + 3)n are positive, we see that
√ √
   
n n−2
2x = (2 + 3)n + (2 − 3)n = 2 2n + 2 · 3 + · · · ≥ 4.
2
Thus x ≥ 2. Hence 2x + 1 < 2x + (2x − 1) and therefore 2x − 1, 2x, 2x + 1 are the sides of a
triangle. By Heron’s formula we have

∆2 = 3x(x + 1)(x)(x − 1) = 3x2 (x2 − 1) = 9x2 y 2 .

Hence ∆ = 3xy which is an integer. Finally, its inradius is


area 3xy
= = y,
perimeter 3x
which is also an integer.

Solution 2: We will first show that the numbers 2xn − 1, 2xn , 2xn + 1 form the sides of a
triangle. To show that, it suffices to prove that 2xn − 1 + 2xn > 2xn + 1. If possible, let
the converse hold. Then, we see that we must have 4xn − 1 ≤ 2xn + 1, which implies that
xn ≤ 1. But we see that even for the smallest value of n = 1, we have that xn > 1. Hence,
the numbers are indeed sides of a triangle.

Let ∆n , rn , sn denote respectively, the area, inradius and semiperimeter of the triangle with
sides 2xn − 1, 2xn , 2xn + 1. By Heron’s Formula for the area of a triangle, we see that
p p
∆n = 3xn (xn − 1)xn (xn + 1) = xn 3(x2n − 1)

 let ∆n be an integer for all n ∈ N. We see that due to the presence of the first
If possible,
term n0 2n , we have 36 | xn , ∀n ∈ N. Hence, we get that 3|x2n − 1. Hence, we can write x2n − 1
as 3m for some m ∈ N. Then, we can also write

∆n = 3xn m

Note that we have assumed that ∆n is an integer. Hence, we see that we must have m to be
a perfect square. Consequently, we get that
∆n ∆n √
rn = = = m∈Z
sn 3xn
Hence, it only remains to show that ∆n ∈ Z, ∀n ∈ N. In other words, it suffices to show
that 3(x2n − 1) is a perfect square for all n ∈ N.

We see that we can write xn as


 
 
1 X n n−2k k 
xn = 2 2 3
2 2k
k≥0
1 √ √ 
= (2 + 3)n + (2 − 3)n
2
2 3 √ √ √ √ 
3xn − 3 = (2 + 3)2n + (2 − 3)2n + 2(2 + 3)n (2 − 3)n − 3
4
3 √ √ √ √ 
= (2 + 3)2n + (2 − 3)2n − 2(2 + 3)n (2 − 3)n
4
√  !2
3 √ n √ n
= (2 + 3) − (2 − 3)
2

We are left to show that the quantity obtained in the above equation is an integer. But we
see that if we define
√ 
3 √ √ 
an = (2 + 3)n − (2 − 3)n , ∀n ∈ N
2
the sequence hak i∞
k=1 thus obtained is exactly the solution for the recursion given by

an+2 = 4an+1 − an , ∀n ∈ N, a1 = 3, a2 = 12

Hence, clearly, each an is obviously an integer, thus completing the proof.

———-000000———-

También podría gustarte